Está en la página 1de 216

RAZONAMIENTO

VERBAL

LINGÜÍSTICA

5°Secundaria
ÍNDICE
RAZONAMIENTO VERBAL
»» MISCELÁNEA I  7
»» ESCRITURA DE UN TEXTO  10
»» ANALOGÍAS  15
»» LA REFERENCIA: TIPOS DE REFERENTES  19
»» LA INFERENCIA  23
»» LOS CONECTORES  28
»» ACENTUACIÓN GENERAL Y ESPECIAL  32
»» TALLER DE REDACCIÓN  36
»» PLAN DE REDACCIÓN I  40
»» TESIS Y ARGUMENTOS  44
»» ELIMINACIÓN DE ORACIONES I  49
»» PREGUNTAS DE REFORZAMIENTO Y DEBILITAMIENTO DE ARGUMENTOS  53
»» COHERENCIA Y COHESIÓN DE UN TEXTO  57
»» LA COMA  62
»» INCLUSIÓN DE ENUNCIADOS  66
»» ORACIONES INCOMPLETAS  70
»» PREGUNTAS POR EXTRAPOLACIÓN  73
»» DIÁLOGOS – PUNTO DE DISCREPANCIA  77
»» ANÁLISIS DE IMÁGENES  82
»» INCLUSIÓN E IMPLICACIÓN  86
»» USO DE LA B, V, G Y J  89
»» LECTURA CRÍTICA: TONO, PARADOJA Y ESTRUCTURA DE UN TEXTO  93
»» CONECTORES LÓGICOS - ORACIONES INCOMPLETAS  98
»» LA REFERENCIA  101
»» INCLUSIÓN E IMPLICACIÓN  103
»» PLAN DE REDACCIÓN II  105
»» ELIMINACIÓN DE ORACIONES II  110
ÍNDICE
LINGÜÍSTICA
»» LA COMUNICACIÓN  117
»» LENGUAJE: CARACTERÍSTICAS Y FUNCIONES  121
»» PLANOS DEL LENGUAJE  126
»» FONÉTICA Y FONOLOGÍA  129
»» MULTILINGÜISMO  134
»» MORFOLOGÍA: FORMACIÓN DE PALABRAS  138
»» TILDE DIACRÍTICA  143
»» EL SUSTANTIVO  146
»» EL ADJETIVO CALIFICATIVO  149
»» DETERMINATIVOS: ADJETIVOS DETERMINATIVOS  153
»» EL ARTÍCULO  157
»» PRONOMBRE Y SUS CLASES  160
»» EL VERBO Y SUS ACCIDENTES GRAMATICALES  164
»» MAYÚSCULAS  168
»» CLASIFICACIÓN DE LOS VERBOS I  172
»» CLASIFICACIÓN DE LOS VERBOS II  176
»» EL ADVERBIO Y SUS CLASES  179
»» LA PREPOSICIÓN  182
»» LA CONJUNCIÓN  185
»» LA ORACIÓN: CLASIFICACIÓN SEGÚN LA ACTITUD DEL HABLANTE  188
»» USO DEL PUNTO Y COMA, EL PUNTO Y LOS DOS PUNTOS  192
»» EL PREDICADO I  196
»» EL PREDICADO II  199
»» ORACIÓN COMPUESTA  202
»» ORACIÓN SUBORDINADA SUSTANTIVA  206
»» ORACIÓN SUBORDINADA ADJETIVA  210
»» ORACIONES SUBORDINADAS ADVERBIALES  213
Razonamiento
Verbal
Colegio

RAZONAMIENTO VERBAL  Mejores Personas, Mejores Familias

M A
TE

01   MISCELÁNEA I

I. SINONIMIA II. ANTONIMIA


Los sinónimos son palabras que tienen significado parecido. Los antónimos son palabras que tienen significado opuesto.
Ejemplo: Ejemplo:
YY bardo : poeta YY díscolo : dócil
YY boato : ostentación YY coloquial : formal
YY jumento : asno YY venusto : feo
YY escueto : breve YY voluble : decisivo

dadivoso generoso tacaño

Recuerda
Tanto antónimos
como sinónimos
deben pertenecer a
la misma categoría adjetivo
gramatical.

III. SERIES VERBALES


Conjunto de palabras que comparten el mismo campo semántico (características comunes).
Ejemplos:
YY Soso – insípido – sabroso
YY Hediondo – fétido – oloroso
YY Bizarría – valentía – coraje

IV. TÉRMINO EXCLUIDO


Consiste en señalar el elemento que no guarda relación con los demás, bajos ciertos criterios.
Ejemplos:

ACICALAR LLEVAR
a) Asear a) Transportar
b) Ordenar b) Enviar
c) Arreglar c) Pasar
d) Meditar d) Guiar
e) Limpiar e) Entonar

5º AÑO DE SECUNDARIA
Mejores Personas, Mejores Familias 7
Colegio

Mejores Personas, Mejores Familias RAZONAMIENTO VERBAL

8. La mayoría de los ríos costeños emanan de la Cordillera


de los Andes.
a) salen
b) nacen
Resuelve los siguientes ejercicios. c) emiten
Término excluido d) exhalan

1. REFORMAR 9. La comida le salió totalmente insulsa.


a) Variar a) zonza
b) Cambiar b) simple
c) Alterar c) inexpresiva
d) Modifica d) insípida
e) Organizar
10. Estoy abrumado con tanto trabajo que no se que hacer
2. CIENCIA esta semana.
a) Conocimiento a) atosigado
b) Método b) aliviado
c) Sistematización c) estimulado
d) Conclusión d) calmado
e) Transformación

3. JUGLAR
a) Vate
b) Poeta
c) Rapsoda Antónimos
d) Gracioso 1. PRESBÍTERO
e) Trovador a) Ateo
b) Lego
4. MÓVIL c) Pecador
a) Causa d) Escéptico
b) Motivo e) Incrédulo
c) Origen
d) Razón 2. AMARGAR
e) Interés a) Cumplir
b) Ejecutar
5. COLOR c) Disimular
a) Rojo d) Esquivar
b) Azul e) Afrontar
c) Verde
d) Gris 3. FACINEROSO
e) Amarillo a) Cortés
b) Benévolo
Series verbales c) Sumiso
6. Completa la serie con los términos. d) Atractivo
a) Abstruso, complicado, difícil, ___________ e) Inocuo
b) Mofletudo, flaco, enclenque, ____________
c) Obeso, adiposo, gordo, ________________ Sinonimia
d) Bisoño, novato, principiante, ___________ Coloca el sinónimo de las palabras resaltadas en el texto.
e) Lúgubre, tenebroso, sombrío, ___________ TEXTO
El sonido es la vibración de moléculas que se transmite a
Sinonimia contextual través de la materia. El oído humano está construido para
Identifica el sinónimo de las palabras resaltadas. recibir sonidos transmitidos a través del aire, en un rango de
frecuencias que va desde los 20 ciclos por segundo hasta los
7. Si conduces sin brevete debes eludir a la policía. 20 000, es decir, desde los «ciclos» más bajos hasta los «ciclos»
a) saltear más agudos que es capaz de registrar. Estas vibraciones son
b) correr transmitidas a través del aire que nos rodea hasta el tímpano,
c) evitar una membrana como el cuero de un tambor, en el extremo
d) desafiar interior del conducto auditivo. Conectados al tímpano hay

8 Mejores Personas, Mejores Familias


5º AÑO DE SECUNDARIA
Colegio

RAZONAMIENTO VERBAL  Mejores Personas, Mejores Familias

una serie de huesecillos (llamados por su forma «yunque» 2. Antes de enviar las cartas por correo, hay que timbrar los
, «martillo», «estribo») que transforman las vibraciones en sobres.
impulsos eléctricos que luego son transmitidos al cerebro por a) franquear c) pagar e) comprar
el nervio auditivo. b) chequear d) preparar

3. La televisión va a diferir la inauguración de las próximas


4. _____________ _____________ Olimpiadas.
5. _____________ _____________ a) adelantar c) poner e) comentar
b) coincidir d) demorar
6. _____________ _____________
Sinónimos
7. _____________ _____________ 4. PROFILAXIA
8. _____________ _____________ a) Limpieza c) Pretensión e) Esterilización
b) Prevención d) Integración
Término excluido
9. CENSURA 5. PROLIJO
a) Crítica a) Detallista c) Productivo e) Audaz
b) Juicio b) Abundante d) Congruente
c) Cenefa
d) Reprobación 6. TRAMAR
e) Murmuración a) Demudar c) Confuso e) Dorar
b) Intrigar d) Malograr
10. EXPANDIR
a) Ensanchar Antónimos
b) Difundir 7. EUFONÍA
c) Propagar a) Afonía c) Cacofonía e) Estruendo
d) Extender b) Fealdad d) Sintonía
e) Calmar
8. HOLGADO
a) Corto c) Pequeño e) Ceñido
b) Escaso d) Cerrado

9. ATIBORRADO
Sinonimia contextual a) Vacío c) Bruñido e) Liberado
1. Es una mujer muy laboriosa, parece una hormiga. b) Limpio d) Despejado
a) aplicada c) activa e) perezosa
b) difícil d) vaga 10. INQUINA
a) Bondad c) Atracción e) Sensatez
b) Avenencia d) Benevolencia

5º AÑO DE SECUNDARIA
Mejores Personas, Mejores Familias 9
Colegio

Mejores Personas, Mejores Familias RAZONAMIENTO VERBAL

MA
TE

02   ESCRITURA DE UN TEXTO

I. COMPRENSIÓN DE TEXTOS
En el campo académico, es la serie de ejercicios de lectura que tiene como objetivo que el alumno comprenda el texto que lee.
Es un aspecto esencial para la posterior comprensión de las demás enseñanzas.
La distribución y orden del texto u obra se presenta de la siguiente manera:

1. El texto (del latín textus).


Documento escrito general; conjunto de palabras que constituyen el contenido o el cuerpo de una obra. Viene a ser una
unidad semántica estructural cuyo elemento básico es el párrafo. El texto es una estructura totalizada con elementos
ordenados que dan a conocer un mensaje.

A.
Estructura del texto
a) El tema
Es la idea o asunto del que trata un texto. Es un enunciado que resume todo lo expuesto, de manera general. En
otras palabras es el motivo, fondo o contenido del texto.
b) La idea principal
Es la que destaca sobre el resto, por ser la más importante del texto, tiene concordancia con el análisis temático.
Se aplica también a la oración o proposición de la que dependen sintácticamente una o más oraciones o propo-
siciones. No formula ejemplos, no entra en detalles o especificaciones.
c) Las ideas secundarias
Son aquellas de menor importancia en comparación con la consideración fundamental, complementan la idea
principal. En un texto pueden existir una o varias, aportan criterios específicos. Su finalidad es argumentar,
verificar, comparar, ejemplificar, repetir y enunciar.
d) El título
Es la palabra o palabras con que se da a conocer el contenido del texto (encabezamiento o cabecera). Una frase
nominal (no tiene núcleo), es más precisa y clara en relación con el tema y la idea principal.

Ejemplo:
Un continente es una gran extensión de tierra que se diferencia de otras menores o sumergidas por conceptos geográficos
y culturales como océanos y etnografía. La palabra viene del latín continere, que significa «mantener juntos» y deriva del
continens terra, «las tierras continuas». Literalmente, el término se refiere a una gran extensión de tierra en la superficie
del globo terrestre. Sin embargo, esta definición estrictamente geográfica es frecuentemente modificada de acuerdo a
criterios históricos y culturales. Así, hay algunos sistemas de continentes que consideran Europa y Asia como dos conti-
nentes, mientras que Eurasia no es más que una extensión de tierra, y otros lo hacen a la inversa.
La definición poco clara de continente ha dado lugar a la existencia de varios modelos y actualmente se reconocen entre
cuatro y siete continentes. Pero esto no ha sido siempre así y esos modelos han variado a lo largo de la historia y el des-
cubrimiento de nuevos territorios.

TEMA IDEA PRINCIPAL TÍTULO

______________________ ______________________ ______________________


______________________ ______________________ ______________________

10 Mejores Personas, Mejores Familias


5º AÑO DE SECUNDARIA
Colegio

RAZONAMIENTO VERBAL  Mejores Personas, Mejores Familias

B. Clasificación por su estructura de ideas


a) Textos Deductivos o Analizantes: Presenta la idea principal al inicio, la cual se desarrolla en varias ideas secun-
darias. Va de lo general a lo particular.
b) Textos Deductivos o Sintetizantes: Se presentan varias ideas secundarias al inicio del texto, las que se resumen
en una idea principal, es decir, la idea principal se encuentra al final del texto, a modo de conclusión. Va de lo
particular a lo general.
c) Textos Inductivos Deductivos o Sintetizantes Analizantes: Se dan a conocer las ideas secundarias, las que se
resumen en una idea principal y a su vez se desarrolla en otras ideas secundarias.
d) Textos Encuadrados: Da a conocer la idea principal al inicio del texto, la que se desarrolla en varias ideas secunda-
rias (explica), para finalizar con una idea principal que resume lo anterior.
e) Textos Alternados: Presenta alternadamente las ideas principales e ideas secundarias.

Lee atentamente cada texto y reconoce las respuestas correctas. 3. La manera como Escribá explica la formación del Opus
Dei obedece a:
TEXTO I a) Una ingenuidad de un partidario del cristianismo.
Fue durante los años veinte, mientras España atravesaba una b) Un mecanismo orientado a generar la compasión.
profunda crisis ideológica, que se fundó el Opus Dei. Un 2 de
c) Una forma de amedrentamiento a los que no son cre-
Octubre de 1928, día en el que, según el propio Escribá, “fue
yentes.
iluminado y recibió las palabras del cielo sobre lo que tenía que
ser la obra”, la única agrupación dentro de la Iglesia católica que d) Una estrategia para afianzar la adhesión de los fieles.
cree haber sido creada por Dios. e) Una asombrosa iluminación que recibió de Jesucristo.
La idea original era que la perfección cristiana no debía ser
patrimonio exclusivo de sacerdotes, monjas o frailes. Cualquier 4. De acuerdo a la lógica de la organización descrita, se po-
persona podía alcanzar la santidad sin usar sotana y viviendo dría afirmar que:
en medio del mundo secular. Se puede ser santo a través del a) Estaría al margen de los profesionales.
trabajo diario. De ahí que el Opus Dei, en teoría, se dirige a
b) Compatibiliza con regímenes aristocráticos.
todos, pero en la práctica se concentra en reclutar a estudiantes
y profesionales de las clases altas bajo el supuesto de que ellos c) Rechaza toda forma de elitismo social.
están en mejores condiciones para comprender el mensaje de d) Surge dentro de una crisis ideológica.
“la obra”. Al respecto, Federico Prieto Celi, conocido periodista e) Fue resultado de la inspiración de la divinidad.
y primer peruano convertido al Opus Dei, declaró hace algún
tiempo que “Dios no es democrático y por eso, la cristiandad 5. El Opus Dei considera que el trabajo es importante ya que:
deber ser elitista, pues la élite es la levadura que hará fermentar
a) Asegura la dignidad del ser humano.
toda la masa. La gente que manda y que influye es la gente que
piensa, y por eso, es lógico que el mensaje del Opus Dei se dirija b) Permite el desarrollo de la sociedad.
a los intelectuales, a los líderes”. Como todo grupo elitista, se c) Mantiene las relaciones elitistas.
afana por alcanzar el poder. Por eso, ejerce influencia en las d) Permite alcanzar la santidad.
esferas políticas, empresariales y en los medios de información. e) Nos acerca a los demás religiosos.
Sus miembros piensan que, cuando más poderosa sea “la obra”,
tanto más católico será el mundo.
TEXTO II
1. El tema del texto gira en torno a: El cerebro es el órgano de la conducta. Todo lo que hacemos,
a) La difusión progresiva del Opus Dei sentimos o pensamos es el resultado de la actividad fisiológica
b) La historia de un grupo cristiano del cerebro. Además, es un órgano plástico, capaz de asimilar
c) Los principios del Opus Dei las experiencias a través de cambios, poco conocidos aún, de su
d) Las jerarquías en el Opus Dei estructura biológica. A esta plasticidad se debe la gran influen-
e) El elitismo del Cristianismo cia que las experiencias pasadas tienen sobre el control de la
conducta y la importancia del aprendizaje en nuestro organis-
2. Desde el punto de vista sociológico, el Opus Dei: mo. Con todo, las propiedades biológicas básicas del cerebro
a) Es parte del sistema religioso cristiano. nos vienen dadas por la herencia, manifestándose poco a poco
b) Se dirige principalmente a los intelectuales. en el curso del desarrollo y de la maduración. Por tanto, la he-
c) Margina a los sectores de estrato popular. rencia y el ambiente determinan las características biológicas
d) Excluye a los que no alcanzan la perfección. del cerebro y lógicamente influyen sobre la conducta y la mente
e) Está logrando un gran éxito social. humana, aún en sus desviaciones.

5º AÑO DE SECUNDARIA
Mejores Personas, Mejores Familias 11
Colegio

Mejores Personas, Mejores Familias RAZONAMIENTO VERBAL

En consonancia con lo anterior, podemos afirmar que las en- 9. Con respecto al cerebro, este importante órgano:
fermedades mentales son enfermedades del cerebro y dado que a) Controla la conducta a través de procesos químicos que
este posee numerosas funciones, habrá también muchas enfer- desconocemos completamente.
medades. Teniendo en cuenta la íntima conexión que, en el ce- b) Posee atributos biológicos básicos que se manifiestan
rebro, se establece entre herencia y ambiente, es natural que en en el proceso de maduración.
algunos casos sea difícil averiguar el origen de la enfermedad. c) Asimila las experiencias gracias a transformaciones que
El principio, las enfermedades más conocidas, como la de- la ciencia ha esclarecido con éxito.
presión, la esquizofrenia o las demencias, parecen debidas a d) Posee una serie de propiedades que solo pueden ser re-
alteraciones químicas del funcionamiento cerebral, mientras cogidas del medio ambiente.
que las formas anómalas de reaccionar ante los problemas de e) Está encargado de un gran número de funciones, entre
la vida pueden tener su origen en experiencias o aprendizajes ellas, producir males mentales.
incorrectos. En el primer caso, el tratamiento será químico, y
reorientativo en el segundo. Sin embargo, el desconocimiento 10. Si consideramos las importantes funciones que cumple el
que aún existe en cuanto al origen de este tipo de males hace cerebro, podemos concluir que:
que el tratamiento no alcance siempre el éxito que desearían a) Si estas disminuyeran, habría enfermedades mentales.
tanto el médico como el paciente. b) Todo lo que hacemos o pensamos es producto de su ac-
tividad.
c) Asimila las experiencias a través de alteraciones estruc-
6. El enunciado que mejor resume la lectura (idea principal) turales.
es: d) Toda nuestra vida interior depende directamente de él.
a) El cerebro es el órgano de la conducta, pues todo aquello e) La depresión podría deberse a ciertos cambios químicos.
que pensamos o sentimos es el resultado de su actividad
fisiológica.
b) Enfermedades mentales conocidas, tales como la de-
presión o la esquizofrenia, se deben probablemente a
alteraciones químicas del cerebro.
c) La ciencia médica desconoce el origen exacto de buena ZZ Lee atentamente cada texto y señalas las respuestas correctas.
parte de las enfermedades que tienen su origen en el ce-
rebro humano. TEXTO Nº 01
Un hombre se confunde, gradualmente, con la forma de su
d) El cerebro es el órgano que determina los fenómenos destino; un hombre es, a la larga, sus circunstancias. Más
mentales humanos y sus desviaciones o enfermedades. que un descifrador o un vengador, más que un sacerdote del
e) El cerebro es un privilegiado órgano capaz de asimilar dios, yo era un encarcelado. Del incansable laberinto de los
las experiencias a través de procesos pocos conocidos sueños yo regresé como a mi casa, a la dura prisión. Bendije su
de su estructura biológica. humanidad, bendije su humedad, bendije su tigre, bendije el
agujero de luz, bendije mi cuerpo doliente, bendije la tiniebla
7. En el texto, el término plástico puede ser reemplazado y la piedra.
por: Entonces ocurrió lo que no puedo olvidar ni comunicar.
Ocurrió la unión con la divinidad, con el Universo no sé si
a) Elástico b) Biológico
estas palabras difieren. El éxtasis no repite sus símbolos; hay
c) Descartable d) Maleable quien lo ha percibido en una espada o en los círculos de una
e) Contaminante rosa. Yo vi una Rueda altísima, que no estaba delante de mis
ojos, ni detrás, ni a los lados, sino en todas partes, a un tiempo.
Esa Rueda estaba hecha de agua, pero también de fuego, y era
8. Una idea falsa con respecto a lo manifestado por el autor
(aunque se veía el borde) infinita. Entretejidas, la formaban
es:
todas las cosas que serán, que son y que fueron. Y yo era una
a) Las experiencias acumuladas en nuestra vida influyen de las hebras de la trama total, y Pedro de Alvarado, que me dio
en nuestra conducta. tormento, era otra.
b) La herencia proporciona al cerebro sus propiedades Ahí estaban las causas y los efectos y me bastaba ver a esa
biológicas fundamentales. Rueda para entenderlo todo, sin fin, ¡Oh dicha de entender,
c) En el cerebro se produce una relación cercana entre he- mayor que la de imaginar o la de sentir! Vi el Universo y vi
rencia y ambiente. los íntimos designios del Universo. Vi los orígenes que narra
d) El tratamiento a pacientes con problemas mentales no el Libro del Común. Vi las montañas que surgieron del agua,
siempre da resultados. vi los primeros hombres palo, vi las tinajas que se volvieron
contra los hombres, vi los perros que les destrozaron las caras.
e) La depresión y la esquizofrenia, ahora se sabe, se deben Vi el dios sin cara que hay detrás de los dioses. Vi infinitos
a malas experiencias. procesos que formaban una sola felicidad y, entendiéndole
todo, alcancé a entender la escritura del tigre.

12 Mejores Personas, Mejores Familias


5º AÑO DE SECUNDARIA
Colegio

RAZONAMIENTO VERBAL  Mejores Personas, Mejores Familias

1. La dicha, según el narrador personaje, está más vinculada 6. El texto trata sobre __________.
al ___________. a) la visión del futuro a través de los sueños.
a) imaginar b) razonar b) explicaciones técnicas de los sueños.
c) sentir d) comprender c) una explicación científica de los sueños.
e) pensar d) el contenido simbólico de los sueños.
e) la teoría psicoanalítica de Freud y de Jung.
2. En el primer párrafo, la palabra «prisión» equivale princi- 7. Según el autor a la gente le interesa más _____.
palmente a ___________. a) cómo se producen los sueños.
b) el porqué de los sueños.
a) felicidad b) sueños c) la explicación científica de los sueños.
c) aposento d) soledad d) el significado de los sueños.
e) arrepentimiento e) la teoría psicoanalítica de los sueños.

3. ¿Qué es lo que no podrá olvidar el narrador personaje? 8. ¿Qué concluye el autor con respecto a las teorías que inter-
pretan los sueños?
a) Una rueda pequeña
a) La teoría de Freud y de Jung aporta pruebas irrefutables.
b) Un resplandor ciego b) La teoría psicoanalítica es la más acertada.
c) Una fusión con Dios c) La teoría del carácter profético de los sueños es la más
d) Una visión del Universo creíble.
e) Una espada y una rosa d) La teoría psicoanalítica es la verosímil.
e) Ninguna teoría aporta pruebas convincentes.
4. La percepción de Dios es ________.
a) omnipotente 9. Se deduce que para la teoría psicoanalítica el contenido del
sueño expresa un deseo _________.
b) providencial
a) agradable b) pueril
c) omnisciente c) vergonzoso d) benéfico
d) única e) inocente
e) disímil
10. Un planteamiento no postulado por la psicoanalítica es
5. ¿Cuál es el tema del texto? que los sueños ______________.
a) La experiencia de la unión del narrador con dios. a) se disfrazan de simbolismo.
b) Dios es inalcanzable para algunos hombres. b) expresan deseos rechazados por la conciencia.
c) tienen un significado.
c) El encuentro con las divinidades.
d) contienen un deseo de realización.
d) Conocer a Dios implica conocer la verdad de todo. e) constituyen fenómenos premonitorios.
e) El narrador y Dios se encontraron de forma extraordi-
naria.

TEXTO Nº 02
Es evidente que el interés de la humanidad por los sueños
estriba más en su interpretación y en su posible significado ZZ Lee atentamente cada texto y marca las respuestas
que en su explicación científica. En este sentido, las teorías correctas.
que han alcanzado más predicamento son dos. Según la
primera, los sueños tienen carácter profético y nos avisan TEXTO I
en general sobre peligros y acontecimientos del futuro. En «El hombre del principio del Paleolítico era nómada y no usó
algunas circunstancias se relacionan también con fenómenos vestidos a causa de la benignidad del clima. Lo prueban los di-
premonitorios que acaban o están a punto de suceder. Nadie bujos y pinturas de esta época que hemos hallado; vivía al aire
sensato parece tomarse en serio esta clase de teorías. libre, cerca del mar o de los ríos, y cazaba animales de mediano
En cambio, mucho más aceptada es la teoría psicoanalítica tamaño, dadas las frágiles armas de que disponía. La principal,
de Freud y de Jung, según las cuales, los sueños nos informan para la defensa y la guerra era el hacha de mano, que consistía
sobre nuestros complejos inconscientes. en un trozo de sílex tallado a golpe con otra piedra. La raza
característica es la llamada de Neanderthal, de baja estatura y
Esta teoría se basa en tres ideas. La primera pretende que
cráneo alargado.
el contenido del sueño tiene un significado. La segunda,
Hacia mediados de este periodo se inicia una glaciación. El
que contiene siempre un deseo de realización; y la tercera,
hombre deja de vivir al aire libre para buscar abrigo en cuevas,
que este deseo es rechazado por la conciencia y por tanto,
preferentemente las situadas en lugares elevados. Con el cli-
debe disfrazarse simbólicamente para poder aparecer. ma cambian también los animales, y el hombre caza entonces
Desgraciadamente, la teoría psicoanalítica carece también de principalmente renos, de los que obtiene pieles, para vestirse y
pruebas convincentes. grasa para pintar, mezclándola con arcilla».

5º AÑO DE SECUNDARIA
Mejores Personas, Mejores Familias 13
Colegio

Mejores Personas, Mejores Familias RAZONAMIENTO VERBAL

1. Según el texto, se afirma que el hombre no usó vesti- 5. ¿Qué no se afirma en el texto?
do y esta afirmación se puede corroborar en la pintura a) Que al delfín le agrada las notas musicales.
__________. b) Tiene la audición muy desarrollada.
a) de los bodegones c) Siente mucho afecto por sus amigos.
b) rupestre d) Huye de la presencia del hombre.
c) de los murales e) Huye de los óbices.
d) del Neolítico
e) del ocaso del Paleolítico TEXTO III
Durante un tiempo la candidez, el determinismo no reconocía
2. Señala la afirmación correcta, según el texto: límites. «Conocida una hora de la vida de un hombre, un
a) El cambio de clima hace más diligentes a los hombres serafín antropométrico podía calcular todo lo que ese hombre
de aquel entonces. había sido y todo lo que habría de ser». (Mortiner Collins). De
b) El hombre se hace sedentario y utiliza vestimenta. manera que en 1871 las ciencias sociales emulaban la máxima
c) En los albores del Paleolítico el hombre no fue errátil. laplaciana del determinismo universal. Laplace habla del
d) El hombre siempre cazó renos. átomo más ligero; aquí Collins habla de un hombre.
e) Los cambios cíclicos hicieron que el hombre fuese errá- Laplace habla del conocimiento cobrado en el instante. Collins
tico. habla de lugares públicos para realizar mediciones de los
transeúntes. Dalton no era ningún ser seráfico, pero compartió
TEXTO II (y contribuyó a popularizar) la idea de que las mediciones
El delfín no es solo un amigo del hombre, sino también del físicas y mentales mostraban la clave de la naturaleza humana.
arte musical. Encuentra placer en escuchar las notas de una
sinfonía y, especialmente, las tonalidades de los instrumentos 6. ¿Cuál de los siguientes enunciados expresa una idea prin-
hidráulicos; no se asusta de la presencia del hombre, sino que cipal?
se le aproxima, sale al encuentro de las embarcaciones, salta a) Laplace fundó el determinismo universal.
alegremente a su alrededor, nada con ella, porfía y se desliza a b) El conocimiento es crucial para el determinismo.
su lado cuando navegan a toda vela. c) El determinismo influenció en las ciencias sociales.
Bajo el reinado del divino Augusto, un animal de esta d) Dalton logró determinar la naturaleza humana.
especie, que moraba en el mar Lucrínico, amaba muy tierna y e) La medición de un átomo es como la de un hombre.
extremadamente al hijo de un hombre que, desde las Bayanas,
iba a la escuela de Piteoli. A la hora del mediodía, se quedaba 7. Para el determinismo, los hechos son _______.
este allí mismo, le llamaba con el nombre de Simón y le hacía a) matemáticos b) predecibles
acercarse, siempre lo hacía, con pequeños pedazos de pan que c) probables d) infinitos
llevaba consigo para este fin. e) Idénticos
Me avergonzaría de contar esta historia si no se encontrara
relatada en los escritos de un mecenas, Fabianus, Flavius y de 8. Se podría decir que el autor asume, en relación al determi-
muchos otros. A cualquier hora del día, solo con que el niño nismo una actitud de ____________.
lo llamara, acudía presuroso, con la mayor celeridad de sus a) perplejidad b) adhesión
aletas, desde las profundidades, por muy apartado y oculto que c) indiferencia d) crítica
se encontrara, y se le acercaba, comía de su mano y le ofrecía e) consecuencia
su dorso para que se sentara sobre él, después de esconder las
puntas espinosas de sus aletas, recogiéndolas como un estuche. 9. Para el determinismo, la medición de un instante es sufi-
ciente para alcanzar _________.
3. ¿Cuál sería el probable título del texto? a) un conocimiento total
a) El delfín b) una deducción parcial
b) El delfín y sus amigos c) una teoría científica
c) El niño y el delfín d) un ideal de la ciencia
d) La fantasía del delfín e) una visión de la naturaleza humana
e) El delfín y el arte musical
10. ¿Cuál es el tema del texto?
4. Según el tema podríamos concluir que ______. a) Las mediciones
a) el delfín aprende lentamente b) El determinismo
b) el delfín es hueco c) Laplace y la ciencia
c) el delfín permite escribir muchas historietas. d) La antropometría
d) el delfín tiene su capacidad de aprendizaje muy desa- e) La naturaleza humanaa
rrollada
e) los mecenas tuvieron delfines

14 Mejores Personas, Mejores Familias


5º AÑO DE SECUNDARIA
Colegio

RAZONAMIENTO VERBAL  Mejores Personas, Mejores Familias

MA
TE

03   ANALOGÍAS

I. ANALOGÍA
El término analogía significa “comparación o relación entre varias razones o conceptos”. El objetivo que se busca alcanzar a
través de estos ejercicios es que los alumnos puedan comparar o relacionar dos o más seres u objetos, mediante la razón, se-
ñalando características generales y particulares, generando razonamientos basados en la existencia de semejanzas entre estos.

II. ESTRUCTURA
Para resolver este ejercicio debemos conocer sus elementos:

III. MÉTODO
Existen muchos métodos para la resolución de estos ejercicios, uno muy completo es el denominado RON, a partir del cual
evalúan la premisa o par base y las alternativas.

A : B : : PREMISA
a)
R
:
S ALTERNATIVA Recuerda
CORRECTA Para resolver fácilmente este
b)
T
:
M
ejercicio procura elaborar
c) V : I
DISTRACTORES una oración con la premisa.
d) A : L
e) P : U

1. Relación
Es la razón por la que los elementos de la premisa han sido vinculados. El mejor modo de hallarla es por medio de una
oración.
En el caso citado, tenemos que A y B son letras, es decir, que ambos son elementos del mismo género, pero ese criterio
no es suficiente, dado que todas las alternativas presentan el mismo caso: son letras. En el caso en el que solo una de las
alternativas reproduzca la relación, esa será la respuesta.

2. Orden
Es el sentido en el que se encuentran emplazados los elementos de la premisa. Es el caso del ejemplo, tenemos que
todas las alternativas eran letras, por lo que ese no era un criterio que determinara o justificara por sí solo algunas de
las respuestas. Sin embargo, al analizar las alternativas, tenemos que las letras A y B no son simples letras, sino que son
consecutivas. Ello nos induce a dar como respuesta la alternativa A, que presenta dos letras que en nuestro alfabeto son
también consecutivas. Es este nivel debemos realizar el grado de generalidad que tienen las alternativas y marcar aquella
que presente un grado similar al de la premisa.

3. Naturaleza
Es, de alguna manera, el tema que desarrolla la premisa. Debemos considerar algunas características que hacen única la rela-
ción entre la premisa y alguna de las alternativas de manera coincidente. Es preciso tener en cuenta, por ejemplo, la categoría
gramatical de la premisa o la relación específica de la misma.

Ejemplo:
VINO : EMBRIAGUEZ :: Si establecemos la relación, encontramos que el exceso
a) agua : sed de vino provoca EMBRIAGUEZ. Y como la relación es
b) hambre : desmayo horizontal, busquemos otro par que sea horizontal. La
c) velocidad : muerte única que sigue es la alternativa D.
d) sol : insolación
e) ingestión : comida

5º AÑO DE SECUNDARIA
Mejores Personas, Mejores Familias 15
Colegio

Mejores Personas, Mejores Familias RAZONAMIENTO VERBAL

Algunos Casos

ORDEN HORIZONTAL Si nos percatamos, la relación se da entre las palabras de
AUTOMÓVIL : VEHÍCULO : : la premisa. Por ello, sigue una horizontalidad. Estas son
a) sillón : butaca las más usadas en los exámenes de admisión.
b) ópera : ballet
c) revista : hemeroteca
d) chalet : vivienda
e) pregunta : enigma

ORDEN VERTICAL
MENARQUIA : MENOPAUSIA : :
En este ejemplo, la relación se da entre una premisa y una
a) aristocracia : poder
alternativa, es decir, la línea a seguir es la verticalidad.
b) música : violín
Aunque no son frecuentes aportan otra visión.
c) familiaridad : amistad
d) principio : fin
e) todo : nada

Advertencia Pre
ZZ Lo principal es establecer la relación en la premisa.
ZZ Tener en cuenta el orden de la premisa, pues el orden de la
premisa debe ser el mismo.

Lee cada ejercicio y señala la alternativa correcta. 4. DELITO : INDULTO : :


1. MÚSICO : ORQUESTA : : a) ignominia : retractar
a) soldado : batallón b) infracción : amonestación
b) persona : individuo c) litigio : concesión
c) poeta : poema d) deuda : exención
d) literato : cenáculo e) pecado : absolución
e) actor : elenco
5. ARIDEZ : FERTILIDAD : :
2. DÁDIVA : REGOCIJO : : a) exuberancia : escasez
a) triunfo : frenesí b) pobreza : opulencia
b) parabién : beneplácito c) lozanía : marchitez
c) injuria : indignación d) torpeza : pericia
d) elogio : placer e) raleza : frondosidad
e) conclusión : dolor
6. ABSTRUSO : COMPRENDER : :
3. EXPOSITOR : AUDITORIO : : a) fácil : asir
a) maestro : enseñanza b) rígido : doblar
b) poeta : cenáculo c) nublado : despejar
c) político : congregación d) indeleble : borrar
d) sacerdote : feligresía e) frágil : quebrar
e) emisor : receptor

16 Mejores Personas, Mejores Familias


5º AÑO DE SECUNDARIA
Colegio

RAZONAMIENTO VERBAL  Mejores Personas, Mejores Familias

7. ACRÓBATA : PLASTICIDAD : : 10. SÍNTESIS : FÓRMULA : :


a) cómico : hilaridad a) astronomía : planeta
b) atleta : ingeniosidad b) cacofonía : canción
c) piloto : copiloto c) autonomía : gobierno
d) soldado : valeroso d) anarquía : caos
e) juez : imparcialidad e) pensamiento : acción

8. FÉRETRO : DIFUNTO : : 11. ENCICLOPEDIA : VOLÚMENES : :


a) chimenea : humo a) librería : textos
b) cenicero : ceniza b) colegio : maestros
c) sobre : carta c) filosofía : corrientes
d) contendor : material d) libro : capítulos
e) vehículo : motor e) ciencia : experimentos

9. CÉLULA : ORGANISMO : :
a) fisiología : función
b) etapa : historia
c) molécula : estructura
d) fibra : tejido
e) átomo : materia

1. COHERENCIA : INTERPRETACIÓN : : 5. MITIGAR : ATENUAR : :


a) sabor : gusto a) obviar : perdonar
b) agua : humedad b) enlazar : exaltar
c) flexibilidad : indisciplina c) aliviar : empeorar
d) longitud : cálculo d) mermar : disminuir
e) claridad : visión e) ondular : ondear
Resolución:
2. ANÁLISIS : CONCLUSIÓN : : Mitigar significa atenuar; por ello, debemos buscar una
a) cisma : cooperación relación de sinonimia. Entonces la clave es la alternativa
b) regionalismo : distribución “d”.
c) corte : resta Rpta: d
d) cálculo : multiplicación
e) evaluación : resultado 6. CADUCIDAD : VIGENCIA : :
a) debilidad : convalecencia
3. CORAZÓN : HOMBRE : : b) infancia : niñez
a) sala : casa c) llegada : arribo
b) página : libro d) decadencia : desarrollo
c) pantalla : televisor e) capacidad : ineptitud
d) motor : vehículo
e) pizarra : salón 7. IMPÚDICO : RECATO : :
a) torpe : inteligente
4. COMERCIO : DINERO : : b) débil : fornido
a) canción : melodía c) honesto : deshonesto
b) transmisión : contagio d) parcial : imparcial
c) herencia : testamento e) hediondo : aromático
d) educación : conocimiento
e) comunicación : lenguaje

5º AÑO DE SECUNDARIA
Mejores Personas, Mejores Familias 17
Colegio

Mejores Personas, Mejores Familias RAZONAMIENTO VERBAL

8. OPULENTO : RIQUEZA : : 10. GERMINAR : SEMILLA::


a) erudito : conocimiento a) estimular : ánimo
b) genio : capacidad b) inaugurar : ánimo
c) fornido : salud c) fecundar : feto
d) craso : carne d) prólogo : obra
e) furioso : ira e) amanecer : día

9. REHÉN : RESCATE : :
a) error : enmienda
b) tedio : solaz
c) acusado : defensa
d) náufrago : salvación
e) triunfador : premio

1. ZEUS : PODER:: 6. OBRERO : FABRIL::


a) Poseidón : tridente a) pastor : rural
b) Ares : guerra b) vecino : vecinal
c) Vulcano : fuego c) ermitaño : monasterio
d) Baco : vino d) comerciante : comercio
e) Cancerbero : perro e) artesano : industri

2. DÉSPOTA : DICTADOR:: 7. DIAFRAGMA : MEMBRANA::


a) liberal : demócrata a) tejido : piel
b) testarudo : terco b) diagonal : línea
c) fanático : religioso c) perímetro : área
d) fascista : militarista d) diccionario : libro
e) demagogo : político e) vestido : bordado

3. JUEZ : CÓDIGO:: 8. ABLACIÓN : EXTIRPACIÓN::


a) sacerdote : catecismo a) auscultación : higiene
b) médico : diagnóstico b) amputación : saturación
c) asociación : estatuto c) distanciamiento : separación
d) árbitro : reglamento d) expulsión : alejamiento
e) científico : teoría e) exilio : destierro

4. INDIGNACIÓN : IMPROPERIO:: 9. DISIDENTE : ESTADO::


a) respuesta : ofensa a) infiel : matrimonio
b) elogio : placer b) traidor : sindicato
c) sorpresa : castigo c) abjurador : institución
d) dolor : pena d) renegado : familia
e) negocio : regalo e) apóstata : Iglesia

5. EJÉRCITO : DESERTOR :: 10. CUERDO : ORATE::


a) matrimonio : soltero a) evidente : falso
b) sociedad : delincuente b) optimista : afligido
c) Iglesia : apóstata c) juicioso : desidioso
d) patria : expatriado d) lúcido : obnubilado
e) ciencia : ignorante e) remilgado : desaliñado

18 Mejores Personas, Mejores Familias


5º AÑO DE SECUNDARIA
Colegio

RAZONAMIENTO VERBAL  Mejores Personas, Mejores Familias

MA
TE

04   LA REFERENCIA: TIPOS DE REFERENTES

I. DEFINICIÓN
Dentro de un texto podemos encontrar elementos (palabras) que nos sirven para referenciar información. Es decir, son palabras
que nos indican algo que se ha dicho o que se dirá. Son como una especie de “recordatorios” o “señales” que mantienen el hilo del
discurso y nos ayudan a seguirlo. También se le conoce como deixis.
Por ejemplo: Nótese que el nombre
Luis es remplazado por
*  Luis y Ana llegaron ayer. Él está muy contento de haber vuelto.
el pronombre él

II. TIPOS DE REFERENTES


1. Anáfora
La anáfora es también considerada como la repetición simple de una palabra cuando esta va al principio de la frase, o tam-
bién recurrir al texto con menciones implícitas mediante pronombres demostrativos.
Ejemplo:
Juan y Pepe fueron a pescar; este pescó una trucha de tres kilos y aquel otra de cinco.

2. Catáfora
La catáfora es la figura retórica contraria a la anáfora. Consiste en la anticipación de una idea que se expresará más ade-
lante.
Ejemplo:
Se llevó absolutamente todo: sus libros de historia, los prototipos de aviones de colección, sus máscaras decorativas, sus foto-
grafías, etc.

3. La elipsis
La elipsis se refiere a ciertas construcciones sintácticas en las que no aparece alguna palabra que se refiera a una entidad
lógica necesaria para el sentido de la frase. Es común que los hablantes eliminen vocablos o frases en su discurso, siempre
que el receptor pueda inferir aquellos elementos del contexto de la oración.
Ejemplo:
(1) Los lobos pastaban en el monte. Tenían hambre y sed. (los lobos)
(2) Lo bueno, si es breve, dos veces bueno.
En la segunda cláusula de (1) “se ha eliminado” el sujeto. Es decir, de interpretación lógica de la frase ‘[Los lobos]
tenían hambre y sed’, el elemento entre paréntesis que hace la función de sujeto no aparece en la segunda oración
de (1). En la segunda cláusula de (2) “se ha eliminado” el verbo ser, es decir, la frase sería “Lo bueno, si es breve,
[es] dos veces bueno.”

4. Expresión sinónima
La expresión sinónima es un tipo de referencia con la diferencia de que no usa los referentes conocidos, sino emplea
frases que actúan como reemplazo.
Ejemplo:
Me gustaría ir de viaje a Arequipa. Sé que en estos días se celebrará el aniversario de la Ciudad Blanca.

III. TIPOS DE REFERENTES


Dentro de los elementos que pueden actuar como referentes están los pronombres (personales, posesivos, demostrativos
entre otros), adverbios y algunas frases. Acá observemos algunos casos de adverbios:

5º AÑO DE SECUNDARIA
Mejores Personas, Mejores Familias 19
Colegio

Mejores Personas, Mejores Familias RAZONAMIENTO VERBAL

CLASE ADVERBIOS LOCUCIONES ADVERBIALES

aquí, allí, ahí, allá, acá, arriba, abajo, cerca, al final, a la cabeza, a la derecha, a la izquierda,
Lugar
lejos, delante, detrás, encima... al otro lado...

de repente, de pronto, a menudo, al amanecer,


antes, después, pronto, tarde, temprano,
al anochecer, en un periquete, con frecuencia,
Tiempo todavía, aún, ya, ayer, hoy, anoche, enseguida,
de tanto en tanto, , por la noche, por la mañana,
ahora, mientras...
por la tarde...

bien, mal, regular, despacio, así, aprisa, como, a sabiendas, a tontas y a locas, a oscuras, de este
Modo adrede, peor, mejor, fielmente, estupendamente, modo, a las buenas, a las malas, por las buenas,
fácilmente... por las malas, de alguna manera...

poco, mucho, bastante, más, menos,


al menos, con todo, más o menos, todo lo más,
Cantidad algo, demasiado, casi, solo, todo, nada,
como máximo, como mínimo...
aproximadamente...

5. “Mi hermano suspendió todo en junio, pues lo despidie-


ron repentinamente; así que no puedo ir contigo”.
a) Adverbio de modo
b) Adverbio de cantidad
Identifica el tipo de referencia y señala la respuesta correcta. c) Catáfora
d) Anáfora
1. “Muchos recuerdan la ocasión en que resbalé por las esca- e) Elipsis
leras en mi fiesta de quince años, eso nunca lo olvidaré”.
a) Anáfora 6. “¿Sacaste diez en matemáticas? ¡Vaya cosa!”
b) Catáfora a) Adverbio de lugar
c) Pronombre demostrativo b) Elipsis
d) Pronombre relativo c) Catáfora
e) Elipsis d) Anáfora
e) Adverbio de tiempo
2. “Alfredo trajo su escopeta. Sergio la suya”.
7. “Me he encontrado a Rubén y el muy maleducado no me
a) Adverbio de modo
ha saludado”.
b) Catáfora
a) Anáfora
c) Pronombre demostrativo b) Adverbio de modo
d) Elipsis c) Adverbio de tiempo
e) Anáfora d) Catàfora
e) Elipsis
3. “Llegaron varias mujeres. Todas traían pañuelos en la ca-
beza”. 8. “Todos ya llegaron a la reunión: mujeres, niños, ancianos
a) Elipsis están en su butaca”.
b) Catáfora a) Adverbio de lugar
c) Pronombre demostrativo b) Adverbio de modo
d) Anáfora c) Catáfora
e) Expresión sinónima d) Anáfora
e) Elipsis
4. “Tengo un libro que trata de Astrología. Este no lo presto a
nadie”. 9. “Los chicos saldrán a jugar fútbol en el estadio Pero creo
que se quedaron sin cupo”.
a) Elipsis
a) Adverbio de modo
b) Anáfora
b) Adverbio de lugar
c) Adverbio de lugar c) Elipsis
d) Catáfora d) Catáfora
e) Pronombre e) Anáfora

20 Mejores Personas, Mejores Familias


5º AÑO DE SECUNDARIA
Colegio

RAZONAMIENTO VERBAL  Mejores Personas, Mejores Familias

10. En: “Le di el libro a Pedrito, pues estaba sobre la mesa” la 7. Los ministros del presidente son todos muy distintos; por
relación que existe entre Le y Pedrito es: ello, cada uno tiene su propia agenda.
a) Catáfora a) Anáfora
b) Adverbio de Lugar b) Adverbio de modo
c) Adverbio de tiempo c) Adverbio de afirmación
d) Anáfora d) Catáfora
e) Elipsis e) Elipsis

8. Luis y Enrique son buenos amigos, ellos siempre están jun-


tos, su amistad es muy sincera.
a) Adverbio de lugar
b) Adverbio de modo
Identifica el tipo de referencia y señala la respuesta correcta. c) Catáfora
d) Anáfora
1. Juan y Pepe fueron a pescar; este pescó una trucha de tres e) Elipsis
kilos y aquel otra de cinco.
a) Anáfora 9. Las serpientes generalmente causan temor en la mayoría
b) Catáfora de las personas; sin embargo, nadie tiene una explicación
c) Elipsis para la causa del miedo desmesurado por los ofidios.
d) Pronombre demostrativo a) Adverbio de modo
e) Pronombre relativo b) Adverbio de negación
c) Elipsis
2. Félix cantaba una canción romántica y sus amigos, unos d) Catáfora
boleros. e) Anáfora
a) Adverbio de modo
b) Catáfora 10. Mi pajarito pía mucho, después come y luego se duerme.
c) Anáfora a) Catáfora
d) Pronombre demostrativo b) Adverbio de lugar
e) Elipsis c) Adverbio de modo
d) Anáfora
3. Yo llevaba las flores y ellos, el incienso. e) Elipsis
a) Elipsis
b) Catáfora
c) Pronombre demostrativo
d) Anáfora
e) Miscelania
ZZ Identifica el tipo de referencia y señala la respuesta correcta.
4. El edificio se desmoronó súbitamente. Este acontecimien-
1. No pienso ir por lo siguiente: no tengo ganas.
to sorprendió a los ingenieros.
a) Anáfora
a) Elipsis
b) Catáfora
b) Anáfora
c) Pronombre demostrativo
c) Adverbio de lugar
d) Pronombre relativo
d) Adverbio de cantidad
e) Pronombre posesivo
e) Catáfora
2. Solo somos dos: tú y yo.
5. Los niños que estaban en el jardín, sonreían.
a) Adverbio de modo
a) Adverbio de modo
b) Catáfora
b) Adverbio de lugar
c) Pronombre demostrativo
c) Elipsis
d) Elipsis
d) Catáfora
e) Anáfora
e) Anáfora
3. La mayor alegría que tengo es saber que estarás en mi ca-
6. Dijo que había estado en el laboratorio, pero no me lo creí.
mino. Por ello, no me preocupo.
a) Adverbio de lugar
a) Elipsis
b) Adverbio de modo
b) Catáfora
c) Elipsis
c) Pronombre demostrativo
d) Catáfora
d) Pronombre personal
e) Anáfora
e) Anáfora

5º AÑO DE SECUNDARIA
Mejores Personas, Mejores Familias 21
Colegio

Mejores Personas, Mejores Familias RAZONAMIENTO VERBAL

4. El edificio se desmoronó súbitamente. Este acontecimien- 8. Los cuadros fueron premiados. Eran hermosos.
to sorprendió a los ingenieros. a) Adverbio de lugar
a) Elipsis b) Adverbio de tiempo
b) Anáfora c) Anáfora
c) Adverbio de lugar d) Elipsis
d) Adverbio de tiempo e) Catáfora
e) Catáfora
9. Las pinturas que se conservan en las cuevas de España repre-
5. Los niños del coro cantaban alegremente. sentan, con increíble exactitud, bisontes, caballos y ciervos.
a) Adverbio de lugar Estas representaciones están realizadas con pigmentos extraí-
b) Adverbio de modo dos de la tierra.
c) Elipsis a) Adverbio de modo
d) Catáfora b) Catáfora
e) Anáfora c) Adverbio de lugar
d) Anáfora
6. Los ministros del presidente son todos muy distintos. e) Elipsis
a) Adverbio de lugar
b) Anáfora 10. La energía producida por los alimentos debe asegurar
c) Elipsis cuatro funciones vitales: el metabolismo basal, el trabajo
d) Sinonimia muscular, el mantenimiento de la temperatura corporal y
e) Catáfora el crecimiento.
a) Catáfora
7. Él los ha seleccionado de entre un grupo amplio de co- b) Expresión sinónima
rrientes políticas, aunque la mayoría son liberales. c) Adverbio de lugar
a) Anáfora d) Adverbio de modo
b) Expresión sinónima e) Elipsis
c) Adverbio de modo
d) Catáfora
e) Elipsis

22 Mejores Personas, Mejores Familias


5º AÑO DE SECUNDARIA
Colegio

RAZONAMIENTO VERBAL  Mejores Personas, Mejores Familias

MA
TE

05   LA INFERENCIA

I. DEFINICIÓN
Las preguntas por inferencia tienen por propósito encaminarlos a establecer una afirmación que no siendo expresamente
propuesta en la lectura resulte concordante y coherente con la información contenida en el texto.
Las preguntas por inferencias, se resuelven a partir de la información propuesta por el autor, pero se dirigen hacia ideas no ex-
presadas de forma textual, ni las que exceden el ámbito tratado en la lectura.

II. CLASES DE INFERENCIA


Las inferencias se realizan cuando, partiendo de su contenido afirmado, se elaboran deducciones o inducciones.
1. La deducción
Las deducciones suponen que si se explica algo general se pueden inferir a un caso particular. Por ejemplo, si se dice que
“los árboles son vegetales” y “el pino es un árbol”, se infiere entonces que “el pino es un vegetal”.

2. La inducción
La inducción se hace presente cuando la ocurrencia de varios
casos equivalentes nos permiten inferir una regla general. Por
ejemplo, “si me asomo a la ventana y veo una persona con una
camiseta y pantalones cortos, veo otra con una gaseosa en la
mano y sudorosa es posible inferir que hace calor.

Formas de plantear este tipo de preguntas.


●● Del texto se infiere que:
●● Del texto se deduce que:
●● Del texto se colige que:
●● Del texto se deriva que:
●● Del texto se desprende:

Lee atentamente los textos propuestos y responde las preguntas. Ribeyo, como los grandes escritores decimonónicos, creía que los
TEXTO I libros se venden por inercia y no gracias a un despliegue publicitario.
Juan Antonio Ribeyro, hermano del escritor, explica que los Por fortuna, sus libros, en los últimos años, están ingresando a
editores presionaban mucho a Julio los circuitos comerciales de Europa, en muy cuidadas y pulcras
Ramón para que este haga publicidad ediciones.
a sus libros, pues de no ser así estos En una conferencia que ofreció en 1986 – todo acto público
se venderían poco. Por una parte del autor de Prosas apátridas colmaba los auditorios y muchos
estaba bien –afirma–, era lógico que los quedaban pugnando por ingresar– relató la siguiente anécdota:
editores se preocuparan por recuperar cierta vez que regresó a Lima recibió la visita de unas colegialas,
su dinero y, por supuesto, ganar, pero a quienes luego de pedirle autógrafos le obsequiaron una bolsa
mi hermano le incomodaba mucho este de plástico, que contenía lapiceros y cintas para máquinas
tipo de cosas, no le gustaba exhibirse. de escribir, acompañada de la siguiente nota que le conmovió
Además –agrega– su figura no se mucho: “Al señor Julio Ramón Ribeyro, con gran admiración y
prestaba: era flaco y tímido. aprecio, para que nos siga deleitando con sus libros”.

5º AÑO DE SECUNDARIA
Mejores Personas, Mejores Familias 23
Colegio

Mejores Personas, Mejores Familias RAZONAMIENTO VERBAL

1. Se deduce respecto al carácter descrito de Ribeyro que TEXTO II


________ La concentración mental no es un estado antinatural que
a) permitió darle poca publicidad, especialmente en los esté en contradicción con nuestro modo de ser. El alumno
últimos años. concentrado, al fin y al cabo, no es más que el hombre en quien
b) las raíces de su comportamiento huraño están en su ni- no ha perecido la genial capacidad del niño para interesarse
ñez. profundamente en su labor.
c) fue un factor que impidió su rápida celebridad literaria. 6. Podemos deducir que para el autor del texto, los niños re-
d) fue un ingenuo en el fondo, pues las obras solo se ven- flejan _________.
den con publicidad. a) la capacidad de sobreponerse a los gustos.
e) asombró a los editores, ya que resultaba extraño que no b) la aptitud para interesarse en algo muy trivial.
quiera vender. c) la superación de las emociones humanas.
d) el estado natural de los seres humanos.
e) el intenso interés en algo concreto.
2. Entendemos que la afirmación: los libros se venden por
inercia, implica:
TEXTO III
a) Una progresiva y lenta venta
De todos los ingleses, el que llevó más lejos la gloria del teatro
b) Una acogida masiva por parte de los lectores cómico, es el difunto Congreve. Escribió pocas obras, pero
c) Una escasa publicidad de la vida del autor todas son excelentes en su género. En ellas, las reglas del teatro
d) Una publicación inédita de libros son rigurosamente observadas. Están llenas de caracteres
e) Una venta por reconocimiento propio matizados con suprema fineza: no hay una sola broma de
mal gusto; se advierte por doquier el lenguaje de las personas
3. Los obsequios de las colegialas a Ribeyro significan que: honestas, unido a acciones de pícaro, lo cual prueba que
a) Notaron que Ribeyro carecía de recursos para conti- Congreve conocía muy bien su mundo y que vivió en lo que
nuar escribiendo. llamamos buena compañía. Estaba enfermo y casi muriéndose
b) Ansiaban leer los cuentos no publicados debido a su in- cuando yo lo conocí; tenía un defecto: no estimar bastante su
timidad. oficio de autor que había hecho su reputación y su fortuna. Me
c) El autor de Prosas apátridas era un literato no poco co- hablaba de sus obras como de cosas sin importancia, ajenas a
nocido. él. En la primera conversación me pidió que no lo considerara,
d) Deseaban seguir conociendo y complaciéndose con los sino como un gentil hombre que vivía como tal. Yo le respondí
cuentos riberianos. que si él hubiera tenido la desgracia de ser nada más que un
e) El cuentista era querido y apreciado de una manera ex- gentil hombre como otro cualquiera, jamás habría yo acudido a
traordinaria. verlo, chocándome fuertemente esa vanidad tan mal empleada.
Sus piezas son las más espirituales y las más verídicas; las de
4. Podemos inferir sobre las ediciones pulcras y cuidadas de Vanbrugh, las más alegres; y las de Wycherley, las más fuertes.
Riberyro que ____________. 7. El término matizados, en el texto, puede reemplazarse
a) evidencian la calidad artística de los últimos cuentos de por:
Ribeyro a) Llenos d) Escritos
b) Colores e) Coloreados
b) demuestran que finalmente el peruano tiene la publici-
c) Combinados
dad que merece
c) hacen notar que los europeos han sabido valorar siem-
8. El propósito central del autor es ____________.
pre a Ribeyro
a) resaltar la gloria del teatro
d) lograron tener la edición digna que el cuentista nunca b) ensalzar el teatro de Congreve
quiso c) describir los problemas del teatro
e) llegaron a tener el mérito suficiente para ser vendidos d) reconocer los defectos del artista
sin publicidad. e) hacer una apología a Congreve

5. Si Ribeyro hubiese tenido el afán de exhibición, sería falso 9. ¿Cuál sería el título del texto?
que ___________ a) Las reglas del teatro de Congreve
a) los cuentos editados hubiesen llegado a más partes del b) Congreve y los rasgos de sus obras
mundo. c) El teatro picaresco inglés
b) los admiradores serían una cantidad mucho mayor a d) El teatro realista de Congreve
que tuvo en 1986. e) El teatro excelso de Congreve
c) el cuento peruano hubiese llegado en menos tiempo a
Europa. 10. Se deduce que la producción teatral de Congreve se carac-
d) los editores hubiesen generado muchas ganancias para teriza por su:
su bolsillo. a) Prolijidad b) Espiritualidad
e) el repertorio riberiano hubiese tenido un reconoci- c) Armonía d) Rigurosidad
miento general. e) Falta de importancia

24 Mejores Personas, Mejores Familias


5º AÑO DE SECUNDARIA
Colegio

RAZONAMIENTO VERBAL  Mejores Personas, Mejores Familias

4. Sobre la política se puede afirmar que _______.


a) cumple un rol orientador de la práctica social
b) hace adquirir conciencia de sí mismo
c) determina el carácter de la población social
ZZ Lee atentamente los textos y responde las preguntas for- d) corresponde a los fines de algunas colectividades
muladas. e) merece una difusión integral para su mejor comprensión

TEXTO I 5. ¿Cuál sería el título más apropiado para el texto?


La política es la ordenación y la conducción consciente de a) La práctica social
la praxis social. En la medida que ella traduce y promueve b) Definición de la praxis social
la autenticidad de la praxis humana como acción solidaria y c) La conciencia política
creadora, la política es un fermento de la vida de la comunidad y d) La política social
un momento esencial de la existencia personal. En la política la e) La sociedad auténtica
sociedad toma conciencia de sus propias exigencias y endereza
sus esfuerzos a realizar por todos los medios de organización TEXTO II
y promoción disponibles en una etapa histórica determinada. La teoría del conocimiento es una explicación e interpretación
Esta toma de conciencia misma de las exigencias y fines de filosófica del conocimiento humano. Pero antes de filosofar
la sociedad y la decisión de realizarlos, que son la levadura sobre un objeto es menester examinar escrupulosamente este
de toda política real, están condicionadas históricamente. objeto. Esto se logra tratando de aprehender los rasgos esenciales
Ellas no se dan por igual en todas las comunidades ni en generales de este fenómeno mediante la autorreflexión de lo
todos los sectores de la colectividad, ni están preparados y que vivimos cuando hablamos del conocimiento. Este método
dispuestos todos por igual a promover el desenvolvimiento de se llama el fenomenológico, a diferencia del psicológico que
la sociedad en conjunto. Hay grupos, clases o estamentos que, investiga los procesos psíquicos concretos en su curso regular y
en los diversos momentos de la evolución de la humanidad, su conexión con otros procesos, el primero aspira a aprehender
constituyen las fuerzas ascendentes de la sociedad, las que la esencia general en el fenómeno concreto; por lo tanto, no
tienen la más clara conciencia del porvenir colectivo y cuyos describe un proceso de conocimiento determinado, sino lo que
fines pueden ser reconocidos y realizados como las formas es esencial a todo conocimiento, en qué consiste su estructura.
óptimas del cumplimiento de las exigencias sociales en
un momento histórico determinado. La política, pese a su 6. ¿Qué idea remarca el texto?
significado humano general, tiene de este modo un contenido a) La teoría del conocimiento aprehende el fenómeno
histórico particular en cada comunidad y en cada época. concreto.
b) Lo apremiante que resulta diferenciar las esencias del
1. Respecto a la sociedad, hay grupos, clases o estamentos método fenomenológico y el psicológico.
que ______________. c) El rasgo aprensivo del método fenomenológico.
a) conducen los destinos del país al crecimiento paulatino d) La necesidad de estudiar la naturaleza del conocimiento
b) tienen una visión clara acerca de las perspectivas socia- a través de la fenomenología.
les e) Los procesos psíquicos son secundarios en la aprehen-
c) tiene una conciencia reconocida por el pueblo y las ins- sión de la esencia del conocimiento.
tituciones
d) hacen de la política una fuerza para el futuro del país 7. Según el texto, ¿qué quiere decir aprehender la esencia?
e) constituyen fuerzas ascendentes que convergen en el a) Aferrarse con seguridad a la idea final de la esencia del
poder conocimiento.
b) Discernir y comprender cabalmente la naturaleza del
2. Gracias a la política, la sociedad es consciente de conocimiento.
______________. c) Investigar los procesos concretos de la esencia.
a) sus fines, sus medios y sus interrogantes d) Interpretar correctamente la evidente esencia del cono-
b) sus necesidades y sus objetivos cimiento.
c) la elevación de sí misma y de la comunidad e) Concebir sin riesgos una noción acerca de la esencia del
d) su influencia en la praxis social y creadora conocimiento.
e) las condiciones históricas de la praxis
8. La fenomenología deja de lado los procesos psíquicos por-
3. ¿En qué se diferenciaría un estamento de otro? que ___________.
a) En su afán desinteresado acerca del bienestar social. a) son esencias impertinentes para la teoría del conoci-
b) En sus órganos históricamente determinados. miento
c) En la medida en que ella traduce y promueve la autenti- b) se trata de lucubraciones enigmáticas
cidad. c) pertenecen a otro género de interpretación de las esencias
d) En la diversidad de reivindicaciones y metas como tam- d) busca la esencia del conocimiento y lo hace a través de
bién en su desigual participación. la abstracción
e) En la escasez de las instituciones solidarias y construc- e) existen diferencias irreconciliables entre las dos disci-
tivas. plinas

5º AÑO DE SECUNDARIA
Mejores Personas, Mejores Familias 25
Colegio

Mejores Personas, Mejores Familias RAZONAMIENTO VERBAL

9. El método psicológico no puede abarcar la teoría del cono- 1. Podemos deducir que Mario Vargas Llosa había leído a
cimiento por _________. Onetti cuando __________.
a) el hecho de carecer de la aptitud para enfrentar ese tema a) se había producido la división literaria
b) distar mucho de las pretensiones del método fenome- b) ya tenía vínculos con autores extranjeros
nológico c) se había relegado la literatura tradicional
c) permanecer aislada de los demás métodos d) intenta escribir su primera novela tradicional
d) no necesitar de reflexiones sobre las particularidades e) ya había publicado sus primeras obras.
del conocimiento teórico
e) estar centrada en el estudio de un proceso en particular 2. Los autores del círculo del boom dan a entender que
__________.
a) poco les interesó la literatura peruana
10. El término «escrupulosamente» se entiende como b) gozaban de un propio estilo novelístico
___________.
c) leyeron a sus precursores tradicionalistas
a) analíticamente d) no quieren ser injustos con los tradicionalistas
b) sintéticamente e) reconocieron a Onetti como su precursor
c) dinámicamente
d) esquemáticamente 3. La vida breve es importante porque _________.
e) pausadamente a) rompe con el silencio del tradicionalismo costumbrista
y criollista
b) se considera la clave de la nueva literatura en América
Latina
c) es una suerte de momento de descubrimiento del pre-
cursor novelístico
d) es el inicio de la nueva etapa en literatura peruana
ZZ Lee atentamente cada texto y responde las preguntas plan-
e) marca el comienzo de la nueva etapa novelística hispa-
teadas.
noamericana

TEXTO I 4. ¿Qué se observa en el proceso de la literatura peruana?


Vargas Llosa manifestó en una oportunidad que la nueva a) División entre los autores tradicionales
novela hispanoamericana comenzaba con las novelas del gran b) Una suerte de despegue de la narrativa.
escritor uruguayo Juan Carlos Onetti. La vida breve, ha sido c) Contradicción entre los creadores artísticos.
señalada como la novela clave. Otros integrantes del llamado d) La falta de solidaridad entre los autores peruanos.
boom de nuestras literaturas coinciden en destacar la función e) La falta de tradición y continuidad artística.
de precursor del autor de Junta cadáveres y en señalar sus
novelas como una suerte de momento de despegue de la actual
5. Los narradores creadores serían _______.
narrativa de América Latina. Lo curioso del caso es que casi
a) los autores originales en sus producciones literarias
todos, sin excepción reconocen que descubrieron a Onetti,
avanzada ya la década del sesenta, cuando ellos –los del círculo b) aquellos que han olvidado los valores más excelsos
más íntimo y valioso del boom – habían producido buena parte c) los autores que perdieron la identidad del indigenismo
de las novelas que con justicia los han lanzado a la popularidad d) los lectores fanáticos de novelistas extranjeros
universal. e) los novelistas influenciados por autores extranjeros
Estas palabras quieren servir nuevamente para que nos
planteemos el problema de la tradición en la novela TEXTO II
hispanoamericana o, más concretamente, la interrogante de Si el Estado tiene su origen en la necesidad que tienen los
si existe una secuencia, una línea de afluencia en la literatura hombres de subsistir, de asociar sus fuerzas y ayudarse
peruana. Si coincidiéramos con la afirmación de Vargas Llosa, mutuamente; la unidad del Estado debe ser el resultado de la
que útilmente ha sido especificada mediante una hipótesis armonía de las voluntades y el equilibrio de los intereses.
que supone una tajante división de los narradores de nuestra En el Estado existen tres clases, que corresponden precisamente
América en primitivos y creadores, una suerte de cordón a las facultades de razón, valor y deseo. La clase animada por
sanitario que impide que se mezclen es que se les confunda entre el deseo, amiga de la ganancia, comprende a los artesanos y
sí, unida a sus muy conocidas influencias literarias, tendríamos labradores, que tienen a la templanza por virtud. Sus almas
que convenir en que Vargas Llosa poco o nada le debe a los están compuestas del hierro y del bronce, y su finalidad en el
escritores peruanos anteriores a él. Algo similar sucedería con Estado es satisfacer la necesidad de vivir.
el otro narrador peruano importante, Julio Ramón Ribeyro, La clase animada por el desprecio al peligro y el amor a la gloria
lector fanático como Mario Vargas Llosa, de la novela francesa es la de los guerreros. Su virtud es el valor, sus almas están
del siglo XIX, que me dejó perplejo hace algunos años; cuando elaboradas con el noble metal de la plata y tienen por misión
me confesó que casi no había leído a López Albújar. satisfacer la necesidad de defenderse que tiene el Estado.

26 Mejores Personas, Mejores Familias


5º AÑO DE SECUNDARIA
Colegio

RAZONAMIENTO VERBAL  Mejores Personas, Mejores Familias

La tercera clase, la de los magistrados, corresponde a la razón; 8. La clase de los labradores y artesanos ______.
su virtud es la prudencia, sus almas están fabricadas con el más a) adopta una postura sumisa
noble metal, el oro; y su misión en el Estado es la de gobernar. b) muestra una actitud servil
Los labradores y artesanos deben obedecer a los guerreros y a c) satisface la necesidad de defensa
los magistrados. d) tiene la condición de subordinado
e) carece de las más nobles virtudes
6. ¿A qué corresponde la jerarquía de las clases al interior del
Estado? 9. En la opinión del autor, ¿quiénes deben gobernar?
a) Al principio de autoridad institucional a) Los que poseen el metal precioso.
b) A la necesidad de la justicia como valor b) Los que luchan por el bienestar.
c) A la diversidad de aptitudes humanas c) Los hombres dotados de sabiduría
d) A las múltiples necesidades insatisfechas d) Los guerreros y los sabios
e) A la voluntad de los gobernantes estatales e) Las personas que buscan la justicia

7. El otorgamiento de una función específica a cada clase so- 10. ¿Cuál sería el título apropiado para el fragmento leído?
cial, es para lograr _______. a) Organización irreal del Estado
a) el equilibrio de intereses b) Constitución ideal del Estado
b) la armonía de voluntades c) Virtudes para gobernar el Estado
c) la unidad del Estado d) Valores humanos en el Estado
d) la defensa del Estado e) Finalidad de todo Estado
e) la convivencia pacífica

5º AÑO DE SECUNDARIA
Mejores Personas, Mejores Familias 27
Colegio

Mejores Personas, Mejores Familias RAZONAMIENTO VERBAL

MA
TE

06   LOS CONECTORES

I. DEFINICIÓN
Se denominan conectores o ilativos a los enlaces que son nexos gramaticales y que unen internamente una oración o más
oraciones entre sí para conformar un discurso.
1. Conectores lógicos

TIPO CONECTIVO FUNCIÓN EJEMPLO


Porque, pues, puesto que,
Señalan el motivo
ya que, debido a que, como Llegué tarde porque no me levanté
1 Causa o la razón de lo que
consecuencia de, a causa de que, lo suficientemente temprano.
precede.
en vista de que, etc.
Luego (= entonces), así que,
de ahí que, de manera que,
entonces, en conclusión, por Anuncian, en lo que Entregaste el trabajo con cinco
2 Consecuencia esto, por esta razón, por (lo) sigue, una deducción minutos de demora, de modo que se
tanto, de modo que, de suerte de lo que precede. te descontará la mitad de tu sueldo.
que, en consecuencia, por
consiguiente, ergo, etc.
Mas, pero, sino, por el (al)
Los borrachos siempre dicen la
Contraste u contrario, sin embargo, no Señalan relaciones de
3 verdad, sin embargo, nadie les hace
oposición obstante, más bien, empero, oposición.
caso.
antes bien, en cambio, etc.
Aunque el gobierno actual ha
Aunque, a pesar de que, aun No es una negación prometido subir los sueldos, es
4 Concesión
cuando, si bien, etc. absoluta sino parcial. probable que no sea capaz de
hacerlo.
Más aun, además, así mismo, Generalmente tienen
No solo es una persona incapaz,
5 Adición inclusive, aparte de ello, también, uso parecido a y, pero
sino que también es irresponsable.
no solo... sino también, etc. más enfático.
En otras palabras, en otros Se anuncian lo mismo Esa chica tiene cinco enamorados,
6 Equivalencia términos, es decir, o sea, vale en términos más en otras palabras, se está divirtiendo
decir, en suma, etc. simples o más técnicos. bastante.
Todos los empleados de esta
Obviamente, como es obvio, empresa debemos trabajar seria y
Se recuerda un punto
7 Evidencia naturalmente, por supuesto, sin profesionalmente; como es obvio,
obvio.
lugar a dudas, etc. esto incluye a los directivos de la
misma.
Primero, en primer lugar, en
En primer lugar, nadie aquí consume
segundo lugar, antes, después,
Se usan para enumerar alcohol; en segundo lugar, nadie
8 Orden finalmente, por último, a
una serie de puntos. aquí lo volverá a hacer; por último,
continuación, seguidamente,
¡salud!
etc.
Debes decirle a la chica que te
Después, luego (= después), Se usan para indicar gusta lo que sientes, directamente;
9 Secuencia
enseguida, etc. una sucesión de ideas. después debes besarla en el acto sin
darle tiempo a reaccionar.

28 Mejores Personas, Mejores Familias


5º AÑO DE SECUNDARIA
Colegio

RAZONAMIENTO VERBAL  Mejores Personas, Mejores Familias

2. Uso de signos de puntuación con conectores


Recuerda que antes de un conector cualquiera se debe usar de modo obligatorio algún signo de pun-
tuación (coma, punto y coma o punto). El uso de la coma se dará cuando no existen ideas completas
a cada lado del conector (las condiciones para la existencia de una idea completa son dos: verbo
conjugado y sentido propio)
Después de un conector, sea simple o compuesto, no se debe colocar punto o punto y coma. Si el co-
nector es simple, tampoco es adecuado el uso de la coma; en cambio, de ser compuesto el conector, se
debe colocar de modo obligatorio una coma, menos en los que terminan en QUE/CUAL o preposición,
dado que son elementos subordinantes.

6. En el siglo de _____ literatura en particular tenían que ser


o estar comprometidos __ debían ponerse al servicio de
una causa, promover una ideología, servir al proletariado
universal, a la raza aria o a cualquier otro mito fundacional
Lee atentamente y completa con el conector adecuado para de porvenires utópicos.
cada oración. a) Sartre, el arte en general y la - , es decir; artistas y escritores
1. El investigador debe ser capaz_______ solo de conceptuar b) Sartre: el arte en general y la - ; es decir, artistas o escritores
el problema, ___________ también de verbalizarlo de for- c) Sartre; el arte en general o la - ; es decir, artistas y escritores
ma clara, precisa _______ accesible. d) Sartre, el arte en general y la - , es decir, artistas y escritores
a) ni – sino – o d) no – si no – y e) Sartre, el arte en general: y la - ; es decir; artistas y escritores
b) no – sino – y e) nunca – si- o
c) no – si no – o 7. El asunto era muy complejo de resolver y alguien acoto:
“quien de una pista será ______ detective _______ brujo”.
2. El poder está en manos _________ políticos rutinarios ______ era tiempo ya de exponer las causas
______ escépticos, manejados ____ una poderosa pluto- a) ni - ni - Antes bien
cracia. b) entonces - no - Como que
a) de - y - desde c) como - pero - Por consiguiente
b) sobre - inclusive - desde d) con todo - y - Ergo
c) aun de - o - desde e) no - sino - Mas
d) de - y - por
e) menos - más - ni 8. Estudié todo el año para postular a la universidad; no in-
gresé. Ahora tendré que esforzarme más, ________ mi ob-
3. No llegó a defender la cuestión agraria ____ él mismo jetivo es ser universitario.
se sentía ideológicamente ligado al viejo orden; ______, a) no obstante - ya que
su esposa sí lo hizo _______ lo consideraba un proyecto b) es decir - y
plausible. c) y - por lo tanto
a) porque - en cambio - ya que d) a su vez - luego
b) por qué - por tanto - entonces e) ergo - es decir
c) porqué - luego - solo
d) por qué - además - aunque 9. Estaba exhausto _________ debía dar su mejor esfuerzo
e) porqué - también - ni __________ en casa lo esperaban sus pequeños retoños
__________ sus hijos.
4. No era buen poeta, ____ un gran ensayista que descolló a) ; sin embargo, - , pues - , es decir,
______ obtuvo un premio literario. b) ; pero, - ; pues - , o sea,
a) pero – por que d) si no – ergo c) ; no obstante - : y - , es decir,
b) si no – por qué e) empero - luego d) , sin embargo - ; pues - ; vale decir
c) sino – y e) , en consecuencia - : pues - , es decir

5. _________ de su destreza, el matador no pudo evitar la 10. Era un experto _______ declamación, hacía brotar lágri-
embestida de la bestia. Hoy los doctores temen por su vida, mas _______ de los corazones mas fríos, ______ lo que
_________ su juventud ________ vigor hacen abrigar es- era requerido en toda ceremonia.
peranzas. a) para la - aún - y
a) A causa - no obstante - y b) para - a causa - en
b) A pesar - aunque - y c) con la - inclusive - para
c) En contra - y - mas d) en - incluso - por
d) A pesar - empero - o e) haciendo - hasta
e) Encontrar - aunque - más

5º AÑO DE SECUNDARIA
Mejores Personas, Mejores Familias 29
Colegio

Mejores Personas, Mejores Familias RAZONAMIENTO VERBAL

5. No consiguió el empleo __________ para colmo, tampo-


co le explicaron el ___________, no lo querían contratar
_______ ocupar el puesto.
a) también – porque – en
ZZ Elige el conector que mejor completa el sentido del enun- b) y – por qué – a fin de
ciado. c) sino – porque – para
d) y – porqué – para
1. No era buen poeta ________ un gran ensayista que desco- e) ni - más - por
lló ________ obtuvo un premio literario.
a) pero – por que d) si no – ergo 6. Comentó el tiempo frío _________ lluvioso ________ en-
b) si no – por qué e) por – si tablar conversación; _________, su compañero estaba tan
c) sino – y distraído que no lo oyó.
a) pero – por – entonces
b) o – a fin de – más
2. En la región de la Costa escasea el agua. ___________
los antiguos peruanos ejecutaron obras hidráulicas que c) y – para – sin embargo
hoy mismo sucitan la admiración de propios y extra- d) y – y – empero
ños. En esta zona, nazcas, chimúes e incas son famosos e) u – ni – mas
___________ construyeron canales, acueductos, reservo-
rios y pozos que permitían aprovechar el agua de los ríos y 7. Bebía poco ____________ en esta ocasión solo quería ol-
de las lluvias para fertilizar el desierto. _________ la falta vidar su pauperismo, ________ lo único que conseguía era
de tierras indujo a los incas a construir los famosos an- recordarlo más; _______, se fue.
denes. Tenían como fin evitar la erosión del suelo de las
a) aunque – por ello – entonces
lluvias y el viento. ________ cortaban el suelo en forma de
b) empero – ergo – pero
escalones, luego levantaban muros de contención y, antes
bien, en esta especie de maceteros, llenaban, tres capas de c) luego – empero - más
piedra menuda, arena y tierra vegetal. d) también – más – así
a) Más aún, - , en consecuencia, - Por el contrario, - Na- e) pero – mas – por eso
turalmente,
b) Por ello, - , porque - Debido a que - Primero 8. No hay nada más lindo que una mirada alegre ______
c) Por lo cual, - , ya que, - Evidentemente, - Primeramente, franca, _______ un saludo amical _____ leal. _______
d) Más aún, - , en consecuencia, - Por el contrario, - Es solo encontramos perfidia.
decir, a) y – o – y – Sin embargo
e) Luego - , mas - Sin embargo - O sea b) pero – o – más – Pero
c) y – y – o – Pese a que
3. El sentimiento, según los románticos, es más importante d) ni - ni - O
que la razón, _______ no se debe escribir siguiendo ciertas e) y – no – y – Empero
normas _____ deben seguirse los impulsos del alma. Los
románticos desdeñaron el orden impuesto por los clásicos 9. Su nerviosismo lo delató, fue detenido. _______, al revisar
_______ proclamaron la libertad de expresión en el em- su equipaje se halló narcóticos ________ su interior.
pleo de cualquier palabra o giro. a) Entonces – por
a) por ello, - , sino - y b) Luego – en
b) pero - ; sin embargo, - , en primer lugar, c) Y - ni
c) aunque - , no obstante -, es decir, d) Por eso – sobre
d) pero - ; sino, - , pero e) A fin – por
e) pese - , sino -, es decir,
10. La actitud de Vallejo es dialéctica _________ sin eludir la
4. Nadie se podía explicar ________ se subió al peñón base trágica __________ la existencia humana, la dignifica
_______ tampoco _______ no quería bajar de ahí. _________ humaniza en realidad concreta del mundo.
a) porqué – o – si a) por qué – de – o
b) porque – sino – porque b) más – o - ni
c) por qué – ni – por qué c) pero – en – y
d) porqué – más – porqué d) pues – de – y
e) porque - ni - más e) porque – en – o

30 Mejores Personas, Mejores Familias


5º AÑO DE SECUNDARIA
Colegio

RAZONAMIENTO VERBAL  Mejores Personas, Mejores Familias

6. Las minas de los lápices son de grafito. ______, contienen


otros materiales que les dan resistencia.
a) En otros términos d) Pues
b) Por último e) Más bien
ZZ Señala el conector que enlaza mejor las ideas. c) Además

1. El mandril tiene un aspecto simpático. ______ , es uno de 7. Los chicos prefieren las lecturas científicas; ________, las
los animales más feroces. chicas eligen textos literarios y psicológicos.
a) Empero a) aparte de ello d) en cambio
b) Por eso b) por consiguiente e) decirnos
c) Además c) a pesar de ello
d) Por que
e) Por otro lado 8. Nuestro Perú es uno de los países __________privilegia-
dos, ___________ le ofrece al mundo los ________ sor-
2. Hay deportes que requieren enviar la pelota al campo con- prendentes paisajes.
trario. ______ , en el tenis, cada golpe debe enviar la pelo- a) mas – ya que – mas
ta, sin rebotar, al terreno del oponente. b) más – pero – mas
a) Por ejemplo c) aún – no obstante – más
b) A causa de d) más – pues – más
c) Obviamente e) sino – luego – sin duda
d) En consecuencia
e) En mención 9. _____________ los sentidos nos engañan, no podemos
dejarlos totalmente de lado, _____ sin ellos no podríamos
3. En algunas regiones llueve mucho. ______, se construyen captar la realidad externa.
varios tejados superpuestos para que no penetre el agua. a) Aunque – ya que d) Pero – aún
a) Ya que d) Después b) Pero – además e) Pese a - sino
b) Por esta razón e) Luego c) A pesar de que – incluso
c) Incluso
10. La producción de café ha aumentado considerablemen-
4. Los peces son animales vertebrados, _____ , tienen esque- te, _________ el consumidor no pagaba más barato
leto. _________ más caro.
a) por ejemplo d) aparte de ello a) pero – sino
b) no obstante e) en mención b) pero – además
c) es decir c) no obstante – por tanto
d) aunque – sino
5. Hay una lluvia torrencial. _____ el agua se empozará en e) luego - primero
pistas y veredas.
a) Sin embargo
b) Puesto que
c) Más aún
d) Obviamente
e) Comúnmente

5º AÑO DE SECUNDARIA
Mejores Personas, Mejores Familias 31
Colegio

Mejores Personas, Mejores Familias RAZONAMIENTO VERBAL

MA
TE

07   ACENTUACIÓN GENERAL Y ESPECIAL

ACENTO C. Esdrújulas o proparoxítonas 


Rasgo prosódico mediante el cual se destaca una sílaba de Son aquellas que llevan acento en la antepenúlti-
una palabra frente a las demás que la componen o una unidad ma sílaba. Estos se acentúan ortográficamente sin
lingüística frente a otras de su mismo nivel. excepción.
Es la mayor inflexión de voz con que se pronuncia una Ejemplos:
determinada sílaba de una palabra. El acento se ubica en la LL Án - fo – ra
sílaba tónica. LL Ál – ge – bra

I. CLASES D.
Sobresdrújulas o superproparoxítonas
1. Acento prosódico Son aquellas que llevan el acento en la anterior a la
Se nota solamente en la pronunciación. Ej.: antepenúltima sílaba. Estas se acentúan ortográfi-
●● Ve- lo – ci - dad camente sin excepción.
●● Su – pe – rior Ejemplos:
LL Con – sí – gue – te – la
2. Acento ortográfico LL Dí – ga – se – le
Se nota por el uso de la tilde (´). Ej.:
●● Cá – ma – ra 2. Tildación de palabras compuestas
●● E- xí – ge – le A. Tildación de vocablos fusionados en
uno solo
II. CLASIFICACIÓN DE LA ACENTUACIÓN LL Décimo + Sétimo = decimosétimo
1. Acentuación general o común LL Balón + cesto = baloncesto
A. Agudas u oxítonas LL Río + platense = rioplatense
Son aquellas que llevan el acento en la última LL Cama + cuna = camacuna
sílaba. Llevan tilde cuando terminan en “n”, “s” o Cuando se fusionan dos vocablos simples para
vocales. formar uno, el primero pierde su acento (prosódi-
Ejemplos: co u ortográfico) y el segundo conserva su acento
LL Ten – sión como siempre lo tenía.
LL In – ten – si – dad
LL In – glés B. Tildación de vocablos unidos por guion
LL Al – fi – ler LL técnico - pedagógico
Excepto terminados en grupo consonántico LL ítalo - germánico
LL Ca – nals LL teórico - práctico
LL I – sern LL anglo - soviético
Los compuestos de dos o más adjetivos unidos
B. Graves o llanas o paroxítonas por guion, cada elemento conserva su acento (or-
Son aquellas que llevan el acento en la penúltima tográfico o prosódico).
sílaba. Llevan tilde cuando terminan en cualquier
consonante excepto “n”, “s” o vocales. C.
Tildación de vocablos adverbializados
Ejemplos: LL dócil + mente = dócilmente
LL Al – mi – bar LL común + mente = comúnmente
LL Di – bu – ja – ron LL rápido + mente = rápidamente
LL Hués – ped LL suave + mente = suavemente
LL U– sas Llamamos vocablo adverbializado a la palabra
Llevan tilde cuando terminan en grupo conso- junta al sufijo–mente, para formar adverbio mo-
nántico. dal.
LL Bí – ceps El primer componente conserva su original.
LL Fór – ceps, etc

32 Mejores Personas, Mejores Familias


5º AÑO DE SECUNDARIA
Colegio

RAZONAMIENTO VERBAL  Mejores Personas, Mejores Familias

D.
Tildación robúrica o disolvente 7. Marque la opción en la que hay escritura correcta.
La que se emplea para indicar el hiato acentual. a) Peruano - aleman
Esto se produce cuando hay en una palabra una b) Ríoplatense
vocal abierta átona y una cerrada tónica. c) Soviético - japonés
Ejemplos: Caída; Geometría d) Décimoseptimo
e) Prioritáriamente

8. Señale lo correcto.
a) Con tu tesis crítico-descriptiva se ha graduado como
1. ¿Qué palabras deben llevar tilde? fisicomatemático.
a) Distraido - calle b) Con tu tesis crítico-descriptivo se ha graduado cómo
b) Veintidos - sapiens físicomatemático.
c) Oimos - Matildita c) Con tu tesis crítico-descriptiva se ha graduado como
d) Friito- realmente físicomatemático.
e) Beneplacito – entreganosla d) Con tu tesis critico-descriptíva se ha graduado como
físicomatemático.
2. Ubique la alternativa que contiene solo palabras paroxítonas. e) Con tu tesis crítico-descriptiva se ha graduádo como
a) Alegre, divertido y noctámbulo físicomatemático.
b) Oraciones, cánticos y ruegos
c) Salud, educación y trabajo 9. Señale la alternativa correctamente tildada.
d) Vino, vio y venció a) Estuvo donde conociste al alferez.
e) Carabina, lápiz y examen b) Dime si traes tu maletín.
c) Se puso botox para borrar sus arrugas.
3. Marque la alternativa donde el acento cumple la función d) Será para Oscar que lo necesita.
distintiva. e) Yo creci tomando leche todos los dias.
a) Devolverán el dinero.
b) Ella usará el hábito. 10. Marque la alternativa donde haya uso correcto del acento
c) Ellos gritaron mucho por lo gélido. ortográfico.
d) Yo solo tomo agua mineral. a) Nunca me gustaron ese tipo de examénes.
e) Deseo jugar muy temprano. b) Discutimos la estratégia en el interín.
c) A la última tentación, Jesús resistió.
4. Marque la alternativa donde todas las palabras están escri- d) No se presentó el jurista italo - americano.
tas correctamente. e) Deberías ser consciente de su estres.
a) Alhelí, aúllar, carnicería, dio
b) Séis, area, ciempiés, ágil
c) Adiós, cráter, paraíso, sílbar
d) Casuística, egoísta, código, apreciáis
e) Nectar, húsares, Juanjuí, dócilmente
1. Marque la alternativa correctamente usado el acento orto-
5. Marque la alternativa correctamente escrita. gráfico.
a) Éste niño es muy locuaz. a) Tocó un solo de piano.
b) Creo que él fue audáz. b) Vive triste desde que te fuíste.
c) No hagas éso nunca más. c) Iré sólamente aunque no lo creas.
d) Con ésto sera suficiente. d) Vive sólo y solamente de sus rentas.
e) Aquella mujer es bonita. e) No hornéa pan con bromato.

6. Señale lo correcto. 2. Seleccione la opción donde se presenta tildación correcta.


a) Tenia un poster de Riky Martin. a) Dijo que no vendria a la reunión.
b) Y se mostro pusilánime en la reunion. b) Me envió veintiséis álbumes.
c) No era un mendígo sino un avaro. c) Se sabe que no hubo reunion.
d) Estuvo difícil el exámen. d) Muy témprano se retiró anoche.
e) Con ejercicios conseguí los cuádriceps de Caicedo. e) Tiene un enorme cicatríz.

5º AÑO DE SECUNDARIA
Mejores Personas, Mejores Familias 33
Colegio

Mejores Personas, Mejores Familias RAZONAMIENTO VERBAL

3. En la siguiente lista de palabras: pasaba, lápiz, públi- 9. Indique la serie en la que las tres palabras deben llevar tilde.
co, ejército, meridiano, acumula, alcohol, beneficio, tene- a) Biceps – fragil – baul
mos__________________________. b) Virgen – examen – arbol
a) 3 agudas, 3 llanas, 2 esdrújulas c) Tiroides – craneo – matriz
b) 1 aguda, 5 llanas, 2 esdrújulas d) Incluido – cesped – area
c) Ninguna aguda, 6 llanas, 2 esdrújulas e) Heroe – cancion – azul
d) 2 agudas, 6 graves, ninguna esdrújula
10. Señale la alternativa en la que aparecen, exclusivamente,
e) 1 aguda, 6 llanas, 1 esdrújula
palabras agudas.
a) Reloj – puerta – abdomen
4. ¿En qué lista todas las palabras deben ser tildadas? b) Papel – regla – tajador
a) Futil, plastificado, asesino, hoja c) Marfil – alcohol – amor
b) Aullido, regimenes, ademan, solar d) Avestruz – oficio – aroma
c) Regimenes, torpeza, camion, temprano e) Feliz – crisis - herida
d) Examenes, futil, interin, razon
e) Folder, senil, procer, condor

5. ¿En qué lista todas las palabras están correctamente escri-


tas? 1. Señala la alternativa que presenta una palabra grave:
a) Rompér, anís, además, héroe a) Profecía c) Adiós e) Vio
b) Maullar d) Solamente
b) Anuncia, cárcel, ebrío, déme
c) Óleo, frágil, jóvenes, démelo 2. Presenta correcta tildación:
d) Nácar, ávaro, táctil, mutuo a) Me muero de vergüénza.
e) Délo, fuiste, petróleo, sepaís b) Que cree un robót.
c) Me comeré un sándwich triple.
d) La ví con su vestido azul.
6. Déficit : crisis
e) Devuélveme a mi «ámix».
a) Sol : Voz
b) Ratón : Amistad 3. En las frases: «El hábito más común de todos» y «Juan ha-
c) Color : Amor bitó tres cuadras», ¿qué función cumple el acento?
d) ínterin : Deseo a) De tono d) Etimológica
b) Fonética e) Fonológica
e) Tarot : Emoción c) Sintáctica

7. Marque la alternativa correcta. 4. ¿Cuál de las siguientes palabras es oxítona?


Las palabras terminadas en -mente son siempre a) Maíz c) Chofer e) Admiró
b) Sastre d) Ópalo
graves o llanas.  ( )
Ningún monosílabo tiene acento. ( ) 5. ¿Cuál de las siguientes palabras está bien silabeada?
Siempre que una palabra aguda termine en “s”, se a) Fa - us-to
tilda.  ( ) b) Bo-hí-o
Cuando la última letra de una palabra grave es “s” c) Ba-hía
d) Cam-bi-áis
jamás se debe tildar.  ( )
e) Trau-ma – to – lo - gía
a) V V V V c) V V F F e) F F F F
b) V F F V d) F F V V 6. En las oraciones:
I. Digo que este día será diferente.
8. Es una palabra ditónica. II. A este le parecía mal el trabajo.
La acentuación de los elementos resaltados
a) Antiguamente es____________.
b) Clemente a) prosódica y especial
c) Rompeolas b) diacrítica
d) Desfibrilador c) general
d) disolvente
e) Antipreuniversitario
e) enfática

34 Mejores Personas, Mejores Familias


5º AÑO DE SECUNDARIA
Colegio

RAZONAMIENTO VERBAL  Mejores Personas, Mejores Familias

7. Señala la palabra que debería llevar tilde. 9. Señala la oración que presenta acentuación incorrecta.
a) Darwin a) Tuvo un problema en el cúbito dorsal.
b) Acentuado b) Usted se vá con Diós.
c) Muleta c) Yo te llevaré a ese hospital.
d) Forum d) Iré esta vez a las 8 o 9 de la noche.
e) Tesis e) Hizo mal cálculo para las compras.

8. Señala la serie con tildación correcta. 10. ¿Por qué la palabra «bahía» lleva tilde?
a) Fuimos, vio, volúmen, violencia. a) Para disolver el diptongo que se formaría si no lo llevara.
b) Albúm, dictámen, querría, parsimonia. b) Porque es una sílaba llana terminada en vocal.
c) Anúnciaba, grúa, parabién, ciempiés. c) Porque es una palabra aguda.
d) Ganzúa, traumático, frágil, veréis. d) Porque necesita de tilde diacrítica.
e) Ciempiés, gracíl, reumático, ágil. e) Porque es un sustantivo que carece de sufijo derivativo.

5º AÑO DE SECUNDARIA
Mejores Personas, Mejores Familias 35
Razonamiento
Colegio Verbal
Mejores Personas, Mejores Familias RAZONAMIENTO VERBAL

MA
TE

08   TALLER DE REDACCIÓN

I. TEXTO EXPOSITIVO – USO DE TILDES


NO OLVIDES TOMAR EN CUENTA QUE…
Un  texto expositivo  es el que presenta de forma objetiva hechos, ideas y conceptos. Su finalidad es
informar sobre un tema determinado (que se desarrolla a través de subtemas); de manera que el autor
en ningún caso plasme sus opiniones, pensamientos y/o sentimientos. Generalmente está escrito en
3ra. persona.

1. Algunas características lingüísticas de los textos expositivos son:


YY La objetividad (tendencia del uso de la 3ª persona verbal, léxico denotativo).
YY Uso de oraciones impersonales y enunciativas.
YY Claridad y precisión.
YY Uso preferente del presente intemporal y del modo indicativo. Puntualmente, el uso del imperfecto si se trata de la des-
cripción de procesos.
YY Uso de conectores
YY Empleo de recursos como las comparaciones, definiciones, enumeraciones, ejemplos.
YY Los textos expositivos son conocidos como informativos en el ámbito escolar. La función primordial es la de transmitir
información, pero no se limita simplemente a proporcionar datos.
2. Tipos de textos expositivos:
A. Analizante o deductivo:
Expone la idea principal al inicio, la desarrolla y detalla a lo largo del texto.
B. Sintetizante o inductivo:
Expone al comienzo los datos o ideas particulares para llegar al final o determinación del tema fundamental.
C. Encuadrada o cuadrativa:
Presenta al principio el tema que se desarrolla a lo largo del texto, y por último da una conclusión que refuerza la
idea inicial.
En los textos con este tipo de estructura las ideas se exponen sin que haya necesidad de coordinarlas entre sí; todos
tienen el mismo nivel de importancia.

II. CLASIFICACIÓN DE LAS PALABRAS POR EL ACENTO

ZZ Palabras agudas: Son las que van acentuadas (prosódicamente) en la última sílaba.
Ejemplos: cantar, beber, alfiler
ZZ Palabras graves o llanas: Son las que llevan el acento prosódico en la penúltima sílaba.
Ejemplos: beso, conde, muchacha
ZZ Palabras esdrújulas: Son las que llevan el acento prosódico en la antepenúltima sílaba.
Ejemplos: cómico, ósculo, barómetro
ZZ Palabras sobreesdrújulas: Son las que llevan el acento prosódico en la sílaba anterior a la antepenúltima.
Ejemplo: arrebatándoselo

36 Mejores Personas, Mejores Familias


5º AÑO DE SECUNDARIA
Colegio

RAZONAMIENTO VERBAL  Mejores Personas, Mejores Familias

2. Relaciona correctamente:
I. Claudio A. Locusta
II. Nerón B. Papa
ZZ Lee atentamente y responde las preguntas formuladas. III.Clemente VII C. Agripina
TEXTO I a) IA, IIB, IIIC b) IC, IIB, III A
Corría el año 54 de nuestra era y el Imperio romano se c) IB, IIA, IIIC d) IC, IIA, IIIB
encontraba en el apogeo de su esplendor. El emperador e) IA, IIC, IIIB
Claudio, máximo pontífice y jefe de los ejércitos, se disponía,
en la soledad de una de las cámaras de Palacio, a cenar un 3. No son afirmaciones verdaderas del texto:
plato de setas, su manjar predilecto. La emperatriz Agripina I. El Imperio romano se encontraba en el apogeo de su
le miraba de lejos, aguantando la respiración y esperando que esplendor alrededor del año 54 a. C.
los efectos de la letal Amanita phalloides hiciesen efecto. Al II. Las setas eran el manjar predilecto de Claudio.
terminar, Claudio se retiró a su alcoba con un fuerte dolor III.La sustancia venenosa usada para matar al empera-
de estómago. dor Claudio fue la letal Amanita phalloides.
Pidió a un esclavo que le suministrase una pluma de ave para a) I y II b) II y III c) I y III
vomitar, pero Agripina lo tenía todo preparado: la pluma iba d) Solo II e) Todas
convenientemente emponzoñada, y así selló el fin de uno de
los emperadores romanos que con más desigual fortuna han 4. No son afirmaciones falsas:
pasado a la historia. I. La pluma de ave que le suministraron a Claudio para
La muerte de Claudio no sería ni la primera ni la última vomitar estaba convenientemente emponzoñada.
provocada por el veneno, una sutil arma política que ha II. El veneno puede tener una utilidad política.
sido utilizada desde los tiempos más remotos para eliminar III.Hasta bien entrado el siglo XIX era imposible detec-
rivales políticos, cambiar regímenes y resolver conflictos tar sustancias tóxicas en los cadáveres.
dinásticos. Y todo con una impunidad casi absoluta. La lista a) Solo I b) Solo II c) Solo III
de los que han sucumbido ante la ponzoña es interminable: d) II y III e) Todas
emperadores como Claudio o Napoleón, Papas como
Clemente VII o Alejandro VI, faraones como Tutankamon, 5. Uno de los siguientes personajes no pertenece a la serie:
reyes como Ciro El grande, o incómodos filósofos como a) Claudio b) Agripina c) Napoleón
Séneca forman parte de esa selecta nómina de grandes d) Alejandro VI e) Tutankamon
hombres que han descendido a la fosa tras probar un amargo
sorbo de veneno. TEXTO II
El hecho es que hasta bien entrado el siglo XIX quien moría El flúor fue el último de los no metales que se preparó en
envenenado moría sin más, porque era imposible detectar estado libre (gases nobles aparte). Desde que fue descubierto
sustancias tóxicas en los cadáveres. Si algún príncipe o un en 1771 por el químico sueco Carl Wilhelm Scheele, pasaron
ministro caído en desgracia moría en extrañas circunstancias 100 años hasta que el químico francés Henri Moissan lo
los rumores circulaban, pero poco podía hacerse, pues no aisló en 1886. Durante este período se realizaron numerosos
había modo de demostrar el magnicidio y, mucho menos, de intentos fallidos para obtenerlo. Entre los que lo intentaron
cargárselo a alguien. No en balde, una obsesión recurrente sin conseguirlo, hay grandes nombres de la historia de la
de los reyes de tiempos pasados era contar con un probador química como Faraday, Davy (descubridor del sodio, potasio,
entre sus asistentes de cámara. El emperador Nerón lo llevó calcio y magnesio), Gay-Lussac y Thénard (descubridores
al extremo. Aparte de una legión de esclavos probadores del Boro). Algunos de los que lo intentaron murieron y la
de alimentos, disponía de un médico, Andrómaco, que le mayoría sufrieron graves envenenamientos por el flúor y sus
preparaba antídotos y de una experta en brebajes venenosos, compuestos. La dificultad que presenta la obtención del flúor
la anciana Locusta, que, caprichosamente, había sido la radica en que gracias a la reactividad, generada al formarse
responsable del envenenamiento de su padre adoptivo. se combina con lo que encuentra a su alrededor. El éxito de
1. ¿Cuál sería la idea principal del texto anterior? Moissan fue consecuencia de utilizar platino, un metal muy
a) Claudio fue el emperador romano que pasó a la histo- inerte, y trabajar a bajas temperaturas reduciendo de esta
ria con desigual fortuna. manera la actividad del flúor. El flúor es un gas de color
b) El veneno ha sido usado en múltiples ocasiones para verde-amarillento, altamente corrosivo y venenoso, de olor
acabar con la vida de personajes importantes. penetrante y desagradable. Es el elemento más reactivo de
c) El veneno ya se usaba durante el año 54 en el Imperio toda la tabla periódica.
romano cuando este se encontraba en el apogeo de su Se combina directamente y, en general, de forma violenta con
esplendor. la mayoría de los elementos.
d) La anciana Locusta había sido la responsable del en- El ácido fluorhídrico (HF) es también una sustancia muy
venenamiento de su padre adoptivo. corrosiva. Su facilidad para atacar al vidrio se utiliza en la
e) El veneno es un «remedio» seguro contra personajes industria para la realización de grabados.
incómodos ya que las sustancias tóxicas no pueden
ser detectadas.

5º AÑO DE SECUNDARIA
Mejores Personas, Mejores Familias 37
Colegio

Mejores Personas, Mejores Familias RAZONAMIENTO VERBAL

6. El texto anterior nos refiere principalmente _______. 10. Es información verdadera según el texto:
a) al origen y desarrollo del flúor I. La naturaleza parece determinar el número de huevos
b) a los grandes científicos investigadores del flúor o crías que van a permitir el correcto desarrollo de la
c) al flúor como el más importante de los elementos quí- especie.
micos
d) al descubrimiento y características del flúor II. Cada hormiga pone 100 huevos.
e) a los usos y peligros del flúor III.Son pocos los huevos de hormigas que prosperan y
dan lugar a individuos adultos.
7. De la lectura se puede entender que ________. a) Solo I b) Solo II c) Solo III
I. el flúor existe desde 1771 d) I y II e) Todos
II. Faraday descubrió el sodio
III. el flúor no se combina con algunos elementos.
a) Solo I b) Solo II c) Solo III
d) I y II e) Todas

8. Relaciona de manera apropiada:


1. Davy A. Boro USO DE TILDES
2. Thénard B. Aislamiento del flúor
3. Moissan C. Descubrimiento del flúor 1. En: De te de menta para los invitados; a mi deme una ga-
4. Scheele D. Calcio seosa mas, ¿cuántas tildes faltan?
a) 1A, 2B, 3C, 4D b) 1D, 2C, 3B, 4A a) 0 b) 4 c) 5
c) 1C, 2B, 3D, 4A d) 1D, 2A, 3B, 4C d) 3 e) 2
e) 1D, 2C, 3A, 4B
2. Completa el siguiente párrafo:
TEXTO III «______ vale pájaro en mano que embarazo adolescente.
Cuando una especie animal encuentra dificultades para ______ que es un refrán mal dicho, pero está bien apli-
reproducirse, la naturaleza pone remedio y permite que sea cado».
inmenso el número de huevos o crías que van a permitir el a) Mas - Si, ya sé b) Más - Sí, ya sé
correcto desarrollo de la especie. c) Más - Sí, ya se d)Mas - Sí, ya sé
Hagamos un pequeño cálculo para demostrar de qué
manera crecería la descendencia de una hormiga y cómo las 3. Indica qué palabras deben llevar tilde:
dificultades que encuentran en el medio, aniquilan millones «(1)Tia, (2)si me (3)rio, me gritas; (4)si no me (5)rio,
de ellas. también. (6)Aun no (7)se (8)como complacerte».
Supongamos que cada hormiga pone 100 huevos y que en el a) 1, 3, 5, 6, 7 y 8 b) 1, 3, 5, 7 y 8
curso de un verano se alcancen seis generaciones de hormigas. c) 2, 3, 5, 6 y 7 d) 1, 3, 5, 7 y 8
En la primera generación saldrán 100 hormigas, de ellas 50
hembras; de estas 50 hembras, en la segunda generación 4. Reconoce la oración correctamente tildada:
salen 5000 hormigas, de las cuales 2500 serán hembras... y a) Dé aquí se va a su cuarto y, dé todas las prendas que
siguiendo el proceso, en la sexta generación aparecerían 1 hay, de a los muchachos las rosadas.
562 500 000 000 hormigas que puestas en fila, cubrirían unas b) De aquí se va a su cuarto y, de todas las prendas que
20 veces la distancia entre la Tierra y la Luna. Está claro que hay, de a los muchachos las rosádas.
las cosas no suceden así. Son relativamente pocos huevos los c) Dé aqui se va a su cuarto y, de todas las prendas que
que prosperan y dan lugar a individuos adultos. hay, de a los muchachos las rosadas.
d) De aquí se va a su cuarto y, de todas las prendas que
hay, dé a los muchachos las rosadas.
9. El texto anterior da respuesta a __________.
a) ¿Por qué existen tantas hormigas en el mundo? 5. «Para que sepas, desde hoy, que no importa lo que can-
te: siempre será todo igual». Del texto anterior señala las
b) ¿Cómo se reproducen los insectos en general y las
afirmaciones correctas.
hormigas en particular?
I. Uno de los «que» debe llevar tilde.
c) ¿Cuál es la razón por la cual las hormigas ponen tan- II. Los dos puntos están mal utilizados.
tos huevos? III. Falta más de una tilde.
d) ¿Cómo la naturaleza suple las deficiencias reproducti- a) I y II b) II y III c) I y III
vas de algunos animales? d) Solo III e) Solo I
e) ¿Por qué las hormigas no han invadido la Tierra y co-
lonizado la Luna?

38 Mejores Personas, Mejores Familias


5º AÑO DE SECUNDARIA
Colegio

RAZONAMIENTO VERBAL  Mejores Personas, Mejores Familias

6. Menciona cuatro palabras ditónicas.


a) __________________________________
b) __________________________________
c) __________________________________ USO DE TILDES
d) __________________________________ 1. ¿Qué palabras deben llevar tilde?
Es probable que (1)cuando (2)cuándo tengas que rein-
7. Escribe cuatro palabras con hiato acentual. ventarte no tengas idea de (3)cómo (4)como hacerlo.
a) __________________________________ a) 1 y 4 b) 2 y 3
c) 1 y 3 d) 2 y 4
b) __________________________________
c) __________________________________ 2. En: Digamosle de una vez por todas donde puede encon-
trar mas emulos de su fe, ¿cuántas tildes faltan?
d) __________________________________ a) 5 b) 4
c) 3 d) 2

TEXTO EXPOSITIVO 3. Señala el enunciado que presenta correcta tildación:
Agrega un párrafo que continúe. a) Acaso crees tu que no tendremos cómo afrontar las di-
ficultades que nos presenta el destino.
8. La energía solar es una fuente de vida y origen de la b) Acaso crees tú que no tendremos como afrontar las di-
mayoría de las demás formas de energía en la Tierra. ficultades que nos presenta el destino.
Cada año la radiación solar aporta a la Tierra la ener- c) Acaso crees tú que no tendremos cómo afrontar las di-
gía equivalente a varios miles de veces la cantidad de ficultades que nos presenta el destino.
energía que consume la humanidad. Recogiendo de d) Acaso crees tu que no tendremos como afrontar las di-
forma adecuada la radiación solar, esta puede trans- ficultades que nos presenta el destíno.
formarse en otras formas de energía como energía tér- e) b y d
mica o energía eléctrica utilizando paneles solares.
___________________________________________ 4. En: El pregunto inutilmente de donde le vino esa idea: ma-
___________________________________________ tar al ciego de la pistola, ¿cuántas tildes faltan?
___________________________________________ a) 4 b) 3
___________________________________________ c) 2 d) 1
___________________________________________
___________________________________________ 5. Identifica el enunciado adecuadamente acentuado:
___________________________________________ a) El aire frio le refresco la cara. Qué bien se esta aquí,
pensó.
9. El virus de la gripe ha sido considerado uno de los más b) El aire frío le refrescó la cara. Que bien se está aqui,
esquivos conocidos hasta ahora por la ciencia médica, pensó.
debido a sus transformaciones constantes para eludir c) El aire frío le refrescó la cara. Qué bien se está aquí,
los anticuerpos protectores que se han desarrollado pensó.
tras exposiciones previas a gripes o vacunas. Cada d) El aire frio le refresco la cara. Que bien se está aquí,
dos o tres años, el virus sufre algunos cambios me- pensó.
nores. Sin embargo, aproximadamente cada decenio,
luego de que una gran parte de la población mundial
ha logrado algún nivel de resistencia a estos cambios
menores, el virus evoluciona drásticamente.
___________________________________________
___________________________________________
___________________________________________
___________________________________________
___________________________________________
___________________________________________

5º AÑO DE SECUNDARIA
Mejores Personas, Mejores Familias 39
Colegio

Mejores Personas, Mejores Familias RAZONAMIENTO VERBAL

MA
TE

09   PLAN DE REDACCIÓN I

Se denomina plan de redacción al esquema de carácter lógico deductivo integrado por un conjunto ordenado de
enunciados que sirven de base para la construcción de un texto completo (ensayo, monografía, relato, etc.) coherentemente
estructurado.

ZZ Ejemplo N° 1
Supongamos que pretendemos redactar un escrito sobre La historia del Perú; pues bien, lo primero que debemos diseñar
es un esquema previo, un «esqueleto», basado en una estructura lógica deductiva con los siguientes elementos básicos
probables.

1. La historia del Perú es muchas veces desconocida por los propios habitantes del
país. Plan
2. Descubrimiento de lo que hoy constituye nuestro país de
3. La Conquista española del Perú redacción
4. La Independencia del Perú
5. Consecuencias de la Independencia del Perú


I. Estructura de un ejercicio de plan de redacción
Un ejercicio de plan de redacción consta de un título a través del cual se propone un tema y cuatro (4) o cinco (5) oracio-
nes o frases formuladas de manera desordenada, las mismas que expresan distintas ideas en torno a la temática a desarro-
llar. Seguidamente se consignan cinco (5) alternativas que señalan diferentes modos de ordenar los enunciados, dentro de
los cuales se encuentra el orden correcto que el evaluado debe establecer en base al análisis de sus componentes, así como
a determinados criterios lógico deductivos.

ZZ Ejemplo N° 2

El alma en el pensamiento platónico Título

I. Particularmente esto generó un deslumbramiento filósofico en el platonismo medieval.


II. Así, toda filosofía de raigambre platónica está siempre centrada en el alma.
III. El concepto de alma tiene primacía en el pensamiento platónico. Enunciados
IV. Para este, el conocimiento de su alma implicaba el encuentro con la felicidad y el acce-
so a la verdad.
V. Inversamente, toda filosofía centrada en el alma es siempre una filosofía de rai-
gambre platónica.

a) II – V – III – IV – I d) III – II – V – I – IV Alternativas


b) II – V – IV – III – I e) III – V – II – IV – I múltiples
c) I – V – IV – III – II

40 Mejores Personas, Mejores Familias


5º AÑO DE SECUNDARIA
Colegio

RAZONAMIENTO VERBAL  Mejores Personas, Mejores Familias

II. Esquema general de redacción IV. En 1665 Londres fue asolada por la peste bubónica.
Es un ordenamiento integral y convencional compuesto V. Gran físico matemático que formuló las leyes básicas de
por tres niveles básicos: introducción, desarrollo y conclu- la mecánica.
sión. Resulta eficaz como una pauta de orientación básica VI. En 1664, a los 21 años, escribe en forma de apuntes su
para resolver los ejercicios. Además, es el esquema que em- primer libro Algunas cuestiones filosóficas.
plean las ciencias para plasmar y difundir el conocimiento. a) V-III-I-VI-IV-II b) III-V-II-I-IV-VI
c) III-I-V-VI-IV-II d) III-VI-I-V-IV-II
1. Introducción e) III-VI-IV-II-I-V
●● Presentación (expresión llamativa o que contenga
palabras del título, una interrogante, etc.) 3. El proceso tecnológico
●● Antecedentes I. Evaluación del objeto tecnológico
●● Etimología II. Construcción del objeto tecnológico
●● Definición / Concepto / Importancia III. Identificación del problema tecnológico
●● Causas / Orígenes IV. Diseño del objeto tecnológico
●● Idea más general a) I-II-III-IV b) IV-III-II-I c) III-IV-II-I
2. Desarrollo d) II-I-III-IV e) II-III-IV-I
●● Análisis o descripción
●● Características 4. Federico García Lorca
●● Clasificación I. En su poesía destacan el simbolismo y la metáfora.
●● Explicación del tema en sus diversas formas II. El teatro de Lorca es variado y de gran valor artístico.
III. Figura fundamental de la generación del 27, nació en
3. Conclusión Granada en 1898.
●● Ejemplos IV. Se deben poner en relieve sus tragedias: Bodas de San-
●● Síntesis gre y Yerma.
●● Recomendaciones V. Su arte se expresa tanto en poesía como en obra dramá-
●● Aplicación, proyecciones tica.
a) V-IV-III-II-I b) III-V-I-II-IV
Empero, la experiencia nos demuestra que una parte signifi- c) IV-V-II-I-III d) II-III-V-I-IV
cativa de los ejercicios no corresponde a este esquema general e) III-V-II-IV-I
por carecer de los componentes básicos, ante esta circunstancia
5. La intencionalidad en la fenomenología de Husserl
se impone aplicar los criterios de ordenación que seguidamente
I. Semblanza biográfica de Edmund Husserl
formulamos. Lee cada ejercicio y plantea el orden lógico de
II. Tesis fundamentales de la fenomenología
las premisas. III. La intencionalidad en la fenomenología de Husserl
IV. Intencionalidad en Brentano, maestro de Husserl
V. La intencionalidad en la filosofía medieval
a) V-II-IV-I-III b) V-IV-I-II-III
c) V-III-I-II-IV d) V-IV-III-II-I
ZZ Indica el orden correcto de las oraciones propuestas. e) I-IV-II-III-I

1. El concepto del universo
6. Joyce: el gran renovador de la novela
I. Ptolomeo dijo que la Tierra era el centro del universo.
I. Esta novela marcó un hito en la literatura: presenta in-
II. El hombre no ha tenido siempre la misma idea del uni-
mejorablemente el monólogo interior y la perspectiva
verso.
múltiple.
III.Copérnico superó a Ptolomeo, sosteniendo que el cen-
II. Al inicio, Joyce escribió obras que no llamaron mucho
tro del universo era el Sol.
la atención: Dublíneses y Retrato del artista adolescente.
IV. Para los griegos, por ejemplo, la Tierra era plana y el
III. Nació en Dublín, fue un hombre polifacético, muy eru-
cielo era una bóveda.
dito y de gran memoria.
V. Hoy en día sabemos que la Tierra no es el centro del
IV. Esto cambió en 1922 cuando publicó Ulises, su obra
universo.
capital.
a) II-I-III-IV-V b) II-IV-I-III-V
a) III-II-IV-I b) I-II-III-IV c) II-IV-III-I
c) II-IV-III-I-V d) V-II-I-IV-III
d) III-I-IV-II e) IV-III-II-I
e) I-III-IV-V-II
7. Camus y el existencialismo
2. Isaac Newton
I. Por su experiencia del dolor humano, se adhirió a la fi-
I. En 1661, cuando tenía 18 años, Newton fue enviado al
losofía existencialista.
Trinity College de la Universidad de Cambridge.
II. Nació en Argelia, cuando este país era una colonia fran-
II. Como consecuencia, permaneció en cama hasta su
cesa.
muerte en 1727, a los 84 años.
III. Pero fue un existencialista sui generis: no aceptaba la idea
III.Nació en la Navidad de 1642.
de la literatura comprometida.

5º AÑO DE SECUNDARIA
Mejores Personas, Mejores Familias 41
Colegio

Mejores Personas, Mejores Familias RAZONAMIENTO VERBAL

IV. Vivió las dos guerras mundiales y reflejó en su obra III. El recinto se comunicaba antiguamente con el Corican-
toda la angustia de los hombres de su tiempo. cha.
a) III-IV-I-II b) I-III-II-IV c) I-II-III-IV IV. Actualmente se celebra allí la fiesta del IntiRaymi cada
d) II-IV-I-III e) IV-II-III-I 24 de junio.
V. La construcción está sobre un cerro y es de planta trian-
8. La novela en el Renacimiento gular.
I. El Lazarillo de Tormes, obra anónima, otro caso exce-
lente Orden lógico: ___________________________
II. Durante el Renacimiento se cultivó la prosa de ficción.
3. Mario Vargas llosa
III. Una obra importantísima al respecto es Don Quijote de
I. Ha sido traducido a numerosísimas lenguas y ha gana-
la Mancha
do los mayores premios literarios internacionales, entre
IV. En España, surge por entonces la novela en el sentido
ellos, el Premio Cervantes.
moderno.
II. Junto a Julio Cortázar, Carlos Fuentes y Gabriel García
a) I-II-III-IV b) III-I-II-IV c) II-IV-III-I
Márquez, es considerado uno de los más grandes nove-
d) IV-II-III-I e) IV-II-I-III
listas hispanoamericanos de la segunda mitad del siglo
9. Las revoluciones científicas XX.
I. Surge entonces una nueva teoría que intenta audazmen- III. Nacido en 1936, en Arequipa. Estudió en Bolivia, Piura
te explicar las anomalías. y Lima, por lo que en 1959 viajó a París y luego a Ma-
II. Gracias a éxitos empíricos, la nueva teoría va imponien- drid, donde cursó estudios superiores.
do una serie de nuevas categorías. IV. Alcanzó la fama por primera vez al ganar el premio Bi-
III. La teoría convencional presenta una multiplicidad de blioteca Breve en Barcelona, con su novela La Ciudad y los
anomalías o problemas sin explicar. Perros.
IV. Las anomalías se resisten a ser explicadas en el marco de V. Incorpora las técnicas narrativas más innovadoras de la
las limitadas categorías tradicionales. novela contemporánea (efectos impresionistas, monó-
a) IV-III-II-I b) I-II-IV-III c) II-III-I-IV logo interior), etc.
d) II-IV-I-III e) III-IV-I-II Orden lógico: ___________________________

10. El Oleoducto Norperuano 4. Lanzamientos espaciales


I. Permiten transportar el petróleo crudo y se emplean I. Lanzamiento del Luna 9 (URSS), primera nave que aluniza y
cuando existen grandes distancias entre el lugar de pro- envía información a la Tierra.
ducción y los centros de consumo. II. Series Mariner y Viking (USA); comienza la explora-
II. Se aprovecha el petróleo producido en la selva. ción de Marte.
III. El Oleoducto Norperuano una de las más extraordina- III. Lanzamiento del Lunik II (URSS), primera nave espa-
rias obras de la ingeniería moderna en el mundo. cial enviada a la Luna.
IV. Los oleoductos son extensas tuberías de acero de gran IV. El 3 de marzo de 1972 el Pioner 10 parte hacia Júpiter.
diámetro. V. El Mariner 4 se sitúa en la órbita de Marte.
V. Actualmente, se aprovecha al máximo en la costa el pe-
tróleo producido en la selva. Orden lógico: ___________________________
a) IV-I-III-II-V b) III-I-II-V-VI
5. Evolución del hombre
c) IV-III-I-V-II d) IV-I-V-II-III
I. Lugar del hombre en la naturaleza
e) III-II-I-IV-V
II. Evolución humana como objetivo de estudio
III. Proceso de hominización
IV. El hombre como vertebrado
V. Principales restos fósiles
Orden lógico: ___________________________
ZZ Indica el orden correcto de las oraciones propuestas.
1. La evolución del universo 6. El teatro y el cine
I. El Bing Bang o la gran explosión I. El teatro no solo se ha mantenido; sino que también ha
II. El Sol, su sistema y la Vía Láctea evolucionado.
III. Estrellas: génesis, desarrollo y muerte II. Entre ellas, la participación del público.
IV. Nebulosas, cúmulos y galaxias III. Cuando nació el cine, se pensó que el teatro desapa-
recería.
Orden lógico: ___________________________
IV. Esto prueba que la suerte del teatro no depende del éxi-
2. Sacsayhuamán to del cine.
I. Formada de grandes muros, escalinatas y bancos de V. El teatro se ha renovado con técnicas que el cine no
piedra. puede usar.
II. Fortificación incaica situada en Cusco (Perú). Orden lógico: ___________________________

42 Mejores Personas, Mejores Familias


5º AÑO DE SECUNDARIA
Colegio

RAZONAMIENTO VERBAL  Mejores Personas, Mejores Familias

ZZ Señala el orden correcto de las oraciones. II. Nuevas enfermedades durante el Perú colonial (viruela,
7. El concepto de estructura gripe, difteria, etc.).
I. Definición de estructura III. Teorías sobre el descenso demográfico: guerras, explo-
II. Clases de estructura tación y epidemias.
III. Etimología de la palabra estructura IV. La disminución de los indígenas costeños fue la más
IV. Estructuras matemáticas y estructuras sociales acentuada en el país.
a) II-I-III-IV b) III-II-I-IV c) I V- I I - I I I - I a) II-I-IV-III b) I-II-III-IV c) IV-I-II-III
d) I-II-III-IV e) III-I-II-IV d) IV-I-III-II e) III-I-II-IV

8. Restricciones a la libertad de expresión e información 2. Las mariposas y la coloración
I. La libertad de expresión e información en los pactos in- I. En otras, para advertir que contienen sustancias tóxicas
ternacionales de derechos humanos. o para imitar depredadores.
II. Restricciones al derecho y la libertad de expresión e in- II. En algunas especies, sirve de camuflaje para pasar inad-
formación. vertidas, confundidas con el paisaje.
III. De la libertad de expresión al derecho a la información: III. Este último es el caso de mariposas cuyas alas abiertas,
antecedentes históricos. vistas desde atrás, tienen dos grandes ojos amarillos
que imitan los de una lechuza.
IV. Tipos de restricciones: por seguridad nacional, orden
IV. La coloración de las mariposas es esencial para su su-
público y moral pública.
pervivencia.
a) I-II-III-IV b) III-I-II-IV c) I I - I - I I I - I V
a) II-IV-I-III b) III-II-I-IV c) IV-I-II-III
d) III-I-IV-II e) III-IV-II-I
d) IV-III-I-II e) IV-II-I-III

9. La hernia 3. El desarrollo moral
I. La forma más habitual es la hernia inguinal y aparece I. Piaget establece que hay un patrón ordenado y lógico
principalmente en el hombre. en el desarrollo de los juicios.
II. La hernia se produce, por lo general, como resultado de II. En la actualidad, las teorías más completas son las de
esfuerzos físicos que coinciden con una pared abdomi- Piaget, Kohlberg y Freud.
nal debilitada. III. Kohlberg ha logrado una ampliación y una revisión so-
III. Los orificios herniarios típicos incluyen la ingle y el bre el desarrollo moral.
ombligo. IV. Muchas teorías del desarrollo moral afirman que este
IV. Se da el nombre de «hernia» a la salida de un órgano procede mediante una secuencia.
interno a través de una parte debilitada de la pared ab- a) III-IV-II-I b) I-IV-II-III c) IV-II-I-III
dominal. d) II-IV-III-I e) III-I-II-IV
V. Los puntos donde la pared abdominal es más vulnera-
ble constituyen los orificios herniarios. 4. Sincretismo
a) IV-II-III-V-I b) IV-II-I-III-V I. También se observa elementos del mundo cristiano eu-
c) V-II-IV-III-I d) III-IV-II-I-V ropeo.
e) IV-II-V-III-I II. Algunas fiestas andinas conservan elementos culturales
del mundo nativo.
10. Newton y la Ley de gravitación universal III. Todo ello podría ser considerado como una concilia-
I. Newton establece las tres leyes del movimiento y la Ley ción o sincretismo.
de gravitación universal. IV. Ambas manifestaciones se expresan a través de la mú-
II. La astronomía, desde la época de los griegos, era la más sica y la danza.
respetada de las ciencias. a) I-III-IV-II b) III-I-IV-II c) IV-III-II-I
III. Los movimientos de la Luna, en la época de Newton, estaban d) I-IV-II-III e) II-I-IV-III
mejor estudiados que en la de Galileo.
IV. La Ley de gravitación universal estudia los movimien- 5. La descentralización
tos de planetas y satélites. I. La noción de descentralización geopolítica
a) II-III-IV-I b) III-II-IV-I c) II-III-I-IV II. Fracaso de los planes de descentralización
d) I-IV-III-II e) I-III-IV-II III. Proyectos para llevar a cabo la descentralización
IV. La permanencia del centralismo geopolítico
a) I-II-III-IV b) I-III-II-IV c) IV-III-I-II
d) III-I-II-IV e) II-III-I-IV

ZZ Indica el orden correcto de las oraciones propuestas.
1. La disminución demográfica indígena
I. Las epidemias destruyen rápidamente una población
numerosa.

5º AÑO DE SECUNDARIA
Mejores Personas, Mejores Familias 43
Colegio

Mejores Personas, Mejores Familias RAZONAMIENTO VERBAL

MA
TE

10   TESIS Y ARGUMENTOS

I. Concepto
Uno de los tipos más frecuentes de preguntas de lectura crítica, son aquellas referidas a los argumentos y tesis de la
lectura. Un argumento está formado por una serie de ideas o proposiciones, que se ordenan y exponen para defender o
sustentar una posición o punto de vista. Esta posición o punto de vista puede ser llamada tesis y es importante distinguirla
del argumento.

ZZ V
eamos un ejemplo:
Yo no necesito ir a la universidad. Se trata de perder cinco años de mi vida o más estudiando. Las personas dicen que
es para capacitarte más, pero yo conozco muchos universitarios que están pateando latas. Lo que manda en el mundo
es el dinero, no el conocimiento. También se dice que en la universidad, se obtiene más formación, más nivel, pero los
conocimientos universitarios no tienen la más mínima utilidad práctica. ¿De qué me sirve estudiar cursos como Filosofía
o Historia? Lo que debo saber lo puedo aprender de frente en el mundo, en la vida. Quiero poner un negocio para hacer
dinero en el menor tiempo posible, y no perder tiempo estudiando para ser el empleado de alguien.

1. Pregunta por la tesis


¿De qué nos quiere convencer el autor? Evidentemente, de que a él no le es necesario ir a la universidad para tener
éxito en la vida. Entonces, podemos enunciar la tesis de la siguiente manera: «El autor cree no necesitar ir a la univer-
sidad para tener éxito en la vida».

2. Pregunta por el argumento central


El argumento consiste en las razones, evidencias y afirmaciones que sustentan la tesis.
Entonces, el argumento del texto sería...
_______________________________________________________________________________
_______________________________________________________________________________

II. Pautas básicas de resolución de preguntas
Para formular bien la tesis, se debe tener en cuenta que esta es una opinión sobre un tema determinado.
YY Debes hacerte estas preguntas básicas:
1. ¿De qué o quién se habla o se opina en el texto? tema
2. ¿Qué opinión o postura tiene el autor del texto sobre este tema? tesis
Una vez determinada la tesis del autor del texto, se debe identificar el argumento central sobre el que esta se sostiene.
YY Las preguntas que pueden ayudarte a determinar el argumento central podrían ser formuladas de la siguiente manera:
1. ¿Por qué el autor del texto cree que… (tesis)?
2. ¿Qué razón principal da el autor para sostener que … (tesis)?

III. Esquema del texto argumentativo

Tesis (postura) A favor o en contra del tema


tratado

Razones que sustentan,


Argumentos ejemplifican o prueban la
tesis del texto

44 Mejores Personas, Mejores Familias


5º AÑO DE SECUNDARIA
Colegio

RAZONAMIENTO VERBAL  Mejores Personas, Mejores Familias

4. Señala el argumento central:


a) La ley de reincorporación forzosa de los trabajadores no
toma en cuenta las necesidades reales de los trabajado-
res.
ZZ Lee atentamente y responde. b) La ley es inaplicable, según la opinión de los especia-
Texto I listas, y además, contradice la decisión que los propios
Los países con democracias fuertes, condenan los movimientos trabajadores tomaron en su momento.
de izquierda sin darse cuenta de que ellos mismos generan c) La ley de reincorporación forzosa exige algo imposible
las injusticias que hacen que personas poco idóneas lleguen de acuerdo con muchos expertos.
al poder. Si no me creen, analicen lo ocurrido en Cuba o si d) La ley es una aberración, pues no ha sido creada por
desean, investiguen el caso de Hugo Chávez o Evo Morales. especialistas en derecho laboral, sino por congresistas
que no conocen nada del tema.
1. Señale la tesis del texto:
a) Algunos países no saben por qué condenan a las demo- Texto III
cracias. ZZ Índice: Yo soy el dedo más importante, porque toda la
b) Los países con democracias fuertes son los responsables fuerza de la mano se concentra en mí; si alguien quiere
de que personas inadecuadas gobiernen algunos países. apretar botones, empujar algo o rascarse una oreja, nece-
c) La democracia no sirve si no hay un real pacto social sitará usarme.
por parte de sus gobernantes. ZZ Pulgar: Las funciones que tú realizas podrían ser hechas
d) Las democracias fuertes no se interesan por las viola- perfectamente por Medio o Anular; en cambio yo soy in-
ciones a los derechos humanos que hay en esta parte del dispensable para que la mano pueda sujetar las cosas que
mundo. utilizarán.

2. Señale qué hechos usa el autor como argumentos:


5. El argumento con el que Pulgar refuta a Índice señala que
a) La llegada al poder de presidentes títere que solo obede-
______.
cen al imperialismo.
a) Índice es un dedo que podría ser reemplazado por una
b) El apoyo de los gobiernos fuertes a democracias poco
prótesis.
idóneas para el bienestar común.
c) El descubrimiento de acuerdos secretos de estado entre b) las funciones que realiza Índice pueden ser asumidas
la Unión Europea y los movimientos de izquierda. por otros dedos.
d) Lo ocurrido en Cuba y otros países. c) los cinco dedos de la mano son tan importantes como
Índice.
Texto II d) la mano no sería lo que es si no fuera por Pulgar.
La ley que busca la reincorporación forzosa de los trabajadores
que fueron despedidos en el régimen anterior es una trampa 6. El argumento empleado por Pulgar es:
sediciosa, porque exige lo imposible, tal como han señalado a) La mano solo puede sujetar objetos gracias a la base que
innumerables especialistas. Además, es una aberración, porque le otorga Pulgar.
pretende incorporar a las planillas gubernamentales a gente que b) Índice es un dedo en el que los anillos no pueden llevar-
aparentemente sobraba y que se acogió a beneficios especiales se.
que se ofrecían a quien optara por retirarse voluntariamente. c) Los dedos de la mano tienen funciones bien diferencia-
Es una ley de una demagogia malintencionada, porque busca das.
enfrentar a todos los desocupados con el Gobierno. En el d) Sin el pulgar, los seres humanos no hubiéramos evolu-
futuro, habrá que pedir cuentas de lo que pueda suceder a los cionado.
congresistas que la impulsaron. Su irresponsabilidad no tiene
límites, sin duda.
Texto IV
3. ¿Cuál es la tesis que postula el autor del texto anterior? ZZ Rosa: La celebración del Día del Amigo es una idea acer-
a) La ley de reincorporación forzosa de los trabajadores tadísima, cómo no se pensó en ella antes. Imagínate que
estatales es una aberración. ahora sí tenemos un día específico para celebrar con nues-
b) Los trabajadores estatales que fueron despedidos du- tros amigos y pasarla bien. Ya no tenemos que esperar el
rante el régimen anterior sabían a qué se atenían cuan- Día de San Valentín -día del amor y la amistad- para tratar
do renunciaron a sus empleos. de reunimos; además eso casi es imposible, porque la ma-
c) La ley que exige la reincorporación al Estado de los tra- yoría prefiere pasarlo con su pareja.
bajadores que fueron despedidos en el régimen anterior ZZ Camila: Lo que dices tiene cierta validez. Es verdad que
es una trampa irresponsable. en el Día de San Valentín preferimos pasarlo con nuestra
d) Los congresistas que crearon la ley que pretende la re- pareja, y no necesariamente con nuestros amigos. Pero hay
incorporación al Estado de los trabajadores despedidos que ser realistas, con los amigos comúnmente salimos a
quieren desestabilizar al actual régimen. divertirnos cada semana o por lo menos una vez al mes, así
que no hay motivo para celebrar un día específico el Día

5º AÑO DE SECUNDARIA
Mejores Personas, Mejores Familias 45
Colegio

Mejores Personas, Mejores Familias RAZONAMIENTO VERBAL

del Amigo porque en el año lo celebramos muchas veces. de la Brea y Fariñas para justificar el cuartelazo del 3 de
La creación de este día es solo una idea consumista plan- octubre. Belaunde, en 1980, el mismo 28 de julio, devolvió los
teada por el mercado para vender más. medios a sus legítimos propietarios, pero la parálisis posterior
para desmontar las reformas militares deslució su segundo
7. La tesis de Rosa sería: mandato. Como recordaremos siempre, en 1985, García apeló
a) Los amigos merecen tener un día de celebración. a la farra fiscal y la cuestionada reducción en la compra de
aviones Mirage. Fujimori hizo exactamente lo contrario a lo
b) La celebración del Día del Amigo es una idea formida-
ofrecido en campaña y provocó el tremendo shock del 7 de
ble.
agosto de 1990. Toledo, prisionero de la frivolidad, privilegió
c) En el Día de San Valentín no se considera a los amigos. su sueldo y la refacción del Palacio de Gobierno. El gobierno
d) La amistad es valorada por las personas, por eso debe actual, al parecer, no se queda atrás.
tener su día.
1. Señale la tesis del texto:
8. La tesis de Camila es: a) El ex presidente Belaunde es un buen ejemplo de lo que
a) La celebración del Día del Amigo solo busca una acti- se llama la «luna de miel presidencial».
tud consumista. b) La farra fiscal del expresidente García no ayudó a que
b) No debemos celebrar el Día del Amigo. subiera en las encuestas.
c) Los amigos no deben exigir un día para celebrar su fun- c) Al parecer, la gente evalúa cada día al gobierno de tur-
ción. no, incluso en los primeros días.
d) Los llamados «cien días» son un elemento valioso para
d) No hay razón para celebrar el Día del Amigo, ya que en
evaluar al Gobierno.
el año se celebra muchas veces.
2. Señale cuál es la finalidad de colocar a Belaunde, García,
Texto V Fujimori y Toledo en el segundo párrafo:
Los hombres solo llegan a ser hombres cuando utilizan la a) Ejemplificar cómo se pueden llegar a tomar acertadas
razón. La razón implica pensar antes de actuar, ser prudentes; medidas en los primeros cien días de gobierno.
es decir, conocer los efectos y las consecuencias de nuestros b) Dar un claro ejemplo de una lamentable paradoja.
actos. c) Ejemplificar cómo en cierto lapso los mandatarios pue-
den llevar a cabo acciones que dejan huella.
9. La tesis del texto es: d) Explicar en qué consistieron todos los errores de los úl-
a) La razón implica ser prudentes. timos mandatarios.
b) Una persona debe razonar sobre los efectos de sus ac-
3. Señala qué alternativas son argumentos del texto anterior:
ciones.
I. Velasco nacionalizó la Brea y Pariñas en los primeros
c) La razón dirige nuestro accionar.
cien días de su gobierno.
d) El uso de la razón hace que seamos hombres. II. Los cien días iniciales de un gobierno suelen tener algu-
na capacidad para impactar en la memoria colectiva.
10. Un argumento del texto sería: III. El gobierno actual continúa con la tendencia de em-
a) Usar la razón simplemente lleva a reflexionar sobre plear los cien primeros días como un vector importante
nuestras actitudes. para que se juzgue su única actuación.
b) El hombre que se deja llevar por el instinto no es hom- a) Solo I b) II y II c) I y II
bre. d) Todas
c) Un indicador del uso de la razón es ser prudentes; pen-
sar antes de actuar. Texto II
Ahora, cumplidos sus primeros cien días, el presidente
d) Todos los hombres somos racionales.
acaba de desdeñar el balance tempranero de los cien días y
lo ha calificado de «hábito mediático y político típicamente
estadounidense», que no tiene motivo alguno para extrapolarse
al Perú. También ha descalificado, aplicando abundancia de
adjetivos, a quienes pretendan poner reparos a la gestión de su
gobierno durante dicho periodo. Y digo su gobierno porque este
ZZ Lee atentamente los textos y responde. es el punto que más debería preocuparnos a quienes pensamos
que el primer mandatario, fiel a sí mismo, sigue padeciendo
Texto I una severa incapacidad para delegar funciones y un insaciable
Los cien primeros días (llamados «luna de miel»), cábalas afán de protagonismo. En otras palabras, el presidente sigue
aparte, son un referente muy bueno para juzgar un régimen. siendo básicamente el mismo veinte años después. Lo que no
Por ello, muchos presidentes tratan de llevar a cabo acciones quiere decir que vaya a cometer los mismos errores; ¡Dios nos
decisivas para ganarse la simpatía del pueblo. encuentre confesos!
En el plazo ya mencionado, a Velasco le bastó la nacionalización

46 Mejores Personas, Mejores Familias


5º AÑO DE SECUNDARIA
Colegio

RAZONAMIENTO VERBAL  Mejores Personas, Mejores Familias

4. Señala la tesis del presidente aludido en el texto: preguntaba cuánto tiempo duró la sacudida, cuántas replicas
a) El actual presidente sale desaprobado si analizamos lo se sintieron, qué daños fueron causados, si los animales
hecho durante los cien primeros días de gobierno. se comportaron de modo extraño antes del sismo y otras
b) El actual mandatario no quiere ser sometido a un aná- interrogantes que hoy son básicas para quienes estudian estos
lisis que ponga bajo la lupa sus cien primeros días de fenómenos.
gobierno.
c) El análisis de los cien primeros días de gobierno no tie- 6. Es verdadero de acuerdo al texto anterior:
ne por qué aplicarse a la realidad peruana. I. El marqués de Pombal es considerado un precursor de
d) Muchos presidentes salen mal parados luego de sus cien la sismología.
primeros días de gobierno. II. Al instalar el primer sismógrafo se evitan los
sismos.
5. El argumento central del texto anterior es: III. El marqués de Pombal no se
a) Evaluar un gobierno tras sus primeros cien días es solo distinguió por llevar una vida digna de elogio moral.
un hábito mediático y político de los Estados Unidos. a) Solo I b) I y II
b) El presidente, al igual que en su primera gestión, no ha c) Solo III d) Todas
aprendido a delegar funciones y gusta del protagonis-
mo. 7. El autor se basa en:
c) El presidente ha señalado que su gestión será mejor que a) Consideraciones legales contemporáneas
la que llevó veinte años atrás. b) Estudios psicológicos del comportamiento de las masas
d) El que ahora el Gobierno muestre señales de protagonis- c) Análisis sismográficos detallados
mo y de incapacidad para delegar funciones no debe ha- d) Ejemplos históricos consignados
cernos pensar que cometerá los mismos errores de hace
veinte años. 8. Señala la tesis del texto.
a) A pesar de ser sanguinarios, muchos líderes se han sabi-
Texto III do ganar el favor del pueblo.
Cuando ocurre una tragedia en cualquier parte del mundo, b) El buen desempeño ante alguna catástrofe y la muestra
la gente puede perdonar los errores de sus dirigentes, pero de sensibilidad pueden hacer que un cuestionable polí-
no su insensibilidad. Por supuesto, con el paso del tiempo, tico como el marqués de Pombal pueda aparecer como
cuando se demuestra gran eficiencia para reparar o reconstruir un héroe.
los estragos espirituales o materiales de una calamidad, los c) El papel del marqués de Pombal durante la catástrofe,
sobrevivientes elevan a un pedestal a quien transformó la demostró su gran talento para ganar simpatías en el
devastación en regeneración. pueblo y hacer que estos olviden su modo sanguinario
Eso ocurrió con uno de los políticos más controversiales de de ser.
Portugal, el marqués de Pombal, el primer ministro del rey José d) Los desastres naturales y las guerras suelen elevar la fi-
I cuando Lisboa fue sacudida por un devastador terremoto que gura de los gobernantes autoritarios y represivos al ni-
causó la muerte de más de 60 000 personas en 1755. vel de héroes nacionales. De este modo, la gente olvida
El marqués fue el primer dirigente que ordenó ejecutar sus fallas.
lo que ahora es algo natural: enviar equipos para rescatar
sobrevivientes, apagar incendios y recuperar los miles de Texto IV
cadáveres para enterrarlos inmediatamente y evitar epidemias. No es cierto que el actual gobierno esté derrochando el
También movilizó al Ejército con el fin de impedir saqueos y erario nacional en obras populistas con miras a las próximas
de que los hombres sanos huyeran de Lisboa para obligarlos elecciones. La oposición es mezquina al negarnos la posibilidad
a despejar las ruinas. Una medida cuestionada fue la orden de de apoyar a los ciudadanos menos favorecidos por el sistema y,
ejecutar a los saqueadores en público para asustar a quien se antes que vernos ayudar, prefieren fiscalizarnos sin ton ni son.
atreviese a imitarlos. Todos desearíamos que entiendan que las buenas ideas no se
Poco después, el marqués y el rey contrataron a arquitectos descartan solo porque no vienen de uno.
e ingenieros y, en menos de un año, Lisboa estaba libre de
escombros y se construyeron las primeras edificaciones para 9. El autor postula centralmente:
que, en lo posible, fueran resistentes a los sismos. Cuestionado a) El Gobierno no está despilfarrando los recursos públi-
por construir amplias avenidas y calles anchas en la ciudad, el cos en aras de fines mezquinos e interesados.
marqués de Pombal contestó con gran visión futurista: «Un día b) Las buenas ideas no deben desdeñarse simplemente
serán pequeñas». porque provengan de otro y no de uno mismo.
El hombre que habría pasado a la historia lusitana como un c) Hay mezquindad por parte de la oposición al negarle al
político maquiavélico y tirano fue considerado por muchos Gobierno la posibilidad de ayudar a los desfavorecidos.
como un héroe nacional por su liderazgo durante la crisis y d) La oposición no debería fiscalizar al Gobierno simple-
reconstrucción. Es más, algunos lo señalan como el precursor mente por gusto.
de la sismología moderna, puesto que ordenó la realización
de una encuesta en todas las parroquias del país, en la cual se

5º AÑO DE SECUNDARIA
Mejores Personas, Mejores Familias 47
Colegio

Mejores Personas, Mejores Familias RAZONAMIENTO VERBAL

10. ¿Cuál es el argumento central del texto anterior? Texto II


a) El gobierno actual solo busca su propio beneficio y por Más que madurar, el presidente ha envejecido. A pesar del
ello apoya medidas populistas contrarias a los intereses cabello sin una sola cana, a pesar del baile del teteo, al que a
de la gente. este paso habrá que añadirle pronto el estribillo: ¡chino, chino,
b) Las buenas ideas pueden venir también de alguien dife- chino!
rente a uno mismo y eso debería ser apoyado. Cien días y más. ¡Qué medianía, qué exceso de temor! Qué
c) Sin importar si las iniciativas son del Gobierno o de la retroceso respecto a sus promesas electorales, ya bastante
oposición, todos deberían apoyar lo que ayude a sacar aguachentas; qué férrea voluntad de no hacer olas; qué
adelante a los más desfavorecidos. conservador luce el gobierno del cambio responsable.
d) La oposición prefiere fiscalizar al Gobierno por gusto y Ataca con razón, pero también alevosía, al avión parrandero.
no reconoce una buena idea solo porque es ajena. Pero las grandes líneas económicas del gobierno de Toledo,
muy parecidas a las de Alberto Fujimori, siguen intactas, salvo
la pasada de sombrero a las mineras y algunos shocks leves
como los recientes temblores limeños.

3. Son afirmaciones ciertas:


Texto I I. Es posible encontrar similitudes entre la gestión actual
Durante mucho tiempo, la música ha servido para representar y Fujimori.
a distintos tipos de personas. Tenemos así a la típica chica II. El llamado «avión parrandero» fue adquirido en el go-
romántica que es fanática de las baladas dulces de telenovela. bierno de García.
Al intelectualoide que proclama a los cuatro vientos su afición III. Existe un pacto secreto entre el presidente y algún ex-
por Schubert y la trova, al rebelde de pelos parados y cadena a mandatario.
un lado del raído pantalón que busca la tan mentada anarquía a) Solo I b) I y III
oyendo ritmos subterráneos de letras inconformes escritas por c) II y III d) Solo III
personas que ya pasaron la época de estupidez adolescente, a
la muchacha hiperactiva adicta al estilo de moda siempre y
4. Según el texto:
cuando la haga bailar para salir del aburrimiento de leer un
a) El presidente ha demostrado haber aprendido de sus
libro o solo pensar. Dicho sea de paso, estas divisiones no son
errores.
dogmáticas ya que podemos encontrar a una jovencita con
b) El presidente ha implementado una economía de shocks
carita de mosca muerta y con toda una colección de metal
que está funcionando.
satánico en su habitación.
c) El gobierno actual quiere quitarse de encima la imagen
1. Es verdadero según el texto anterior: de conservador.
I. Leer o solo pensar puede provocarle aburrimiento a d) Las promesas electorales del actual gobierno no se han
una joven. cumplido totalmente.
II. A todos los pseudointelectuales les gusta la trova.
III. Las telenovelas son musicalizadas con baladas de gran 5. El autor postula centralmente:
impacto juvenil. a) El actual gobierno no parece propiciar el cambio, sino
a) Solo I b) I y II más bien una tendencia regresiva en todos los órdenes
c) Solo II d) II y III de la vida social.
b) Las más importantes líneas económicas del gobierno
2. ¿En qué basa el autor su postura? actual son continuistas con respecto a los gobiernos an-
a) En estudios de comportamiento teriores.
b) En la falta de criterio de los jóvenes c) Cien días son más que suficientes para haber consegui-
c) En varios casos representativos do el gran cambio responsable y no esta medianía inso-
d) En la capacidad de la música portable.
d) El Gobierno ha pedido algunas donaciones a las empre-
sas mineras y ha producido algunos shocks.

48 Mejores Personas, Mejores Familias


5º AÑO DE SECUNDARIA
Colegio

RAZONAMIENTO VERBAL  Mejores Personas, Mejores Familias

MA
TE

11   ELIMINACIÓN DE ORACIONES I

I. Definición
Ejercicios de Aptitud Verbal en los que se evalúa la capacidad del alumno para reconocer el tema y el modo adecuado de
desarrollarlo en cuanto a su forma de discurso, generalidad u otros criterios para eliminar un dato (dado en una oración)
que es innecesario para la correcta comprensión de un fragmento, segmentado en una serie de oraciones, o agregar alguna
información al mismo.

II. Criterios para eliminar oraciones


1. Inatingencia o impertinencia
Según este criterio debemos eliminar la oración que aporte una información ajena o contradictoria con el tema trata-
do en el fragmento.
Ejemplo:
(I) Los primeros años de vida tienen una influencia decisiva en la conformación psíquica de una persona. (II) Así, un
niño que fue constantemente criticado y escarnecido seguramente tendrá una débil autoestima. (III) En cambio, un
niño elogiado y estimulado tendrá gran seguridad en sí mismo. (IV) También la alimentación recibida influirá en la
conformación ósea del futuro adulto. (V) Por último, un ambiente de hostilidad puede provocar desconfianza en el
infante.
¿Cómo titularías el fragmento? _____________________________________________________
a) I b) II c) III d) IV e) V

2. Redundancia
Según este criterio debemos eliminar la oración que reitera una idea mejor expresada en otra.
Ejemplo:
(I) El pueblo Maya veneró a una gran cantidad de dioses y los personificó en ciertos astros y en algunos elementos
de la naturaleza. (II) Estos dioses estaban jerarquizados en divisiones mayores y menores. (III) En el grupo de las
divinidades mayores destacaban Itzamná y Cuculcán. (IV) En el grupo de los dioses menores figuraban Yum Kaax e
Ixchel. (V) Una de las particularidades de la religión maya fue su carácter politeísta.
¿Cómo titularías el fragmento? _______________________________________________________
a) I b) II c) III d) IV e) V

Observaciones:
1. El orden de las oraciones no implica mayor o menor importancia, excepto cuando la información de dos de ellas sea
idéntica; en este caso, se elimina la segunda oración dentro del orden mostrado.
2. Es recomendable que luego de leer el fragmento este sea titulado, para la mejor aplicación de los criterios de elimina-
ción.
3. En el caso de redundancia se puede observar las siguiente variantes:
a) Cuando el contenido de dos oraciones son totalmente equivalentes se elimina siempre la segunda.
b) Cuando una de dos oraciones repite un dato, pero a la vez da mayor información se elimina la que tiene menos
información.
c) Cuando una oración hace un breve resumen de las otras, se elimina.

5º AÑO DE SECUNDARIA
Mejores Personas, Mejores Familias 49
Colegio

Mejores Personas, Mejores Familias RAZONAMIENTO VERBAL

a) II b) IV c) I
Tarea d) III e) V

6. I. En el capitalismo surge la separación entre el productor
ZZ Identifica, en cada caso, la oración que no es necesaria para
y los medios de producción. II. Se desarrolla ampliamen-
el desarrollo del tema:
te la división social del trabajo. III. El capitalismo origina
1. (I) Cada cierto tiempo se convoca al pueblo para elegir un desarrollo vertiginoso de las ciencias. IV. Además, el
a sus gobernantes. (II) Muchos postulantes al Congreso objetivo no es satisfacer necesidades colectivas, sino indi-
procuran ubicarse en un buen lugar en la lista de cada viduales. V. La caza de ganancias es la fuerza motriz.
agrupación política. (III) Contratan asesores de imagen a) I b) III c) V
que los puedan hacer «simpáticos» ante el electorado. (IV) d) IV e) II
Se agencian grandes sumas de dinero para competir en
el mercado electoral. (V) Buscan aparecer en la televisión, 7. I. El lenguaje constituye una capacidad humana. II. Exis-
aunque sea a costa de su persona. te una estrecha relación entre lenguaje y otras facultades.
a) III b) IV c) I III. Así, por ejemplo, el lenguaje exige la participación del
d) II e) V pensamiento. IV. Tanto el lenguaje como el pensamiento
son productos sociales. V. Pensar es, en última instancia,
2. (I) La palabra souffle viene del verbo francés que significa usar los signos del lenguaje.
«soplar». (II) Para la preparación del souffle se necesitan a) I b) III c) V
huevos, harina y azúcar. (III) El souffle se caracteriza por d) II e) IV
tener una textura ligera. (IV) Para conseguir la textura del
souffle la clave es batir a punto de nieve las claras de los 8. I . La vicuña es un camélido sudamericano. II. Ella mide
huevos. (V) Para conseguir la textura que hace famoso al apenas un metro de estatura al nivel del lomo. III. Un kilo
souffle, este debe subir e hincharse. de la fina fibra de la vicuña está valorizado en 920 dólares.
a) I b) II c) III IV. Las vicuñas se adaptan muy bien a las alturas. V. Este
d) IV e) V camélido posee una de las lanas más finas del mundo.
a) II b) IV c) I
3. (I) La metonimia es una figura literaria diferente de la si- d) III e) V
nécdoque y hay varios tipos. (II) La sinécdoque manifiesta
la relación entre parte y todo. (III) La metonimia revela, en 9. ¿Cuál de las siguientes oraciones se puede eliminar sin al-
algunos casos, el lazo que existe entre causa y efecto. (IV) terar el sentido de un texto que desarrolle como tema la
La metáfora es una figura semántica distinta de la meto- valoración cultural del color?
nimia. (V) Existe otro tipo de metonimia que actualiza la a) Por ejemplo, el negro, en nuestra cultura se asocia al
relación entre continente y contenido. duelo, a la muerte.
a) II b) III c) I b) En otras culturas, en cambio, se identifica la muerte con
d) IV e) V la presencia de animales como el búho.
c) El vestido blanco las novias es una antigua reminiscen-
4. (I) Al despertar, Gregorio yacía en su cama convertido en cia de la virginidad que era uno de los bienes que era
un extraño ser. (II) Intentó levantarse, pero sus miembros requisito en el matrimonio.
no soportaban el peso de su cuerpo. (III) Entonces, se apo- d) En el cine, es usual que una mujer apasionada y sensual
yo en el borde de su cama y dio un salto hasta alcanzar la use vestidos rojos.
puerta. (IV) Gregorio, acababa de dormir. (V) Gritó hasta
que su hermana vino en su auxilio; entonces, se sintió ali- 10. ¿Cuál de las siguientes oraciones se puede eliminar sin al-
viado. terar el sentido de un texto que desarrolle como tema los
a) II b) V c) III procesadores de texto?
d) IV e) I a) Los procesadores de texto son los programas más usa-
dos pues están especializados en facilitar la tarea de es-
5. (I) Nuestras corridas de toros, como las de todas partes cribir.
del mundo, proceden de la original versión española. (II) b) Reemplazaron a las viejas máquinas de escribir porque
Esta se trasladó tempranamente al Perú, como atestigua la dividen el proceso de escritura en fases.
antigüedad de la plaza de toros de Acho, la más antigua de c) Los procesadores editan e imprimen los escritos
América. (III) Pero en el Perú, además, se incorporaron a d) Por eso es un programa muy difundido.
las costumbres indígenas y se crearon versiones extrañas
como las corridas de búfalos de los pueblos serranos. (IV)
Incluso en algunos poblados serranos son las mujeres las
únicas que torean, a diferencia de lo que ocurre normal-
mente en Lima. (V) Sin embargo, lo que está fuera de toda
discusión, en su naturaleza cruel y brutal.

50 Mejores Personas, Mejores Familias


5º AÑO DE SECUNDARIA
Colegio

RAZONAMIENTO VERBAL  Mejores Personas, Mejores Familias

5. (I) Todo el mundo tiene derecho a entrar a la red. (II) Aun-


que a veces se debería poder bloquear algunas páginas.
(III) La cuestión no es negar, sino utilizar la mente crítica.
(IV) Por ejemplo la web de Rage against the machine. (IV)
ZZ Identifica la oración que se puede eliminar sin alterar la
En esa página, cuatro gringos desadaptados afirman que
estructura lógica del texto.
Abimael Guzmán es un héroe.
1. ¿Cuál de las siguientes oraciones se podría eliminar sin
a) I b) III c) V
alterar el sentido de un texto que inicie con la oración: El
d) II e) IV
Romanticismo no fue solo un movimiento literario, sino una
expresión total del espíritu?
6. (I) Recientemente se han multiplicado los estudios que
a) Se manifestó tanto en la literatura como en la música. algunos psicólogos denominan opciones sexuales no con-
b) El individualismo, la melancolía y la exaltación por lo vencionales. (II) Eso se comprueba, por ejemplo, cuando
natural encontraron en el romanticismo su expresión en los estantes de las librerías encontramos títulos referi-
literaria más desarrollada. dos a la homosexualidad y el lesbianismo. (III) En las cáte-
c) Grecia y la Edad Media fueron dos puntos de referencia dras de estudios de género es usual discutir sobre la validez
para el hombre romántico. de una opción sexual distinta. (IV) Este reciente interés no
d) El Romanticismo fue un movimiento básicamente lite- debe implicar que la sociedad permita el libre ejercicio de
rario e individualista. estas repudiables perversiones. (V) Diversas universidades
norteamericanas han abierto estudios de posgrado en los
2. Elimina la oración innecesaria para el desarrollo del texto: que se pretende abordar toda la sutil complejidad de esta
a) La conexión de Jauja con sus distritos se produce a través práctica humana.
de una carretera afirmada de solo 9 km de extensión. a) II b) IV c) I
b) El carnaval de Marco es una fiesta popular de caracterís- d) III e) V
ticas agrícolas, ganaderas y paganas.
c) Marco suele ser un apacible villorrio rodeado de cam-
7. (I) El fenómeno muestra la esencia, y al mismo tiempo, la
pos de cultivo ubicado en el valle de Yanamarca.
oculta. (II) El análisis fenomenológico determina el senti-
d) El 28 de febrero comienza la celebración del carnaval
do de las esencias. (III) El fenómeno tiene su estructura, su
que se prolonga a lo largo de cinco días de festejos con
propio orden y su propia legalidad que puede ser revelada
música y folclore.
y descrita. (IV) El fenómeno es algo que se manifiesta in-
mediatamente, primero y con más frecuencia. (V) La com-
3. Identifica cuál de los enunciados es incompatible en un prensión del fenómeno marca el acceso de la esencia.
párrafo que inicie con la oración: En nuestra sociedad, el a) II b) V c) I
homosexual aún debe enfrentar una situación general de d) III e) IV
discriminación.
a) Sobre la base de un prejuicio secular, algunos individuos
8. (I) El surrealismo fue una tendencia artística y literaria.
le niegan al homosexual su calidad de persona humana.
(II) Proclamó la primacía de los valores poéticos sobre los
b) En ocasiones, este rechazo, al hacerse abierto y grupal, principios lógicos, afirmando que el arte nacía del sub-
se agrava aún más, dando lugar a situaciones de vio- consciente. (III) Los representantes del surrealismo qui-
lencia. sieron plasmar los sueños y las asociaciones de ideas au-
c) El problema de la discriminación étnica o racial es otra tomáticas de la psique humana planteadas por Freud. (IV)
muestra de la intolerancia irracional de ciertos grupos. Fueron influenciados por la obra de Freud. (V) Surgió en
d) La homosexualidad no debería verse en una dimensión, el siglo XX en Francia.
exclusivamente privada e individual, sino que también a) V b) IV c) II
deberían tomarse en cuenta sus aspectos sociales d) III e) I

ZZ Identifica la oración que debe o puede ser eliminada. 9. (I) El nylon es utilizado para la confección de prendas,
4. (I) Ante la subida en la intención de voto de Toledo: ¿cuál como medias y camisas. (II) Se emplea también gran can-
ha sido el motivo de su ascenso?. (II) Existen razones de tidad de nylon para hacer alfombras. (III) Pese a que exis-
carácter circunstancial y otras personales. (III) En el Perú, ten muchos tipos de nylon, todos son poliamidas. (IV) Su
desde 1990, el voto se constituye como una identificación gran resistencia lo hace ideal para la confección de redes
fuertemente personalizada, dejando de lado la adhesión a de pescar. (V) El nylon es empleado en la elaboración de
ideas. (IV) Esto hace que las lealtades electorales sean muy engranajes de trituradoras caseras.
frágiles y altamente volátiles. (V) Es fácil saber cual es el
a) II b) V c) I
motivo del ascenso del candidato Toledo.
d) III e) II
a) I b) V c) IV
d) II e) III

5º AÑO DE SECUNDARIA
Mejores Personas, Mejores Familias 51
Colegio

Mejores Personas, Mejores Familias RAZONAMIENTO VERBAL

10. (I) El traje espacial de los astronautas está formado por va- 3. (I) Las bacterias han llegado a constituirse en un impor-
rias capas distintas. (II) La capa junto a la piel puede ser tante instrumento genético. (II) En un principio se creyó
enfriada con agua, lo que evita que el astronauta tenga mu- que su mecanismo hereditario y su aparato genético eran
cho calor. (III) Otra capa es hermética y contiene aire para muy sencillos. (III) Ahora se sabe que causan muchas en-
que el astronauta pueda respirar. (IV) Sobre la capa de aire fermedades. (IV) Se ha demostrado que su reproducción
existe una capa de presión similar a la presión terrestre. es un medio para provocar nuevas combinaciones genéti-
(V) La propia NASA confecciona los trajes espaciales de cas. (V) Existe entonces una mayor posibilidad de varia-
sus astronautas. ción en los organismos.
a) V b) III c) I a) II b) III c) V
d) IV e) II d) I e) IV

4. (I) Los dientes son órganos blancos y duros. (II) En ellos
podemos distinguir la corona, que es la zona trituradora,
el cuello y la raíz. (III) Según su forma y función, los dien-
tes se clasifican en caninos, premolares y molares. (IV) En
ZZ Identifica la oración que se puede eliminar sin alterar la la parte exterior, los dientes están revestidos por el esmalte
estructura lógica del texto. y constituidos por la dentina. (V) Con el paso del tiempo,
1. (I) La glándula pineal es una estructura ubicada en el in- los dientes se caen y los hombres suelen reemplazarlos por
terior de nuestro cerebro. II) Tiene forma de piñón y pesa una dentadura postiza.
unos 150 gramos. III) Descartes creía que el alma se origi- a) II b) III c) I
naba en la glándula pineal. IV) A pesar de que se ignoran d) V e) IV
muchas de sus funciones, se sabe que actúa sobre las glán-
dulas sexuales. V) Además, la glándula pineal actúa como 5. (I) El pez parecía jugar entre las plantas que se hallaban
un «reloj» biológico, poniendo en marcha los procesos que en el fondo marino. (II) Una medusa se deja transpor-
desencadenan la pubertad. tar por la fuerza de la corriente marina. (III) Un tiburón
a) II b) I c) IV velozmente viaja en busca de algún alimento que pueda
d) III e) V aplacar su voracidad. (IV) Un destruido submarino casi
oculto entre peñascos submarinos es un triste recuerdo de
2. (I) Los verbos modales son universales: reflejan la manera la Segunda Guerra Mundial. (V) El ballenato nada al lado
cómo funciona la inteligencia humana, el sentido. (II) de su madre aprendiendo el arte de sobrevivir en el acuoso
Un examen detallado muestra que la oposición ser-estar y extenso mundo marino.
contra hacer es el punto de partida para la constitución de a) I b) III c) V
un inventario básico. (III) A partir del ser-estar se articula d) II e) IV
una modalidad del tener vinculada a los valores objetivos.
(IV) La universalidad de los verbos modales está en el he-
cho de que cualquier relato, sea de la cultura o de la época
que fuera, está modalizado. (V) Lo esencial de la semiótica
narrativa es la estructura modal.
a) I b) III c) II
d) V e) IV

52 Mejores Personas, Mejores Familias


5º AÑO DE SECUNDARIA
Colegio

RAZONAMIENTO VERBAL  Mejores Personas, Mejores Familias

MA
TE

12   PREGUNTAS DE REFORZAMIENTO Y
DEBILITAMIENTO DE ARGUMENTOS

Para reforzar o debilitar un argumento, en principio, debemos seguir estos pasos:


ZZ ¿De qué trata la lectura? Tema
ZZ ¿Qué sostiene la lectura acerca del tema? Tesis
ZZ ¿Cómo sostiene la lectura su tesis? Argumento
Si se tratara de algún diálogo, considera cada uno de los textos de manera independiente.

Reforzar y debilitar un argumento 2. ¿A qué apela fundamentalmente Lucía en su argumento?


ZZ Una opinión debilita o refuerza, pero muy poco. a) A la relatividad del patrón de belleza femenino.
ZZ Un hecho debilita o refuerza más que cualquier opinión. b) Al aumento de la desesperación de las mujeres por no
ser gordas.
ZZ Un estudio científico debilita o refuerza más que un he- c) Al desfase entre las mujeres de ayer y de hoy.
cho simple. d) Al gusto artístico de los pintores del Siglo de Oro.
Por otro lado
ZZ Una afirmación argumentada debilita o refuerza más Texto II
que una afirmación simple. ZZ Ejecutivo: El gobierno de un país, para mantener el or-
den y la paz, debe establecer el principio de autoridad. El
medio para lograrlo es darle todos los poderes al Ejecu-
tivo para que gobierne con firmeza, sin interferencias, y
evite el caos y el desorden.
ZZ Legislativo: Darle todos los poderes al Ejecutivo sería per-
ZZ Lee atentamente cada texto y marca la alternativa correcta.
judicial para el orden en el país y significaría que se instaure
Texto I un gobierno autoritario y dictatorial, pues tendría poderes
ZZ Lucía: No me importa lo que digan. ¿Para qué andarme sin límite. Para evitar esos peligros el Congreso debe tener
con dietas y ejercicios como tú? Solo tengo que esperar mayores poderes, para poder fiscalizar al Ejecutivo.
un tiempo y, tal vez hasta pueda sacar mi calendario. Eso
de la belleza o fealdad son prejuicios que la cultura va 3. ¿Cuál es el punto central de la discrepancia entre Ejecu-
creando y cambiando, según lo que se ponga de moda. Si tivo y Legislativo?
estuviéramos en el Renacimiento, esa gordura que ator- a) Cómo encontrar una solución al problema al caos y el
menta a tantas mujeres, sería su principal valor estético. desorden social.
Las rechazadas de hoy habríamos sido las modelos de b) Cuál es el origen de todas las dictaduras.
Botticelli y Raphael. c) Si el poder se debe concentrar en el Ejecutivo o el
ZZ Fabiola: ¿Y cuánto tiempo vas a esperar? ¿Un año? ¿Cinco? Legislativo.
¿Diez? ¿Un siglo? Vivimos en una cultura que aprecia las fi- d) Si el Congreso debe tener todos poderes o no.
guras esbeltas y, si queremos ser atractivas, tenemos que ade-
cuarnos a eso que tú llamas «modas». No comer lo que te gusta 4. ¿Qué premisa, de ser verdadera, reforzaría más al argu-
y matarte con ejercicios puede ser angustiante, pero mira, ¿a mento de Legislativo?
quién crees que está mirando ese chico allá? ¿A ti o a mí? a) La corrupción se da tanto en el Poder Ejecutivo como
en el Legislativo.
1. ¿Cuál es el punto de discrepancia central entre ambas in- b) El Congreso pone límites al Poder Ejecutivo en los
terlocutoras? países en que hay más orden y paz.
a) La belleza femenina c) Al concentrar todos los poderes, el Poder Ejecutivo pue-
b) La universalidad del concepto de belleza, siendo el de lograr la pacificación interna.
caso que este es una convención al igual que el de feal- d) El Congreso puede dictar medidas que sean tan autorita-
dad. rias como las del Poder Ejecutivo.
c) Los escrúpulos que se tienen que tener para poder lle-
var adelante una dieta imperante. 5. ¿En qué concuerdan Ejecutivo y Legislativo?
d) La validez de seguir las modas en su afán de decir y a) El Gobierno de un país debe ser dictatorial.
determinar qué es lo bello. b) El orden interno debe ser logrado por el Legislativo.

5º AÑO DE SECUNDARIA
Mejores Personas, Mejores Familias 53
Colegio

Mejores Personas, Mejores Familias RAZONAMIENTO VERBAL

c) El orden interno es deseable en un país. ZZ Adriana: ¡Qué fresca! ¿Con qué cara se atreve Mónica a
d) Un gobierno sin interferencias se logra sin la inter- vetarme el acceso a su celda? No solo parece creerse la
vención del Legislativo. directora del penal, sino que además se olvida de que ella
recibió collares de oro. Ella sí aceptó que se sobornaran
Texto III jueces para que la justicia la favoreciera y ahora, cree que
ZZ Pilar: La clonación afecta el orden natural de las cosas, puede decidir con quién compartir su celda. ¡Está loca!
especialmente, de la naturaleza humana, pues puede lle- Ya hablé con Baldomero y él me ha asegurado que voy a
gar a impedir que la reproducción se desarrolle según su estar en su celda.
curso natural.
ZZ Pamela: La razón de la clonación es mejorar las condi- 9. ¿Qué semejanza hay en el razonamiento de Mónica y
ciones de vida del ser humano. Ya son muchas las ven- Adriana?
tajas y vendrán otras. Con la clonación, por ejemplo, se I. Pensar que la otra tiene derecho a decidir con quién
podría llegar a producir órganos vitales que, con un tras- compartir su celda.
plante, salvarían muchas vidas. II. Que ambas están dispuestas a hacer uso de sus in-
fluencias.
6. ¿Cuál es el punto central de la discrepancia entre Pilar y III. Que ambas sostienen que la otra es una corrupta.
Pamela?
IV. Ambas sugieren ser inocentes del delito imputado.
a) Las ventajas o desventajas de la clonación, según su
naturaleza y sus efectos. a) I, II y IV b) I, II y III
c) I y II d) II y III
b) Si el trasplante de órganos vitales atenta contra el or-
den natural de las cosas.
10. ¿Cuál es la contradicción en el discurso de Mónica?
c) Si la clonación es ya una realidad o es solo una espe-
culación para el futuro. a) Descalificar a Adriana por hacer uso de sus influen-
cias cuando está dispuesta a hacer lo mismo.
d) La conveniencia de que la reproducción animal se desa-
rrolle lo más naturalmente posible. b) Hablar de derechos cuando está presa, porque ya per-
dió sus derechos.
7. ¿Qué información, de ser cierta, debilitaría más el punto c) Negarse a recibir a alguien en su celda por ser una
de vista de Pamela? persona de conducta impropia.
a) Los animales clonados suelen padecer defectos gené- d) Acusar de conducta impropia a Adriana por recibir
ticos no previstos. los regalos de Baldomero.
b) La mayoría de verduras y frutas que consumimos son
resultado de clonaciones.
c) La medicina está a punto de encontrar una vacuna
contra el sida.
d) Existe una política para facilitar más la donación de ZZ Lee atentamente los textos y responde las preguntas
órganos vitales. planteadas.
Texto I
8. La tesis de Pilar es: ZZ Luis: No entiendo en realidad la utilidad de la Historia.
a) El hombre no puede ser clonado. Es cierto que es entretenida y que es interesante saber so-
b) Los seres clonados presentan defectos. bre el origen de los actuales órdenes sociales. Pero, ¿cuál
c) La clonación afecta el orden natural de las cosas. es la utilidad de esto? Alguien podría pasar su vida sin
d) La clonación va en contra de Dios. leer jamás un libro de Historia y no sería comparativa-
mente más infeliz que un historiador.
ZZ Rodrigo: ¿Por qué una disciplina debería ser útil? La
Textro IV ciencia nos demuestra que el conocimiento siempre es
ZZ Mónica: ¿Cómo que la van a poner en mi celda? ¡De nin- mejor que la ignorancia y por ello debemos cultivarlo.
guna manera! Ella no tiene derecho a estar en mi celda. Me Incluso el contenido de la Historia.
rehuso a compartir mi celda con una mujer de conducta
impropia que se aprovechó de sus influencias y recibió sin 1. ¿Cuál de las siguientes oraciones expresa mejor la tesis
ningún escrúpulo un departamento, motos acuáticas y via- de Luis?
jes alrededor del mundo. ¿Dónde se ha visto esto? ¡Voy a a) El orden social es el producto de la Historia.
hablar con el encargado del pabellón, con el alcaide y con b) Todo conocimiento debería mostrar utilidad.
el ministro si es necesario! ¡Berta no sabe con quién se ha c) La Historia es inútil.
metido! d) El saber es a menudo inútil.

54 Mejores Personas, Mejores Familias


5º AÑO DE SECUNDARIA
Colegio

RAZONAMIENTO VERBAL  Mejores Personas, Mejores Familias

2. ¿Cuál es el punto de discrepancia entre ambas opiniones? 7. Un punto de discrepancia entre Sandro y Gonzalo, sería:
a) La utilidad de la Historia a) Si los mayores deben guiar a los jóvenes en sus vidas.
b) La utilidad del saber a nivel teórico b) Si los jóvenes están o no en la capacidad de cometer
c) Si todo conocimiento debe ser útil o no errores.
d) La utilidad de los historiadores c) Si los jóvenes tienen la experiencia de las personas
mayores.
3. Escoge entre las siguientes alternativas una que ejempli- d) Si las personas mayores pueden o no pensar con gran
fique lo dicho por Luis: serenidad.
a) Un historiador feliz trabajando en su gabinete.
b) Un exitoso comerciante que no sabe si las cruzadas Texto IV
fueron antes o después de Cristo. ZZ Mafalda: Es el colmo que en pleno siglo XXI muchas
c) Las alternativas de trabajo de un historiador. mujeres sufran de acoso sexual en su centro de traba-
d) Un ignorante que siente respeto por un libro de Historia. jo. Lo peor del caso es que la mayoría mantiene silencio
para no perder su trabajo, pues las que se animan a de-
Texto II nunciar a su jefe, terminan siendo echadas. Eso tiene sus
La vida no tiene sentido. No sabemos exactamente a dónde vamos raíces en una sociedad machista, la gente debería tomar
ni de dónde venimos. Solo si Dios (Padre, Hijo y Espíritu Santo) conciencia de ello y el Estado debería crear leyes adecua-
existiera, la vida tendría sentido. das para punir a los acosadores.
ZZ Manolito: Es verdad que hay casos de acoso sexual, pero
4. La tesis del autor sería: la mayoría de las veces son las mujeres las que incitan
a) No sabemos nada de nada. a los hombres, que dada su naturaleza instintiva no
b) El sentido de la vida depende de Dios. pueden controlarse. Por eso, deberían prohibir el uso
c) La vida no tiene sentido. de minifalda y maquillaje en los centros de trabajo, de
d) La vida acaba con la muerte. esa manera se resolvería el asunto sin tanta ley ni tanto
alboroto. En realidad, la solución está en manos de las
5. Señala la opción que debilitaría la postura del autor. mujeres, si no quieren que las acosen, que no se vistan
a) Buscar el sentido de la vida en un ser superior de- tan llamativamente.
muestra falta de imaginación y autoestima.
b) Dios es el responsable de muchas tragedias en todo el 8. ¿Cuál es el punto central de la discusión entre Manolito
mundo, pero tiene un plan. y Mafalda?
c) La vida es fútil con Dios o sin Dios. a) Si el problema del acoso sexual hacia las mujeres en su
d) Las creencias religiosas son divergentes entre sí, a pe- centro de trabajo tiene su origen en el hombre o en la
sar de algunas similitudes. mujer.
b) Si la solución del problema de acoso sexual hacia las
6. ¿Qué acto sería repudiado por el autor? mujeres en su centro de trabajo está en manos de la
a) La existencia de un profesor metalero. sociedad o del Estado.
b) La quema de edificios paganos. c) Si el problema de acoso sexual hacia las mujeres en su
c) La afirmación de los principales científicos de lo acer- centro de trabajo es un problema real o aparente.
tada de la teoría evolutiva. d) Si el problema del acoso sexual hacia las mujeres en su cen-
d) La proyección de una cinta en la cual se muestra que tro de trabajo se puede resolver o no.
Jesús fue un loco.
9. ¿Cuál es la solución que postula Mafalda?
Texto III a) El problema se resolvería si las mujeres denunciaran a
ZZ Gonzalo: Los jóvenes carecen de la experiencia que te- los acosadores.
nemos las personas mayores. Por eso, es nuestro deber b) Se pondría fin al problema si las mujeres se vistieran
guiarlos para que no cometan los mismo errores que he- diferente.
mos cometidos nosotros. Debemos indicarles el camino c) El problema se resolvería si la sociedad dejara de ser
correcto, impidiendo con nuestra serenidad que sus ju- machista.
veniles arrebatos los lleven por sendas que les impidan d) Se pondría fin al problema si se establecieran leyes
hacer algo de provecho en la vida. punitivas.
ZZ Sandro: Es evidente que tú subestimas a los jóvenes.
¿Con qué derecho consideras que debes guiarlos hacia 10. La discrepancia entre Manolito y Mafalda es:
lo que a ti te parece correcto o adecuado? Es posible que a) Total b) Parcial
esos jóvenes sepan más del mundo actual de lo que toda c) Mínima d) Casi inexistente
tu anquilosada experiencia te pueda decir. Además, si no
se equivocan por su cuenta, nunca aprenderán realmente
cómo es esta vida. Dales el derecho a equivocarse, quizás
acierten mejor que tú a su edad.

5º AÑO DE SECUNDARIA
Mejores Personas, Mejores Familias 55
Colegio

Mejores Personas, Mejores Familias RAZONAMIENTO VERBAL

c) La difusión de una conciencia ecológica es la mejor


forma de solucionar el problema de la destrucción de
los recursos naturales.
d) La legislación es el arma más adecuada para combatir
ZZ Lee atentamente los textos y responde las preguntas el agotamiento de los recursos naturales.
planteadas.
Texto I 3. ¿Cuál es el punto de coincidencia entre Lucho y Erika?
ZZ Erika: El sistema económico actual y la intención de a) El problema del agotamiento de los recursos naturales
preservar la ecología son incompatibles, pues el capita- se centra en la actitud de las personas ante el mismo.
lismo se caracteriza por su intento de mantener un cre- b) La conciencia ecológica está muy difundida entre los
cimiento indefinido y ello está en contradicción con los Estados, las empresas y los individuos.
recursos limitados de la Tierra. Hasta que no cambie el c) El sistema económico capitalista debería ser reempla-
sistema económico capitalista, los recursos naturales se zado por uno más beneficioso para el hombre.
encontrarán en grave peligro y nadie podrá hacer nada d) El capitalismo, por su naturaleza intrínseca, no es
al respecto. El futuro del ser humano sobre la Tierra es compatible con la preservación de la ecología.
incierto.
ZZ Lucho: Es cierto que, por esencia, el capitalismo no tiene 4. ¿Cuál es el punto de discrepancia entre Lucho y Erika?
conciencia ecológica, sin embargo, en los últimos tiem- a) Lo beneficioso o perjudicial que puede llegar a ser el
pos hemos podido ver esfuerzos de los Estados y las em- sistema económico capitalista.
presas capitalistas por preservar los recursos naturales. b) La viabilidad o no de difundir una conciencia ecológi-
Aún falta mucho por hacer, pero creo que la difusión de ca entre los Estados, las empresas y los individuos.
una conciencia ecológica, además de leyes que eviten la c) Lo incierto o no del futuro del ser humano sobre el
destrucción de los recursos, nos llevará a la solución del planeta.
problema. d) La posibilidad o no de lograr la preservación de la
ecología dentro de la economía capitalista.
1. ¿Cuál es el argumento principal de Erika?
a) El reemplazo del sistema económico capitalista por 5. ¿Qué premisa debilitaría mejor el argumento de Erika?
otro más beneficioso para el ser humano. a) Los recursos naturales son limitados, pero su explo-
b) El capitalismo, al buscar un crecimiento indefinido, tación puede darse indefinidamente porque estos se
es incompatible con la preservación de la ecología. renuevan permanentemente.
c) No se puede hacer nada para evitar el agotamiento de b) El capitalismo es el sistema económico que más ha
los recursos naturales, pues estos son limitados. perjudicado al ser humano a lo largo de la historia.
d) La falta de una conciencia ecológica difundida entre c) La preservación de los recursos naturales solo se pue-
los Estados y la población. de dar en un contexto económico diferente.
d) La preservación de la ecología supone una inversión
2. ¿Cuál es la postura de Lucho en la discusión? muy grande de dinero; y las empresas capitalistas no
a) La conciencia ecológica no es propia de la economía están dispuestas a invertir.
capitalista, más bien se opone a ella.
b) Los Estados y las empresas capitalistas han hecho
grandes esfuerzos por poner fin al problema de la so-
breexplotación de recursos naturales.

56 Mejores Personas, Mejores Familias


5º AÑO DE SECUNDARIA
Colegio

RAZONAMIENTO VERBAL  Mejores Personas, Mejores Familias

MA
TE

13   COHERENCIA Y COHESIÓN DE UN TEXTO

La coherencia y cohesión son propiedades esenciales y muy importantes que debe poseer cualquier tipo de texto para lograr su
intención comunicativa.

I. La coherencia
La coherencia se refiere a la relación que se establece entre las distintas partes de un texto. Se busca que el texto tenga una
estructura, que esté bien organizado, que tenga sentido lógico y no presente contradicciones. Por el contrario, un texto se
considera incoherente cuando la estructura no es clara, no está bien organizado, presenta contradicciones y repeticiones in-
necesarias, y no tiene sentido lógico.

II. La cohesión
La cohesión se refiere a los recursos y procedimientos que permiten la correcta relación, conexión y organización entre las
oraciones para la construcción de un texto. Es decir, que tanto las palabras como las oraciones deben estar unidas con es-
tructuras pertinentes al tema que se esté desarrollando para lograr coherencia y sentido. Así, cuando un texto no posee una
correcta relación entre las oraciones o enunciados, o no son adecuados, se habla de un texto sin cohesión.
Para lograr cohesión en un texto, y por ende coherencia, es necesario utilizar distintos recursos o procedimientos. A conti-
nuación exponemos los más importantes:

1. Conectores
Los conectores textuales se definen como nexos o elementos relacionantes entre enunciados. Dependiendo de la corres-
pondencia que se quiere establecer, se utiliza uno u otro conector. No tienen género ni número, por lo tanto son invaria-
bles.
Tipo de coordinación Conector Definición Ejemplo

Copulativa y, e, ni, que Indican adición o suma Ella escribe y él lee.

Indican exclusión, No comas tanto o vas a


Disyuntiva o, u, bien
alternancia o equivalencia engordar.

mas, pero, sino, sin embargo,


aunque, excepto, antes, antes Indican oposición entre las El joven llegó aunque ella
Adversativa
bien, salvo, sino (que), no dos proposiciones se fue.
obstante, por lo demás
Entregan una explicación,
Aliméntate, es decir, come
Explicativa esto es, es decir, o sea (que) una proposición de la otra
bien.
oración
bien... bien...; ora..., ora..., este...
aquel; ya..., ya...; unas veces..., Indican la distribución o
No para de bailar, bien
Distributiva otras...; tan pronto... como...; alternancia de las acciones
cueca, bien tango.
sea ... sea; unos ... otros; aquí de los verbos
... allí

2. Correferencias
Recurso utilizado para ir reemplazando las palabras por referentes o sinónimos (personajes, sucesos, lugares, ideas o
procesos) a lo largo del texto, para evitar la redundancia y permitir la secuencia de significado. Esto ayuda a que no se
repitan las mismas palabras al hablar de un tema, sino que se utilicen expresiones distintas al tema original.

3. Pronominalización
Se refiere a la utilización de pronombres personales, relativos o demostrativos en el transcurso del texto para evitar la reitera-
ción de las palabras. Para ello se utilizan los pronombres demostrativos, personales y posesivos.
Para ejemplificar la correferencia y la pronominalización, se considerará el siguiente texto:

5º AÑO DE SECUNDARIA
Mejores Personas, Mejores Familias 57
Colegio

Mejores Personas, Mejores Familias RAZONAMIENTO VERBAL

Estoy verdaderamente desolada. Todo puedo sopor-


tar, pero, ¡ver llorar a mi hermana, eso sí que no! Y Rpta. : _____ / _____ / _____ / _____ / _____
por primera vez en mi vida la he visto sollozar. Ha sido
algo terrible para mí el sentirme incapaz de consolar a 4. Mate de cedrón
Perfecta. I. Nada más agradable que un mate de cedrón.
Hoy, después del almuerzo, llegó una carta para ella. II. Sobre todo si es servido calientito.
Se fue a leerla a su pieza; pasó mucho rato y ella no me III. El cedrón es una planta no originaria de América.
llamó. IV. Finalmente, es un arbusto de hojas verdes, olorosísimas
y lanceoladas.
La emisora de la carta, al hablar de su hermana, pri- V. Probablemente sea una más de las llevadas por los ára-
mero la nombra como tal: «pero ¡ver llorar a mi her- bes a Europa.
mana, eso sí que no!» Posteriormente, dice su nom-
bre: «de consolar a Perfecta» y, finalmente, la identifica Rpta. : _____ / _____ / _____ / _____ / _____
con el pronombre personal ella: «y ella no me llamó».
También utiliza el artículo definido femenino para 5. La Constitución
identificarla cuando dice: «la he visto sollozar». I. Todo Estado, democrático o no, tiene Constitución.
II. Ley fundamental de orden normativo de una comuni-
dad política.
III. Organiza el Estado y enuncia los derechos ciudadanos.
IV. Documento único y sistemático en que se apoyan todas
ZZ Ordena coherentemente las siguientes oraciones, de tal las demás leyes.
manera que se recupere el párrafo original.
Rpta. : _____ / _____ / _____ / _____
1. Rito matrimonial
I. Luego de cinco años de enamorados, decidieron por fin 6. Características de la alpaca
comprometerse oficialmente. I. Algo que ya hacían los antiguos peruanos 4000 años an-
II. Fueron saludados en el atrio de la iglesia por los parien- tes.
tes y amigos invitados. II. Animal típico de la cordillera de los Andes, la alpaca es
III. La fiesta de la boda fue muy alegre, y duró varias horas. pariente de la llama.
IV. Enviaron los partes a sus invitados, entregándolos ellos III. Sin embargo, no posee la mansedumbre de esta, sino
mismos a sus más próximos familiares. que es muy difícil de domesticar.
V. La ceremonia de petición de mano fue muy formal y IV. Ambas provienen del guanaco salvaje que vive hasta
respetuosa de la tradición. hoy.
V. Su aspecto es semejante al de la oveja, con cabeza y cue-
Rpta. : _____ / _____ / _____ / _____ / _____ llo similares a los del camello.
Rpta. : _____ / _____ / _____ / _____ / _____
2. Estadios de la memoria
I. La atención es fundamental en cualquiera de los tres es- 7. La araña y su presa
tadios. I. Aun cuando las pegajosas hebras dificulten la huida, los
II. En el siguiente, organizamos mentalmente la informa- insectos fuertes suelen tener éxito en sus intentos por li-
ción recabada y la almacenamos en la memoria de largo berarse....Mientras la araña no haya llegado hasta ellos.
plazo. II. Le infligirá a la vez algunos mordiscos con sus fuertes
III. Se puede definir la memoria como un proceso que quelíceros, en cuyos extremos va a desembocar una
consta de tres estadios. glándula venenosa. Ese veneno provoca la muerte a una
IV. Y en el último recuperamos dicha información cuando mosca en pocos minutos.
queremos o nos es necesaria. III. Cuando una presa se ha enredado en su tela, lo prime-
V. En el primero, vemos, oímos, olfateamos, etc., la nueva ro que hace una araña es ir hacia ella y asegurarla.
información. IV. La araña corta entonces los hilos que atan a sus víctimas
a la tela y se la lleva, envuelta como está, a la atalaya, y
Rpta. : _____ / _____ / _____ / _____ / _____ allí la cuelga inmediatamente de un hilo corto.
V. En ese momento, inundará a su víctima con un ancho
3. El innatismo torrente de filamentos y la enlazará con rápidos movi-
I. Descartes, filósofo moderno, y la doctrina de las ideas mientos hasta tenerla al poco tiempo envuelta y comple-
innatas tamente indefensa.
II. El innatismo de las ideas en Platón (filosofía antigua)
III. Chomsky: la lingüística contemporánea y el innatismo Rpta. : _____ / _____ / _____ / _____ / _____
IV. La crítica de Locke al innatismo cartesiano
V. Objeciones al innatismo chomskiano

58 Mejores Personas, Mejores Familias


5º AÑO DE SECUNDARIA
Colegio

RAZONAMIENTO VERBAL  Mejores Personas, Mejores Familias

8. Gutenberg y la Biblia
I. Hacia 1450 había comenzado a imprimir pequeñas
obras para luego dedicarse a su más grande objetivo: la
impresión de la Biblia.
II. Así, la Biblia se comenzó a imprimir en 1454 y estuvo ZZ Ordena las oraciones de tal manera que se recupere el párrafo
lista en marzo de 1455. original.
III.Natural de Mainz, Alemania, Johannes Gutenberg, ha 1. El transplante
pasado a la historia como el hombre que inventó la im- I. El trasplante más frecuente es la transfusión de sangre.
prenta. II. El trasplante es la sustitución quirúrgica de ciertos teji-
IV. Para este propósito contó con el apoyo financiero de Jo- dos u órganos por otros similares.
hann Fust, un acaudalado abogado de su ciudad natal. III. Tiene dos problemas: las características propias de toda
V. Esta obra se conoce como la Biblia de 42 líneas por el nú- intervención y la posibilidad de rechazo.
mero de filas que forman las dos columnas de cada una IV. Para evitarlo, se debe seleccionar muy bien el tejido del
de las 1282 páginas de 42 por 62 centímetros en que está donante.
impreso el libro. V. Pueden ser procedentes del mismo individuo, de otro, e
Rpta. : _____ / _____ / _____ / _____ / _____ incluso de un animal.

9. Los instrumentos musicales de la Prehistoria Rpta. : _____ / _____ / _____ / _____ / _____
I. Del período Lítico, por ejemplo, no existen referencias
iconográficas que demuestren que usaron instrumentos 2. Evolución del dibujo
musicales. I. El uso posterior del pincel estaba condicionado a un
II. Tal vez se descubrieron las cualidades sonoras de estos tipo de superficie.
instrumentos por casualidad, por necesidad o por otros II. Los dibujantes que realizaban su labor con tinta china
motivos. aún se servían, en algunos casos, del pincel.
III. Al estudiar los instrumentos musicales arqueológicos, no- III. Esto se dio gracias al descubrimiento del papiro y el
tamos que conforme retrocedemos en el tiempo, estos son pergamino.
cada vez más escasos. IV. Así, durante la Edad Media, lograron elaborar superfi-
IV. Los primeros instrumentos debieron ser cáscaras, caraco- cies para el uso del pincel.
las, fragmentos de huesos, cañas y cualquier otro material V. Probablemente, las técnicas de los dibujos hechos en
que produjese sonido. carbón fueron los más antiguos.
V. No dudamos, sin embargo, que debieron confeccionar-
los rudimentariamente en piedra, o tal vez con material Rpta. : _____ / _____ / _____ / _____ / _____
orgánico.
3. Las várices
Rpta. : _____ / _____ / _____ / _____ / _____ I. Las várices son dilataciones patológicas de las venas.
II. Por los vasos sanguíneos circulan la sangre al corazón.
10. El papel de la universidad III. Las várices también pueden presentarse en otras partes
I. Uno de sus fines es el desarrollo de la investigación. del cuerpo.
II. En el futuro, la universidad debe ser el ente rector del IV. Las venas afectadas son generalmente las de las piernas.
destino del país. V. Esta dilatación se produce en los vasos sanguíneos.
III. Además, debe contribuir con propuestas a la solución
de la problemática nacional. Rpta. : _____ / _____ / _____ / _____ / _____
IV. La universidad es un centro superior de formación pro-
fesional. 4. La Gran Muralla
V. Hoy en día no se puede concebir que la universidad se I. Se formó de esta manera una muralla de más de 5000
desvincule de la tecnología. kilómetros de largo, sobre la que se alzaban, a interva-
los, torres con puestos de guardia: era un baluarte ex-
Rpta. : _____ / _____ / _____ / _____ / _____ traordinario.
II. Para defender el país de las incursiones de las tribus nó-
madas que hacían presión en las fronteras septentrio-
nales, el emperador hizo unir entre sí las distintas forti-
ficaciones erigidas, antes de la unificación, por algunos
estados del norte de China.
III. En China, durante el reinado del emperador Qin Shi
Huangdi, se construyó la llamada Gran Muralla.
IV. A los Ming se les atribuye el trazado actual de la muralla,
que hoy se puede visitar y recorrer a pie en los tramos me-
jor conservados.

5º AÑO DE SECUNDARIA
Mejores Personas, Mejores Familias 59
Colegio

Mejores Personas, Mejores Familias RAZONAMIENTO VERBAL

V. Más tarde fue consolidada por las siguientes dinastías 8. La pena de muerte
y, durante la dinastía Ming, se procedió a su restaura- I. El fundamento racional de la pena de muerte procede
ción total. del principio de la legítima defensa.
II. El criminal es un enemigo de la sociedad y merece la
Rpta. : _____ / _____ / _____ / _____ / _____ pena de muerte.
III. La pena de muerte es una sanción por la que se priva
5. Cleopatra de la vida a los reos por determinados delitos.
I. Cuando César se hallaba en Egipto tras Pompeyo, ella IV. Beccaria es una representante de la teoría abolicionis-
fue amante del general, con el cual, según la leyenda, ta de la pena de muerte.
se encontró en sus apartamentos secretos. V. Los delitos que merecen la pena de muerte implican una
II. Después de la muerte de César, Cleopatra se unió a Marco violación grave de las leyes sociales.
Antonio, a quien dio mellizos.
III. De su amor nació Cesarión, un niño que Cleopatra llevó Rpta. : _____ / _____ / _____ / _____ / _____
a Roma para que el pueblo lo reconociera como herede-
ro legítimo de César. 9. Matrimonio de luto
IV. Cuando Marco Antonio se suicidó, Cleopatra se hizo I. En este contexto fueron las mujeres, sobre todo las ca-
morder por una serpiente mortal, el áspid, y puso fin saderas, las que evidenciaron el sentimiento de dolor
a su propia vida. que la nación entera sentía: se casaban de luto.
V. Cleopatra era hija del rey de Egipto Tolomeo Aulete; II. El archivo fotográfico de algunas familias de la épo-
desde pequeña estuvo asociada al trono con su her- ca guarda para la posteridad algunas placas de lo que
mano menor, Tolomeo XIII, con quien había sido quedará como una de las formas más patéticas de
unida en matrimonio según la costumbre egipcia. mostrar su amor al país.
Rpta. : _____ / _____ / _____ / _____ / _____ III. Casarse de negro no fue solo una actitud de las clases
altas o de los sectores populares de la época. Novios
6. El tiranosaurio de toda condición social iban al altar vestidos de ro-
I. El célebre y terrible tiranosaurio (su nombre significa pas negras.
«lagarto tirano») vivía en América del Norte, hace 70 IV. Luego de la pérdida del monitor Huáscar y la ocupa-
millones de años. ción de Tarapacá por parte de las tropas chilenas, un
II. Se trata de un cráneo macizo que probablemente podía sentimiento de desolación por la patria humillada se
resistir un choque brutal, como cuando el animal embes- apoderó de la población.
tía a más de 30 km/h contra una presa: el monstruo carga-
ba contra sus víctimas y las tumbaba en el suelo para luego Rpta. : _____ / _____ / _____ / _____
desgarrarlas con sus enormes dientes afilados.
III. Se encontraron huesos de este dinosaurio a partir de 10. La necesidad de una lengua universal
1902; más tarde, incluso un esqueleto completo, que I. Como no todos estos idiomas modernos son comu-
permitió una reconstrucción fiel. nes, ha cobrado fuerza un movimiento a favor de la
IV. El enorme cráneo mide 1,40 m y tiene mandíbulas ar- creación de un idioma artificial que todo el mundo
madas con dientes de 15 cm. debería conocer.
V. Los más grandes tiranosaurios medían 13 metros de II. Esta base grecolatina, sin embargo, ha sido el prin-
largo y pesaban 9 toneladas. cipal reparo para considerar al esperanto como una
lengua verdaderamente universal.
Rpta. : _____ / _____ / _____ / _____ / _____ III. En algunos casos, ciertos idiomas cumplen esta función,
como el inglés o el francés, que muchas personas hablan
7. El infarto como segundo idioma.
I. Este dolor se localiza debajo del esternón pasando IV. Este proyecto tiene un antecedente: el esperanto, lengua
luego al brazo izquierdo. artificial que posee una gramática sin irregularidades,
II. El síntoma más frecuente es el dolor intenso y de larga una pronunciación sencilla y un léxico basado en el grie-
duración en el pecho. go y el latín.
III. La causa de tal obstrucción suele ser generalmente un V. Con el fin de resolver los problemas de comunicación
coágulo sanguíneo. que hay en el mundo, debido a la enorme diversidad
IV. Este coágulo obedece a un proceso arterioesclerótico lingüística existente, se ha propuesto establecer el em-
descrito por los especialistas. pleo de un lenguaje internacional.
V. El infarto se produce cuando se obstruye alguno de
los vasos que llevan sangre al miocardio. Rpta. : _____ / _____ / _____ / _____ / _____

Rpta. : _____ / _____ / _____ / _____ / _____

60 Mejores Personas, Mejores Familias


5º AÑO DE SECUNDARIA
Colegio

RAZONAMIENTO VERBAL  Mejores Personas, Mejores Familias

4. Un príncipe que se extingue


I. El guepardo es el más peculiar y extraño entre todos
los felinos.
II. Es precisamente esta especialización lo que está situando
1. Sobre un asesinato contra las cuerdas al príncipe.
I. Las pistas apuntan a incriminar al esposo. III. Algo que no es nuevo, pues desde el Pleistoceno han
II. Se encuentra el cadáver de una joven. desaparecido al menos cuatro especies.
III. El verdadero asesino es juzgado y sentenciado.
IV. Su forma a cazar es también exclusiva, se basa en una
IV. Los primeros sospechosos son interrogados.
veloz carrera.
V. Una prueba cambia todas las conclusiones.
V. Tanto su morfología como sus costumbres resultan
bastante diferentes.
Rpta. : _____ / _____ / _____ / _____ / _____
Rpta. : _____ / _____ / _____ / _____ / _____
2. La novela cumbre de Umberto Eco
I. Valorativamente, la novela es un ensayo novelado so- 5. Así se halló el título Trilce
bre cuestiones de filosofía medieval. I. Tres, trisss, triess, tril, trilssss. Se le trabó la lengua, y en
II. El miedo y la incertidumbre se apoderan de los mon- ceceo salió trilssece ... ¿trilce?, ¿trilce?
jes ante las misteriosas muertes en la abadía. II. Mortificado, César Vallejo repitió por varias veces:
III. El nombre de la rosa es una novela histórica de detec- «Tres, tres, tres...»
tives que se desarrolla en el siglo XIV. III. Las primeras carillas del libro ya habían sido impresas
IV. El franciscano Guillermo de Baskerville trata de arro- y los cambios agregarían tres libras más al costo.
jar luz sobre los terribles sucesos. IV. Se quedó unos instantes en suspenso y exclamó: «Bue-
V. El escenario es un monasterio benedictino del norte no, llevará mi nombre, pero el libro se llamará Trilce».
de Italia. V. Con esa insistencia que tenía para repetir palabras y
deformarlas.
Rpta. : _____ / _____ / _____ / _____ / _____
Rpta. : _____ / _____ / _____ / _____ / _____
3. ¿Es peligroso comer roedores?
I. Ahora bien, el consumo de ratas trae consigo también
graves riesgos.
II. En principio, la carne de rata es tan nutritiva y sabrosa
como la del resto de mamíferos.
III. Ya que estos animales son portadores de numerosas en-
fermedades infecciosas y parasitarias.
IV. De hecho, este roedor es muy preciado en épocas de
penuria y conflictos bélicos.
V. Además, muchos pueblos africanos y países exóticos
del Extremo Oriente la consumen.

Rpta. : _____ / _____ / _____ / _____ / _____

5º AÑO DE SECUNDARIA
Mejores Personas, Mejores Familias 61
Colegio

Mejores Personas, Mejores Familias RAZONAMIENTO VERBAL

M A
TE

14   LA COMA

Signo de puntuación (,) que indica normalmente la existencia IV. COMA DE CONECTOR
de una pausa breve dentro de un enunciado. Se escribe junto a Se escribe coma delante de las conjunciones pero, mas,
la palabra o el signo que la precede y separada por un espacio sino, aunque.
de la palabra o el signo que la sigue. A continuación se exponen Ejemplos:
los usos normativos de la coma.
YY Quisiera acompañarte, mas estoy cansada.
I. COMA ENUMERATIVA YY Escribe bien, aunque puede redactar mejor.

Se emplea para separar los elementos que forman una serie V. COMA ELÍPTICA
o para separar miembros gramaticalmente equivalentes. Se escribe coma para sustituir el verbo en los casos en que
Ejemplos: se omite porque ya se mencionó anteriormente o se so-
brentiende.
YY Sherlock Holmes ordenó al doctor Watson que guar- Ejemplos:
dara la pipa, el sombrero, los libros y las joyas.
YY Visité la biblioteca, anoté la bibliografía, seleccioné la YY Pamela es bella; Patricia, también.
información y revisé varias veces mi borrador antes de YY Víctor es delgado; Pamela, gorda.
escribir este ensayo. YY Julio perdió su billetera; Juan, su mochila.

II. COMA VOCATIVA VI.COMA INCIDENTAL


Se escribe coma para separar los incisos que se incrustan
YY S e usa para separar el vocativo y el mensaje. en el discurso. Se trata de una palabra, una frase o una ora-
Ejemplo: Patricia, apoya a tus hermanos. ción que…
YY Cuando el vocativo va en medio de la oración se escri-
be entre dos comas. YY Explica el sujeto o el verbo, o el objeto.
Ejemplo: No olvides, querido Francisco, que te llevo YY No forma parte de la esencia, tanto que, si se elimina,
diez años de ventaja. la idea queda incólume.
YY Va siempre después de lo explicado.
III. COMA HIPERBÁTICA YY Va entre comas.

Cuando se invierte el orden sintáctico de la oración, se es- En suma, puede usarse las siguientes situaciones:
cribe coma después del complemento anticipado. a) Incidentales en general: este inciso es el que le da a la
Ejemplos: oración el carácter de explicativa.
Ejemplos:
YY De acuerdo con la decisión del grupo, usted no parti-
cipará del campeonato. ●● Francisco, al darse cuenta de que Pamela no esta-
YY Según la experiencia del chavo del ocho, la venganza ba, corrió furioso hacia el parque.
no es buena porque mata el alma y la envenena. ●● Los grupos de redacción, incluidos los alumnos
YY Dinero, ya no le queda. del taller, deberán presentar un texto expositivo.
YY En estos casos la Academia recomienda la siguiente ●● Las manzanas chilenas, cuyo rojo encendido es
norma para verificar la exigencia de coma: una de sus características, son mis preferidas.
a) Si el elemento antepuesto admite una paráfrasis ●● El genio literario, dice Harold Bloom, es difícil de
con «en cuanto a»: definir.
Ejemplo: Dinero, ya no le queda. (En cuanto al
dinero, ya no le queda) b) Las aposiciones explicativas ofrecen información o
b) Si, por el contrario, admite una paráfrasis con «es explicaciones sobre un nombre o persona.
lo que» o «es el que», no se empleará coma: Ejemplos:
Ejemplo: Vergüenza debería darte. (Equivalente a ●● Gabriel García Márquez, autor de Cien años de
«Vergüenza es lo que debería darte») soledad, nació en Colombia.
●● Alonso, el profesor, es un gran hombre.

62 Mejores Personas, Mejores Familias


5º AÑO DE SECUNDARIA
Colegio

RAZONAMIENTO VERBAL  Mejores Personas, Mejores Familias

c) Coma delante y después de las conjunciones «y», «o», III. No la llamó Filemón, el que la despertó ayer; por ello,
«e», «u» está triste.
Los incisos explicativos van entre comas, aunque haya IV. Los alumnos, que están muy motivados, sí aprobaran
«y», «o», «e», «u». en el examen.
Ejemplos:
a) II y IV b) I, II y IV c) Todas
●● Mario Vargas Llosa, el autor de Conversación en
d) I y III e) I, II y III
la catedral, y Gabriel García Márquez se reunirán

en el Museo de Arte.
5. Tiene correcta puntuación.
●● Al encuentro con los escritores asistirán los alum-
nos de la reunión y, desde luego, los docentes.
a) Le sonreí mucho, mas, no me prestó atención.
b) Por fin, trajo, la pelota, la camiseta y los chimpunes.
d) Cuando la secuencia que encabeza la conjunción expre-
c) Puedes traer, por ejemplo, un tarro de leche.
sa un contenido distinto al elemento o elementos an-
d) Le pediste, mucho más amor, no le importas ni un tantito.
teriores, y cuando la conjunción enlaza la proposición
e) Alegre como nunca, Rebeca abandonó, el albergue.
anterior.
Ejemplos:
6. Ese día ___ al terminar la historia ___ las primas de Doro-
●● Compró un libro, un diccionario y una revista, y
tea se sintieron muy asustadas.
salió de la librería muy contenta.
●● Pintaron las paredes de la habitación, cambiaron a) : – , b) : – . c) , – ,
la disposición de los muebles, y quedaron encan- d) , – : e) ; – ,
tados.
●● La Edad Media identificaba la belleza (además 7. No estaremos ocupados el sábado así que puedes visitar-
que en la proporción) con la luz y el color, y este nos Camilo. Tu tía Eustaquia no parece estar a pesar de
color era siempre elemental, una sinfonía de ro- todo muy entusiasmada. Si puedo convencerla nos acom-
jos, azules, oros, platas, blancos y verdes. pañará. Faltan ___ comas.

a) dos b) tres c) cuatro


d) cinco e) seis

8. Lamentablemente la ciudad que visitamos resultó ser en
1. Ponga la carne de cerdo en un tazón grande. Mezcle el je- realidad un pueblo fantasma; no obstante no estoy entris-
rez la fécula de maíz la salsa de soya el jengibre el aceite tecido: tú eres mi fuerza Pantaleón. Faltan ___ comas.
de oliva y el jugo de limón; vierta sobre la carne; sazone
bien. Deje macerar 30 minutos. ¿Cuántas comas faltan en a) dos b) tres c) cuatro
el fragmento anterior? d) cinco e) seis

a) Una b) Cinco c)Ocho 9. Tiene correcta puntuación.


d) Tres e) Cuatro
a) Le entregué todo señor.
2. Señala la alternativa que necesita coma hiperbática. b) Candy, mi amorcito, vendrá a visitarme hoy.
c) Nobulina cómo es posible, este engaño.
a) No crea don Fernán que me gustó. d) Luego del partido iremos, a la fiesta.
b) Para poder acercarse cambió su personalidad. e) Todos los días Juanacha, levanta pesas.
c) Hoy no verás a Inés la novia de tu primo.
d) Todos incluso Damián irán a las olimpiadas. 10. Está puntuada inadecuadamente.
e) Amiga David José el padre de tu hijo volvió.
a) En textos antiguos, se predijo esta realidad.
3. Venturo ___ si te alejas de mí ___ no sazonaré bien mi b) Hasta que lo descubra, no parará con su ilícito.
vida. c) Hay un solo camino hasta aquí.
d) Manu, cuando tu ex esté conmigo, no quiero verte llorar.
a) , – : b) , – ; c) , –, e) Ven, a mi casa esta Navidad.
d) , – . e) : – ,

4. Hay coma(s) incidental(es) en ____________

I. Roxy, amor de mi vida, llamaste al gasfitero, el amigo que


siempre te ayuda. 1. Nos habrías escuchado mejor (1) si hubieras salido. Más
II. Luis Ramón, tu hermano mayor, canta tan bien como tarde (2) cuando te llamé (3) me dijiste que (4) volve-
su padre. rías aquí (5) pero (6) no volviste. Se requieren comas en
________.

5º AÑO DE SECUNDARIA
Mejores Personas, Mejores Familias 63
Colegio

Mejores Personas, Mejores Familias RAZONAMIENTO VERBAL

a) (2) (5) y (6) b) (2), (3) y (5) c) (3) y (6) 9. Recientemente los resultados de las elecciones resultaron
d) (1), (2), (3) y (6) e) (3), (4), (5) y (6) sin explicación modificados por tercera ocasión pese a ello
tenemos la esperanza viva: confiamos en el pueblo. Fal-
2. Hay uso correcto de la coma. ta(n) ___ coma(s).

a) No saldré de este socavón querido, amigo. a) una b) tres c) cinco


b) No te pude traer la guitarra, el equipo, las luces, ni la d) dos e) cuatro
batería.
c) Lucy, sorprendida, solicitó que le devuelvan la contri- 10. ¿En qué oración se hace uso de la coma explicativa?
bución al benefactor.
d) Siempre revisé cada, pregunta con cuidado: podré a) El ambiente, desde una visión holística, es el aire, las
aprobar sin dificultad. plantas, los animales, las personas, los ríos, etc.
e) Llegaron productos donados desde, Finlandia, Francia b) En cambio, a los que piensan en forma superficial les
y Alemania. preocupa la piedra que cae.
c) La reflexión en voz alta, siempre ejercito.
3. En un par de semanas ___ la inculpada ___ a la cual nadie d) Si nos aprecian, que nos inviten.
soporta ___ volverá. e) El ambiente incluye a los edificios, las calles, los puentes
y muchas otras cosas más.
a) ; – , – , b) , – , – , c) , – , – ;
d) , – ; – , e) , – , – :

4. No necesita una coma.

a) Creo que te llevaré a caminar por el viejo callejón. 1. Señala los tipos de comas en las siguientes oraciones, res-
b) Sí acércate a ella conquistador. pectivamente:
c) Inés mi ex entiende los cursos de Física Álgebra y Lite-
ratura. ZZ Mañana, partido.
d) Le sugerí una salida mas no me hizo caso. ZZ Hija, ese no te conviene.
e) Debido a que te amo te pienso siempre. a) Enumerativa - vocativa
b) Explicativa - apositiva
5. Hay uso correcto de la puntuación. c) Elíptica - hiperbática
d) Enumerativa - elíptica
a) Después cuando vuelva le, avisaré. e) Elíptica - vocativa
b) Norma, si no me llamas, no habrá regalos.
c) Llegó sumamente feliz, el que te «extraña», tanto. 2. ¿Cuál de las siguientes oraciones presenta coma elíptica?
d) Descartes dijo, «Pienso luego existo».
e) Encontré una salida en, consecuencia, soy el ganador. a) Llegaron a Ica, Luis y Juan.
b) Tiéndeme la mano, vida mía.
6. Sinceramente mi vida siempre que puedas debes llamar- c) Ana, ven pronto.
me: te extraño mucho y ese es el principal motivo de mi d) De día divisamos el Sol; de noche, la Luna.
tristeza. La primera coma que falta es ____. e) Lázaro, levántate.

a) hiperbática b) incidental c) enumerativa 3. ¿En qué alternativa se encuentra correctamente empleada


d) vocativa e) apositiva la coma?

7. Le faltan más comas. a) Nosotros, compramos atún; tus padres, pan.


b) Mi hermano, el menor, ingresó a la UNMSM.
a) Sabe que mentiste yo se lo dije hace un rato. c) Tu acción me honra hija, mía.
b) Finalmente lo pude conseguir pero fue con tu ayuda. d) Va a caja, la abre, saca el dinero, y, cancela la cuenta.
c) Rómulo llegará si tú lo ayudas a salir lindura. e) Vendió, por último su, casa.
d) Vine a las cinco; ella a las seis y media.
e) Anteayer el resultado no las convenció totalmente. 4. Señala el tipo de coma que falta en la siguiente oración:
Hasta que vuelvas conmigo, yo daré la media vuelta.
8. Al final de la disputa ___ Marcelino ___ nuestro mejor
comprador ___ estuvo tranquilo. a) Coma vocativa
b) Coma elíptica
a) ; – , – , b) , – , – ; c) , – , – , c) Coma hiperbática
d) , – ( – ) – ; e) , – (–) – , d) Coma explicativa
e) Coma enumerativa

64 Mejores Personas, Mejores Familias


5º AÑO DE SECUNDARIA
Colegio

RAZONAMIENTO VERBAL  Mejores Personas, Mejores Familias

5. ¿En qué alternativa la coma elíptica está omitida? 8. Identifica la oración que precisa del uso de la coma.

a) Tú irás por aquí; ustedes por allá. a) El rumor de las aguas nos despertó.
b) En aquella ciudad te esperaba. b) Fue una película emocionante hasta el final.
c) José el ingeniero trabaja en Lima. c) Cada vez tiene más interés por el estudio.
d) Ana Luisa María y Lucía son amigas. d) No se le otorgó el diploma a ella.
e) Isabel te esperamos en el colegio. e) No te alejes por ningún motivo de mí mi vida.

6. El punto y coma y la coma elíptica deben estar presentes 9. ¿Qué alternativa ofrece un mejor uso de los signos de pun-
en la alternativa _________. tuación?

a) El trabajo paga las deudas la ociosidad las aumenta. a) Messi según mi opinión, fue el mejor futbolista.
b) Su esposo es un gran arquitecto ella una destacada dentista. b) Messi, según mi opinión fue, el mejor futbolista.
c) Tengo un hambre intensa. c) Messi, según mi opinión fue el mejor futbolista.
d) ¡Qué hermoso ramo de rosas y qué bella es esta flor del d) Messi, según mi opinión, fue el mejor futbolista.
cambio! e) Messi, según mi opinión: fue el mejor futbolista.
e) En las manos jubilosas de los obreros flameaba el viento.
10. Señala el número de comas que se han omitido en la si-
7. En el enunciado: Como si fuera fácil me piden que te olvide guiente expresión: «Mañana chicos vendrá Hildebrando
mi amor, se ha omitido el empleo de _______. nuestro aclamado profesor».

a) dos comas a) Una b) Dos c) Tres


b) el punto y coma d) Cuatro e) Cinco
c) los dos puntos
d) una coma
e) los puntos suspensivos

5º AÑO DE SECUNDARIA
Mejores Personas, Mejores Familias 65
Razonamiento
Colegio Verbal
Mejores Personas, Mejores Familias RAZONAMIENTO VERBAL

M A
TE

15   INCLUSIÓN DE ENUNCIADOS

En esta nueva forma de ejercicio, debes de buscar la alternativa II. Procedimiento para incluir una oración al texto:
que complete mejor el texto que se presenta con un vacío en 1. Determina con claridad el tema del texto.
su desarrollo. En otras palabras, se presenta un texto al que le 2. Determina el estilo discursivo: formal, coloquial o téc-
falta un enunciado, y se debe encontrar ese enunciado entre las nico.
alternativas. 3. Determina el tipo de texto: narrativo, argumentativo,
Véase el ejemplo: descriptivo, instructivo o informativo.
Instrucción 4. Determina el nivel de generalidad de la oración a
Elige el enunciado que complete adecuadamente el texto. incluir: ejemplo, características, definición, causa, etc.
5. Escoge entre las alternativas aquella que completa
1. I. _________________. II. En cada colmena, existe una el texto y verifica leyéndolo con la oración agregada
sola abeja fértil, la reina, que es fecundada por un solo (toma en cuenta que la información agregada no debe
macho, el zángano. III. La mayoría de las abejas llamadas ser redundante).
obreras son estériles. IV. La misión de las abejas obreras
es recolectar el polen y el néctar de las flores. V. Con estos
elaboran la miel, alimento de las larvas, y la cera con la que
construyen el panal.
a) El ser humano cría abejas para obtener beneficios.
b) Las abejas obreras buscan la fuente del alimento. ZZ Completa los espacios en blanco con la oración pertinente.
c) En cada vuelo nupcial, el zángano fecunda a la reina.
d) Las abejas se agrupan en colonias llamadas colmenas. 1. Ebenezer Howard postuló por primera vez en el mundo
industrial moderno la idea de movilizar a los habitantes
El enunciado que completa correctamente el párrafo es la de las urbes densamente pobladas hacia ciudades jardín
opción d. autosuficientes.
Encontrar un enunciado específico que complete el senti- Señala cuál de las siguientes alternativas es la más adecua-
do lógico de un texto puede ser difícil si no se tiene claro da para completar la oración anterior:
cuándo un texto está completo ni cómo mantiene su cohe-
rencia interna. En vista de este potencial problema es que a) De esta manera, en la visión de Howard, cuando una
se plantean algunos criterios iniciales que pueden servir ciudad jardín alcanzara unos 32 000 habitantes, sería
como guía al alumno. hora de construir la siguiente.
I. Criterios de resolución b) Es decir, vivirían en casas con jardines en el centro, ca-
1. Coherencia lógica minarían para trabajar en las fábricas y serían alimenta-
Aunque al texto le falte una oración o enunciado, igual dos por granjas.
es posible entender que tema está tratando. En ese c) El sueño de Howard resultó contagioso: no pasaron
sentido, puedes descartar las alternativas que se alejan demasiados años antes de que surgiera Letchworth , la
del tema del texto o que no guardan relación con él: primera ciudad jardín bajo este concepto.
son inatingentes. Es importante que te fijes en la for- d) En 1907, una delegación de 500 esperantistas visitó la
ma en que se presentan los enunciados. Es importante primera ciudad jardín y el propio Howard les dedicó un
que reconozcas el contexto donde se podría estar ubi- interesante discurso (en esperanto, por supuesto).
cando el texto, así como la información más específica
y coherente con la totalidad del texto. 2. Completa el siguiente fragmento de modo que la alternati-
va presente el mismo nivel de generalidad.
2. Conexión por referencia interna En su documental Roma fantástica, Luigi Cozzi demuestra
Debes ubicar los referentes dentro del texto. Conec- lo adelantados que se encontraban los cineastas del giallo
tores y palabras que hagan referencia a conceptos ya italiano a través de una celebrada escena del director Ma-
mencionados son absolutamente necesarios de identi- rio Bava en Kill, Baby, kill en la que la niña protagonista
ficar. aparece y desaparece junto con una pelota; dicha idea sería
retomada por Federico Fellini en Toby Dammit.
3. Sucesión y enumeración
Muchas veces existe una progresión de eventos, vale a) Este documental recorre la capital italiana para home-
decir, un proceso. En ese sentido, debes buscar las cau- najear a los grandes cineastas forjadores del giallo y
sas y las consecuencias de los conceptos que el tema para conocer las locaciones utilizadas en sus películas.
del texto trate. Por otro lado, debes estar atento a posi- b) Con sus anotaciones y fragmentos de películas, Cozzi
bles enumeraciones. Las puedes identificar por comas recuerda, para quienes no la conocen, a la triada de
sucesivas o por los conectores enumerativos (primero, celebérrimos cineastas: Riccardo Freda, Mario Bava y
en primer lugar, en segundo lugar, etc.). Antonio Margheriti.

66 Mejores Personas, Mejores Familias


5º AÑO DE SECUNDARIA
Colegio

RAZONAMIENTO VERBAL  Mejores Personas, Mejores Familias

c) De Antonio Margheriti, Cozzi destaca el hecho de que 6. I. Cuando las ondas se propagan y chocan contra una su-
Klaus Kinski encarne a un pistolero que se asemeja a perficie, rebotan. II. El fenómeno es similar cuando se agita
Drácula en Y Dios hizo a Caín. De aquí, Werner Herzog una cuerda atada a una pared. III. ____________________
vería ya en Kinski a su futuro Nosferatu. IV. En este caso se dice que la onda se reflejó totalmente. V.
d) Brevemente se refiere a Lucio Fulci y a su influencia. En cambio, si la pared es una tela, una parte se refleja y la
Lamenta su muerte y la obra que quedó pendiente, afec- otra será absorbida.
tada por la baja en la producción cinematográfica en
Italia desde los años ochenta. a) Esta reflexión se denomina reflexión parcial.
b) La pared obliga a la onda a regresar a través de la cuerda.
3. Indica el fragmento que se puede suprimir sin afectar el c) La superficie metálica, por el contrario, se resiste a las
sentido del texto: ondas de luz.
d) El agua refleja, además, una pequeña porción de luz.
a) La campeona mundial de boxeo, Kina Malpartida, vol- e) Esta reflexión en diversas direcciones se denomina di-
vió a verse involucrada en un cuestionable hecho a bor- fusa.
do de su vehículo.
b) Según Ricardo de la Cruz, la boxeadora intentó darse a 7. I. ________________________ II. Argumentar es plan-
la fuga luego del impacto. tear una serie de razones o de pruebas en apoyo de una
c) Malpartida habría intentado escapar por la Carretera idea. III. Su propósito es convencer o persuadir. IV. Debe
Central, pero el tráfico le impidió esta posibilidad. El evitarse la intromisión de lo personal y conviene que pre-
chofer logró alcanzarla y, con ayuda de la Policía y el dominen las proposiciones subordinadas causales y conse-
serenazgo de Ate, pudo reclamarle por el hecho. cutivas.
d) De acuerdo con ciertos programas de televisión, la
boxeadora peruana ha protagonizado diferentes inci- a) Aunque en la teoría de la argumentación se incluye el
dentes similares. debate y la negociación.
b) En el trabajo intelectual resulta fundamental el uso de la
4. I. Altamira fue descubierta de manera casual en 1869. II. argumentación.
Su descubridor fue Marcelino de Santuola. III. Las prime- c) Desde la Antigüedad, la argumentación ha tenido el
ras pinturas encontradas reproducían imágenes de bison- propósito de persuadir.
tes. IV. Inicialmente, hubo escepticismo sobre la fidelidad d) En la actualidad, el estudio de la argumentación ha re-
de las pinturas debido a su increíble estado de conserva- cobrado vigencia debido a la gran influencia de los me-
ción. V. _____________________ dios de comunicación.
e) Se reconocen tres tipos de discursos persuasivos: la de-
a) La cueva de Altamira es una cavidad natural en la roca mostración, la argumentación y la descripción.
en la que se conserva uno de los ciclos pictóricos y ar-
tísticos más importantes de la Prehistoria. 8. I. Como todos los jueves, después de la reunión, el gru-
b) Mercelino de Santuola fue su descubridor. po de amigos salió a comer. II. Fueron a un restaurante
c) De esta forma se puede asegurar que la cueva fue uti- no lejos de donde había sido la reunión. III. La comida se
lizada durante varios periodos, al menos, unos 22 000 veía bien, pero el servicio era un poco lento. IV. Hicieron
años, desde hace unos 35 600 hasta hace 13 000 años su pedido. V. _________ VI. Entonces, incómodos por la
cuando menos. demora, algunos de ellos prefirieron retirarse.
d) Tras años de estudio, las pinturas al fin fueron cataloga-
das como auténticas. a) La comida llegó inmediatamente.
e) Fue declarada Patrimonio de la Humanidad en 1985. b) Mientras esperaban, disfrutaron un aperitivo.
c) La comida consistía en un plato de fondo y una copa de
5. I. Se llama responsabilidad social a la carga, compromiso vino.
u obligación de los miembros de una sociedad. II. Ya sea d) Pasó media hora y la comida no llegó.
como individuos o como miembros de algún grupo, así e) Pocas veces podían compartir en grupo una comida.
como para la sociedad en su conjunto.III. El concepto in-
troduce una valoración positiva o negativa que tiene un im- 9. I. Los mochica tuvieron una especial preocupación por
pacto en la sociedad. IV___________________________ el desarrollo agrícola. II. En este sentido cultivaron maíz,
V. Se diferencia de la responsabilidad política porque no se camote, yuca, papa, calabaza; frutas tales como tuna, lúcu-
limita a la valoración del ejercicio del poder a través de una
ma, chirimoya, tumbo y papaya. III. Como debieron llevar
autoridad estatal.
agua para cultivar tierras secas, construyeron canales que
se muestran como notables obras de ingeniería hidráulica.
a) Hasta hoy en día no se encuentra una definición acep-
IV. ____________________________ .
tada.
b) Ya sea como individuos o como miembros de subgru-
a) Relacionada con una erupción del volcán Krakatoa.
pos.
b) Obras como el canal de Ascote y el de La Cumbre
c) La responsabilidad social es la ética de un gobierno.
c) Construyeron represas.
d) Esta idea proviene de los filósofos griegos y romanos.
d) Quedó registrada una sequía de varios años.
e) Esa valoración puede ser tanto ética como legal.
e) Generó un invierno de dos años en todo el mundo.

5º AÑO DE SECUNDARIA
Mejores Personas, Mejores Familias 67
Colegio

Mejores Personas, Mejores Familias RAZONAMIENTO VERBAL

10. I. En una cena en 1957, Jim Muir, gerente de ventas de a) Por tal medida, el árbitro fue expulsado de la FIFA.
Wolverine World Wide, recibió unas bolitas fritas de maíz. b) Y todos felices y contentos.
II. Estas bolitas eran conocidas en el sur de los Estados c) En el minuto 60 Kuwait realiza un tiro de esquina.
Unidos como hush puppies. III. Cuando preguntó sobre el d) En España se realizó el mundial de fútbol.
nombre, su anfitrión le explicó que los granjeros locales e) Hubo una anulación de gol en el partido del mundial.
usaban esta comida para callar a los perros que ladraban.
IV. Muir decidió que era el nombre perfecto para un nuevo 4. I. Como todos los jueves, después de la reunión, el grupo
tipo de zapato de piel de cerdo que su compañía estaba de amigos salió a comer. II. Fueron a un restaurante cerca
desarrollando. V. __________________ . de donde había sido la reunión. III. La comida se veía bien,
pero el servicio era un poco lento. IV. Hicieron su pedi-
a) Los hush puppies eran unas bolitas de maíz.
do. V. _________ VI. Entonces incómodos por la demora,
b) La comida chatarra abunda en Norteamérica.
algunos de ellos prefirieron retirarse.
c) Jim Muir tenía una empresa muy prestigiosa.
d) Pues el zapato «podía aliviar los pies adoloridos de los
clientes», o sea, sus «perros ladradores». a) La comida llegó inmediatamente.
e) Los obreros locales disfrutaban realizar campeonatos b) Mientras esperaban, disfrutaron un aperitivo.
deportivos. c) La comida consistía en un plato de fondo y una copa de
vino.
d) Pasó media hora y la comida no llegó.
e) Pocas veces podían compartir en grupo una comida.

5 I. Todos los relojes miden el tiempo mediante un mo-


ZZ Completa los espacios en blanco. vimiento regular. II. En el reloj de Sol, se utiliza el mo-
1. I. __________________. II. A los seis, ya escribía música vimiento aparente del astro alrededor de la Tierra. III.
y se iniciaba en el órgano. III. A los siete, comenzaron sus ___________ IV. En el reloj de arena, se mide el flujo re-
clases de violín. IV. Un día llegó al pueblito un grupo de gular de un chorro de esta. V. Los relojes mecánicos tienen
músicos vagabundos. V. Pablo escuchó, por primera vez, un péndulo que oscila de lado a lado.
el instrumento que lo llevaría a la fama mundial: el violon- a) El dispositivo que mide el tiempo en intervalos se llama
chelo. escape.
b) En el reloj de agua, se tiene en cuenta el goteo constante
a) El niño, a los cuatro años, ya sabía patinar. del líquido.
b) El joven a los siete podía cantar ópera. c) Antiguamente, para medir el tiempo se empleaba
c) El niño, a los cuatro años, cantaba en el coro parroquial. relojes de Sol.
d) El niño, a los tres años, componía sinfonías enteras. d) El reloj con mecanismo de rueda apareció en el siglo
e) El niño, a los nueve años, jugaba fútbol. XIV.
e) Los primeros relojes llevaban un escape de corona con
2. I.__________ II. La gente todavía cree en el mito de que un vástago.
los antibióticos son remedios milagrosos e inofensivos. III.
Lo cierto es que tomar antibióticos cuando no se necesi- 6. I. Cuando las ondas se propagan y chocan contra una su-
tan, no solamente carece de utilidad, sino que puede resul- perficie, rebotan. II. El fenómeno es similar cuando se agi-
tar muy perjudicial. IV. Levy agrega algo muy importante: ta una cuerda atada a una pared. III. _____________ IV.
V. Con demasiada frecuencia, los médicos recetan antibió-
En este caso se dice que la onda se reflejó totalmente. V. En
ticos indebidamente.
cambio, si la pared es una tela, una parte se refleja y la otra
a) Stuart Levy, autor del libro La paradoja de los antibióti-
será absorbida.
cos, dice:
b) Stuart Levy, autor del libro La contradicción de las pasti-
llas, dice: a) A esta reflexión se le denomina reflexión parcial.
c) El autor que escribe sobre los antibióticos malignos, b) La pared obliga a la onda a regresar a través de la cuer-
cuenta: da.
d) La verdad acerca de los antibióticos es una sola. c) La superficie metálica, por el contrario, se resiste a las
e) N.A ondas de luz.
d) El agua refleja, además, una pequeña porción de luz.
3. I. En 1982, en el mundial de España, jugaban Kuwait y e) Esta reflexión en diversas direcciones se denomina
Francia. II. En el minuto 80 Francia marcó el 4 a 1 a su difusa.
favor, pero los jugadores de Kuwait reclamaron haber oído
un pito antes del gol. III. Entonces el jeque Fahid Al Ah- 7. I. Se encuentran en la discoteca.
mad, presidente de la confederación de fútbol de Kuwait, II. Luego, se miran fijamente a los ojos.
entró a la cancha con sus guardaespaldas y amenazó con III. _________________________________
retirar al equipo. IV. El árbitro soviético Miroslav Stupar III.Posteriormente acepta la propuesta de baile.
decidió, ¡anular el gol! V. __________________. IV. Le roba un beso

68 Mejores Personas, Mejores Familias


5º AÑO DE SECUNDARIA
Colegio

RAZONAMIENTO VERBAL  Mejores Personas, Mejores Familias

a) Amor a primera vista.


b) Se acerca a invitarla a la pista de baile.
c) Le manda un beso volado.
d) Se molesta y se va.
e) No era para ella, sino para la mesera.
ZZ Completa los textos con la oración adecuada.

8. I. Juan y Roxana son amigos de trabajo. 1. ( I) Los efectos inmediatos al fumar marihuana incluyen
II. Ambos son solteros y les gusta salir los fines de semana, taquicardia, desorientación, falta de coordinación física, a
ya que su trabajo lo permite. menudo seguidos por depresión o somnolencia. (II) Al-
IV. Juan tiene vergüenza de invitar a salir a Roxana porque gunos consumidores sufren ataques de pánico o ansiedad.
es tímido. (III) El humo de la marihuana contiene del 50% al 70% más
V. __________________________________ sustancias que causan cáncer que el humo del tabaco. (IV)
VI.Roxana le menciona a Juan, que si ella no hubiera roto Un estudio de investigación importante informó que un
el hielo; ese miedo seguiría distanciándolos. porro de cannabis puede causar el mismo daño a los pul-
mones que cinco cigarrillos fumados uno detrás del otro.
V) ___________________________________________
a) Roxana conoce a otro chico más interesante y sale con ______________________________________________
él.
b) Su timidez hace que invite a salir a Roxana 2. ( I) Existen cuerpos de bomberos en que sus miembros no
c) Juan le manda una carta inspirada en el trabajo. reciben una remuneración por esta actividad y la realizan
d) Roxana, al percatarse de la timidez de Juan, le propone como un servicio comunitario. (II) En países como Argen-
salir. tina, Ecuador, Perú, Chile y Paraguay, solo existen cuerpos
e) Juan duda de su hombría y desiste. de bomberos voluntarios. (III) Estos funcionan como una
sola institución jurídica dividida en varias compañías y
cuerpos, sin recibir salario y obteniendo recursos a través
9. I. Alexis se encuentra en la esquina de su barrio reunido
de donaciones. (IV) Además, en otros países existen los
A con sus amigos.
bomberos voluntarios junto con los bomberos permanen-
II. Era un día como cualquier otro. tes, que reciben la misma capacitación. (V) ____________
III.Realizaban sus palomilladas cuando de repente. ______________________________________________
IV.___________________________________ ______________________________________________
V. Que por favor le ayuden a bajar un mueble del sexto
piso. 3. ( I) El descanso es el reposo, la quietud o la pausa que se
hace en medio del trabajo o de otra actividad. (II) Por
a) Vino la policía y se los llevó; para luego pedir. ejemplo: «Me tomo un descanso de cinco minutos y sigo
con el informe». (III) «Lamento informarles que hoy ten-
b) Salió la mamá de Alexis y lo mandó a comprar fideos al
dremos que trabajar sin descanso para cumplir con los re-
mercado.
querimientos del nuevo cliente». (III) _______________
c) Se encontraron una bolsa de dinero que un ladrón arro- ______________________________________________
jó. ______________________________________________
d) Una abuelita les pidió ayuda para cruzar la pista y les
pagaría 1000 dólares. 4. ( I) La mayoría de las muertes por hambre se deben a la
e) Un señor de cabellera blanca y tez trigueña se acercó a desnutrición permanente. (II) Las familias sencillamente
pedirles. no tienen suficientes alimentos. (III) O tal vez no cuentan
con recursos para adquirirlos debido a su carestía. (IV)
10. I) Yo asistía a la iglesia de mi comunidad junto con mis Esto, a su vez, se debe a la extrema pobreza. (V) ________
amigos. II) ________________ III) Como yo era el mayor ______________________________________________
de todos oía la misa de rodillas para dar el ejemplo. IV) ______________________________________________
No levantaba los ojos del devocionario, cayendo a veces en
una profunda meditación. V) Que al parecer inspiraba a 5. ( I) El sueño es un estado fisiológico de autorregulación y
mis amigos para hacer lo mismo. reposo uniforme de un organismo. (II) En contraposición
al estado de vigilia que se da cuando el ser está despierto.
(III) El sueño se caracteriza por los bajos niveles de activi-
a) El padre daba un sermón muy aburrido.
dad fisiológica. (IV) Es decir, por presión sanguínea, respi-
b) A mis padres no les gustaba ir a misa. ración y por una respuesta menor ante estímulos externos.
c) La iglesia tenía adornos muy llamativos. (V) ___________________________________________
d) También me gustaba ir de paseo por el parque. ______________________________________________
e) Éramos un grupo de cinco los que íbamos a comulgar ______________________________________________
a esta iglesia.

5º AÑO DE SECUNDARIA
Mejores Personas, Mejores Familias 69
Colegio

Mejores Personas, Mejores Familias RAZONAMIENTO VERBAL

M A
TE

16   ORACIONES INCOMPLETAS

YY El esforzado corredor era piernicorto, pero _____.

a) lento b) veloz c) aburrido


d) flemático e) apresurado

III. PRECISIÓN LÉXICA


Entre las opciones que nos ofrecen, podemos encontrar
términos que son sinónimos entre sí, en cualquier contex-
to. En esos casos, debemos escoger el término más preciso
para la ocasión. Veamos los siguientes ejemplos:

DEFINICIÓN YY Los argumentos de mi abogado fueron ____.


Las oraciones incompletas son ejercicios que se componen de bre-
ves textos en los que se han omitido una o varias palabras. a) deshechos b) refutados c) demolidos
El ejercicio consiste en escoger las palabras que completen la ora- d) destrozados e) anulados
ción de la manera más adecuada.
Existen tres criterios que deben ser considerados para la reso- YY Joselo ofrece una _______ a quien encuentre (y de-
lución de las oraciones incompletas. vuelva) su zapato.

I. GRAMATICALIDAD a) paga b) plata c) subsidio


Toda respuesta de una oración incompleta debe dar como d) billete e) gratificación
resultado una estructura gramatical bien construida desde el
punto de vista sintáctico. Es decir, que tenga concordancia
entre sus términos.

Observa los siguientes ejemplos:


YY El heavy metal y la música selecta son ______. ZZ Completa las oraciones aplicando los criterios de resolución.
a) atractivos b) alegre c) solemne 1. E
ntre el sofisticado equipo con que contaban los brigadis-
d) placenteros e) horrible tas, destacaba un visor de rayos ______ que les permitía
ver aún en medio de la más cerrada _________.
YY La minoría de participantes ______ a la meta.

a) encontrará b) subirán c) llegarán a) equis – multitud


d) encontrarán e) llegará b) catódicos – pantalla
c) infrarrojos – oscuridad
Las gramaticalidades más frecuentes se producen por falta d) láser – defensa
de concordancia.
2. H
ubo un tiempo en que la universidad era una institución
II. COHERENCIA CONTEXTUAL de _____. Solo iban a ella los hijos de los titulados.
En toda oración incompleta encontramos diferentes térmi-
nos que pueden «encajar» en el espacio en blanco; pero de a) verdad b) nivel
acuerdo con la información proporcionada por el sentido c) calidad d) élite
contextual, solo una palabra resultará la más clara y lógica.
Observa los siguientes ejemplos: 3. Si pensamos que errar es _____, para dar cuenta de _____
hechos, podemos concluir que nuestra sociedad es un de-
YY Aquel espantoso fantasma ________, todas las no-
chado de humanidad.
ches, al pobre Juanito.

a) aparece b) sorprende c) asusta a) malo – muchos b) humano – los


d) levita e) enfurece c) bueno – pocos d) mundano – todos los

70 Mejores Personas, Mejores Familias


5º AÑO DE SECUNDARIA
Colegio

RAZONAMIENTO VERBAL  Mejores Personas, Mejores Familias

4. C
uando el objeto nos asusta, la mención de su nombre 10. T
odo en él es_____: sus ademanes ________, su voz_____, etc.
también genera _____, debido a que nombre y objeto sue-
len ser _____. a) precipitado – súbitos – parsimoniosa
b) mesurado – imperceptibles – pastosa
a) temor – identificados c) excesivo – controlados – chillona
b) pánico – vinculados d) hiperbólico – desaforados – estentórea
c) susto – relacionados
d) sensaciones – fundidos

5. T
odos los hospitales de Londres tenían sus entradas de
emergencia a mi disposición._____ eran modernos y
otros, más bien,_____edificios victorianos. Pero me acos- 1. Q
uien critica las _____de los demás sin _____bien de-
tumbré a no hacer _____. muestra con ello palmariamente su ___.

a) Pocos – reducidos – preferencias a) creencias – catalogarlas – descuido


b) Todos – anticuados – selecciones b) iniciativas – practicarlas – pesimismo
c) Varios – desagradables – distinciones c) ideas – conocerlas –ignorancia
d) Algunos – vetustos – distingos d) acciones – actuar – contradicción

2. U
na _____ libertad de prensa puede generar atropellos a
6. E
n el Perú, el problema de la pobreza está imbricado con
los derechos _____ .
un problema de prejuicio racial, que hace más _____ la
_____de los sectores pobres a la sociedad formal.
a) amplia – civiles
a) débil – tendencia b) amarilla – mundanos
c) malentendida – ciudadanos
b) improbable – aceptación
d) desmedida – societarios
c) lenta – ayuda
d) difícil – integración
3. S iempre es conveniente que, de cuando en cuando, nos
demos tiempo para _____sobre nuestras acciones, sobre
7. El que domina _____, domina _____. nuestras metas, sobre el _____último de nuestras _____.

a) su fuerza – su guerra a) meditar – día – existencias


b) su alegría – su paz b) pensar – acto – pasiones
c) sus ímpetus – su cultura c) escribir – fin – preocupaciones
d) su cólera – su peor enemigo d) reflexionar – sentido – vidas

8. A
nte aquel terrible diagnóstico –su mal era incurable– 4. L
os profesionales de la salud tienen la responsabilidad de
Juan quedaba en medio de una _____ crítica: o la desespe- _____ de las enfermedades, pero también de _____.
ración o la _____.
a) curarnos – sanarlas
a) elección – terapia b) restablecernos – prevenirlas
b) resolución – paciencia c) alejarnos – solucionarlas
c) opción – acción d) librarnos – neutralizarlas
d) disyuntiva – resignación
5. L
a mayoría de las _____sabe si a sus _____ les ocurre algo
malo aun antes de que estos se lo digan; la intuición guía a
9. _
________, observaba cómo el cielo, de manera paradójica,
aquellas y les permite interpretar gestos y expresiones no
lucía _____.
verbales.

a) Meditabundo – sereno
a) personas – familiares
b) Abatido – lúgubre
b) esposas – maridos
c) Feliz – encapotado c) mujeres – padres
d) Entristecido – cubierto d) madres – hijos

5º AÑO DE SECUNDARIA
Mejores Personas, Mejores Familias 71
Colegio

Mejores Personas, Mejores Familias RAZONAMIENTO VERBAL

6. A
pesar de que disponía del maravilloso instrumento de
la _____, el invasor europeo elaboró poquísimos informes
sobre las culturas_____ del Perú que todavía vivían ante
sus ojos.
1. E
l surrealismo dio entrada a lo inmotivado del sueño y
a) traducción – monolingües del inconsciente (a lo ilógico), algo así como por ejemplo
b) religión – idolátricas _____y _____ haciendo_____sobre una mesa.
c) imprenta – rudimentarias
a) una cocinera – su hija – tortas
d) escritura – prehispánicas
b) un ladrón – su compinche – negocios
c) un jefe – su secretaria – planes
7. P
uede ser más cómodo no ____nunca que _____ muchas d) un paraguas – una máquina de coser – el amor
veces; pero sirven mejor a la humanidad los hombres que,
por acertar una vez, aceptan los inconvenientes de equivo- 2. L
a _____aplicada a nuestros actos cotidianos es el mejor
carse mil. antídoto contra la inmovilidad que trae consigo la _____ .

a) preocuparse – intentar a) gimnasia – obesidad


b) imaginación – rutina
b) hipotetizar – fallar
c) ciencia – ignorancia
c) esforzarse – tropezar d) virtud – flojera
d) equivocarse – errar
3. T
rabajar _____no quiere decir actuar _____todo el tiem-
8. L
a hipótesis_____postula que los seres vivos_____ unos de po. Ciertas actitudes artificiales y afectadas ocultan con
otros por _____sucesivas debidas a procesos naturales. frecuencia un sentimiento de inseguridad frente a las ca-
pacidades propias.
a) religiosa – se derivan – transformaciones
a) en grupo – bulliciosamente
b) conductista – aprenden – generaciones b) seriamente – solemnemente
c) evolucionista – descienden – modificaciones c) rápido – suspicazmente
d) biológica – se alimentan – hambrunas d) eficientemente – frívolamente

9. E
specialista es quien está dispuesto a _____muchas cosas 4. E
n nuestra época, los acontecimientos se _____de mane-
con tal de saber _____sobre una sola. ra tan rápida e inusitada que dan lugar a planteamientos
_____ inusitados.

a) investigar – poco a) presentan – inusualmente


b) saber – bastante b) dan – contrariamente
c) aprender – hablar c) establecen – inevitablemente
d) ignorar – todo d) suscitan – igualmente

5. N
os regocijamos con las hazañas de nuestros _____, olvi-
10. El desarrollo del lenguaje y del pensamiento se implican
dando que nosotros también podemos ser, para alguien,
mutuamente; en ese sentido, el lenguaje debe ser _____
seres _____ .
con el cual el maestro _____la inteligencia y el pensamien-
to del niño.
a) días – del pasado
b) próceres – bienintencionados
a) el mejor medio – programa c) héroes – extraordinarios
b) el importante objeto – eduque d) compañeros – como ellos
c) el principal instrumento – desarrolle
d) el adecuado mecanismo – edifique

72 Mejores Personas, Mejores Familias


5º AÑO DE SECUNDARIA
Colegio

RAZONAMIENTO VERBAL  Mejores Personas, Mejores Familias

M A
TE

17   PREGUNTAS POR EXTRAPOLACIÓN

Las preguntas de extrapolación son aquellas que tienen como está presente. El único medio de escapar a este círculo sin fin
finalidad calibrar la capacidad del lector para deducir las po- (vida – muerte – reencarnación) es alcanzar el nirvana.
sibles modificaciones que se operarían en el pensamiento del Acceden al nirvana los sabios que, como Buda, llegan al
autor si, hipotéticamente variaran las premisas, condiciones, conocimiento perfecto, tras una larga serie de existencias
circunstancias, etc., del texto. terrestres. Para ello es preciso haberse desprendido de todo
apego al mundo, ser capaz de difundirse en el gran Todo
Formas de plantear este tipo de preguntas universal, de fusionarse con el Cosmos.
El nirvana no es paraíso, un cielo como el de la religión
ZZ S i se negara________, sucedería que _________ cristiana. Es más bien, un estado de reposo absoluto, de eterna
ZZ Si se invirtiera la siguiente idea: ____ se pensaría que ___ calma. Por tanto, quien entra en el nirvana ya no se reencarna,
ZZ Si creamos el supuesto de ___, la consecuencia sería ____ no conocerá más el dolor. Se habrá librado para siempre del
mal, del error y de todos los sufrimientos de la vida terrestre.
Ejemplo:
Preguntas por extrapolación por condición negada
TEXTO I Si el autor tomara como punto de partida lo contrario de lo
Roger Bacon heredó de su maestro, Roberto Grossete, su afi- afirmado, entonces se podría deducir que ____.
ción a las ciencias y su inquietud por la metodología.
En la edad de oro de la filosofía y tecnología escolástica, cuan- a) el nirvana es un estado de sosiego
do prevalece en todas las escuelas el método racional deduc- b) el nirvana permitiría el acceso de los sabios
tivo y se desdeña la utilidad práctica del saber, Roger Bacon
c) los católicos anhelan la desaparición del averno
constituye una excepción notable. Dos ideas resultan especial-
mente revolucionarias a sus contemporáneos. La primera su d) los cristianos aspirarían alcanzar el nirvana
actitud antiliberal y antiaristotélica de que la ciencia debe po- e) los seres humanos se convertirían en entes inertes
nerse al servicio de la vida práctica. La segunda, el hincapié en
la experimentación como punto de partida para la ciencia, en
detrimento de las «autoridades» científicas sin discusión, y de
la deducción como método universal del saber.
Roger Bacon fustiga las lacras presentes en los intelectuales de
su época: la ocultación de la propia ignorancia con la retórica y ZZ Lee atentamente los textos y responde las preguntas
dialéctica verbales, la falta de inquietud científica que se refu- propuestas.
gia en lo dicho por los antiguos, los prejuicios que entorpecen
el avance verdadero del saber. TEXTO I
El síntoma inicial de la apendicitis suele ser el dolor abdomi-
Preguntas por extrapolación por condición supuesta nal, provocado por las contracciones del apéndice o la disten-
Si Bacon tuviera la ocasión de conocer el desarrollo tecnológico sión de su luz. Se trata de un dolor de difícil localización, sordo,
del mundo actual, probablemente afirmaría lo siguiente: difuso y profundo, que en el 50 a 60% de los casos se localiza
a) Tanto la deducción como la experimentación son los prin- en el estómago o alrededor del ombligo, y en el curso de las
cuatro o seis horas siguientes, cuando la inflamación ya se ha
cipales métodos del saber.
extendido hacia el peritoneo, se convierte en un dolor cons-
b) La filosofía es actualmente un actitud superflua para el lo- tante e intenso en el cuadrante inferior derecho del abdomen
gro de algún avance científico. (fosa ilíaca derecha). Este dolor empeora con la tos y los movi-
c) Es deplorable que tanto avance científico haya sido utiliza- mientos (deambulación, respiración, etc.) Y está acompañado
do con fines bélicos y depredadores. de malestar, náuseas, vómitos y anorexia (falta de apetito). La
d) Su tesis acerca de los fines de la ciencia están palmaria- fosa ilíaca derecha es muy sensible y la presión en ella desenca-
mente demostrado en dicho desarrollo. dena un dolor intenso. El abdomen se encuentra tenso y duro
e) Es absurdo que hoy se siga estudiando escrupulosamente a mientras la pierna derecha se mantiene ligeramente flexionada
Aristóteles en las universidades. a la altura de la cadera para aliviar el dolor. Si se intenta exten-
der la pierna, el dolor se agudiza. En el 40 a 50% de los casos
TEXTO II restantes, las manifestaciones clínicas varían en función de la
localización anatómica del apéndice. En los ancianos y los ni-
Pero, ¿qué es nirvana, que siempre se evoca al hablar del
ños menores de dos años, el cuadro clínico suele ser atípico, y
budismo? Según esta religión, todo ser vivo se reencarna, tras es posible que el dolor sea moderado y no se observe rigidez
su muerte, en un nuevo cuerpo para llevar en él una nueva en el abdomen.
existencia más o menos feliz; sin embargo, el dolor siempre La anorexia, o pérdida de apetito, es tan frecuente en la

5º AÑO DE SECUNDARIA
Mejores Personas, Mejores Familias 73
Colegio

Mejores Personas, Mejores Familias RAZONAMIENTO VERBAL

apendicitis que su ausencia obliga a dudar de este diagnóstico. diferencias importantes entre los países.
El 50 a 80% de los pacientes presentan náuseas y vómitos Históricamente, la frecuencia de estas enfermedades disminu-
(ni copiosos ni constantes) cuando ya ha aparecido el dolor yó de manera considerable con la introducción de la penicilina
abdominal. En la mayoría de los casos no se modifica el hábito en 1940, pero hacia los años sesenta la sífilis y la gonococia em-
defecatorio. Algunos pacientes presentan estreñimiento, y pezaron a aumentar. En los años ochenta se produjo un incre-
otros, diarrea. mento de las ETS de origen vírico, principalmente el herpes y
Cuando el apéndice se localiza cerca de la vejiga puede provocar el papiloma, debido a que son enfermedades incurables y muy
disuria (dolor al orinar) y polaquiuria (micciones frecuentes y recurrentes. Como consecuencia del gran impacto psicosocial
poco abundantes). del síndrome de inmunodeficiencia adquirida (SIDA) en la dé-
cada de los ochenta la población en general parece haber to-
1. S i la falta de apetito fuera un elemento presente en todos mado conciencia de la importancia de las medidas preventivas
los casos de apendicitis: y, como consecuencia de ello, ha disminuido el número de las
ETS, exceptuando el SIDA.
a) La anorexia no sería un síntoma destacado de la Actualmente se reconoce el carácter de transmisión sexual de
apendicitis. al menos 25 microorganismos y 50 síndromes clínicos, que
b) Más del 50% de pacientes tendrían náuseas y vómitos. representan un conjunto de manifestaciones clínicas localiza-
c) La apendicitis generalmente produciría disuria. das en un órgano y que pueden estar producidas por diferen-
d) Los ancianos no padecerían realmente de apendicitis. tes microorganismos. La mayoría de las ETS se diagnostican
e) La ausencia de anorexia significaría que no se trataría en la población de 20 a 25 años, asociadas a factores como la
de un caso de apendicitis promiscuidad, la prostitución, la adicción a las drogas por vía
parenteral y el nivel socioeconómico bajo.
2. El mejor título para el texto sería: Las infecciones bacterianas más habituales son las producidas por
Chlamydia trachomatis (clamidiasis), Neisseieria gonorrhoeae o
a) Síntomas y causa de la apendicitis gonococo (gonococía o gonorrea), Treponema pallidum (sífilis o
b) El dolor abdominal y sus distintas ubicaciones lúes) y Haemophilus ducrey (cancroide o chancro blando).
c) Cómo reconocer y combatir la apendicitis
d) Síntomas y signos de la apendicitis 6. El contenido textual podría titularse:
e) Diagnóstico y terapia de la apendicitis

3. Se afirma en el texto que ________. a) Definición y causas principales de las ETS


b) El SIDA y otras ETS
a) el dolor abdominal es de fácil ubicación c) Generalidades acerca de las enfermedades de transmi-
b) al presentarse el dolor abdominal, un número conside- sión sexual
rable de pacientes sufre náuseas y vómitos d) ETS que se reconocen en la actualidad
c) generalmente la apendicitis provoca dolor al orinar e) Síntomas y clases de ETS
d) las causas de la apendicitis aún no se conocen con
exactitud 7. Si la penicilina pudiera curar el SIDA:
e) la tos y los movimientos empeoran la apendicitis y
agudizan las náuseas y vómitos a) Se produciría un gran impacto psicosocial.
4. C
on relación a los síntomas del apendicitis, se desprende b) Aparecería por primera vez en la década de los ochenta.
del texto que _____. c) Sería la principal enfermedad de transmisión sexual.
d) Se generaría la disminución de otras enfermedades de
a) la ubicación del dolor influye en el grado de intensidad transmisión sexual.
que presenta e) No representaría un grave peligro para la salud pública.
b) las náuseas y vómitos ocurren casi siempre
c) el dolor abdominal no se presenta 8. El autor afirma en el texto que ______.
d) la disuria es el dolor al orinar
e) no existen en algunos pacientes a) la promiscuidad y la prostitución son factores principa-
les de las ETS
5. La palabra curso en el texto alude a _____.
b) las relaciones sexuales no son el único medio para
a) tema b) transcurrir c) momento transmitir una enfermedad infecciosa
d) diferencia e) posteriormente c) hoy solo se reconocen 25 mil microorganismos nocivos
d) la infección bacteriana más frecuente entre todas es la
TEXTO II clamidiasis
Se denomina enfermedades de transmisión sexual (ETS) a las e) un factor importante de la aparición de una ETS es el
infecciones producidas por contacto sexual, aunque a menudo factor sociocultural
esta no es la única vía de transmisión. La mayoría de las ETS se
manifiestan en los órganos genitales, pero pueden tener reper-
cusión en cualquier parte del organismo. Su frecuencia de apa- 9. E
l término repercusión, en el contenido textual, se podría
rición y su distribución geográfica son muy variables, puesto reemplazar por el término:
que factores como el nivel sociocultural, las prácticas sexuales
de la población, el grado de control sanitario, la política sani- a) Influencia b) Explosión c) Expresión
taria del país, el consumo asociado de drogas, etc., determinan d) Continuidad e) Efectividad

74 Mejores Personas, Mejores Familias


5º AÑO DE SECUNDARIA
Colegio

RAZONAMIENTO VERBAL  Mejores Personas, Mejores Familias

10. Una idea concordante con lo dicho por el autor es: 3. Se concibe la dislexia como _______.
a) La clamidiasis se contagia fácilmente. a) un problema de velocidad para escribir y leer
b) En 1940, con la penicilina, se detuvo el efecto de las b) un problema de autoestima a causa de una mala lectura
enfermedades infecciosas. c) un trastorno en la esfera de la palabra leída
c) La prevención es importante para combatir la d) el mayor factor de abandono escolar
proliferación de ETS. e) un trastorno que dificulta la lectura, el habla y al escritura
d) Los que consumen drogas por vía parenteral son
personas cuyas edades oscilan entre 20 y 25 años. 4. Puede ocurrir en un niño disléxico que _____.
e) Las parejas promiscuas no pueden evitar contraer
alguna ETS. a) llegue a detestar las matemáticas tanto como la lectura
b) tenga dificultades para realizar operaciones matemáticas
c) sean veloces leyendo pero confundan letras y lean
monótonamente
d) posean una inteligencia superior y por tanto su
capacidad comprensiva sea muy elevada
ZZ Lee atentamente los textos y responde las preguntas e) escriban dificultosamente, pero que en los cálculos ma-
propuestas. temáticos sean sobresalientes

TEXTO I 5. Un efecto destacado por el autor que produce la dislexia es:


La dislexia es una dificultad en la precisión, la velocidad o la
comprensión de la lectura, que se sitúa por debajo del nivel a) Problemas de tartamudez y de escritura
esperado por la edad, el coeficiente de inteligencia y la escola- b) Diferencias en el estudio de las ciencias
ridad de la persona. c) Coeficiente intelectual notablemente inferior
A veces el trastorno se acompaña de problemas en la escritura o en el d) Deserción escolar infantil
cálculo matemático. El trastorno es másfrecuente en los varones, que e) Problemas de visión y por tanto de percepción
representan el 60% u 80% de los casos. La prevalencia se estima en
un 4% en los niños en edad escolar. Asimismo, se ha observado un TEXTO II
patrón familiar, ya que el trastorno es más frecuente en personas Pese a la trascendencia que el pensamiento platónico ha tenido
que tienen familiares biológicos de primer grado (padres, abuelos en la reflexión sobre la literatura, como advierte García: «Platón
y tíos) con este trastorno. no ofrece un volumen sistemático de aportaciones que permitan
Muchos de los niños afectados presentan una inteligencia nor- considerarlo una alternativa doctrinal en poética».
mal o superior a la media y una visión y audición normales, Fue su discípulo Aristóteles el que –mucho menos moralista
pero tienen problemas para percibir letras o palabras en el or- que su maestro– dedicó obras completas a esos dos fenómenos
den adecuado. También confunden letras, leen de un modo discursivos que Platón criticó como distorsionadores de la verdad:
monótono, no prestan atención a la puntuación o deletrean La Poética y la Retórica.
incorrectamente las palabras. Las diferencias entre los pensamientos platónicos y aristotélicos
Las dificultades para progresar en la lectura pueden provocar nos sirven para advertir ya desde esos primeros momentos
en estos niños una enorme desmoralización, con aislamiento en la historia de las teorías literarias que la polémica es un
y baja autoestima, que con frecuencia desembocan en fracaso componente inevitable de la misma. Los conflictos que se
escolar. manifiestan al nivel de las definiciones y de las valoraciones de
La tasa de abandonos escolares en niños afectados de trastornos lo que aparentemente es un mismo «objeto», la poesía, son en
del aprendizaje se sitúa en torno del 40%. Posteriormente realidad diferentes posibilidades de «ser» de la poesía. No se
este déficit puede traducirse en dificultades laborales o de trata de que Platón y Aristóteles estén en desacuerdo sobre un
adaptación social. mismo objeto, sino de que están hablando de objetos diferentes.
Aristóteles ha superado ya ese concepto mítico de la poesía como
1. S i nos encontramos con un niño disléxico este podría fruto de la inspiración, que podía corresponderse con la poesía
presentar ______. homérica, y forma un solo cuerpo con los poetas de su tiempo
proporcionándoles la teoría definidora que las viejas tradiciones
a) dificultad para sumar restar o multiplicar le negaban.
b) insociabilidad y mostrar hostilidad a sus compañeros
c) el no cumplimiento de sus deberes en su centro laboral 6. S i estuviéramos de acuerdo con el autor del texto, entonces
d) su dificultad para mantener la atención en forma sostenida afirmaríamos que las reflexiones de Platón y Aristóteles:
e) la no consideración de los signos de puntuación cuando
se lee un texto a) Tratan desde posiciones encontradas el «objeto» de la
poesía.
2. El mejor título para el texto es: b) Son la piedra angular sobre la que descansa la actual
teoría literaria.
a) Problemas de escritura y lectura en el niño disléxico c) No coinciden pues se acercan a la poesía desde diferen-
b) Carácter de la dislexia infantil y cómo superarlo tes perspectivas.
c) La velocidad y la compresión de la lectura d) Presentan diferentes enfoques teóricos sobre la poesía.
d) Naturaleza de la dislexia y su presencia en la infancia e) Hablan acerca de posiciones coincidentes desde el
e) La disminución de la inteligencia de un niño disléxico punto de vista teorico.

5º AÑO DE SECUNDARIA
Mejores Personas, Mejores Familias 75
Colegio

Mejores Personas, Mejores Familias RAZONAMIENTO VERBAL

7. ¿Cuál es la idea que mejor resume el texto leído? de su criterio el historiador que para sondear el fondo de un
personaje acudiera únicamente a las informaciones de las hojas
a) El desacuerdo entre las teorías de Platón y Aristóteles cotidianas. El diario puede revelar la psicología de un pueblo,
frente al hacer poético mas rarísima vez servirá de testimonio fidedigno para juzgar
b) Los estudios de poética entre los filósofos griegos de la a los hombres públicos. El diarista posee su verdad que no es
Antigüedad siempre la verdadera.
c) La teoría literaria aristotélica frente a las lucubraciones
platónicas 1. En la primera parte, el autor:
d) La Poética y la Retórica como objeto de reflexión de los
filósofos griegos a) Menciona el merecimiento que poseen diferentes
e) El discípulo de Platón erró en su reflexión sobre la diarios en el mundo.
poética b) Nombra cuatro gacetas y su carácter internacional.
8. En el texto, el vocablo volumen significa: c) Censura severamente a diferentes diarios distorsionan-
do su veracidad.
a) Surtido b) Compendio c) Libro d) Pone en evidencia el mercantilismo y la falta de
d) Conjunto e) Espacio principios de varios diarios.
e) Cuestiona de forma indulgente la actitud negativa de
9. Según el autor del texto, para los estudios literarios: los diarios.

a) La reflexión aristotélica es más importante que la 2. E


n el texto, el término consignan puede ser reemplazado
platónica. por _____.
b) Las discusiones entre Platón y Aristóteles son
improductivas. a) confirman b) sustraen c) obtienen
c) El «objeto» de la poesía varía en función de la cultura d) incluyen e) invierten
del poeta.
d) Tendrían su manifestación en la escritura realizada por 3. ¿A qué se refiere el autor con «las informaciones de las
Homero. hojas cotidianas»?
e) Nos dan las pistas suficientes para afirmar que estamos
ante un problema. a) Al contenido informativo de los diarios, periódicos o
gacetas
10. Del texto se deduce que las reflexiones acerca de la poesía b) A la muestra pobre del criterio del historiador que
de Platón: sondea el fondo
c) A la fuente inadecuada de cualquier estudioso que
a) No se materializaron en libros orgánicos. busca conocimientos
b) Impiden considerarlo una alternativa teórica. d) Al carácter falaz de la información de los documentos
c) Influenciaron en la escritura homérica. de estudio
d) Fueron rechazadas airadamente por los poetas e) A los principios inadvertidos de periodistas falaces y
e) Impedían que la poesía floreciera en Grecia. mercantilistas

4. S i estuviéramos de acuerdo con el autor, también


mostraríamos:

a) Disconformidad con los medios de información


ZZ Lee atentamente los textos y responde las preguntas b) Descontento por la actitud de las editoriales
propuestas. c) Su acuerdo con la minoría de diarios o periódicos
TEXTO I d) Desacuerdo con el contenido de algunos periódicos
Desde Le Figaro de París hasta The Times de Londres y desde e) Rechazo a la pobreza intelectual de los historiadores
The New Herald hasta la Gaceta de Colonia, algunos merecen
llamarse tenduchos de compra y venta, covachas de embuste 5. El título más apropiado es:
por mayor y menor. En las grandes potencias, así como en a) Cuestionamiento indirecto a cuatro diarios, periódicos
los pequeños estados, los presupuestos consignan sumas o gacetas en el mundo
destinadas a los periodistas oficiales y oficiosos, lo que se llama b) Los diarios Le Figare, The Times, The New Herald y la
el fondo de los reptiles. Cavour y Bismarck no vacilaron en Gaceta de Colombia
confesar lo mucho gastado por Italia y Alemania con el fin de c) Denuncia del carácter mercantilista y falaz de algunos
ganarse las simpatías o el silencio de las prensa internacional. diarios
Si en cuarenta o cincuenta diarios leemos hoy la narración de d) Crítica despectiva a cuatro revistas a nivel de todo el
algún hecho acaecido ayer, difícilmente sabremos en limpio mundo
la verdad cuando el hecho se relaciona con los intereses de e) Condición mercantilista y comercial de los norteamericanos
la banca o la política del gobierno. Muy pobre muestra daría

76 Mejores Personas, Mejores Familias


5º AÑO DE SECUNDARIA
Colegio

RAZONAMIENTO VERBAL  Mejores Personas, Mejores Familias

M A
TE

18   DIÁLOGOS – PUNTO DE DISCREPANCIA

una persona dice que para alcanzar la justicia es necesario


tener un gobierno dictatorial y otra persona afirma que lo
ideal para alcanzar la justicia es tener un gobierno permi-
sivo, entendemos que hay un desacuerdo evidente, pero
también descubrimos que hay un punto en común (alcan-
zar la justicia).

Recuerda
Eventualmente, se puede pedir el punto de discrepancia
DIÁLOGO subyacente o el presupuesto de la discusión. Este es un
Todos hemos creado textos argumentativos alguna vez, pues siempre elemento necesariamente implícito que permite que
debemos fundamentar las diversas opiniones que tenemos. Sin em- ocurra la discusión. Vale decir, que de no existir esa
bargo, no siempre nuestros puntos de vista van a coincidir con los de diferencia de opinión, la discusión no podría ser posible.
otros y en muchos casos debatimos para defender nuestras ideas. Un
debate, lejos de enervar o destruir ideológicamente a nuestro interlo-
cutor, puede enriquecer nuestras ideas y hasta mejorarlas. Si analiza-
mos un debate con los puntos que ya han sido tocados en el curso, en-
contraremos que en toda discusión hay solo un tema, y por lo general,
dos tesis enfrentadas. Las preguntas de debates o diálogos son cuatro. ZZ Lee atentamente los textos y responde las preguntas
propuestas.
ZZ R
eforzar un texto: Aquí utilizaremos todos los criterios
vistos anteriormente, pero no debemos caer en el facilis- TEXTO I
mo de pensar que reforzar una postura es igual a debilitar ZZ PEDRO: Uno de los más graves problemas que vivimos a
a otra. Se recomienda que se trabaje el texto en cuestión de diario es el caos vehicular. No cabe duda de que el desarrollo
manera separada al texto antagonista. trae problemas muy complejos. ¿Quién iba a decir, hace quin-
ZZ Debilitar un texto: De la misma manera, usaremos los mis- ce años, que una de las razones principales del caos sería la
mos criterios y recordaremos que no siempre podemos de- abundancia de autos nuevos? Yo considero que este es el ma-
sarrollar puntos por simple oposición al texto antagonista. yor problema. Hay horas en que se forman grandes colas y ahí
Además, se recomienda trabajar de forma separada al tex- puedes notar que hay más autos que micros o «combis». Pero
to que nos piden minar. el problema se agrava cuando la gente se acostumbra tanto al
auto que ya ni caminar quiere. Es increíble que hasta para ir a
ZZ P
unto de discrepancia central: Este es el primer tema una tienda prefieren usar el auto que ir a pie. Eso incluso trae
nuevo que proponemos. El punto de discrepancia cen- como daño colateral el sedentarismo y los problemas de salud
tral es el centro neurálgico de la discusión. Podemos derivados del poco ejercicio.
hallarlo al impactar o fusionar ambas tesis. No debemos ZZ GIANNY: El caos vehicular es un problema más complejo
olvidar que el tema central de la discusión debe estar y no se reduce solo al incremento de autos nuevos. Es un
presente, pues el punto de discrepancia central es bas- problema estructural que viene desde hace mucho y está
tante específico. sentado en la escasez de vías rápidas, en la mala semafo-
rización y en la tugurización de algunas zonas que debe-
rían estar libres de tráfico. Considero que sí es uno de los
ZZ P
unto de concordancia central: En este caso buscaremos problemas más graves que los ciudadanos vivimos en la
el punto de acuerdo entre los interlocutores. Es poco co- actualidad, pero sospecho que la razón no es el incremento
mún que este acuerdo se encuentre en la tesis, pues de ser y uso excesivo de autos nuevos, sino la falta de un sistema
así, no habría discusión. Cierto es que, a pesar del debate, integrado de transporte. Es necesario que alguien se haga
dos personas pueden concordar en puntos secundarios. Si cargo de eso inmediatamente.

5º AÑO DE SECUNDARIA
Mejores Personas, Mejores Familias 77
Colegio

Mejores Personas, Mejores Familias RAZONAMIENTO VERBAL

1. ¿ Cuál de las siguientes alternativas, de ser cierta, refuerza 4. ¿Cuál es el punto de discrepancia central?
mejor la argumentación de Pedro?
a) La modernidad como factor de eliminación de las casas
antiguas.
a) China ha sido el principal exportador de autos nuevos
b) Lo acertado de convertir la eliminación de casas antiguas
en los dos últimos años.
como norma urbana limeña.
b) En el último reporte anual sobre venta de vehículos, se
c) Las casas antiguas y palacetes en su función embellece-
puede notar que el incremento de mototaxis ha sido
dora de la ciudad de Lima.
13% mayor que al año pasado.
d) La importancia de tener centros como Larcomar o
c) A nivel mundial, el desarrollo de la producción de autos
edificios como en San Isidro.
ha sido significativa, sobre todo, en Latinoamérica.
d) La venta y circulación de autos nuevos se ha incremen- 5. ¿ Cuál es el punto de discrepancia implícito entre ambos
tado en un promedio de 43% cada año desde el 2001. dialogantes?

2. Pedro y Gianny coinciden en que ______. a) El obstáculo que constituyen para la modernidad de la
ciudad los palacetes y las casas antiguas.
b) La consideración hacia los vecinos de Lima que tienen
a) hay un incremento desmedido de vehículos casas antiguas como las de Barranco.
b) para todos es insoportable el caos que viene soportando c) La valoración de Barranco y La Punta como distritos de
Lima la Lima de antaño y su valor como patrimonio cultural
c) el caos vehicular es uno de los más graves problemas de la ciudad.
que se viven d) Los centros comerciales como representativos de la cul-
d) nadie, a la fecha, hace nada para solucionar el problema tura moderna de una ciudad.

3. Ambos dialogantes discrepan en _______. 6. ¿Qué alternativa de ser cierta debilitaría más a Moderno?
a) Los capitalistas extranjeros han afirmado que solo
a) las razones que exponen para defender sus posturas. invertirán en el país en aquellos negocios que vayan de
b) quién debe solucionar el problema. la mano de la actual globalización.
c) si es o no el crecimiento económico una razón válida. b) Los vecinos de Miraflores organizaron una marcha
d) la postura que cada uno defiende. cuando el parque Salazar iba a ser destruido.
c) Existen distritos que son patrimonio cultural y que, en
consecuencia, no pueden ser destruidos.
TEXTO II
d) Las recientes demoliciones de casas antiguas y palacetes
ZZ MODERNO: La ciudad de Lima está llena de casas an-
no ha afectado el embellecimiento de Lima.
tiguas que le quitan modernidad. En todas las ciudades
del mundo se pueden ver edificios de más de 40 pisos y TEXTO III
residencias inmensas, y en todo caso, si hay casas, estas ZZ MARÍA LUCÍA: Cuando una mujer es violada, sufre un
son modernas. Todas las casas antiguas que se encuentran trauma de por vida. Qué derechos, es mi pregunta, puede
entre edificios y casa modernas deben ser derribadas: los conservar aquel que ha vejado a una mujer inocente: nin-
distritos sin excepción deben ser modernizados. Cuando guno. Por lo tanto, yo propongo la pena de muerte para los
la Municipalidad Metropolitana de Lima se encargue de violadores, sea por el abuso de una menor o una mayor de
esto, tendremos una ciudad moderna y atractiva con más edad; no importa quien sea, las mujeres deben ser protegi-
centros como Larcomar o la zona comercial de San Isidro, das de estos monstruos. La pena de muerte debe ser lenta, ya
y menos casas que ya no concuerden arquitectónicamente sea mediante la silla eléctrica con descargas periódicas o se le
con la época que vivimos. quema vivo, pero jamás cadena perpetua.
ZZ TRADICIONALISTA: Hace poco he visto como han de- ZZ MARÍA FERNANDA: Lo que propones es animal. El vio-
rribado un palacete de principios de siglo que estaba junto lador, si bien ha cometido una bestialidad, no tiene por qué
a la Alianza Francesa de la avenida Arequipa. Supongo que sufrir una condena inhumana. La pena de muerte es un cas-
en su lugar van a colocar un edificio o más de uno, pues tigo que no solo incluye el dolor de la muerte lenta, también
el terreno es grande. Esto no me parece enteramente con- es la angustia de los días de espera para la condena, todo
denable, pues creo que, aunque no comparto la decisión es realmente infrahumano. Lo mejor que se puede hacer es
de los dueños, estos pueden hacer lo que quieran con su condenarlo a cadena perpetua para que, con los años que le
propiedad. Sin embargo, abogar como lo haces para que quedan de vida, trate de arrepentirse y expíe su culpa.
se destruyan todas las casa antiguas me parece despropor-
cionado. Estas embellecen la ciudad y recuerdan toda una 7. ¿Cuál es el punto de discrepancia central?
época; si una casa antigua o un palacete se encuentra en-
a) La piedad ante el sufrimiento de saberse condenado a
tre edificios modernos, pues que se quede ahí, ¿acaso el
pena de muerte.
mismo contraste no te parece representativo? Imagínate si
b) La mejor pena para un violador.
Barranco y La Punta fueran arrasados, y en su lugar hubie-
c) Lo insuficiente de la pena privativa de la libertad para
ra solo edificios; la nostalgia sería tan grande como la que
un violador.
nos produce el Centro Histórico de Lima o el desaparecido
d) El trauma de una mujer como justificación para la pena
parque Salazar.
de muerte a los violadores.

78 Mejores Personas, Mejores Familias


5º AÑO DE SECUNDARIA
Colegio

RAZONAMIENTO VERBAL  Mejores Personas, Mejores Familias

8. ¿Cuál es un punto de discrepancia implícito? 1. ¿ Cuál de las siguientes opciones expresa la postura de
MODIFICADOR?
a) El respeto a la vida de un delincuente.
b) La igualdad de los seres humanos ante Dios. a) Las técnicas de mejoramiento genético permitirán una
c) La deshumanización que significa la pena de cárcel. mayor productividad de las semillas.
d) La expiación como posibilidad de reformación. b) El que las personas se opongan al desarrollo de semillas
transgénicas, es un separación.
9. ¿ Qué argumento, de ser cierto, reforzaría la opinión de c) La elaboración de semillas transgénicas podría acabar
María Lucía? con el hambre mundial.
d) El modificar la naturaleza para nuestros intereses
a) Según el Departamento Federal de los EE.UU., la pena evolutivos como seres humanos, es una práctica reali-
de muerte ha disminuido los crímenes, incluyendo las zada desde siempre.
violaciones.
b) Las feministas han organizado una marcha mundial en 2. ¿ A qué apela fundamentalmente NATURALISTA en su
pro de la pena de muerte para violadores. argumento?
c) En las tribus de África, las violaciones se resuelven con
el asesinato del violador. a) A las alteraciones que generan en la naturaleza.
d) La vida de todo ser humano está por encima de los sen- b) A las proteínas que tienen estos nuevos alimentos.
timientos de venganza. c) A los probables riesgos a los que se expondrían la salud
10. ¿Qué argumento de ser cierto reforzaría más la opinión de de los seres humanos.
María Fernanda? d) A que las nuevas técnicas incorporarán características
existentes en las plantas.
a) El gobierno de EE.UU. respeta la opinión de los Estados
que aplican la pena de muerte. 3. ¿ Cuál de las alternativas, de ser cierta, podría debilitar el
b) Los Derechos Humanos y la ONU imposibilitan la punto de vista de NATURALISTA?
práctica universal de la pena de muerte.
c) Ningún país en el mundo practica la pena de muerte a) Los alimentos que con mayor frecuencia causan reac-
para ladrones. ciones alérgicas son los cereales que contienen gluten, la
d) La pena de muerte para violadores depende de la edad soya, la leche, los huevos, los crustáceos, el maní y otros
de la mujer afectada. frutos secos.
b) Las acciones humanas deben ser contempladas no solo por
las consecuencias que puedan tener sobre los individuos
hoy existentes, sino que deben considerarse también desde
la perspectiva de los derechos de las generaciones futuras.
c) Las mismas personas que argumentan que modificar
ZZ Lee atentamente los textos y responde las preguntas genéticamente los cultivos alimentarios equivale a «ac-
propuestas. tuar como si uno fuera Dios», no suelen oponerse al
empleo de ingeniería genética para producir insulina, la
TEXTO I hormona de crecimiento humano y muchos otros fár-
ZZ MODIFICADOR: Es absurdo que las personas se opon- macos de uso común.
gan al desarrollo de semillas transgénicas ya que las técni- d) Las alergias alimentarias son causadas, para un grupo
cas que se usarán incorporarán características inexisten- de personas, por una gran variedad de alimentos que
tes en las plantas, las que pueden permitir aumentar la consume sin problemas el resto de la población. Esto no
productividad y el valor nutritivo de los productos, esto es algo nuevo de los cultivos transgénicos.
permitiría ayudar a la reducción del hambre en el mundo.
Además, las técnicas modernas de mejoramiento genético 4. ¿ Qué punto de concordancia podríamos encontrar entre
solo son un método alternativo al mejoramiento conven- ambos interlocutores?
cional para obtener mejores alimentos, puesto que nunca
hemos dejado de modificar la naturaleza para nuestros in- a) Las semillas transgénicas causan polémica.
tereses evolutivos como seres humanos. b) El hambre en el mundo es un problema existente.
ZZ NATURALISTA: Quienes insisten en la idea y práctica c) La salud humana es preocupante.
de desarrollar semillas transgénicas enarbolan como ban- d) Las personas hoy en día están mal alimentadas.
dera la reducción del hambre en el mundo, pero no toman
en cuenta los posibles riesgos, impredecibles, para la sa- TEXTO II
lud humana, que en un largo plazo podrían producir los ZZ ANDRÉS: En una gran ciudad, cuando el automóvil reco-
transgénicos, debido a las alteraciones que se realizan en la rre una distancia de 20 km permanece parado por culpa
naturaleza. Uno de los riesgos para la salud asociado a los de atascos durante una media de 15 minutos, en los que,
transgénicos es la aparición de alergias, ya que estos ali- sin embargo, continúa consumiendo combustible y conta-
mentos introducen en la cadena alimentaria nuevas pro- minando el aire. Con la bicicleta, no se pierde ese tiempo y
teínas que nunca antes habíamos consumido. esta no consume combustible ni contamina. Este vehículo

5º AÑO DE SECUNDARIA
Mejores Personas, Mejores Familias 79
Colegio

Mejores Personas, Mejores Familias RAZONAMIENTO VERBAL

es el medio de transporte más rápido y más económico ZZ JAVIER: De ninguna manera esta resolución beneficia a
para una gran ciudad; deberíamos utilizar cada vez me- Galán García, pues él no tiene condición de investigado,
nos el automóvil, el cual solo contribuye al detrimento de ni testigo en este proceso. Lo que sucedió en el Frontón
nuestra salud. fue una reacción del Estado frente a un acto de violencia,
ZZ RAMIRO: No estoy de acuerdo contigo. La bicicleta es un recordemos que reos terroristas se amotinaron y tomaron
vehículo diseñado para pasear y no para ir, por ejemplo, al como rehenes a miembros de la Guardia Republicana.
trabajo, al que uno debe llegar presentable. Es verdad que,
a veces, el tráfico se pone pesado y nos dificulta llegar al 8. ¿ Cuál es el punto de discrepancia central entre GLORIA y
destino a tiempo, pero son los costos que hay que pagar JAVIER?
para aprovechar la principal ventaja de los automóviles: la
comodidad de viajar sin hacer esfuerzos físicos agotadores a) El Tribunal Constitucional tiene intereses particulares
y con rapidez. Además, existen ya los automóviles híbri- en el caso del Frontón
dos, los cuales no contaminan, porque se pueden usar con b) Si Galán García se beneficia o no con el fallo del TC.
un sistema eléctrico o con un sistema de gas. c) Si la matanza del Frontón se debe considerar un crimen
de lesa humanidad.
d) Lo ocurrido en el Frontón tiene carácter imprescriptible.
5. ¿Cuál es la tesis de RAMIRO?
9. ¿Cuál es la tesis de GLORIA?
a) El automóvil es un medio de transporte preferible a la
bicicleta. a) Es extraño que con escasos votos se prohíba la investi-
b) La bicicleta es un vehículo diseñado para pasear. gación de la matanza del Frontón.
c) Debemos llegar presentables al trabajo; con la bicicleta b) Dicho caso es una grave violación de los DD.HH.
sería inevitable no hacerlo. c) Está claro que el fallo está dado para beneficiar a Galán
d) La comodidad sin hacer esfuerzo es lo mejor del uso del García.
automóvil. d) Que el TC haya dado un fallo que no considera la ma-
tanza como un delito.
6. ¿ Cuál es el punto de discrepancia central entre ANDRÉS y
10. ¿Cuál de las siguientes alternativas expresa mejor la tesis
RAMIRO?
de JAVIER?

a) Qué tan económico es usar el automóvil con respecto a a) Lo sucedido en el Frontón no fue un acto de violencia.
la bicicleta. b) Los terroristas se amotinaron y tomaron como rehenes
b) Si la pérdida de tiempo debido al tráfico justifica el uso a efectivos policiales.
del automóvil. c) Galán García no tiene condición de inculpado en dicho
c) Qué medio de transporte es preferible, el automóvil o la caso.
bicicleta. d) La resolución del TC no es beneficiosa para Galán García.
d) Si la bicicleta ha sido diseñada para pasear o acudir al
trabajo.

7. ¿Cuál es el argumento central de RAMIRO?


ZZ Lee atentamente los textos y responde las preguntas propuestas.
a) El automóvil evita esfuerzos físicos innecesarios y es
más rápido.
TEXTO I
b) La bicicleta está hecha para pasear y no para ir a trabajar. ZZ CHARLIE: El proyecto de ley sobre unión civil entre per-
c) Viajar en automóvil es más cómodo que ir en bicicleta. sonas del mismo sexo soluciona un problema de «discri-
d) Existen automóviles que no contaminan el ambiente. minación» que vive este grupo de ciudadanos. Ocurre que
personas gays y lesbianas que quieren tener un proyecto
TEXTO III de vida junto con su pareja, no tienen la protección del
ZZ GLORIA: Es extraño que con solo dos votos de los magis- Estado que sí tienen los heterosexuales que se casan y tie-
trados del Tribunal Constitucional, se prohíba, básicamen- nen la institución del matrimonio que los protegen para
te, una futura investigación contra los posibles responsa- herencias, seguro social, pensión. Sin tocar el tema de la
bles, material o intelectualmente de la matanza ocurrida institución del matrimonio, que no lo abordamos, estamos
en el Frontón. Este hecho fue una grave violación de dere- creando una figura con nombre distinto, con un registro
chos humanos, aunque el TC haya dado un fallo contrario, distinto, que se llama unión civil. Esta figura es la de per-
y es por ello imprescriptible. Es evidente que esta resolu- sonas del mismo sexo que quieren buscar un hogar, una
ción tiene nombre propio, creo que este fallo es solo para familia, derecho que no se les puede negar.
lograr la impunidad de quien sería el autor intelectual de ZZ SANTINO: Si se da luz verde a este proyecto, se estarían
este execrable hecho, obviamente me refiero a Galán García. distorsionando las bases sobre las que se construye la fa-

80 Mejores Personas, Mejores Familias


5º AÑO DE SECUNDARIA
Colegio

RAZONAMIENTO VERBAL  Mejores Personas, Mejores Familias

milia, Dios creó al varón y a la mujer y les mandó crecer y 3. ¿Cuál de las opciones podría debilitar lo dicho por CHARLIE?
multiplicarse. La ley debe contribuir siempre a su consoli-
dación y fortalecimiento sobre la base de los principios y a) El matrimonio es una institución esencialmente hetero-
normas constitucionales que obligan a la comunidad y al sexual y esto implica desnaturalizar el concepto de ma-
Estado a protegerla como institución natural y fundamen- trimonio, lo que implica pervertir la «naturaleza» del
tal de la sociedad. La unión civil para personas del mismo mismo.
sexo, bajo el falso argumento de no discriminar, altera el b) Dios creó al varón y a la mujer y les mandó crecer y
orden natural. multiplicarse, obviamente los homosexuales no pueden
multiplicarse.
1. ¿Cuál es la tesis de CHARLIE? c) Para evitar el desamparo legal a parejas homosexuales,
lo cual se considera discriminatorio, no hace falta apro-
a) Hay personas del mismo sexo que desean formar una bar la unión civil, ya que la mayoría de los beneficios de
familia. una relación puede regularse a través de acuerdos lega-
b) La unión civil contempla el derecho de personas del les (por ejemplo, en relación a herencias, transmisión
mismo sexo a formar un hogar y tener una familia. de bienes, propiedades compartidas, etc.).
c) El Estado protege a los heterosexuales, pero no a los ho- d) Todo niño tiene derecho a un padre y una madre para su
mosexuales. desarrollo integral como persona. Así, conceder la adop-
d) La figura de unión civil es un proyecto antidiscriminación. ción a homosexuales sería perjudicial para el menor.

4. ¿Cuál es el punto de discrepancia entre CHARLIE y SANTINO?


2. ¿ Cuál es el argumento central de CHARLIE?
a) Los gays tienen derecho a formar una familia y ser pro-
a) La pertinencia de aprobar ley a favor de la unión civil
tegidos por el Estado.
para personas del mismo sexo
b) No se está abordando el tema de la figura institucional b) El amor entre homosexuales
del matrimonio. c) La tolerancia de la sociedad con respecto a opciones
c) El proyecto de ley sobre unión civil entre homosexuales distintas de las de la mayoría
soluciona un problema de «discriminación» que vive d) La falta de conocimiento de la sociedad respecto a la
este grupo de ciudadanos unión civil entre personas del mismo sexo
d) La unión civil contempla el derecho de personas del
mismo sexo a formar un hogar y tener una familia.

5º AÑO DE SECUNDARIA
Mejores Personas, Mejores Familias 81
Colegio

Mejores Personas, Mejores Familias RAZONAMIENTO VERBAL

M A
TE

19   ANÁLISIS DE IMÁGENES

Las imágenes, como los escritos, tienen un autor que intenta


comunicar impresiones, ideas, sentimientos, informaciones. Pautas para la interpretación
El autor concibe, produce o ejecuta la imagen, ayudándose de 1. Descripción de las imágenes y personajes: Se trata
materiales, técnicas y útiles determinados: suministrar una in- de describir la primera impresión que genera la
formación, incitar a la compra, convencer, denunciar, agradar, imagen.
la cual culmina al ser exhibida públicamente en un libro, una 2. Explicaciones de los elementos que aparecen en
revista, una valla publicitaria, por televisión, etc. cada imagen: Requiere precisar a cada uno de los
personajes y establecer la situación o realidad en
Cuando nosotros observamos esa imagen, estamos participan- que se encuentran.
do, como receptores, en el proceso de comunicación. 3. Identificación de los elementos que podrían pro-
Tanto las imágenes como los textos transmiten un mensaje, y en vocar comicidad, ridículo o humor
ellas, entonces, podemos reconocer ideas principales o secunda- 4. Localización del autor en cuanto a su postura: Es
rias, del mismo modo que inferir supuestos, intenciones o con- necesario reconocer la posición del autor frente a
clusiones. Leer, en estos casos, corresponde a observar las imáge- un tema.
5. Reconocimiento del mensaje, lo cual implica el
nes, identificar los elementos que la configuran y explicar cómo
reconocimiento del tema, título e idea principal.
estos se relacionan para confirmar el significado total.

ZZ Observa cada imagen, analízalas y responde las preguntas propuestas.

IMAGEN 1

Mafalda y Susanita.
1. ¿Cuál es el tema central de la caricatura?

a) Qué hacer con la pobreza b) Los múltiples sentimientos que provocan los pobres.
c) Las grandes dificultades para solucionar la pobreza d) La indignación que produce la pobreza.

2. E
s correcto a partir de la viñeta:
I. Susanita y Mafalda comparten los mismos sentimientos hacia los pobres.
II. Los pobres siempre están sentados en las calles.
III. Mafalda tiene una posición socialista.

a) Todas b) Solo III c) II y III d) Ninguna

82 Mejores Personas, Mejores Familias


5º AÑO DE SECUNDARIA
Colegio

RAZONAMIENTO VERBAL  Mejores Personas, Mejores Familias

3. S e puede afirmar a partir de la imagen:


I. Algunos quisieran ayudar a los pobres para que dejen su estado de indigencia.
II. Algunos sienten que los pobres retrasan el desarrollo de la nación.
III. Habría quienes quisieran evadir la existencia de los pobres.
IV. Algunos quisieran que los pobres desaparezcan.

a) I, III y IV b) Solo III c) III y IV d) II y IV

IMAGEN 2

4. O
rdena cronológicamente los siguientes eventos de acuerdo con lo mostrado en el texto anterior:
I. La Apolo se acopla al modulo lunar.
II. Correcciones de mitad de camino de regreso a la Tierra.
III. El módulo lunar dispara sus cohetes de descenso.
IV. El módulo lunar se desacopla.
V. Amarizaje del módulo de mando.

a) III –II–I–IV–V b) I–IV– III–II–V c) IV–I – III–II–V d) II–III–IV–I–V

5. Señala la idea verdadera según la infografía:

a) El módulo lunar aluniza acoplado al módulo de mando.


b) El Saturno puede llegar por sí mismo desde la Tierra a la Luna en una sola etapa.
c) La misión realiza correcciones a medio camino dos veces en el viaje.
d) El módulo de servicio amariza unido al módulo de mando.

5º AÑO DE SECUNDARIA
Mejores Personas, Mejores Familias 83
Colegio

Mejores Personas, Mejores Familias RAZONAMIENTO VERBAL

IMAGEN 2

ZZ Observa cada imagen, analízala y responde las preguntas


propuestas.

IMAGEN 1

5. Es correcto según el texto:

a) Para la realidad del texto la procreación no implica un


acto sexual.
b) No es fácil llevar a un recién nacido por los cielos du-
rante mucho tiempo.
c) El anciano está desnudo.
d) El pájaro ha estado vagando por años por el mismo lugar.

6. Señala el dicho que hubiera tenido que aplicar la cigüeña:

a) No por mucho madrugar se amanece más temprano.


b) Querer es poder
c) De tal palo tal astilla
d) Preguntando se llega a Roma.
1. Se puede afirmar a partir de la imagen anterior:
IMAGEN 3
a) Quien atiende a la puerta recibe una visita inesperada.
b) La respuesta busca eludir a la muerte.
c) La muerte es bienvenida.
d) La muerte es inevitable.

2. ¿Cuál es el sentido humorístico de la viñeta?

a) Quien trabaja como empleada del hogar es un travesti


que desconcierta a la muerte.
b) Que la muerte tenga que tocar la puerta y preguntar por
la persona que está buscando.
c) Que para eludir a la muerte se disfrace de empleada
doméstica.
d) Que sea un empleada del hogar, de cuestionable belleza,
quien atiende a la muerte.

3. S i la muerte fuera omnisciente, entonces: 7. Se puede afirmar a partir de la imagen:


I. Esperará el momento exacto en que vuelva la persona
que busca. I. Colombia es un país empeñado en sobresalir en el ám-
II. No se dejará engañar por el intento evasivo. bito económico.
III. No habría ido en ese momento a buscar a la persona II. La noticia propagada en algunos medios de comunica-
que quiere encontrar. ción podría ser discordante con la realidad.
III. Los periódicos pueden ser usados más de una vez.
a) Solo I b) I y II IV. Un periódico podría cobijar contra el frío durante la
c) Solo II d) II y III noche.
V. Mientras soñamos, la realidad puede ser otra
4. L
a imagen anterior se podría titular:
a) Una visita inesperada a. I, II y II
b) Engañando a la muerte b. II y IV
c) La muerte prematura c. I, III y V
d) Dialogando con la muerte d. II y III

84 Mejores Personas, Mejores Familias


5º AÑO DE SECUNDARIA
Colegio

RAZONAMIENTO VERBAL  Mejores Personas, Mejores Familias

8. La imagen se podría titutal: 2. Un buen título para la caricatura anterior sería:

a. La indigencia es un problema prioritario en Colombia. a) El conejito acróbata


b. El desarrollo económico americano no llega a todos en b) Elaborado suicidio del conejito
la misma proporción. c) Un conejito que ya no quiere vivir
c. Paradoja en Colombia en cuanto a la desigualdad del d) Adivinanza del conejo
crecimiento económico. 3. De la caricatura se infiere con certeza:
d. Colombia rumbo a ser la primera economía en latinoa-
mérica. a) El conejito ha llevado a cabo unas operaciones muy
complejas.
9. El autor de la imagen pretende: b) El conejito es inconsciente de sus actos.
c) El conejito confía en la fuerza de la inercia.
a. Develar la verdadera situación de crisis que vive Co- d) El conejito tensó las palmeras con sus propias patitas.
lombia.
IMAGEN 2
b. Criticar la falta de humanidad y compasión por parte de
las autoridades colombianas ante su gente.
c. Evidenciar irónicamente una noticia sobre Colombia y
la contradicción de la realidad.
d. Promover una campaña para ayudar a los más necesita-
dos en Latinoamerica.

10. ¿Qué titulo le pondrías a la imagen?

_____________________________________________
______________________________________________

4. Se puede afirmar:

IMAGEN 1 a) Los empleados tienen posición humillante frente a los


dueños de las empresas.
b) Las diferencias entre empleados y empleadores son
muy grandes.
c) Sin la presencia de los jefes algún subordinado podría
no trabajar.
d) El empleador tiene una actitud despótica frente a los
empleados.

5. El mejor título para la imagen sería:

a) Diferencias laborales
b) Cuestionamientos sociales
c) El jefe y los empleados
d) Haciendo meritos en el empleo
1. Si el conejito serrucha la cuerda, ¿qué pasará?

a) Será feliz.
b) Irá al cielo conejil.
c) Morirá.
d) No se puede determinar.

5º AÑO DE SECUNDARIA
Mejores Personas, Mejores Familias 85
Colegio

Mejores Personas, Mejores Familias RAZONAMIENTO VERBAL

M A
TE

20   INCLUSIÓN E IMPLICACIÓN

I. INCLUSIÓN ¿Qué opción está incluida en el concepto de los


1. Definición demás?
Mediante este tipo de ejercicios, donde se emplean a) Tanque
preguntas, se evalúa la capacidad del alumno para de- b) Avión
finir con precisión los conceptos indicados y sus rela- c) Vehículo
ciones de inclusión.
d) Automóvil
Por ejemplo: e) Helicóptero
Dado los conceptos RELOJ y MINUTERO, razonare-
mos la inclusión de conceptos del siguiente modo: Si ●● T
ercera forma:
el concepto RELOJ es absolutamente necesario para Señala la alternativa en la que el primer término
definir el concepto MINUTERO, entonces podemos incluye necesariamente al segundo.
decir: a) campo – tractor
b) pista – peatón
●● «El concepto RELOJ está incluido en el concepto c) seda – pañuelo
MINUTERO» o lo que es lo mismo:
d) chompa – vestir
●● «El concepto MINUTERO incluye al concepto
e) balón – fútbol
RELOJ»

II. IMPLICACIÓN
2. Pautas de solución
1. Definición
1. Lee atentamente la formulación de la pregunta. La implicación de conceptos es un caso particular de los
2. Define una de las palabras para ver si la otra for- ejercicios de inclusión de conceptos, en el cual un térmi-
ma parte de su definición. no se encuentra incluido en la definición de los cuatro
restantes; es decir, debemos precisar qué concepto está
3. Luego de determinar la verdad o falsedad de cada
siempre dentro de la definición de los otros conceptos.
enunciado, verificas qué opción concuerda con tu
2. Pautas de solución
conclusión.
1. Hecha la primera lectura, intenta determinar qué
palabra podría estar implicada en el resto.
3. Tipos de ejercicios
2. A continuación, pregúntate, opción por opción, si
●● Primera forma:
dicha palabra forma parte de la definición de los
¿Qué conceptos están necesariamente incluidos restantes.
en el concepto OCÉANO?
3. Si al hacer ese cotejo inicial encuentras que su
1. Inmenso 4. Continente
concepto elegido no es la respuesta, elije otra op-
2. Mar 5. Ola ción y repite el proceso.
3. Agua
Ejemplo:

¿Cuál concepto está implicado en los restantes?
●● Segunda forma:
a) Olla
¿Qué opción incluye conceptualmente al resto?
a) Alcohol b) Canasta
b) Uva c) Recipiente
c) Licor d) Taza
d) Vino e) Vaso
e) Líquido Resolución:
Una rápida revisión nos advierte que «recipiente» es el
concepto que está incluido.

86 Mejores Personas, Mejores Familias


5º AÑO DE SECUNDARIA
Colegio

RAZONAMIENTO VERBAL  Mejores Personas, Mejores Familias

10.
a) Médico b) Abogado c) Profesional
d) Contador e) Ingeniero
INCLUSIÓN

1. Señala qué conceptos están necesariamente incluidos en el


concepto MOTOR.
INCLUSIÓN
1. Automóvil 2. Vehículo 3. Generador
4. Aparato 5. Gasolina 1. ¿Qué término incluye en su concepto al resto?

a) 2; 3 y 4 b) 1; 3; 4 y 5 c) 1; 3 y 4 a) Benéfico b) Desvalidas c) Asilo


d) Todas e) 3 y 4 d) Personas e) Establecimiento

2. Señala qué conceptos están necesariamente incluidos en el 2. Señala qué conceptos están necesariamente incluidos en el
concepto CÁRCEL. concepto PETRÓLEO.

1. Centro 2. Cadena 3. Barrotes 1. Viscoso 2. Hidrocarburo 3. Gasolina


4. Crimen 5. Reclusorio 4. Oscuro 5. Costoso

a) 1; 4 y 5 b) 1; 2; 3 y 5 c) 1; 3 y 5 a) 2; 4 y 5 b) 1; 2 y 4 c) 1; 2; 4 y 5
d) Todas e) 1 y 5 d) Todas e) 1; 2 y 5

3. ¿Qué término incluye en su concepto al resto? 3. Señala qué conceptos están necesariamente incluidos en el
concepto COMPRA.
a) Cuadrilátero b) Boxeo c) Asalto
d) Lucha e) Disputa 1. Bien 2. Billete 3. Adquisición
4. Precio 5. Tienda
4. Señala la alternativa en la cual el primer término incluye al
segundo en su concepto. a) 1; 3 y 4 b) Solo 1 y 3 c) 1; 2 y 3
d) Todas e) 1; 3; 4 y 5
a) remo – mar b) calzado – pasador
c) vestido – cartera d) correa – sujetar IMPLICANCIA
e) masa – pan ZZ ¿Indique el concepto que está implicado en el resto?
4.
5. Señala la alternativa en la cual el segundo término incluye a) Escultura b) Grabado c) Poesía
al primero en su concepto. d) Arte e) Pintura

a) habla – conversación b) corbata – camisa 5.


c) pulsera – muñeca d) arete – oreja a) Bondad b) Sinceridad c) Virtud
e) lentes – vista d) Perseverancia e) Optimismo

IMPLICANCIA 6.
ZZ ¿Cuál concepto está implicado en los restantes? a) Caballa b) Pez c) Sardina
6. d) Atún e) Bonito
a) Cocina b) Lustradora c) Televisor
d) Aspiradora e) Artefacto 7.
a) Número b) Racional c) Entero
7. d) Natural e) Imaginario
a) Lluvia b) Mar c) Agua
d) Laguna e) Río 8.
a) Anzuelo b) Pez c) Pesca
8. d) Piscicultura e) Piscicultor
a) Venta b) Trueque c) Comercio
d) Compra e) Transacción 9. Señala qué conceptos están necesariamente incluidos en el
concepto MOTOR.
9.
a) Árbol b) Arbusto c) Hierba 1. Automóvil 2. Vehículo 3. Generador
d) Vegetal e) Planta 4. Aparato 5. Gasolina

5º AÑO DE SECUNDARIA
Mejores Personas, Mejores Familias 87
Colegio

Mejores Personas, Mejores Familias RAZONAMIENTO VERBAL

a) 2; 3 y 4 b) 1; 3; 4 y 5 c) 1; 3 y 4 3. ¿En cuál de las alternativas el tercer término incluye en su


d) Todas e) 3 y 4 concepto a los restantes?

10. Señala la alternativa en la cual el primer término incluye al a) cincel – martillo – escultor
segundo en su concepto. b) médico – homicidio – morgue
c) calle – foco – poste
a) remo – mar b) calzado – pasador d) lugar – encierro – jaula
c) vestido – cartera d) correa – sujetar e) vela – iluminación – mecha
e) masa – pan
4. ¿Qué término incluye en su concepto al resto?

a) Cuadrilátero b) Boxeo c) Lucha


d) Asalto e) Disputa
IMPLICANCIA
5. Señale la alternativa en la cual el segundo término incluye
1. Señala qué concepto está implicado en los restantes. al primero en su concepto.
a) Funeral b) Cementerio c) Muerto
d) Ataúd e) Luto a) habla – conversación b) corbata – camisa
c) pulsera – muñeca d) arete – oreja
INCLUSIÓN e) lentes – vista
2. ¿En qué alternativa el segundo término incluye en su
concepto a los dos restantes?

a) gimnasta – atleta – agilidad


b) incolora – agua – humedad
c) acuático – pez – río
d) ser – herbívoro – hierba
e) aguja – tejido – hilo

88 Mejores Personas, Mejores Familias


5º AÑO DE SECUNDARIA
Colegio

RAZONAMIENTO VERBAL  Mejores Personas, Mejores Familias

M A
TE

21   USO DE LA B, V, G Y J

I. PALABRAS QUE SE ESCRIBEN CON B YY Regla Nº 9


YY Regla Nº.1 Las palabras que comienzan con lab- rab- sab- tab-.
Las palabras que contienen bla- ble- bli- blo- blu-. Ejemplos: Labor, laboratorio, labio, laberinto, labrado.
Ejemplos: Tiembla, tembló, habla, cable, tabla, Rabia, rabino, rábano, rabadilla, rabo. Sábado, sábana,
biblioteca, bloque, blusa, bledo, Biblia. sabana, saber, sabotaje. Tabla, taburete, tabaco, taba-
Excepciones: Vladimir. quismo, tabú.
Excepciones: Lavar, lava ( de volcán), lavanda, ravio-
YY Regla Nº. 2 les, savia.
Antes de consonante se escribe b y no v.
Ejemplos: Libre, abdomen, obligatorio, lombriz, obvio, YY Regla Nº 10
brigadier, ombligo, cabra, tabla, absoluto. Las palabras que comienzan con ob-.
Ejemplos: Objeto, obispo, obrero, obeso, obelisco, obli-
YY Regla Nº 3 cuo, obtuso, obturador, obedecer, obcecado, obligatorio,
El pretérito imperfecto de los verbos terminados en -ar y óbolo, observatorio.
del verbo ir. Excepciones: Oveja, Oviedo, Ovidio, overol, ovillo.
Ejemplos: Cantabas, bailabas, estudiábamos, ordena-
ban, hablabais, miraban, caminabas, apuntaba, iba, YY Regla Nº 11
ibas, íbamos, ibais, iban. Las palabras que terminan en -bilidad.
Ejemplos: Amabilidad, aplicabilidad, contabilidad,
YY Regla Nº 4 habilidad, solubilidad, inviolabilidad, imposibilidad,
Los prefijos bi- bis- biz- que significan dos o dos ve- probabilidad.
ces. Excepciones: Movilidad, civilidad.
Ejemplos: Bimotor (dos motores), bimestre (dos
meses), bisectriz (dos partes iguales), bisabuelo (dos YY Regla Nº 12
veces padre), bisnieto (dos veces hijo), bizcocho (pan Los verbos que terminan en -bir y -buir, así como to-
sin levadura que se cuece dos veces), bizco (persona das sus formas conjugadas.
que ve doble). Ejemplos: Recibir, distribuir, contribuir, concebir, im-
buir, atribuir, retribuir, escribir, describir, suscribir, su-
YY Regla Nº 5 bir, exhibir, prohibir, sucumbir.
Las palabras que comienzan con alb-. Excepciones: Hervir, servir, vivir, convivir, revivir.
Ejemplos: Alba, albañil, albaricoque, álbum, albur,
albatros, albedrío, alberca, albergue, albino, albóndiga, YY Regla Nº 13
alborada. Las palabras que terminan en -bunda, -bundo.
Excepciones: Álvarez, Álvaro, álveo, alveolo, alverja, Ejemplos: Tremebundo, nauseabundo, moribundo,
alveario. abunda, vagabundo, furibundo.

YY Regla Nº 6 II. PALABRAS QUE SE ESCRIBEN CON V


Las palabras que comienzan con bu-. YY Regla Nº.1
Ejemplos: Buque, burro, bufanda, bueno, búho, búfalo, Después de d y n se escibre v.
buey, buitre, buche, bufé. Ejemplos: Adverbio, invierno, envidia, envase, adver-
Excepciones: Vudú, vuelco, vuelo, vuelta, vuestro, sario, investigación, envoltura, convulsión, convivir, in-
vulcanizar, vulcanología, vulgar, vulgo, vulnerable. vitar, invento, envejecido, advertencia, invicto.

YY Regla Nº 7 YY Regla Nº.2


Las palabras que comienzan con cub-. Después de ol se escribe v.
Ejemplos: Cubo, cúbico, cuba, cubilete, cubierto, cubí- Ejemplos: Olvidar, inolvidable, resolver, polvo, polvo-
culo, cubrir. riento, solvente, disolver.

YY Regla Nº 8 YY Regla Nº.3


Las palabras que empiezan con hab- y heb-. Los pretéritos indefinido e imperfecto y el futuro de
Ejemplos: Haber, habitación, hebra, hebreo, Habana, subjuntivo de los verbos estar, tener, andar y sus com-
habano, hábil, hebilla. puestos.
Excepciones: Hevea (caucho y nombre de un produc- Ejemplos: Tuve, estuve, tuviera, estuviera, anduve, an-
to). duvo, retuvo, contuvimos, obtuviese, tuviere, tuviese.

5º AÑO DE SECUNDARIA
Mejores Personas, Mejores Familias 89
Colegio

Mejores Personas, Mejores Familias RAZONAMIENTO VERBAL

YY Regla Nº.4 Ejemplos: Recoger, recoges, recogí, recogiste, recogimos,


Las palabras que comienzan por vice- (en vez de, hace coger, acoger, escoger, encoger, proteger, emerger, dirigir,
de) y villa- (pueblo, casa de campo). corregir, exigir, urgir, surgir, regir, rugir, erigir, refrigerar,
Ejemplos: Vicerrector, viceministro, vicealmirante, vi- exagerar, aligerar.
llancico, villano, villorrio. Excepciones: Tejer, crujir.
Excepciones: Bíceps, billar.
YY Regla Nº. 5
YY Regla Nº.5 Las palabras terminadas en gio, gia, gía.
Las palabras que comienzan con eva- eve- evo- evi-. Ejemplos: Contagio, privilegio, colegio, refugio, prodi-
Ejemplos: evitar, evidencia, evidente, evadir, evasor, gio, subterfugio, presagio, adagio, prestigio, litigio, sorti-
evaporar, evacuar, evocar, evangélico, eventualmente, legio, sufragio, sacrilegio, elogio, regio, nostalgia.
evolución, evento. Excepciones: Apoplejía, bujía, crujía, herejía, lejía, te-
Excepciones: Ebanista, ébano. jía.

YY Regla Nº.6 YY Regla Nº. 6


Las palabras terminadas en -ava -ave -avo -eva -eve Las palabras terminadas en gión, gionario, gioso, gi-
-evo -iva -ivo. noso y gismo.
Ejemplos: Octava, clave, grave, nueva, leve, suave, co- Ejemplos: Región, religión, legión, legionario, religioso,
municativo, legislativa, llamativa, fugitivo, paliativo, contagioso, prodigioso, prestigioso, elogioso, vertiginoso,
medioevo, activo, pasivo, pavo, pensativa, permisivo, oleaginoso, neologismo, silogismo.
recursivo, bravo. Excepciones: Espejismo, salvajismo.
Excepciones: Baba, haba, sílaba, traba, árabe, jara-
be, cabo, lavabo, nabo, menoscabo, rabo, ceba, prueba, YY Regla Nº. 7
mancebo, placebo, recebo, sebo, criba, giba, arribo, estri- Las palabras terminadas en -gésimo, -gesimal, gésico,
bo, recibo. gético.
Ejemplos: Vigésimo, trigésimo, cuadragesimal, sexage-
YY Regla Nº.7 simal, analgésico, energético.
Las palabras terminadas viro- vira- ívoro-ívora.
Ejemplos: Elvira, carnívoro, herbívora, omnívoro,
triunviro, revira. IV. PALABRAS QUE SE ESCRIBEN CON J
Excepciones: Víbora. YY Regla Nº. 1
Las palabras que comienzan por adj y por obj.
III. PALABRAS QUE SE ESCRIBEN CON G Ejemplos: Adjetivo, objetivo, objeción, objetar, objeto,
YY Regla Nº.1 objetividad.
Después de al, an y ar se escribe con ge, gi y no con je,
ji. YY Regla Nº. 2
Ejemplos: Álgebra, algebraico, álgido, algidez, ángel, Ángela, Las palabras terminadas en aje y en eje.
Angélica, angelical, angelito, Ángel, Argelia, Argentina, argen- Ejemplos: Lenguaje, personaje, embalaje, despeje, he-
tar, evangelio, evangélico. reje, eje, tejemaneje.
Excepciones: Aljibe. Excepciones: Ambages, proteger, proteges.
YY Regla Nº. 2
YY Regla Nº. 3
Las palabras que contienen la combinación gen.
Las formas verbales pretéritas y subjuntivas de los ver-
Ejemplos: General, genial, generar.
bos cuyo infinitivo termina en decir, ducir, traer.
Excepciones: Jengibre, ajenjo, berenjena, comején, y
Ejemplos: Dije, deduje, reduje, traduje, predije, bendije.
la tercera persona de plural de imperativo de los verbos
cuyo infinitivo termina en -jar: trabajen, viajen, dibu-
YY Regla Nº. 4
jen, rebajen.
Las formas verbales de los verbos cuyo infinitivo
termina en jar o jear.
YY Regla Nº. 3
Ejemplos: Trabaje, trabajen, forcejeo, forcejeamos,
Las palabras que comienzan por legi, legis, gest.
hojean, cojea, canjean, granjearon, trajear, baje, rajen.
Ejemplos: Legible, legión, legislar, legislatura, legítimo,
legitimar, gesto, gestar, gesticular, gesta, gestión, gesta-
YY Regla Nº. 5
ción, gestor.
Los sustantivos y adjetivos terminados en jero, jera,
Excepciones: Lejía, lejos, lejanías, lejísimos.
jería.
YY Regla Nº. 4 Ejemplos: Extranjero, agujero, consejero, granjero, pasa-
Las formas verbales de los verbos cuyo infinitivo ter- jero, flojera, tijera, consejería, relojero, relojería, cerrajero.
mina en ger, gir o gerar se escriben con G cuando lle- Excepciones: Ligero.
van ge o gi.

90 Mejores Personas, Mejores Familias


5º AÑO DE SECUNDARIA
Colegio

RAZONAMIENTO VERBAL  Mejores Personas, Mejores Familias

7. Oración con uso inadecuado de grafías:


a) Un sexagenario es una persona de entre 60 y 69 años.
b) Un gesto es un movimiento de las manos o cara que
expresa algo.
1. Presenta correcta redacción. c) La conserjería es el lugar donde suele estar el conserje
a) Una gestoría es una oficina donde te resuelven asuntos de una entidad.
de empresas, seguros, trabajo, etc. d) Generación conjunto de personas de edades aproxima-
b) Un parage es un lugar lejano y aislado. das que viven en una misma época.
c) El Gobiernos Autónomos se dividen en Consegerías: de e) Hogear con «h» es pasar las hojas de un libro y ogear sin
Educación, de Industria... «h» es mirar algo atentamente.
d) El verbo jesticular significa hacer gestos y muecas.
e) Aflijir a alguien es causarle tristeza. 8. Oración con uso inadecuado de grafías:
a) Hacer la gestión de un asunto es hacer lo necesario para
2. Presenta incorrecta redacción. solucionarlo.
a) Homenajear a alguien es hacer algo en su honor. b) Ambajes es una excepción a la regla y significa ‘rodeo’.
b) Tú eres de la generación de mis abuelos. c) Las cosas homogéneas tienen características comunes y
c) Una norma, costumbre o ley que está vigente es la que las heterogéneas, distintas.
está en uso. d) Tener un presagio es creer haber adivinado lo que va a
d) Vimos el suelo mojado y dedugimos que había llovido. pasar.
e) No traduje nada del texto que nos habían mandado. e) Un político actúa con demagogia cuando promete cosas
que no puede cumplir.
3. Presenta uso correcto de grafías.
a) Un potage es un caldo de legumbres con verduras, pata- 9. Oración con uso adecuado de grafías:
tas u otras cosas. a) Llamamos pedijueño a una persona que pide mucho.
b) Algo vertijinoso es algo muy rápido o que produce vér- b) Pedro flogeó mucho al final del curso y empezó a sacar
tigo. malas notas.
c) El hijo primojénito es el primero de una familia. c) Un jenerador es un aparato que transforma el movi-
d) Una barra alrededor de la cual gira un cuerpo es un eje. miento en energía eléctrica.
e) Me digiste que no te llamara, así que no te enfades. d) He comprado tres egemplares de este libro.
e) No confundas conserjería con consejería.
4. Presenta correcta redacción.
a) La linjüística es la ciencia que estudia el lenguaje. 10. Presenta correcta escritura en relación a las grafías G y J
b) Tragimos poca comida para tantos. a) Un egecutivo es una persona que desempeña un cargo
c) Lo que no está claro es ambiguo. de dirección en una empresa.
d) Egercitar es aprender algo repitiéndolo mucho. b) Extremadura tiene riqueza cinejética, es decir, tiene
e) Se llama jestación al tiempo que está el feto en el vientre mucha caza.
de la madre de los mamíferos. c) Decimos que una persona es virjen cuando no ha teni-
do relaciones sexuales.
5. Presenta incorrecta redacción. d) Por lo que me contaste deduge quién era el culpable.
a) Algo que sucede generalmente es que sucede frecuente- e) Cometió plagio durante el examen.
mente.
b) Algo que produce una enfermedad es patójeno.
c) Una exigencia es una reclamación fuerte o caprichosa
de algo.
d) Hablar sin ambages significa hablar sin rodeos, es decir,
hablar claramente, ir al grano. 1. Es una palabra que debería escribirse con –b, pero es una
e) Una gesta es el conjunto de hazañas de un personaje o excepción a las reglas.
de un pueblo. a) relevo b) alvéolo c) leve
d) veintena e) vecina
6. Presenta uso correcto de grafías.
a) Un egemplar es cada uno de los libros, periódicos o dis- 2. Señala la opción que está inadecuadamente escrita.
cos de una misma edición. a) La ciencia está imbuida en la investigación sobre los
b) Al darse el golpe, Ana hizo un jesto de dolor. herbívoros.
c) Quien tiene un litigio con alguien tiene un problema b) ¿La víbora es un animal ovíparo? De qué estás hablan-
que requiere un juicio. do…
d) No digimos absolutamente nada. c) La bacteria, la viruela, la bocina, tu vecina, acabábamos.
e) Una característica jenérica es la que tienen todos los d) Siempre la exuberante mujer buscaba que pasáramos
seres de la misma especie. juntos.
e) Mi mobilidad no llegó en su debido momento: me atrasé.

5º AÑO DE SECUNDARIA
Mejores Personas, Mejores Familias 91
Colegio

Mejores Personas, Mejores Familias RAZONAMIENTO VERBAL

3. Señala la opción correcta. 10. Alternativa que presenta una palabra incorrectamente es-
a) Una bívora deboró a un animal carníboro. crita.
b) Hayaron proyectiles a una milla. a) Juan sintió un sabor acervo en la lengua.
c) El forajido usaba una jerga deshilachada. b) El acervo cultural del Perú es impresionante.
d) El arapiento no osó en abrir la alcancía. c) Al autor le incomoda la crítica acerba de la prensa.
d) Melgar soportó el dolor que le produjo la ausencia de
e) Cogeaba el caballo por la mala colocación del herraje.
Silvia.
e) La acidez se produce por exceso de iones hidrógeno.
4. En una alternativa, le hacen falta más bes.
a) La a_lución nos sir_ió para que la sir_ienta se _ea y _
ista mejor.
b) Mi _isnieto no come _iscochos; es un _arón omní_oro
y _aliente.
c) La _esuquea_a y a_raza_a; después se fue al la_adero. 1. ¿Qué palabra presenta uso incorrecto de grafías?
d) Ha_rá que tener más ci_ilidad y ama_ilidad con la ca- a) Bisílaba b) Hervir c) Civilidad
ter_a. d) Alabavas e) Virrey
e) Se i_a al su_ur_io con el al_acea y la cachim_a a _er la
rum_a. 2. ¿Qué palabra presenta uso incorrecto de grafías?
a) Violencia b) Ambulansia c) Insecticida
5. La confusión entre la b y la v se debe sobre todo d) Herbívora e) Bicolor
a_________.
a) la ignorancia de las personas 3. ¿Qué palabra presenta uso correcto de grafías?
a) Subllugar b) Correjir c) Herege
b) que ambas son fonéticamente idénticas
d) Suabe e) Crujió
c) que no leemos y por eso nos confundimos
d) la mala enseñanza de los maestros 4. ¿Qué palabra presenta uso incorrecto de grafías?
e) que nos da igual escribir con b o con v a) Bicéfalo b) Movilidad c) Servir
d) Andubiste e) Viceministro
6. Serie que contiene más errores de escritura.
a) cabezazo – herbolario – embidioso 5. ¿Qué palabra presenta uso incorrecto de grafías?
b) simbergüenza – advervio – serbiremos a) Subversivo b) Obvio c) Sentencia
c) convivió – lambada – labanda d) Hervíboro e) Vudú
d) lobezno – lavaplatos – desherbar
6. ¿Qué palabra presenta uso correcto de grafías?
e) caballero – relebo – proclive
a) Escases b) Escojer c) Conduje
d) Lebe e) Hospedage
7. Señala la opción incorrecta.
a) La b es conocida como una bilabial sonora. 7. Marca la alternativa que no presente errores de grafías
b) La v, fonéticamente, se representa como b. a) Adverbio, dulsura, altives
c) La v también es una bilabial sonora. b) Cavilaba, sutilesa, desagüe
d) La b y la v tienen diferencias en el sonido. c) Quizo, contagiar, marginal
e) La v puede llamarse uve o v corta. d) Confesión, absolber, tensión
e) Trigésimo, adyacente, rasgo
8. Señala la opción incorrecta.
a) El bullicioso río atraviesa la vertiente. 8. Marca la alternativa con correcto uso de las grafías g y j
a) Exigir, legitimo, garage, deduje
b) Tuvo que absolver varias preguntas.
b) Exijir, legítimo, garaje, deduge
c) Sin brújula no hubiese llegado al lóbrego océano.
c) Exigir, legítimo, garaje, deduje
d) El bagavundo obsequió alvorozado un álbum. d) Exigir, lejítimo, garaje, deduge
e) Recibió un golpe en el tobillo. e) Exijir, lejítimo, garage, deduge

9. Señala la opción que está inadecuadamente escrita. 9. ¿Qué palabra está incorrectamente escrita?
a) El bisnieto estuvo cerca del biombo. a) Elogiar b) Garaje c) Esbeltez
b) Compró bencina barata. d) Lisongear e) Atrasar
c) Sintió vergüenza al saber el beredicto.
d) Por envidioso el novicio es aborrecido. 10. ¿Qué palabra está incorrectamente escrita?
a) Subversivo b) Hervíbora c) Ilación
e) El movimiento subversivo no fue violento.
d) Vascuence e) Quiso

92 Mejores Personas, Mejores Familias


5º AÑO DE SECUNDARIA
Razonamiento
Verbal Colegio

RAZONAMIENTO VERBAL  Mejores Personas, Mejores Familias

M A
TE

22   LECTURA CRÍTICA: TONO, PARADOJA


Y ESTRUCTURA DE UN TEXTO

I. ESTRUCTURA DE TEXTO la sangre de los hombres que él mismo había ordenado ahorcar,
Por estructura debemos entender: andamiaje, conjunto y continúa con un recuerdo de historia común, «la odisea del
ordenado de partes articuladas de tal forma que dan un cadáver de Juan Lavalle, que se iba pudriendo a medida que los
sentido o una forma clara y específica. soldados trataban de preservarlo de los enemigos llevándolo
Las estructuras de los textos son de diversos tipos; por por la quebrada de Humahuaca». La tendencia sigue con un
ejemplo, podemos dar inicio con una idea general y luego hecho insólito: en 1841, un cierto capitán García cuenta cómo
ir descendiendo hasta una idea particular y viceversa. esa muerte tarda: «Los ojos se revuelven, la cabeza cortada se
agita durante varios minutos en el suelo, el cuerpo se desgarra
II. ALGUNOS ELEMENTOS con sus propias uñas, ya decapitado». En una imagen digna de
1. Tono Stendhal, «Una matrona llamada Fortunata García de García
Se refiere al estado emocional en el que se encuentra el recupera la cabeza, la lava con perfume y, supuestamente, la
autor al emitir o producir el mensaje. En estos casos es deposita en un nicho del convento de San Francisco».
importante reconocer la intención y el tema del texto, El proceso de necrofilia se extiende a lo largo del siglo XIX y se da
luego analizarlo en busca de palabras claves y pista que en el siglo XX de infinitas maneras. Por un lado, el culto a Rosas
puedan darnos una idea del estado de ánimo. y la repatriación de sus restos y, por otro lado, en la Recoleta, un
Existe una gran diversidad de tonos; pero los más re- cementerio que es una exposición de las infinitas variantes del
mismo tipo de situaciones. Para Martínez, resulta notable esa
currentes son:
especie de reivindicación de la necrofilia en los últimos años.
Burlón, amenazante, alegre, triste, colérico, irónico, sar-
Así, se profanó la tumba de fray Mamerto Esquelú y se robaron
cástico, amoroso, afable, jubiloso, indiferente, corrosivo,
el cuerpo del padre Martínez de Hoz (entre 1978 y 1988). Poco
adulador, suplicante, impositivo, reflexivo, ingenuo,
más tarde, en 1991, cuando se volvía incierta la elección de
analítico, esperanzado, quejoso, reivindicador, etc.
«Palito» Ortega —como candidato del partido oficial—, en un
gesto efectista, el presidente Menem se presentó en Tucumán
2. Paradoja con los restos del prócer Juan Bautista Alberdi y los ofrendó a la
Es una aparente contradicción en un discurso. Se da una provincia. Así, garantizó la elección de Ortega.
relación causa–efecto. Sin embargo, si una es verdadera,
la otra no debería serlo. Adaptado de Santa Evita, de Tomás Eloy Martínez
Se busca reconocer la paradoja o resolverla
dando una solución que resuelva lógicamente la 1. Señala lo que se afirma en el texto.
paradoja. I. Ulrico Schmidt inició una tradición literaria necrofílica
en Argentina.
II. El robo de un cadáver es un ejemplo de la necrofilia ar-
gentina.
III. La presentación de un cadáver favoreció una elección
política.
IV. El cadáver del capitán García no fue sepultado.
a) I, III y IV
b) II y IV
c) I, II y III
d) II y III

2. ¿Cuál es la idea principal del segundo párrafo?


Lee atentamente los textos y responde las preguntas pro- a) Después de los siglos XIX y XX, la necrofilia desapare-
puestas. ció en Argentina.
b) Entre los siglos XIX y XX, se repatrió un cadáver, se
TEXTO I robó otro y se honró a un tercero.
Para Tomás Eloy Martínez, «La necrofilia argentina es tan vieja c) La necrofilia continuó, en Argentina, durante los siglos
como el ser nacional». Sus inicios son tormentosos. Comienza XIX y XX.
cuando Ulrico Schmidt, el primero de los cronistas de Indias d) Fueron muchas las maneras de rendir culto a los muer-
que llega al Río de la Plata, narra cómo don Pedro de Mendoza tos en Argentina.
pretende curarse de la sífilis que padecía aplicándose en sus llagas

5º AÑO DE SECUNDARIA
Mejores Personas, Mejores Familias 93
Colegio

Mejores Personas, Mejores Familias RAZONAMIENTO VERBAL

3. ¿Cuál es la estructura del texto? c) Novedoso diseño en el campo de la tecnología biomé-


a) Costumbre nacional – desarrollo – manipulación polí- trica
tica d) Los celulares y sus nuevos ámbitos de seguridad
b) Presentación de un proceso – inicios – desarrollo
c) Inicios – despliegue de una tendencia – culminación 7. ¿Cuál de las siguientes alternativas presenta la estructura
d) Tesis – etapas iniciales – etapas finales adecuada para el texto?
a) Afirmación – propuesta – explicación – posible ventaja
4 . Señala el significado de la frase: «La necrofilia argentina es – otras propuestas
tan vieja como el ser nacional». b) Antecedentes – medida – experimento – hipótesis –
a) La naturaleza de la necrofilia argentina está ligada a sus comparación
orígenes nacionales. c) Limitación – innovación – resultado – beneficios direc-
b) La antigüedad de la necrofilia, en Argentina, se remon- tos – beneficios indirectos
ta a la aparición de la nación. d) Método – estudios – conclusiones – resultados – conse-
c) El amor por los muertos es tan antiguo como la Repú- cuencias
blica Argentina.
d) En la medida que pasa el tiempo, la necrofilia se pro- TEXO III
fundiza en Argentina. ZZ GUISELA: El destino no existe bajo ningún criterio del
azar. Soy de las personas que opina que el destino es la
5. ¿Cuáles serían las palabras más apropiadas para reempla- consecuencia de nuestra forma de actuar ante la vida, y
zar la frase «gesto efectista» de las últimas líneas del texto? que todo depende de las decisiones que tomemos en cada
a) Deferencia llamativa situación que se presenta a lo largo de esta.
b) Efecto mediático Creo que si quiero algo en esta vida tengo que sudar la gota
c) Estratagema impactante gorda para conseguirlo, además está comprobado que el
d) Actitud hipócrita poder de la mente es el motor de la realidad humana.
ZZ SUSY: El destino sí existe, ya todo está determinado en
TEXTO II nuestra vida. Muchos estudios que se han hecho sobre her-
Muchos de los servicios a los que manos gemelos, separados en el momento del nacimiento,
accedemos desde los smartphone educados en países diferentes, y sin ningún tipo de comu-
requieren de nuestra identificación. nicación entre ellos, concluyen que han seguido una idén-
Frente al tradicional código PIN, que tica trayectoria en la vida: misma profesión, misma familia
puede ser fácilmente adivinado o co- tipo, mismo número de hijos, hasta se han casado con mu-
piado, un equipo de investigadores jeres que tienen el mismo nombre. ¡Increíble!
propone la autenticación mediante ZZ FLORCITA: No había pensado antes en eso de que la ge-
una firma en el aire. nética influye en nuestro destino y creo que efectivamente,
Esta técnica biométrica se basa en la en gran parte. Si por genética heredas enfermedades como
realización de una firma en el aire su- hipertiroidismo, diabetes, cáncer, artritis, etc., pues sí, ya
jetando el teléfono móvil. Los autores, pertenecientes al Centro estamos marcados por el destino en nuestros genes.
de Domótica Integral de la Universidad Politécnica de Madrid, Entonces, sí podemos concluir que es la genética nuestro
han realizado diversos experimentos para testear la precisión de destino, no somos libres del todo al tomar nuestras deci-
esta nueva técnica. Para ello solicitaron a cien participantes que siones, ya que «debemos» seguir las indicaciones de nues-
realizaran su firma manuscrita en el aire sujetando un teléfono tros genes, ¿depende de ellos nuestro destino?
móvil mientras eran filmados por una videocámara. A partir de Espero que no, fíjense lo que sucedería con los descen-
estas grabaciones, otro grupo de personas debía intentar imitar dientes de psicópatas, de asesinos, de estafadores.
las firmas. Como resultado, se obtiene una tasa de falsificación en
torno al tres por ciento, una cifra muy baja teniendo en cuenta que 8. ¿Cual es la discrepancia central entre Guisela y Susy?
quienes copiaban las firmas disponían de las grabaciones origina- a) No debemos fiarnos de nuestro destino.
les como ayuda adicional. b) El destino es creación de dios o del hombre.
El método aportaría una mayor seguridad a la hora de realizar c) La existencia del destino
operaciones en las que se accede a información personal como d) Existe un destino genéticamente demostrado.
cuentas bancarias, correo electrónico o compras online. Ade-
más, el equipo está investigando también otras técnicas biomé- 9. Son argumentos empleados por Susy:
tricas como el reconocimiento por la geometría de la mano, I. Existen hermanos gemelos que han tenido vidas sepa-
iris, huella, cara, forma de caminar u olor. radas y aun así han coincidido en algunas cosas.
6. ¿Cuál de las siguientes alternativas corresponde mejor al II. Los hijos de asesinos terminan siendo despiadados cri-
título del texto? minales.
a) Innovaciones tecnológicas en los smartphone actuales
b) La firma en el aire como posible nueva técnica de autoi-
dentificación

94 Mejores Personas, Mejores Familias


5º AÑO DE SECUNDARIA
Colegio

RAZONAMIENTO VERBAL  Mejores Personas, Mejores Familias

III. Para conseguir lo que soñamos, no basta con trabajar. una aleación de oro, plata y cobre que brillaban al sol en señal
a) I y II de poder». La pirámide de Abu Rawash se podrá visitar a partir
b) Solo I de 2009, pero solo por su parte exterior.
c) Solo II
d) Solo III 1. Resulta incompatible con el texto
I. El material era traído desde 8000 kilómetros de distan-
10. La tesis de Florcita sería: cia por el Nilo y alcanzó Abu Rawash a través de gran-
a) El destino ya está determinado en la vida de todo ser des canalones.
humano. II. La pirámide de Abu Rawash se podrá visitar a partir de
b) No podemos escapar a nuestro destino. 2009, pero únicamente por su parte exterior.
c) Solo el hombre es capaz de determinar su destino. III. Una profunda grieta en la tierra, que fue el pasaje de ba-
d) La genética influye en nuestro destino. jada a la cámara subterránea, está ahora al acceso libre.
IV. Cada una de las piezas de ese inmenso puzzle pesaba
25 toneladas y se necesitaban hasta 370 personas para
moverla
a) I y II
b) III y IV
Lee atentamente cada texto y responde las preguntas propues- c) I y III
tas. d) II y IV

TEXTO I 2. El asunto abordado en el texto es:


El faraón Dyedefra, que reinó en Egipto hacia el año 2556 antes a) La construcción de la cuarta pirámide, más alta y mag-
de Cristo, hace casi cinco milenios, construyó la pirámide con nífica.
más altura y magnificencia de las cuatro que se veían desde las b) Detalles de cómo se edificó la cuarta pirámide en Egip-
orillas del Nilo. La pirámide, a tenor de los últimos hallazgos, to.
superaba en 7,62 metros de altura a la de Keops, que tiene 146 c) Pormenores y significado de una pirámide desconocida
metros. Cada una de las caras, en su base, medían 122 metros y hasta hoy.
el ángulo de inclinación era de 64 grados, si bien una variación d) Descubrimiento y presentación de los hallazgos de la
impidió que se colapsara. cuarta pirámide de Egipto.
Fue levantada con caliza y granito rojo de Asuán, como la de su
padre. El material llegaba desde 800 kilómetros de distancia por 3. El término precisa significa en el texto:
el Nilo y alcanzó Abu Rawash a través de grandes canalones. a) Única
Allí, utilizando una dura piedra de dolerita, se partían los b) Milimetrada
bloques, a los que daban la forma precisa con una especie de c) Exacta
sierra de cobre con el filo de cuarzo. Cada una de las piezas de d) Pulida
ese inmenso puzzle pesaba hasta 25 toneladas y se necesitaban
hasta 370 personas para moverla. 4. Se infiere del texto:
La mayoría de los trabajadores que llegaron a Abu Rawash, I. Para mover una de las piezas de la pirámide en men-
unos 15000 en los ocho años que duró la construcción, fueron ción, podían participar 360 personas.
agricultores que realizaban este trabajo cuando las crecidas del II. La pirámide del padre del faraón Dyedefra fue construi-
Nilo inundaba sus tierras de cultivo. «Cuando comenzaban el da únicamente con piedra caliza y granito rojo de Asuán.
trabajo, realizaban marcas en las piedras con un tinte rojo», III. La mayoría de las piezas de la pirámide pesaban 25 to-
explica sobre el terreno el egiptólogo Hassan Abd El-Razek. neladas.
Una profunda grieta en la tierra, que fue el pasaje de bajada IV. Ninguna persona puede caminar por la parte interior
a la cámara subterránea, está ahora al aire libre y ha servido de la pirámide a partir del 2009.
para comprobar que en la primera fase se utilizó argamasa para a) I y III
consolidar el interior donde debía descansar el faraón en su b) III y IV
viaje a la otra vida. c) Solo I
En el exterior aún el suelo está plagado de fragmentos de d) I, III y IV
pequeñas vasijas donde se llevaban las ofrendas a Dyedefra.
El-Razek muestra también el lugar donde aparecieron varias 5. El tono del texto es:
estatuas rotas del faraón, así como las ruinas de lo que debió a) Asombrado
ser el templo, cuyas obras de reconstrucción fueron paralizadas b) Desconcertado
por Zahi Hawass. c) Informativo
Anthony Geffen, también aporta detalles de la gran obra d) Incrédulo
mientras, bajo un sol abrasador, pasea entre las milenarias
piedras. «La pirámide estaba recubierta de granito pulido y por

5º AÑO DE SECUNDARIA
Mejores Personas, Mejores Familias 95
Colegio

Mejores Personas, Mejores Familias RAZONAMIENTO VERBAL

10. El tono del texto es:


TEXTO II a) Conservador
Yo estoy a favor del aborto, pero dependiendo del contexto. b) Revolucionario
Creo que la ley está muy bien hecha, ya que prohíbe el aborto a c) De respaldo
partir de un tiempo de gestación concreto. Creo que si alguien d) Indiferente
quiere abortar, tendrá sus razones porque a nadie se le hace
fácil acabar con una vida, pero debería hacerlo antes de que el
feto sienta dolor. Si se pudiese hacer un cambio en la ley sobre
este tema, creo que se debería dejar un poco más de tiempo
para decidir. Todos los que tienen esas ideas conservadoras Lee cada texto y responde las preguntas propuestas.
(más bien del Neolítico) tendrían que pensar en lo difícil que
es educar a un niño en condiciones desfavorables. Es mejor no TEXTO I
crear una vida que crear una vida de sufrimientos y problemas, La ubicación actual de Osama Bin Laden, una figura importante
y para que la sociedad avance, hay que dejar de estancarse en involucrada en terrorismo internacional y una de las personas
las ideas conservadoras o religiosas (como las del papa que más buscadas por Estados Unidos y otros gobiernos, es
desaconseja el uso de preservativos en países como Etiopía, en desconocida. Se habla mucho de su muerte, pero tanto el FBI
los que hay un alto nivel de personas afectadas por el virus del como la red terrorista Al Qaeda han desmentido dicha muerte.
sida) que nos impiden avanzar en todos los aspectos. Actualmente, existe un gran número de reclamaciones no
verificadas acerca de su estado y ubicación, incluidos los
6. ¿Cual es la tesis del texto anterior? rumores de su muerte en varios años, y las reivindicaciones
a) El aborto debe ser legal en el mundo. de sus visitas a diversos países. Sin embargo, aunque hay
b) El contexto determina que el aborto exista. grabaciones de vídeo donde aparece Bin Laden no se puede
c) Estoy a favor del aborto, dependiendo del contexto. saber con exactitud su localización.
d) Existen razones muy fuertes para tomar la decisión de Después de los Atentados del 11 de septiembre de 2001, los
abortar. Estados Unidos pidieron a las autoridades talibanes entregar
a Bin Laden para que enfrentara cargos por terrorismo. Los
7. No son argumentos del autor: talibanes se negaron a entregar a Bin Laden sin pruebas o
I. Para que la sociedad avance, hay que dejar de estancarse en indicios de su implicación en los atentados del 11 de septiembre
las ideas conservadoras o religiosas. e hizo una contraoferta para que Bin Laden fuese a un tribunal
II. Es mejor practicar el aborto cuando el feto no siente do- islámico o lo extraditaran a otro país. Ambas ofertas fueron
lor. rechazadas por el gobierno de los Estados Unidos.
III. No es posible educar a un niño en condiciones desfavo- Los rumores de su muerte siguieron, se decía que estaba
rables. muerto o fatalmente herido durante los bombardeos de
IV. Es mejor no crear una vida que crear una vida de sufri- Estados Unidos después de los atentados del 11 septiembre,
mientos y problemas. o que había muerto por causas naturales. De acuerdo con
a) I , II y III Gary Berntsen, en su libro de 2005, Jawbreaker, un número de
b) II y IV AlQaeda detenidos más tarde confirmó que Bin Laden había
c) Solo III escapado de Pakistán, mediante de una ruta oriental a través de
d) Todos son argumentos montañas cubiertas de nieve en el área de Parachinar, Pakistán.
Los medios de comunicación informaron de que Bin Laden
8. El término concreto, en el texto, significa: sufría de una enfermedad renal que le obligaba a tener acceso
a) Duro b) Macizo a servicios médicos avanzados, posiblemente diálisis renal.
c) Exacto d) Medido Ayman al-Zawahiri, quien es el segundo Jefe al mando de Al
Qaeda, es quien ha brindado atención médica a Bin Laden.
9. ¿Qué enunciado debilitaría más lo propuesto por el autor? La CIA afirma que Osama Bin Laden está vivo y escondido en
a) La santa Iglesia católica ha determinado que todo aquel el noroeste de Pakistán, en gran parte aislado de las operaciones
que prive de la vida a un ser humano merece ser exco- diarias de Al Qaeda.
mulgado. Por otra parte, en el mes de enero de 2010 el FBI divulgó unas
b) La universidad de Florida, en EE.UU. ha determinado imágenes virtuales de Osama Bin Laden, en las que proyectaba
que el embrión (feto) tiene sensibilidad desde el instan- el aspecto que tendría actualmente el líder de Al Qaeda. Los
te mismo de la concepción. expertos forenses del FBI aseguran que Bin Laden seguiría
c) En algunos países americanos se permite el aborto y teniendo barba, además, se especula que el líder de Al Qaeda
esta práctica mejora su economía. caminaría con un bastón.
d) La Iglesia nunca tiene ideas asertivas en cuanto a temas
sexuales y reproductivos.

96 Mejores Personas, Mejores Familias


5º AÑO DE SECUNDARIA
Colegio

RAZONAMIENTO VERBAL  Mejores Personas, Mejores Familias

1. Del texto anterior es posible deducir correctamente: Al ser combinados con figuras y signos más o menos, estos
I. Bin Laden fue requerido por los Estados Unidos a las términos indican un punto en el tiempo que precede o es
autoridades talibanes por el atentado terrorista de las posterior a una acción específica. Por lo tanto, H-3 significa
torres gemelas. 3 horas antes de la Hora H y D+3, 3 días después del Día D.
II. Tanto el FBI como la red terrorista Al Qaeda han des- Asimismo H+75 minutos significa la Hora H más 1 hora y 15
mentido rotundamente la muerte de Obama bin Laden. minutos.
III. Un 11 de septiembre ocurrió un atentado terrorista en La planificación de operaciones a gran escala se elabora en detalle
alguna parte del mundo. mucho antes de que se definan fechas específicas, por lo que las
a) I y II b) II y III órdenes son emitidas para los distintos pasos a realizar en el Día
c) Solo II d) Solo III D o la Hora H más o menos un determinado número de días,
horas o minutos. En el momento apropiado, se emite una orden
2. La idea central del texto es: subsecuente que establece la hora y día reales.
a) Luego de los atentados del 11 de septiembre de 2001, los El Día D de la invasión de Normandía por los aliados, estaba
Estados Unidos requirieron a las autoridades talibanes definido originalmente para el 5 de junio de 1944, pero las
entregar a Bin Laden para que enfrentara cargos por te- malas condiciones climáticas y marítimas, hicieron que el
rrorismo. general Dwight. Eisenhower la aplazara hasta el 6 de junio,
b) Los rumores de la muerte de Bin Laden y las afirmacio- debido a ello esta última fecha se conoce popularmente como
nes que estaba muerto o fatalmente herido durante los el pequeño Día D (en francés se llama Jour o Le Choc)
bombardeos de Estados Unidos después de los atenta-
dos del 11 septiembre. 4. El tema central es:
c) La ubicación de Osama bin Laden, una figura relevan- a) Breve explicación sobre el término Día D: La invasión
te involucrada en terrorismo internacional y una de las de Normandía
personas más buscadas por Estados Unidos y otros paí- b) Explicación sobre el desembarco de Normandía duran-
ses, es desconocida actualmente. te la Segunda Guerra Mundial
d) Osama bin Laden sufría de una enfermedad renal que le c) Los términos Día D y Hora H: su uso para señalar el día
obligaba a tener acceso a servicios médicos avanzados, po- y la hora de un gran suceso
siblemente diálisis renal. d) La planificación de operaciones a gran escala basadas
en el uso del «Día D»
3. En el texto, el término especula significa:
a) Sospecha b) Afirma 5. Según el texto podemos concluir:
c) Calcula d) Formula I. El término Día D siempre se usa para referirse al 6 de
junio de 1944, fecha en que comenzó a realizarse la lla-
mada Operación Overlord.
TEXTO II
II. La marea alta pudo ser un factor determinante para que
El término Día D (traducción del inglés D-Day) lo utilizan
se aplazara el desembarco de Normandía para el 6 de
genéricamente los militares para indicar el día en que
junio de 1944.
se debe iniciar un ataque o una operación de combate.
III. El término Día D (traducción del latín D-Day) lo utili-
Históricamente, dicho término se utiliza para referirse al
zan únicamente los militares para indicar el día en que
6 de junio de 1944, día en el que comenzó a ejecutarse la
se debe iniciar un ataque o una operación de combate.
denominada Operación Overlord. Dicha operación consistía
IV. El planeamiento de operaciones a gran escala se elabo-
en llevar a cabo el desembarco de Normandía, aunque
ra minuciosamente con mucho tiempo de antelación a
comenzó de madrugada, con el lanzamiento de miembros de
la definición de fechas específicas.
las divisiones aerotransportadas estadounidenses 101.ª y 82.ª
a) I y II
y de la 6.ª división aerotransportada británica, que trataron de
b) II y IV
neutralizar parte de la barrera defensiva alemana para facilitar
c) III y IV
el desembarco. Este día, en que las tropas aliadas se adentraron
d) I y IIII
en la costa francesa, marcó el inicio de la liberación de la
Europa occidental ocupada por la Alemania nazi durante la
Segunda Guerra Mundial.
Los términos Día D y Hora H se usan para señalar el día y la
hora en que se iniciará un ataque u operación, en circunstancias
en que el día y la hora aún no han sido determinados o es
absolutamente secreto. El Día D y la Hora H son únicos
para todas las unidades que participan en una operación
determinada.

5º AÑO DE SECUNDARIA
Mejores Personas, Mejores Familias 97
Colegio

Mejores Personas, Mejores Familias RAZONAMIENTO VERBAL

M A
TE

23   CONECTORES LÓGICOS -
ORACIONES INCOMPLETAS

I. CONECTORES LÓGICOS
De manera general, los conectores son partículas gramaticales que se emplean para unir frases u oraciones.
El discurso, el texto y cualquier otro conjunto de palabras organizadas sintáctica y semánticamente, necesitan elementos de
enlace que estructuren dichos conjuntos. Estos enlaces se denominan «conectores», que son nexos gramaticales que unen
internamente frases u oraciones.
Ejemplo:
YY El examen de las variables evolutivas en las diversas especies animales es difícil ______ son innumerables, por no decir
infinitas, ________todas ellas comparten el mismo fin; la perpetuación de las especies vivas.
a) ya que – pero
b) porque – aunque
c) por ejemplo – y
d) pues – puesto que
e) sí – pero

II. ORACIONES INCOMPLETAS


Las oraciones incompletas son ejercicios que están formados por textos breves, a los cuales se le han suprimido de manera
deliberada uno o más de sus elementos; debiendo restablecerse el sentido de la expresión, escogiendo la palabra o palabras
más adecuadas, de una relación de cinco (5) opciones.
Estructura
YY S i hay algo más fuerte que el hierro y más duradero que el granito es la _________ de un hombre _______ (ENUNCIA-
DO).
a) reputación – converso DISTRACTOR
b) conducta – testarudo DISTRACTOR
c) opinión – intolerante DISTRACTOR OPCIONES
d) palabra – honrado RESPUESTA
e) voz – fornido DISTRACTOS

3. Había dicho que no vendría otra vez, ______ un día, va-


rios años más tarde, se presentó en mi oficina, _____ de-
duje que se hallaba en problemas.
CONECTORES LÓGICOS a) más – así d) y – mientras
1. El gran sueño de todos los emperadores del Medioevo fue b) no obstante, –sino que e) pero – entonces
la reconstrucción del Imperio romano, ________, ningu- c) pues – , en efecto
no pudo lograrlo.
a) sin embargo d) por supuesto 4. Hoy en día, una nueva sociedad ha surgido en China
b) pero e) por último _____ progresa ______ pasos agigantados.
c) por lo tanto a) , sin embargo, – si bien d) porque – con
b) y – a e) y – en
2. Desde que el niño nace va sufriendo cambios __________ c) en – para
tienen como finalidad la adaptación al mundo exterior,
_________ es preciso crear un ambiente adecuado para que 5. No pudo viajar el sábado _____ al día siguiente _____ lo hizo
se desarrolle. el lunes _____ pudo conversar con él.
a) así – para– por
a) y – por así
b) ni – así – , aunque
b) que – por eso que c) tampoco –, aunque – de ese modo
c) , pero sí – ya que d) si bien – nunca – , por ejemplo,
d) y por ende – esto es e) ni – , sin embargo, – y
e) , es decir, – entonces

98 Mejores Personas, Mejores Familias


5º AÑO DE SECUNDARIA
Colegio

RAZONAMIENTO VERBAL  Mejores Personas, Mejores Familias

ORACIONES INCOMPLETAS 2. Una lluvia torrencial impidió el viaje programado


6. El principal producto del hombre es el hombre, por lo que _________ como habían sido compradas todas las provi-
todo el proceso de satisfacción de las necesidades materia- siones, se llegó al acuerdo _________ reunirse, todos, en
les de la vida se debe destinar a ________ en las mejores una casa.
condiciones y permitirle al hombre su ________ como tal. a) , sin embargo, – en d) , no obstante, – donde
a) satisfacerlas – desarrollo b) , pero – de e) mientras – para
b) supervisarlas – existencia c) para – de
c) conservarlas – reproducción
d) proveerlas – mejoramiento 3. Se puede conseguir un préstamo automático ______ el va-
e) saturarlas – extinción lor equivalente de sus acciones y bonos ____ el interés que
se recarga es bastante alto.
7. El profesional de la salud debe estar armado de los a) para – el cual d) , porque – así
________ necesarios para no interponer una ________ b) si bien – con e) , aunque – , por tanto,
entre él y el poblador del campo. c) por – , sin embargo
a) instrumentos técnicos–traba institucional
b) elementos antropológicos – barrera cultural 4. Vivió desolada _________ el nacimiento de su hijo,
c) medios sofisticados – frontera tecnológica
d) mecanismos psicológicos – terapia traumática _________ era frecuente que lo descuidara.
e) pertrechos bélicos – relación conflictiva a) más – hasta
b) sin embargo – también
8. Las sociedades en vías de desarrollo, por lo general son c) por – como también
________ carentes de industria. Por lo tanto, la actividad d) hasta – mas
________ predominante es la explotación de recursos na- e) aunque – a veces
turales.
a) Estados – comercial d) países – fabril 5. El agotamiento nervioso era desconocido hace pocos años,
b) pueblos – económica e) regiones – rural ______ hoy crece vertiginosamente.
c) naciones – básica a) inclusive d) por lo tanto
b) y e) cuando
9. Pensando en lo que le esperaba al día siguiente no pudo c) en cambio
_____ el sueño, la _____ le causó el insomnio.
a) consumar – dificultad ORACIONES INCOMPLETAS
b) conseguir – desesperación
c) tener – exaltación 6. La palabra creatividad es demasiado amplía en su ______,
d) conciliar – preocupación pues tiene también un _______ artístico.
e) lograr – exasperación a) contexto – valor
b) contenido – matiz
10. La última ________ unida a la mala campaña, determina- c) dimensión – tamaño
ron el ________ definitivo del equipo. d) vocabulario – rasgo
a) humillación – hundimiento e) léxico – variedad
b) pérdida – alejamiento
c) goleada – descenso 7. Saber contar un cuento es darse _________ para despertar
d) derrota – fracaso la atención del _________ y para acicatearla hasta que el
e) ubicación – rompimiento cuento acabe.
a) tiempo – auditorio d) importancia – grupo
b) prisa – lector e) maña – oyente
c) por entero – crítico

CONECTORES LÓGICOS 8. Sus actitudes _________ y autoritarias contradecían sus


vibrantes discuros en defensa _______.
1. El niño estaba sentado mirando pasar los automóviles a) conciliadoras – de la libertad
___________ la multitud de personas _________ transi- b) dictatoriales – de la guerra
taba, por aquel lugar, en ese día de verano caluroso y seco. c) dogmáticas – del poder del Estado
a) , porque – así d) impositivas – de la democracia
b) y – que e) agresivas – de una política armamentista
c) , pues – ni
d) así – no obstante
e) junto a – desde luego

5º AÑO DE SECUNDARIA
Mejores Personas, Mejores Familias 99
Colegio

Mejores Personas, Mejores Familias RAZONAMIENTO VERBAL

9. Aún hay científicos que defienden la vieja teoría de que las 5. Carece de seguro contra incendios, _________ la cons-
aves _________ se orientan en sus prolongados vuelos por trucción de su inmueble es sólida, _________ pensamos
ciertas fuerzas magnéticas. que debería tenerlo.
a) migratorias d) rapaces a) sin embargo – por lo tanto
b) perdidas e) en la noche b) pues – entonces
c) en peligro c) como – desgraciadamente
d) ya que – aún así
10. Los grupos humanos que no se _________ como tales, e) como – o
desaparecen e igualmente aquellos que no _______ lo que
sus hijos necesitan. ORACIONES INCOMPLETAS
a) relacionan – interaccionan
b) desarrollan – proveen 6. En esos momentos en que debían tomarse ______ inme-
c) crían – crean diatas, la sola _______ era censurable.
d) mantienen – mantienen a) reprimidas – consolación
e) reproducen – producen
b) decisiones – vacilación
c) formulaciones – indeterminación
d) determinaciones – retroceso
e) decisiones – inseguridad
CONECTORES LÓGICOS
7. Fue un factor _________, fue la gota de agua que colmó el
1. Cabalgó solo _________ el límite de la frontera, _________ _________.
sentirse solo, regresó. a) detonante – cansancio
a) por – al b) considerable – vaso
b) hasta – al c) importante – cántaro
c) desde – hasta d) detonante – vaso
d) con – sin e) culminante – pensamiento
e) sin – hasta
8. El mundo necesita _________ y no espinas; puentes y
2. _________ el espacio nocturno, el hombre primitivo tem- no_________; ternura y no rencor.
blaba, _________ monstruos que había creado su insufi- a) rosas – mares
ciencia racional. b) alegres – mares
a) En el – por los c) pétalos – caminos
b) Sin – debido a d) rosas – muros
c) Con – por e) rosas – abismos
d) Durante – hasta por
e) Sin duda – por los 9. Deslizó una mordaz ironía en la reunión, no había duda
que era _________ hasta en los ________.
3. Fue insultado, escarnecido y _______ ejecutado. a) bella – insultos
a) además b) inteligente – rencores
b) sin embargo c) sagaz – ofensas
c) por tanto d) sutil – agravios
d) en consecuencia e) vulgar – insultos
e) hasta
10. El desarrollo social y __________ de las naciones depen-
4. La producción industrial de un país _________ aumenta de en gran medida del aprovechamiento de los recursos
_________ existe un mercado amplio para comercializar __________.
sus productos. a) jurídico – éticos
a) del mismo modo – cuando b) educativo – renovables
b) solo – en tanto c) regional – explotables
c) siempre – mientras d) políticos – tradicionales
d) pocas veces – aunque e) económico – naturales
e) no siempre– aunque

100 Mejores Personas, Mejores Familias


5º AÑO DE SECUNDARIA
Colegio

RAZONAMIENTO VERBAL  Mejores Personas, Mejores Familias

M A
TE

24   LA REFERENCIA

Los elementos anafóricos y catáforicos son mecanismos gramaticales de referencia que evitan las repeticiones inncesarias y permiten la
reaparición de un referente en el texto mediante sustitutos (pronombres, adverbios, etc).

I. ANÁFORA
La anáfora se produce cuando unas palabras del texto se refieren a otras que han sido mencionadas anteriormente.

II. CATÁFORA
Es un tipo de referencia en la cual unas palabras aluden a otras que serán mencionadas posteriormente en el texto, adelantan-
do algo no dicho aún.

III. ELIPSIS
Se produce cuando se eliminan palabras porque están sobrentendidas. Se suprime la expresión, pero permanece el significa-
do. Esta es una de las formas más habituales.
Por ejemplo:
YY Vallejo viajó a París resuelto a probar fortuna. El poeta no sospechó lo que podía ocurrir. Allí encontraría la muerte.
(ANÁFORA)
YY Hay que prevenir las siguientes enfermedades contagiosas: sarampión, difteria, tifoidea.(CATÁFORA)
YY Carlos terminó de leer el libro. (...) se quedó pensativo un rato y luego (...) se durmió.(ELIPSIS)

Completa los siguientes enunciados con palabras anafóricas


referidas a los términos destacados.
� Identifica el tipo de referencia y señala la respuesta correc-
ta. 6. Esta mañana ocurrió un accidente de tránsito. Felizmen-
1. He hablado con María, pero no le he dicho que venga. te, _____ no dejó víctimas que lamentar.
a) Anáfora a) el c) este
b) Catáfora b) eso d) esto
c) Pronombre demostrativo
d) Pronombre relativo 7. Mi médico me ha aconsejado que haga deporte, sobre
todo caminatas matutinas.
2. Todos estaban en casa. Mi padre, mi madre, mis hermanos.. Creo que____ me conoce muy bien. Le haré caso.
a) Adverbio de modo a) ello b) eso
b) Catáfora c) aquel d) él
c) Pronombre demostrativo
d) Elipsis 8. El profesor de Ciencias será _________apruebe el pro-
yecto de biohuerto.
3. Llegaron varias mujeres. Todas traían pañuelos en la cabe- a) quien b) el
za. c) pues d) quienes
a) Elipsis b) Catáfora
c) Pronombre demostrativo d) Anáfora 9. Desde lejos, la cabaña parecía abandonada. A medida que
me acercaba, _________ ya me parecía familiar.
4. María estudia Química; Juan, Psicología. a) esta b) eso
a) Elipsis c) esa d) esto
b) Anáfora
c) Adverbio de función 10. En el debate se plantearon propuestas, ________se harán
d) Catáfora factibles con el apoyo del Estado.
a) ello c) ella
5. Estaba cansado de esperarle una y otra vez. Por eso me fui. b) ellas d) esto
a) Adverbio de modo b) Adverbio de cantidad
c) Catáfora d) Anáfora

5º AÑO DE SECUNDARIA
Mejores Personas, Mejores Familias 101
Colegio

Mejores Personas, Mejores Familias RAZONAMIENTO VERBAL

10. Dieron las tres. Entonces se marchó.


a) Anáfora
b) Catáfora
Identifica el tipo de referente y señala la respuesta correcta. c) Pronombre demostrativo
1. Coge tú este libro; yo me llevaré otro. d) Pronombre relativo
a) Anáfora
b) Catáfora
c) Pronombre demostrativo
d) Pronombre relativo

2. A quien yo vi es a Luis. Vuelve a escribir las siguientes oraciones evitando las repeti-
a) Adverbio de lugar b) Elipsis ciones de palabras.
c) Catáfora d) anáfora
1. Ana Frank fue una niña judía. Ana Frank escribió un dia-
3. Pablo y Alba se tutean. rio al que le puso el nombre de «Kitty».
a) Anáfora ______________________________________________
b) Catáfora _____________________________________
c) Pronombre demostrativo 2. Los barcos de vela transportaban personas y mercan-
d) Pronombre relativo cías. Los barcos de vela eran muy utilizados en la Anti-
güedad.
4. Solo somos dos: tú y yo. ______________________________________________
a) Adverbio de modo b) Catáfora ____________________________________
c) Pronombre demostrativo d) Elipsis 3. El langostino puede pesar hasta 24 gramos. El langostino ne-
cesita vivir con una temperatura media en las pozas.
5. Alfredo trajo su escopeta. Sergio la suya. ______________________________________________
a) Anáfora _____________________________________
b) Catáfora 4. El desierto piurano no es estéril del todo. En el desierto
c) Pronombre demostrativo piurano crecen algarrobos.
d) Pronombre relativo ______________________________________________
______________________________________
6. Tengo un libro que trata de Astrología. 5. Por las calles oscuras no caminaré solo. Puesto que por las
a) Anáfora calles oscuras uno se somete al peligro.
b) Catáfora ______________________________________________
c) Pronombre demostrativo ______________________________________
d) Pronombre relativo
Rodea la palabra que pueda sustituir a la expresión subraya-
7. Le di el libro a Chispita. da en cada caso.
a) Adverbio de persona 6. este – estas
b) Catáfora El puente de vigas está hecho de una manera muy sencilla.
c) Pronombre demostrativo Sin embargo, el puente de vigas solo permite salvar una
d) Elipsis distancia corta.

8. Luisa estudia y Cris hace lo mismo. 7. Los – Estos


a) Anáfora En un disco duro existen miles de archivos. Los archivos se
b) Catáfora agrupan en carpetas.
c) Pronombre demostrativo
d) Pronombre relativo 8. Acá – Allí
Abre el cajón de la cómoda. En el cajón de la cómoda en-
9. Todos estaban en casa. Allí hacía menos frío. contrarás una gran sorpresa.
a) Anáfora
b) Catáfora 9. estos – este
c) Pronombre demostrativo Una de las materias primas es el petróleo. Pero debemos con-
d) Pronombre relativo siderar que el petróleo se puede agotar.

10. la – nos
Quiso encontrar a la mujer de sus sueños, pero no pudo
hallar a la mujer de sus sueños.

102 Mejores Personas, Mejores Familias


5º AÑO DE SECUNDARIA
Colegio

RAZONAMIENTO VERBAL  Mejores Personas, Mejores Familias

M A
TE

25   INCLUSIÓN E IMPLICACIÓN

INCLUSIONES CONCEPTUALES 4. El concepto LUNA incluye o supone necesariamente:


Concepto a) Astro – menguante – radiante
El presente ejercicio consiste en procurar establecer el b) Satélite – sol – galaxia
significado del concepto (pregunta del ejercicio) c) Satélite – astro – Tierra
Ejemplo: d) Tierra – universo – luz
ZZ El concepto ODIO incluye o supone necesariamente: e) Astro – inmenso – lejano
a) Violencia – pasión
b) Agresión – guerra 5. El concepto CERVEZA incluye o supone necesariamente:
c) Locura – donosura a) Bebida – embriaguez –resaca
d) Despecho – inconstancia b) Alcohol – cebada – botella
e) Oposición – sentimiento c) Líquido – caña – fermentación
d) Alcohol – bebida – vaso
Recomendaciones resolutivas e) Bebida – cebada – fermentación

1.° Establecer el significado de la premisa.
� Marca la alternativa que por su concepto esté implicada en
2.° Examinar las alternativas para delimitar la posibilidad de
las demás.
correspondencia.
6.
3.° La respuesta correcta es la que contenga palabras
a) Sueldo b) Jornal c) Honorarios
semánticamente conexas a la premisa.
d) Emolumento e) Remuneración
7.
a) Ánimo b) Alegría c) Pesimismo
d) Optimismo e) Tristeza
8.
1. I. La palabra ESTUDIAR incluye el concepto de LEER a) Oda b) Elegía c) Soneto
II. El concepto de BEODO incluye el de LICOR. d) Poesía e) Égloga
III.Los conceptos de PERRO y CAN aluden a la misma
especie animal. 9.
Es (son) incorrecta(s): a) Rozar b) Refregar c) Palpar
a) II y III b) I; II y III c) Solo I d) Sobar e) Tocar
d) I y II e) Solo III
10.
a) Pueril b) Puericultorio c) Pediatra
2. Dados los conceptos PUENTE – COLUMNA, se puede
d) Niño e) Infantil
afirmar:
I. El concepto PUENTE puede incluir al de COLUM-
NA.
II. Ambos se complementan.
III.El concepto de COLUMNA incluye al de PUEN-
TE. IMPLICACIÓN
a) I y II b) II y III c) Solo III Uno de los siguientes conceptos está siempre implicado en los
d) I y III e) Solo I otros cuatro; señala cuál es.

3. I. El concepto ISLAM está incluido en el de TERRORIS- 1.
MO. a) Carnada b) Sedal c) Anzuelo
II. El concepto DOCENTE está incluido en el de UNI- d) Pesca e) Arpón
VERSIDAD.
III. El concepto MIOPÍA incluye al de VISTA 2.
Son correctas: a) Movimiento b) Traslación c) Viaje
a) Solo I b) I y II c) Solo II d) Transporte e) Terremoto
d) II y III e) Solo III
3.
a) Roble b) Álamo c) Ciprés
d) Sauce e) Árbol

5º AÑO DE SECUNDARIA
Mejores Personas, Mejores Familias 103
Colegio

Mejores Personas, Mejores Familias RAZONAMIENTO VERBAL

INCLUSIÓN 2. I. El concepto ALCOHÓLICO incluye al de LICOR.


4. El concepto PRAGMÁTICO incluye o supone necesaria- II. El concepto MARINO está incluido en el de PEZ.
mente: III. El concepto CHIMPÚN incluye el de DEPORTIVO.
a) Filosofía, lógica, experiencia Son correctas:
b) Juego, necesidad, logro a) Solo I b) I y II c) Solo II
c) Experiencia, conocimiento, práctica d) I y III e) Solo III
d) Agnóstico, ciencia, leyenda
e) Dogma, doctrina, teoría 3. Señala qué concepto(s) no se incluye(n) en la definición de
ABJURAR.
5. El concepto DECADENCIA incluye o supone necesaria-
I. Renuncia III. Dogma
mente:
II. Acción IV. Reino
a) Ocaso, veleidad, caída.
a) Solo IV b) I y II c) Solo III
b) Daño, necedad, maldad.
d) I, II y III e) II y III
c) Alicaimiento, lozanía, desventura.
d) Desdicha, enclaustración, detrimento.
En los siguientes ejercicios, señala los enunciados ciertos.
e) Principio, ruina, desgracia.
6. ¿Qué conceptos están incluidos necesariamente en el con- 4. I. El concepto BALONCESTO está incluido en el de DE-
cepto de LECTURA? PORTE.
1. Costumbre 4. Escritura II. TENISTA no es un concepto incluido en el de DEPOR-
2. Decodificación 5. Biblioteca TISTA.
3. Concentración III. FUTBOLISTA y BASQUETBOLISTA son conceptos
a) 1, 3, 4 y 5 b) 1, 2, 3 y 5 c) 3, 4 y 5 que incluyen al de DEPORTISTA.
d) 1 y 5 e) 2 y 4 a) I y II b) I y III c) II y III
d) Todos e) III
7. Dados los conceptos, EDUCACIÓN–APRENDIZAJE, es
correcto afirmar que _____.
5. I. El concepto BICICLETA no está incluido en el de VE-
I. APRENDIZAJE requiere necesariamente de EDUCA-
HÍCULO.
CIÓN
II. VEHÍCULO es un concepto de mayor generalidad que
II.La EDUCACIÓN supone, en su propia intencionalidad,
AUTOMÓVIL.
el concepto de APRENDIZAJE
III. YATE es un concepto de menor generalidad que BU-
III. El APRENDIZAJE, aún siendo menos intencional que la
QUE.
EDUCACIÓN, es más vasto que esta
a) I y II b) I y III c) II y III
IV. El APRENDIZAJE y la EDUCACIÓN son conceptos
d) Todos e) Solo III
excluyentes
a) Solo II b) I y II c) Solo III
6. I. El concepto BEBIDA no está incluido en el de LICOR.
d) II y III e) Solo I
II. LICOR es un concepto que presupone BEBIDA.
8. ¿Qué palabras incluye necesariamente INGENIO? III. Tanto SIDRA como CERVEZA son conceptos incluidos
a) Agudeza, suspicacia en el de LICOR.
b) Habilidad, inteligencia a) I y II b) I y III c) II y III
c) Precocidad, experiencia d) Solo II e) Solo III
d) Actitud, rapidez
e) Éxito, fortaleza IMPLICACIÓN
Marca la opción que por su concepto implique a las demás.
9. ¿Qué palabras incluye necesariamente CONDENA?
a) Juicio, sentencia, culpa 7.
b) Perdón, indulto, sanción a) Meteoro b) Huracán c) Lluvia
c) Falta, defensa, fallo d) Rayo e) Nieve
d) Pena, juzgado, veredicto 8.
e) Cárcel, reo, arrepentimiento a) Penitencia b) Expiación c) Excomunión
10. Señala el concepto que incluye a los demás en su defini- d) Escarmiento e) Castigo
ción:
a) Deidad b) Tritón c) Marina 9.
d) Pez e) Hombre a) Frejol b) Legumbre c) Haba
d) Garbanzo e) Pallar

10.
a) Sismo b) Movimiento c) Terrenal
INCLUSIÓN d) Suave e) Telúrico
1. ¿Cuál de las siguientes palabras incluye a las demás en su
definición?
a) Persona b) Dictado c) Escribir
d) oficio e) Amanuense

104 Mejores Personas, Mejores Familias


5º AÑO DE SECUNDARIA
Colegio

RAZONAMIENTO VERBAL  Mejores Personas, Mejores Familias

M A
TE

26   PLAN DE REDACCIÓN II

I. DEFINICIÓN III. CRITERIOS DE ORGANIZACIÓN LÓGICO-DEDUCTI-


Se llama plan de redacción al esquema lógico-deductivo VOS
conformado por un conjunto de conceptos propuestos so-
bre un determinado tema que se va a desarrollar. 1. Criterios de lo general a lo particular

Ejemplo N.º 1 Ir de la idea más A la idea más
La Inquisición y los libros general específica o
(I) «El Secreto»; así llamaban al lugar donde iban a parar particular
los libros que prohibía la Santa Inquisición en el Perú
virreinal. 2. Criterio cronológico
(II) Los inquisidores habían destinado el lugar más seguro de
esta prisión a los novelistas, herejes con un poder capaz Partir de los hechos A los más actuales
de desestabilizar el régimen. más remotos o recientes
(III) Esos objetos, sin embargo, no eran los principales in-
quilinos de esta mazmorra.
(IV) En sus oscuros ambientes se podían encontrar las «In-
IV. CONSIDERACIONES PARTICULARES
dulgencias Papales» de Martín Lutero, algún opúsculo
I) La forma de organizar las ideas dentro de cada uno
de Juan Calvino y por supuesto el «Diálogo» de Galileo
de los niveles dependerá del tipo de tema plantea-
Galilei.
do en el título. Por lo tanto, es imperativo leer con
(V) De ahí que, durante 300 años, los españoles prohibie- atención el título del ejercicio, en razón a que de
ron la importación y la publicación de novelas por él depende la organización de los enunciados pro-
considerar que «sus argumentos disparatados y absur- puestos.
dos podían ser perjudiciales para la salud espiritual II) Los ejercicios con títulos simples casi siempre se ini-
indígena». cian con la definición del objeto o del hecho. En cam-
bio, los ejercicios con títulos compuestos comienzan
citando generalmente antecedentes vinculantes o una
I, II, III, IV, V → Plan de redacción idea general que sirve para presentar el asunto.

II. ESTRUCTURA DE UN EJERCICIO DE PLAN DE RE-


DACCIÓN

Ejemplo N.º 2 Indica el orden lógico.


La teoría copérnica → TÍTULO 1. EL TEMOR
(I) La simplicidad fue lo que determinó que la teoría de Co- I. Pero la reacción del temor falta cuando se destruyen las
pérnico reemplazara a la de Ptolomeo. amígdalas.
(II) Aristarco de Samos es un antecesor de Copérnico. II. La reacción del temor y miedo puede ser reproducida
(III) Las relaciones establecidas ordenaron un nuevo modo en animales por estimulación del hipotálamo y de los
de pensar que influyó en varios ámbitos del saber. núcleos amigdaloideos.
(IV) Aristarco consideró que el Sol, al ser más grande, no III. Un ejemplo espectacular es la reacción de los monos
podía girar alrededor de la Tierra. ante las serpientes.
(V) La simplicidad se refería no solo al menor número de IV. Pero, después de una lobotomía bilateral temporal, se
hipótesis, sino a las relaciones establecidas entre ellas. acercan a ellas sin temor, las levantan e incluso se las
comen.
I, II, III, IV, V → son enunciados o subtemas
V. Los monos, normalmente, se aterrorizan frente a las
I – II – IV – III – V serpientes.
IV – I – III – V – II a) III – V – IV – I – II
IV – III – V – I – II Distractores b) II – I – III – V – IV
II – V – I – III – IV c) II – I – III – IV – V
II – IV – I – V – III orden correcto d) II – III – V – I – IV

5º AÑO DE SECUNDARIA
Mejores Personas, Mejores Familias 105
Colegio

Mejores Personas, Mejores Familias RAZONAMIENTO VERBAL

2. EL TACTO 6. HACIA UN NUEVO CONCEPTO DEL HOMBRE


I. La concentración más abundante la encontramos en la I. Si el hombre fuera racional, no existirían horrores como
punta de la lengua que es particularmente rica en estos la guerra, la injusticia, etc.
receptores. II. Hasta nuestros días, se ha definido al hombre como un
II. En el hombre, el tacto es captado por receptores locali- «animal racional».
zados cerca de la superficie de la piel. III. Parece que el concepto de «animal racional» ya no sirve
III. En segundo lugar la encontramos en la piel de las yemas y hay que buscar otro.
de los dedos que puede contener unos 100 receptores IV. Pero si no es racional, se explican estas miserias.
por centímetro cuadrado. V. Sería bueno preguntarse: ¿Es el hombre verdaderamente
IV. La concentración de receptores en otras regiones tiende racional?
a disminuir. a) I, II, V, III, IV b) III, V, IV, I, II
V. Los receptores del tacto no están distribuidos uniforme- c) II, V, I, IV, III d) V, IV, III, II, I
mente sobre la superficie de su cuerpo. e) IV, V, I, II, III
a) II – V – I – III – IV b) II – III – I – IV – V
c) II – V – III – IV – I d) II – I – III – V – IV 7. LA TEORÍA DE BERNE
I. La personalidad del ser humano estaría dividida en tres
3. LA FONÉTICA partes: Padre, Adulto y Niño.
I. Tiene un aspecto acústico y otro articulatorio o fisioló- II. Es el primer psiquiatra canadiense que obtiene renom-
gico. bre universal.
II. Es el estudio de los sonidos del lenguaje. III. Esta división la estableció después de un cuidadoso es-
III. Por lo tanto, es una rama de la lingüística que solo se tudio de sus pacientes.
interesa por el lenguaje. IV. Eric Berne fue un psiquiatra canadiense que planteó
IV. El segundo se ocupa de nuestro aparato fonador y de una teoría revolucionaria.
la manera mediante la cual producimos los sonidos del V. El Padre, es el concepto aprendido de la vida; el Adulto,
lenguaje. la capacidad reflexiva; y el Niño, el concepto de la vida.
V. El primero estudia la estructura física de los sonidos a) I, V, III, IV, II b) IV, I, V, III, II
utilizados y la manera en que el oído reacciona ante los c) I, III, V, IV, II d) IV, II, III, V, I
sonidos.
a) I – II – III – V – IV b) II – I – II – V – IV 8. LA SEÑAL CIENTÍFICA
c) II – III – I – V – IV d) II – I – V – IV – III I. Las metáforas inducen a confusión, por lo que rara vez
deben utilizarse al redactar artículos de investigación.
II. La redacción científica no tiene necesidad de adornos
4. LA GERMINACIÓN
ni cabida para ellos.
I. Para que pueda ocurrir se debe disponer de una tempe- III. Las palabras deben ser tan claras, sencillas y ordenadas
ratura y humedad adecuadas. como sea posible.
II. Las tres condiciones mencionadas anteriormente deben IV. La redacción científica es la transmisión de una señal
llevarse a cabo con algún grado de eficiencia. clara al redactor.
III. La germinación consiste en el restablecimiento del cre- a) IV – III – II – I b) I – III – II – IV
cimiento del embrión al interior de la semilla. c) IV – II – III – I d) III – IV – I – II
IV. Así como también de un suministro apropiado de oxí-
geno. 9. AUSTRALIA
V. Por último, también es importante considerar que en I. La historia de Australia es la historia de un aislamiento,
algunos casos la exposición a la luz puede ser también siempre ha estado demasiado lejos, separada por una
una condición necesaria para que se produzca la ger- inmensidad de agua.
minación. II. El aislamiento físico se ha visto potenciado además por
a) III – II – I – IV – V b) III – V – II – IV – I el aislamiento psicológico.
c) III – I – IV – V – II d) I – III – IV – II – V III. Porque más allá de Australia y Nueva Zelanda solo
queda un océano vacío.
5. EL QUIJOTE IV. Australia, colonia británica hasta 1901, ha tenido siem-
pre vocación europea y alma inglesa.
I. Por lo menos, es la mejor novela de la lengua española.
V. Aunque su posición geográfica corresponde al Pacífico
II. Su protagonista es, en verdad, un antihéroe.
asiático, nunca ha querido relacionarse con sus vecinos
III. Cuenta las aventuras de un caballero andante. naturales, hasta que Gough Whitlam acabó con la po-
IV. Con eso, Cervantes logró una parodia burlesca de las lítica blanca.
novelas de caballería. a) I – II – III – V – IV b) I – III – II – IV – V
V. Esta obra de Cervantes es una de las mejores del mundo. c) V – I – II – III – IV d) I – II – III – IV – V
a) V, III, I, II, IV b) V, I, IV, III, II
c) V, III, II, IV, I d) V, I, III, II, IV

106 Mejores Personas, Mejores Familias


5º AÑO DE SECUNDARIA
Colegio

RAZONAMIENTO VERBAL  Mejores Personas, Mejores Familias

10. FORMACIÓN DE LAS FAMILIAS LINGÜÍSTICAS a) 1, 3, 4, 5, 2 c) 1, 3, 4, 2, 5


I. División de las dos lenguas formadas en variantes dia- b) 4, 1, 3, 2, 5 d) 5, 4, 1, 3, 2
lectales.
II. Constitución de todas las ramas o dialectos en familias 4. VENTISQUERO
de lenguas. 1. El agua, al congelarse en las grietas de las rocas, aumenta
III. Comunidad lingüística con una lengua inicial. de volumen y forma pequeñas agujas de hielo que actúan
IV. Acrecentamiento de las diferencias entre los dos dialec- como cuñas desintegrando la piedra.
tos hasta construir lenguas. 2. El ventisquero es un fenómeno típico de la erosión gla-
V. Fragmentación de la comunidad en dos grupos con sus ciar, que, mediante gliptogénesis, modela el terreno; el
respectivos dialectos. agua tiene mucho que ver.
a) III – V – IV – I – II 3. Oquedad cóncava que produce la acumulación de nieve
b) IV – III – V – I – II y hielo en las cumbres de las montañas, especialmente en
c) II – IV – III – I – V aquellas de composición caliza.
d) II – IV – III – V – I 4. Se produce siempre en las cotas más altas de las monta-
ñas, donde la nieve es arrastrada por el viento y recogida
por accidentes topográficos como cumbres o picachos.
a) 1, 2, 3, 4 b) 3, 1, 4, 2
c) 3, 2, 1, 4 d) 3, 1, 2, 4

1. CAUSAS Y EFECTOS DE LA ÚLTIMA INUNDACIÓN 5. VENTRILOQUIA


1. Cuando comenzó el deshielo, el agua formó torren- 1. Arte de hablar sin mover los labios de modo que una
tes. vez proyectada la voz, parece originarse en otro lugar
2. Son numerosos los damnificados. con la finalidad de dar voz ilusoriamente a un muñeco.
3. El año pasado fue muy lluvioso. 2. Su nombre proviene del latín venter (‘estómago’) y oqui
4. El agua arrastró todo a su paso: casas, animales, vehí- (‘hablar’), ya que antes se pensaba que era resultado de un
culos, etc. uso poco corriente del estómago.
5. Se acumuló nieve en la cordillera. 3. Esta destreza, de la que ya se tiene noticia en antiguos ritos
a) 3, 5, 1, 4, 2 b) 5, 3, 1, 4, 2 religiosos, conlleva modificar la voz a través de una exha-
c) 3, 1, 4, 2, 5 d) 3, 5, 4, 2, 1 lación lenta mientras se habla, minimizando el movimien-
to de la lengua y los labios, y manteniendo inmóviles los
2. LAS MUSAS EN LOS POETAS GRIEGOS músculos faciales de modo que la atención del observador
1. La invocación de la musa en el Renacimiento se convir- se centre sobre la fuente de sonido ilusoria.
tió luego en un puro adorno retórico. 4. Por ejemplo, cuando el ventrílocuo habla con el muñeco
2. La música, la poesía, la escritura, la elocuencia y la filo- alterna entre su voz proyectada (artificial) –o sea, la voz del
sofía eran dones de las musas para los antiguos griegos. muñeco– y su voz normal.
3. El poder divino e importancia de las musas en la antigua 5. Se utiliza una combinación de presión en las cuerdas
Grecia. vocales y los músculos de la garganta para modificar el
4. Grandes poetas griegos como Homero, Hesíodo y Pín- sonido de la voz del ventrílocuo.
daro, al comenzar sus poemas, solían invocar a las mu- a) 1, 2, 3, 4, 5 b) 2, 1, 4, 5, 2
sas. c) 5, 4, 3, 2, 1 d) 2, 1, 5, 3, 4
5. Existen diversas formas de invocación de las musas.
a) 5, 3, 2, 1, 4 b) 5, 2, 3, 4, 1 6. LOS HOMBRES DE BIEN
c) 3, 2, 5, 4, 1 d) 4, 2, 3, 5, 1 I. Otra de sus virtudes consiste en formar una corriente
moral.
3. LAS CATARATAS DEL NIÁGARA II. Los hombres de bien prefieren el beneficio social al per-
1. Constituyen una de las maravillas naturales del univer- sonal.
so. III. Por lo general, son creadores de una forma admirable
2. El poder energético de las cataratas es utilizado como de conducta.
fuente de energía hidroeléctrica. IV. A su vez, quienes siguen dicha corrientes, educan a sus
3. Es uno de los centros turísticos más concurridos y apre- semejantes.
ciados del planeta. V. Incluso, son ideólogos no declarados de una actitud
4. El volumen del agua, la altura de la caída y su ancho perfeccionista.
la convierten en una de las cataratas más grandes del a) III – I – II – V – IV b) II – III – V – I – IV
mundo. c) II – III – V – IV – I d) IV – II – I – V – III
5. Abarca territorios de EE.UU. y Canadá y nace en los
grandes lagos.

5º AÑO DE SECUNDARIA
Mejores Personas, Mejores Familias 107
Colegio

Mejores Personas, Mejores Familias RAZONAMIENTO VERBAL

7. EL PERRO HEROICO
I. En su aliento final, veía los ojos húmedos y agradecidos
de su amo.
II. En tiempo futuro tendría la ocasión de mostrar gratitud 1. LA PACIENTE Y PROVECHOSA OVEJA
a su amo. 1. Un animal domesticado desde la más remota prehisto-
III. A sus tres años tuvo un accidente y raudo fue llevado al ria, junto con la cabra.
veterinario. 2. Gran desarrollo de la ganadería ovina en la Edad Media
IV. Habían ingresado a casa dos ladrones que atacaban a su europea, con el correspondiente aumento de la produc-
amo. ción de tejidos de lana.
V. Fue acuchillado mortalmente en la defensa de su amo; 3. Rumiante arciodáctilo (pezuña hendida) pariente de los
quedó tendido. bovinos (bueyes).
a) V – IV – III – I – II b) III – II – IV – V – I 4. Avanzada tecnología moderna para mejorar las razas
c) III – II – IV – I – V d) V – I – IV – II – III productoras de lana, carne y leche.
5. Orígenes: las mesetas de Anatolia y las estepas del Asia
8. LOS HERALDOS DE LA MUERTE Central.
I. Entre ellos, la voz del instinto le decía, «algo malo está a) 1 – 5 – 3 – 2 – 4 b) 5 – 1 – 3 – 2 – 4
pasando». c) 1 – 2 – 3 – 4 – 5 d) 3 – 5 – 1 – 2 – 4
II. Un dolor en el pecho fue el primer heraldo que le envió
la muerte. 2. EL NACIMIENTO DE LA QUÍMICA MODERNA
III. Se desmayó un día en plena calle, pero tampoco le dio 1. Desde la antigüedad se obtienen, mediante la práctica,
importancia. muchas sustancias naturales.
2. Lavoisier, francés del siglo XVIII, funda la química mo-
IV. También los huesos le causaron dolor y creyó que era
derna cuando comienza a pesar las sustancias que com-
solo el frío.
bina.
V. Mas irresponsablemente, la desoyó y finalmente murió.
3. Prácticos y alquimistas no llegan a explicarse el porqué
a) V – IV – II – I – III b) V – IV – III – I – II de los fenómenos ni a formular leyes generales.
c) II – IV – III – I – V d) II – III – IV – I – V 4. Dos orígenes: a) el trabajo práctico, y b) las teorías mez-
cladas de magia de los alquimistas.
9. LOS ÚLTIMOS CIENTÍFICOS 5. La manipulación de materias por alquimistas medie-
I. No necesitan, por ello, consideraciones que los haga vales en busca de oro, produce en ocasiones descubri-
sentir superiores. mientos valiosos.
II. Hombres de espíritu altruista dedicados a la búsqueda a) 5 – 3 – 2 – 4 – 1 b) 4 – 1 – 5 – 3 – 2.
de la verdad. c) 4 – 2 – 1 – 5 – 3 d) 1 – 2 – 3 – 4 – 5
III. Con tales cualidades se desarrollan los científicos de
3. EL TOMATE
esta generación.
1. Fruto originario del Perú, México y Ecuador.
IV. Sus actitudes están reñidas con la hegemonía social y 2. Introducción en Europa a mediados del siglo XVI.
cultural. 3. Atribución de virtudes mágicas, afrodisiacas y medici-
V. La única superioridad, dicen, es la del amor a la ciencia nales por parte de los conquistadores europeos.
y la verdad. 4. Utilizado en Italia como planta ornamental, con el nombre
a) III – II – IV – V – I b) III – II – I – V – IV de pommodoro (manzana de oro).
c) V – IV – II – I – III d) II – III – IV – I – V 5. En Europa es considerado fruto comestible solo a partir
del siglo XVIII.
10. LA RELIGIÓN CRISTIANA a) 2 – 4 – 3 – 5 – 1 b) 3 – 2 – 1 – 5 – 4
I. Claro que inicialmente luchó no solo ideológicamente c) 2 – 1 – 3 – 4 – 5 d) 1 – 3 – 2 – 4 – 5
contra el politeísmo pagano.
4. UNA BACTERIA CAMBIA DE ACTITUD
II. El cristianismo se constituyó en la esperanza de una
1. A un ratón con tumores, le es introducida la bacteria;
vida justa para los desamparados.
esta los busca y al multiplicarse disminuye el crecimien-
III. Su fundamento doctrinario descansa en el principio de to del tumor.
igualdad entre los hombres. 2. Los científicos aseguran que la bacteria Salmonella ha
IV. El mismo que se resistía a que se admita la existencia de enmendado su rumbo.
un solo Dios. 3. Luego de manejar su cadena genética, han descubierto
V. Y tal vez ese haya sido uno de sus mayores aportes a la sus nuevos objetivos.
humanidad. 4. Aunque no se conocen los mecanismos que utiliza.
a) III – II – IV – V – I b) II – III – V – I – IV 5. Ahora ha sido ocupada para curar el melanoma y cán-
c) II – III – IV – I – V d) III – II – V – I – IV ceres diversos.

108 Mejores Personas, Mejores Familias


5º AÑO DE SECUNDARIA
Colegio

RAZONAMIENTO VERBAL  Mejores Personas, Mejores Familias

a) 1 – 2 – 3 – 4 – 5 b) 2 – 1 – 3 – 4 – 5 8. DEL TEMOR AL PÁNICO


c) 2 – 3 – 5 – 1 – 4 d) 2 – 3 – 1 – 4 – 5 I. Una profusa cantidad, entonces, de ella recorre por sus
arterias.
5. EXCESO DE EJERCICIO II. La inseguridad ante una situación repentina genera en
1. Un punto insólito de la moda es estar en forma. la mente una ligera ansiedad.
2. Los síntomas de la adicción son varios. III. Así, los músculos faciales se contraen, los ojos parecen
3. Ello redunda en la práctica compulsiva del ejercicio fí- crecer y la voz se alista para gritar.
sico. IV. A causa de ella el cerebro estimula en las glándulas supra-
4. Deportes realizados en solitario, exigencia inmoderada, rrenales la producción de adrenalina.
etc. V. Los gritos son los precedentes de la acción desesperada
5. Esto trae como consecuencia múltiples perjuicios a la e incontrolable llamada «pánico».
salud, producto de la alta autoexigencia física. a) II – IV – I – III – V b) I – II – IV – V – III
a) 5 – 4 – 3 – 2 – 1 c) 5 – 3 – 2 – 4 – 1 c) V – I – II – IV – III d) II – IV – III – I – V
b) 1 – 3 – 2 – 4 – 5 d) 1 – 5 – 4 – 2 – 3
9. LA FIGURA FEMENINA
6. LA INQUISICIÓN I. La idea de una silueta perfecta es vital en la mente de
1. Sin embargo, tal exterminio no terminó con la herejía muchas mujeres.
pues ella vive en la conciencia social de la cual aprenden II. Lo aprenden en la pubertad como medio eficaz de atraer
las nuevas generaciones. al varón.
2. Asimismo deberían de ser eliminados sus simpatizan- III. Inclusive tal castigo las lleva al extremo de la letal ano-
tes. rexia.
3. Exterminar a estos es imposible si no son aniquilados a IV. Por tal motivo, van a gimnasios y se pasan horas for-
la vez que sus encubridores. mando su silueta.
4. Pero no es posible acabar con ellas si no se elimina pri- V. Se autoflagelan, además, con el látigo de la inapetencia.
mero a los herejes. a) II – I – III – IV – V b) II – I – III – V – IV
5. La Inquisición tiene por objeto destruir doctrinas que c) I – V – IV – II – III d) I – II – IV – V – III
ponían en peligro a la fe católica y que eran conocidas
como herejías. 10. LOS PERGAMINOS DEL MAR MUERTO
a) V – IV – III – II – I b) III – IV – V – II – I 1. Una serie de vasijas de barro muy antiguas en el interior
c) III – IV – I – II – V d) V – IV – III – I – II de la cueva.
2. Dentro de las vasijas, varios rollos de pergaminos ma-
7. EL CIRCO nuscritos.
I. Las luces se apagaron y un silencio absoluto inundó el 3. Joven beduino, descubridor casual de la cueva.
interior del circo, provocando susto y llanto en uno de 4. Su ubicación, al sur de Jericó, cerca del mar Muerto.
los párvulos. 5. Otra visión sobre los conceptos religiosos del Antiguo
II. ¡Señoras y señores...! Anunció un hombre alopécico Testamento.
bajo un cono de luz que caía sobre su chaleco rojizo de a) 4 – 3 – 2 – 1 – 5 b) 1 – 2 – 3 – 5 – 4
bordes dorados. c) 5 – 3 – 1 – 2 – 4 d) 3 – 4 – 1 – 2 – 5
III. Los inquietos espectadores, cuyas voces emulaban el
zumbido de un enjambre, esperaban el inicio de la fun-
ción.
IV. Tal vez no haya sido la oscuridad lo que asustara a la
criatura sino el barritar del viejo elefante o quizá el ru-
gido del león desdentado.
V. Entre ellos, una infinitud de globos se agitaban con
acompasados movimientos en las manos infantiles.
a) I – III – IV – V – II b) III – II – I – IV – V
c) III – V – I – IV – II d) II – I – IV – V – III
e) II – I – III – V – IV

5º AÑO DE SECUNDARIA
Mejores Personas, Mejores Familias 109
Colegio

Mejores Personas, Mejores Familias RAZONAMIENTO VERBAL

M A
TE

27   ELIMINACIÓN DE ORACIONES II

ELIMINACIÓN DE ORACIONES posibilidad. El chofer logró alcanzarla y, con


(oraciones eliminadas, supresión de oraciones, información ayuda de la policía y serenos de Ate, pudo re-
prescindible) clamarle por el hecho.
d) De acuerdo con ciertos programas de televi-
1. Definición sión, la boxeadora peruana ha protagonizado
Ejercicios de Aptitud Verbal en los que se evalúa la capacidad diferentes incidentes similares.
del alumno para reconocer el tema y el modo adecuado de
desarrollarlo en cuanto "a" su forma de discurso, generalidad YY Con númeración
u otro criterios para eliminar una información (dada en una (Eventualmente, estas preguntas pueden evaluar el que
oración) que sea innecesaria para la correcta comprensión de no se elimine ninguna oración, aunque la casuística de
un fragmento (dado en una serie de oraciones) o agregar al- exámenes de admisión en nuestras principales univer-
guna información a la misma. sidades no haya presentado esa posibilidad).
I. Las acllas permanecían solteras mientras estaban
2. Criterios para eliminación o supresión de oraciones cautivas.
a) Inatingencia o impertinencia: Según este criterio, debe-
II. En los acllahuasis, había mujeres escogidas de to-
mos eliminar la oración que aporte una información aje-
das las clases sociales.
na o contradictoria con el tema tratado en el fragmento.
III. Las acllas eran mujeres cautivas que vivían en los
b) Redundancia: Según este criterio, debemos eliminar
acllahuasis dedicadas a la producción textil o al
la oración que reitera una idea mejor expresada en
arte.
otra.
IV. Las acllas no solo prevenían del pueblo, sino tam-
bién de la nobleza.
Observaciones
1. El orden de las oraciones no implica mayor o menor V. En los acllahuasis, la producción era cooperativa y
importancia, excepto cuando la información de dos especializada.
de ellas sea idéntica; en este caso se elimina la segunda a) I b) IV c) III
oración dentro del orden mostrado. d) V e) II
2. Es recomendable que luego de leer el fragmento este
sea titulado, para la mejor aplicación de los criterios
de eliminación.
3. En el caso de redundancia se puede observar las si-
guiente variantes: Marca el fragmento que se pueda suprimir sin afectar el sentido
a) Cuando el contenido de dos oraciones son total- del texto que comienza del siguiente modo:
mente equivalentes se elimina siempre la segunda.
b) Cuando una de dos oraciones repite un dato, pero a 1. «Desde que en 1886 un veterano farmacéutico de 54 años
la vez da mayor información, se elimina la que tiene llamado John Pemberton creara la fórmula de la Coca-Cola,
menos información. una leyenda negra intenta empañar el éxito de este popular
c) Cuando una oración hace un breve resumen de refresco».
las otras, se elimina. a) Se dice que si se pone a remojo un bistec en Coca-Cola,
desaparecerá en dos días, y que en una sola hora el conte-
3. Modelos de pregunta
nido de una lata es capaz de acabar con los gérmenes de un
YY Con alternativas
inodoro.
Marca el fragmento que se pueda suprimir sin afectar
el sentido del texto: b) Es cierto que contiene algunos ingredientes perjudiciales
a) La campeona mundial de boxeo, Kina Malparti- para el organismo: edulcorantes, conservantes, colorantes
da, volvió a verse involucrada en un cuestiona- y aditivos, como el ácido fosfórico, que daña los huesos.
ble hecho a bordo de su vehículo. c) Además, la combinación de este ácido con azúcar refinada
b) Según Ricardo de la Cruz, la boxeadora intentó y fructuosa dificulta la absorción de hierro, lo que puede
darse a la fuga luego del impacto. generar anemia y mayor facilidad para contraer infeccio-
c) Malpartida habría intentado escapar por la Ca- nes.
rretera Central, pero el tráfico le impidió esta

110 Mejores Personas, Mejores Familias


5º AÑO DE SECUNDARIA
Colegio

RAZONAMIENTO VERBAL  Mejores Personas, Mejores Familias

d) Contiene también cafeína y es un estimulante del sistema 6. I) La risa sarcástica es propia de las personas burlonas. II)
nervioso, alivia las jaquecas, cura las afecciones nerviosas La sonrisa parece ser una expresión innata en los seres hu-
y facilita la digestión. manos. III) Los niños sonríen durante las primeras sema-
nas de vida. IV) La sonrisa evoluciona y llega a ser la risa.
2. (I) Los rumores son afirmaciones que circulan de persona a V) Hay diferencias en las sonrisas de las personas.
persona, y que se transmiten como si fueran verdaderos aun- a) I b) III c) V
que no estén verificados. (II) Se requieren dos condiciones d) II e) IV
básicas para el surgimiento y transmisión de un rumor. (III) 7. I) Los mochicas fueron eximios agricultores. II) Los mo-
En primer lugar, el rumor se debe referir a un asunto sobre chicas fueron eminentemente guerreros. III) Tuvieron una
el que exista cierta ambigüedad. (IV) Uno de los fenómenos organización social vertical. IV) Se constituyeron como es-
más interesantes estudiados por la psicología social es el de la tados independientes. V) Fueron liderados por una aristocra-
propagación del rumor. (V) En segundo término, el rumor cia militar.
debe tratar de un tema importante o de interés general. a) V b) III c) I
a) I b) II c) III d) II e) IV
d) V e) IV
8. I) El arte prehistórico tiene ante todo una finalidad má-
3. (I) El francés George Mélies, primer mago del cine, rea- gica. II) Durante el periodo Neolítico, adquiere gran im-
lizó Viaje a través de lo imposible (1904) y Viaje a la Luna portancia el culto a los muertos. III) Con la representa-
(1902). (II) Después de Mélies hubo pocos cineastas que ción plástica de animales, creían posible que la caza fuese
pudieron evitar el viaje como tema de sus películas. (III) abundante. IV) Su manifestación más representativa es el
La saga de Indiana Jones es un máximo exponente de los arte rupestre. V) También aparecen estatuillas femeninas,
viajes de aventuras. (IV) Un viaje por una vida es el que donde se ha querido ver el símbolo de la fertilidad y de la
realizó Orson Welles en su película El ciudadano Kanc, fil- vida.
mada en 1940. (V) 2001, Odisea del espacio es un viaje a a) II b) IV c) I
través de la historia, del espacio y del alma humana. d) V e) III
a) I b) II c) V
d) III e) IV
9. (I) Es importante la función fiscalizadora del Parlamento
4. (I) Los efectos de edición son todos los efectos que se rea- sobre la corrupción de los últimos años en el país. (II) Es
lizan con posterioridad a la filmación. (II) En Hombre lobo un tema prioritario del Parlamento actual a diferencia del
americano, el pelo que le crecía a M. Fox fue realizado con que funcionó en la última década. (III) Será fundamental
efectos. (III) Se le colocaba una pechera sobre la que había el aporte de las comisiones investigadoras referidas a de-
sido cosido pelo por pelo. (IV) Cada uno de estos pelos se litos en agravio del Estado durante esa época. (IV) Este
unían al resto por atrás y con hilo se tiraba. (V) Los pelos Parlamento debe señalar medidas expresas para que los
se metían adentro del pecho y luego se proyectaba la pelí- delitos perpetrados no se repitan jamás. (V) El pleno del
cula en reversa. Congreso ha creado la Unidad de Inteligencia Financiera
a) III b) IV c) I como parte de la lucha contra el narcotráfico.
d) II e) V a) IV b) V c) III
d) II e) I
5. (I) La Angelología es una nueva disciplina que inten-
ta explicar la existencia de los ángeles. (II) En la Biblia 10. (I) El aumento del volumen de un cuerpo se denomina
existen diversos ejemplos del tipo de poderes que se les «dilatación cúbica». (II) La dilatación cúbica se eviden-
atribuye a los ángeles. (III) Sophy Burnham en El Libro cia por medio del anillo de Gravesande. (III) El aparato
de los Ángeles sostiene que estas criaturas se remontan consta de una esfera de cobre que, estando fría, pasa
a miles de años antes de Cristo. (IV) El 74% de la po- exactamente por un anillo del mismo metal (IV) Cuan-
blación adolescente de EE.UU. cree en los ángeles. (V) do se calienta la esfera ya no puede pasar por el anillo al
Bajo un enfoque u otro el hombre debe ser consciente haber aumentado de volumen. (V) Solo cuando se tiene
de que solo él es el constructor de su propio destino y no en cuenta la dilatación del recipiente se puede hablar de
esperar ángeles salvadores. una dilatación verdadera de un líquido.
a) I b) IV c) V a) IV b) II c) V
d) II e) III d) I e) III

5º AÑO DE SECUNDARIA
Mejores Personas, Mejores Familias 111
Colegio

Mejores Personas, Mejores Familias RAZONAMIENTO VERBAL

3. «El Romanticismo es un movimiento cultural y político


originado en Alemania y en el Reino Unido» a finales del
siglo XVIII.
Marca el fragmento que se pueda suprimir sin afectar el senti- a) Su característica fundamental es la ruptura con la tradi-
do del texto que comienza del siguiente modo: ción clasicista basada en un conjunto de reglas estereo-
tipadas.
1. «La última apuesta de la empresa Runtastic, conocida por b) La libertad auténtica es una búsqueda constante de la
sus aplicaciones para corredores, es la báscula Libra» Humanidad.
a) El salir a «rodar» suave es una actividad que nos va a c) Debido a que el Romanticismo es una manera de sentir
permitir disfrutar más de la carrera: podemos disfrutar y concebir la naturaleza, la vida y al hombre mismo que
del paisaje que cuando vamos haciendo series no nos se presenta de manera distinta y particular en cada país
damos cuenta. donde se desarrolla
b) La báscula Libra, similar en su apariencia a una báscula d) Incluso dentro de una misma nación se manifiestan
doméstica, no solo registra información sobre nuestro distintas tendencias proyectándose también en todas
peso, sino también su composición de grasa, músculo, las artes.
agua y tejidos.
c) El aparato tiene en cuenta, además, la hora a la que nos 4. «La castidad no es lo opuesto a la sexualidad».
pesamos, para limitar las variaciones derivadas de la in- a) La castidad pertenece a la fenomenología del sexo, o del
gesta de alimentos o la retención de líquidos. erotismo para ser más exactos, y por eso requiere una
d) Tras pesarnos, solo habremos de conectar la báscula a comprensión lo más profunda posible de esa dimensión
nuestro iPhone o iPad a través de Bluetooth. tan poderosa y maravillosa de la vida.
e) La aplicación Libra realizará entonces una estimación, b) Además, la sexualidad es una estructura, un todo muy
según la edad, sexo y estilo de vida, de la cantidad máxi- complejo, es cierto, y sin embargo armonioso y esen-
ma de calorías que debemos ingerir para mantenernos cialmente positivo.
en forma. c) Creo que es imposible acceder a un alto nivel erótico
creando contradicciones o antagonismos entre sus di-
2. «Para detectar anemia ya no será necesario someterse a un versos aspectos.
análisis de sangre». d) La castidad es sexo, es una actitud sexual que depende
a) Un nuevo método de medición desarrollado por la Uni- de un alto nivel de comprensión, y no, creando antago-
versidad de Burgos (España) permitirá conocer los ni- nismos o contradicciones entre sus diferentes elemen-
veles de hierro presentes en el suero sanguíneo en solo tos constitutivos.
15 minutos.
b) La anemia se define como una concentración baja de 5. «Las funciones básicas de la ciencia son: la descripción, la
hemoglobina en la sangre. predicción y la explicación».
c) Se detecta mediante un análisis de laboratorio en el que a) La retrodicción es un concepto relativo a la predicción,
se descubre un nivel de hemoglobina en la sangre me- que opera con un signo temporal inverso.
nor de lo normal. b) Por explicación se suele entender el establecimiento de
d) Bastará con depositar una gota de sangre sobre una fina regularidades en términos de causa o efecto.
lámina que cambia de color según la concentración del c) La descripción científica es la presentación verbal o es-
metal y comparar el tono con un sistema de referencia. crita de entidades naturales y/o sus propiedades.
e) El grupo de investigadores ha empleado un material d) La predicción le permite a la ciencia anticipar resulta-
inteligente que cambia de color según la presencia de dos o fenómenos.
hierro en un fluido, de modo que es aplicable también
al agua o al vino.

112 Mejores Personas, Mejores Familias


5º AÑO DE SECUNDARIA
Colegio

RAZONAMIENTO VERBAL  Mejores Personas, Mejores Familias

6. «La escolástica o filosofía medieval se caracteriza por ex- 9. «Mi infancia estuvo llena de juegos y travesuras, salpicada
presar un pensamiento determinado, fundamentalmente por las reprimidas correctivas de mis padres».
por el fenómeno religioso». a) Por entonces, no comprendida por qué los adultos se que-
a) Los pensadores árabes, con sus comentarios e interpreta- jaban de mis gritos y de mis saltos.
ciones de Aristóteles, contribuyeron a conformar el perío- b) En ocasiones fantaseaba sobre mi futuro. ¿Qué sería?
do más fértil de la escolástica. ¿Escritor, deportista, abogado, profesor?
b) La Summa Teológica de santo Tomas de Aquino (1224- c) La idea que más me cautivaba era crecer al lado de mis
1275) corresponde al periodo de oro de la escolástica. padres y protegerlos en su vejez.
c) Los universales, la querella de las investiduras, la exis- d) La vida es cruel, en ocasiones los padres parten antes
tencia de Dios son algunos de los problemas típicos de que podamos devolverles el amor que nos prodigaron.
la escolástica.
10. (I) Nancy es una estudiante americana de lenguas románti-
d) La escolástica, además de ser valiosa por sí misma, lo es
cas que pasa un año en Andalucía preparando su tesis doc-
también porque estableció las condiciones para su su-
toral. (II) En La tesis de Nancy, el autor utiliza un personaje
peración.
extranjero para dar soporte a las conversaciones acerca de los
gitanos andaluces. (III) Se nota en la novela cierto valor lin-
7. «Continúa la polémica entre los paleontólogos con rela-
güístico porque la protagonista es estudiante de lenguas. (IV)
ción a la manera en que se originó el vuelo en la naturale-
Es una novela documento porque expone datos con valor
za».
documental y científico. (V) La reproducción del habla de
a) El que hoy día haya más especies voladoras que pla- los gitanos revela en esta obra una serie de variantes en
neadoras en el mundo, no zanja necesariamente la relación con el español normativo.
cuestión de cuál fue la más primitiva modalidad de a) II c) V e) IV
desplazamiento aéreo en la naturaleza. b) I d) III
b) Están los que piensan que los dromaeosaurios –antece-
sores directos de las aves modernas– vivían en el suelo
cuando evolucionaron para despegar mediante una ca-
rrera o un salto.
c) Los científicos opinan que las extremidades aladas del Mi- Indica la oración que debe eliminarse.
croraptor gui le servían más bien para planear en el aire 1. (I) La teoría de la complementariedad de los sexos con-
desde las copas de los árboles. sidera a estos iguales en valor pero diferentes. (II) Pre-
d) Microraptor gui es el nombre de una especie cuyo fósil dominó en Europa durante el siglo XII y parte del XIII,
recién descubierto muestra que tenía plumas en las cua- y significó libertad y conocimiento para las mujeres. (III)
tro extremidades. A mediados del s. XIII se instaló la teoría de la polaridad
entre los sexos de raíz aristotélica. (IV) El precio que las
8. «El cristianismo, en sus orígenes, fue considerado por los mujeres pagaron por su amor a la libertad fue alto: fingir
romanos como una secta judía más». en aras de la igualdad y desconocer la autoridad materna.
a) La cuna del cristianismo fue Palestina; se constituyó – (V) Los hombres, según esta, son superiores a las mujeres,
en parte– como reacción ante la persecución de los fari- idea que dio pie a una profusa literatura misógina.
seos. a) II b) IV c) I
b) Se estima que los esenios, religión judía poco difundida d) III e) V
y poco conocida, tuvo mucho que ver con el surgimien-
to del cristianismo.
2. (I) Lee Hsiao Lung nació en San Francisco en noviem-
c) Gracias a Constantino, el cristianismo se constituyó en
bre de 1940, hijo de un famoso cantante de ópera chino.
la religión predominante en todo el imperio romano.
(II) Visitó Shunde a 60 kilómetros del noroeste de Hong
d) Sin escritos, pero con una enseñanza viva, Jesús de Na-
Kong en una sola ocasión, a los 5 años. (III) Regresó a
zaret sentó las bases de una religión universal y durade-
ra. Hong Kong a comienzos de la década de los años 70. En
esta ciudad fue donde creció. (IV) Por ello, sus seguidores
hongkoneses cuestionaron a sus autoridades por no haber-
le hecho un museo. (V) Murió de forma repentina de un
edema cerebral el 20 de julio de 1973.
a) I b) IV c) II
d) V e) III

5º AÑO DE SECUNDARIA
Mejores Personas, Mejores Familias 113
Colegio

Mejores Personas, Mejores Familias RAZONAMIENTO VERBAL


3. (I) La isla de Pascua se encuentra a casi 4000 km. de la costa 7. (I) Conocimiento es una relación sujeto congnoscen-
chilena. (II) La isla de Pascua ofrece lugares de alto interés te-objeto congnoscible. (II) A los filósofos, psicólogos y
arqueológico como Hangaroa o el museo Sebastián Englert. sociólogos los reúne, entre otras cosas, el tema del conoci-
(III) El principal atractivo de la isla de Pascua son las colosales miento. (III) El filósofo estudia la posibilidad de esa relación.
estatuas de piedra. (IV) En la base del volcán Rano Raraku (IV) El psicólogo se interesa en los mecanismos responsables
se halla la cantera de donde se extrajeron las piedras para la de esa relación. (V) Para el sociólogo, importa el marco cultu-
prehistórica construcción de dichas estatuas. (V) Estos mega- ral a través del cual se hace efectiva esa relación.
litos de la isla de Pascua son enormes y construidos antes de a) IV b) III c) II
Cristo. d) V e) I
a) IV b) II c) I
d) III e) V 8. (I) La prevención, al adelantarse a la enfermedad, es
más valiosa que curar. (II) Para prevenir un problema
4. (I) El avión mirage F1 OR está equipado con un radar que en general hay que conocer las causas del mismo. (III)
proporciona imágenes tridimensionales del terreno. (II) Prevención es mejor que tratamiento. (IV) Si se conoce
Los nuevos misiles son guiados por rayos láser hacia su lo que propició un accidente, se puede tomar previsiones
blanco. (III) La CIA desarrolló un avión espía capaz de para que no se repita. (V) La posibilidad de prevenir males
volar sin piloto y obtener información sin ser detectado ilustra el valor práctico del saber científico.
por radar alguno. (IV) Los radares de trayectoria estudian a) V b) I c) II
la trayectoria de un proyectil y detectan la posición desde d) IV e) III
donde fue disparado. (V) Hay aviones con sistema especial
que burlan el sistema de detección de redares. 9. (I) Los países no afiliados al COI no pueden inscribir sus
a) V b) II c) III participantes para las olimpiadas. (II) El Comité Olímpi-
d) I e) IV co Internacional (COI) está integrado por 30 federaciones
internacionales y 185 comités olímpicos nacionales. (III)
5. (I) Todos los vertebrados tenemos un envoltorio llamado Solo a través de estos comités olímpicos, los países pueden
«piel». (II) La piel es un órgano que representa el 15 por inscribir a sus representantes para las olimpiadas. (IV) La
ciento del peso en el hombre. (III) Es un órgano sensorial sede del COI está en Suiza, y la ocupan 95 miembros que el
primario que registra el dolor, la presión y la temperatura. mismo COI elige. (V) El presidente electo entre los miem-
(IV) La temperatura del hombre puede variar según facto- bros del COI permanece en su puesto por ocho años.
res biológicos o ambientales. (V) Los órganos internos son a) V b) II c) I
cubiertos y protegidos de las agresiones del exterior por d) IV e) III
este envoltorio.
a) IV b) II c) V 10. (I) En 40 años, nuestro corazón ha latido más de 150 millo-
d) III e) I nes de veces. (II) El corazón se contrae unas 70 veces por
minuto. (III) En ese lapso envía entre 65 y 100 centímetros
6. (I) Una opción política no tiene por qué ser racional. (II) La cúbicos de sangre, es decir, de 5 a 7 litros por minuto. (IV)
acción política es tema de una ciencia social, la politología. En 75 años, un corazón humano ha latido unas 300 mi-
(III) La participación en relaciones de autoridad subordina- llones de veces y ha bombeado 200 millones de litros de
ción se designa acción política. (IV) La acción política supone sangre. (V) El bombeado se realiza gracias al miocardio,
el logro, entre otras posibilidades, de un objetivo social. (V) un importante músculo cardiaco.
La insurgencia es un tipo de acción política. a) I b) III c) V
a) I b) II c) III d) II e) IV
d) IV e) V

114 Mejores Personas, Mejores Familias


5º AÑO DE SECUNDARIA
Lingüística
Colegio

LINGÜÍSTICA  Mejores Personas, Mejores Familias

MA
TE

01   LA COMUNICACIÓN

La comunicación es la base de la cultura. Cuanto más eficaz sea 4. Código


esta, más rica y diversa la cultura. Es un sistema de signos comunes al emisor y el receptor.
En la actualidad, la telefonía, Internet, el fax y otros medios Pueden ser movimientos, gestos, juegos de luces, un sis-
permiten una comunicación eficaz en menos tiempo. tema de lengua o su equivalente (Morse, Braille, etc.). El
código es un elemento concreto que puede ser también de
Definición naturaleza auditiva o táctil.
La comunicación es un proceso por medio del cual damos a
conocer a otros nuestras experiencias, necesidades, deseos, etc. 5. Canal
a través de un sistema de signos, propio de los seres vivientes. Es el medio portador del mensaje, un conducto. Puede ser
En su sentido amplio podemos clasificarla en humana y no hu- una hoja de papel, las ondas sonoras, el teléfono, etc.
mana.
6. El referente o realidad
Es el elemento concreto que es objeto del mensaje. Ha-
Clases de Comunicación
bíamos señalado que el mensaje era un conjunto de ideas
Existen dos clases de comunicación:
organizadas sistemáticamente, por lo tanto, el mensaje es
abstracto, mental y psíquico, pero cuando este mensaje
1. Comunicación no humana se concretiza recibe el nombre de referente. Por ejemplo,
Se da en las sociedades no humanas (animales y vegetales). Renzo le dice a Milagros: «Te invito un pastel a la salida»,
Hoy se sabe que la comunicación entre los organismos vi- el referente en el momento en que Renzo habla no existe;
vos puede realizarse, según sea el caso, mediante signos y se presentará como tal en el momento en que el pastel sea
señales acústicas, visuales, etc. por ejemplo: los variados adquirido y percibido.
sonidos emitidos por los simios, los ovinos, los camélidos
y otras especies de animales constituyen señales acústicas 7. Circunstancia o contexto
de comunicación. Toda comunicación se produce en un determinado lugar
o espacio y en un momento o tiempo. Estos elementos,
2. Comunicación humana constituyen la circunstancia y permiten la decodificación
Se da entre seres humanos, y para ello el hombre emplea de la manera más cercana a lo que el emisor ha pretendido
un sistema de signos: verbales, gestuales, movimientos, so- transmitir. Por ejemplo, si alguien emplea la palabra arco y
nidos, etc. estamos en una clase de Geometría, lo más probable sería
que el significado sea «porción de circunferencia». Si la
Elementos de la comunicación humana circunstancia fuera un estadio deportivo, la palabra arco
La comunicación como proceso presenta los siguientes ele- tendría otro significado.
mentos:
Otros Aspectos
1. Emisor 1. El ruido
Llamado también interferencia, es considerado como todo
Es la persona o grupo de personas que transmite el mensa-
aquello que pueda perturbar a cualquier elemento del
je. El emisor se encarga de realizar la codificación; que es
proceso: decir una cosa por otra, una mancha que hace
la disposición del código para transmitir el mensaje. ilegible una palabra, una tilde mal colocada, el sonido del
ambiente, etc.
2. Receptor
Es la persona o personas que reciben y descifran el mensa- 2. El feedback
je. Se encarga de la decodificación, que consiste en extraer Se define como retroalimentación. Es un fenómeno inver-
el significado(s) a partir del signo o conjunto de signos es- so al proceso: Es toda respuesta o estímulo que permite al
pecíficos recibidos. emisor autorregularse o modificar su mensaje.

3. Mensaje 3. La redundancia
Son los contenidos, es decir; las ideas, pensamientos, senti- Es la parte del mensaje que podría omitirse sin que se pro-
mientos, y todo aquello que se transmite. El mensaje es de duzca pérdida de información. Se utiliza para asegurar que
naturaleza abstracta, es decir, es conceptual y la única mane- no haya pérdida de información esencial causada por el
ra de que este pueda ser percibido es mediante el código. ruido.

5º AÑO DE SECUNDARIA
Mejores Personas, Mejores Familias 117
Colegio

Mejores Personas, Mejores Familias LINGÜÍSTICA

El proceso de la comunicación

ruido ruido

mensaje mensaje

emisor codificación canal decodificación receptor

mensaje
retroalimentación
ruido

Tipos de comunicación c) Gráfica: Cuando se usan dibujos e imágenes.


a) Por el tipo de código d) Audiovisual: Comunicación en la que se usan signos
YY Lingüística: Cuando se usa el lenguaje articulado, sea sonoros y visuales.
en el nivel escrito como en el nivel oral.
YY No Lingüística: Cuando se usan gestos, señales,
dibujos y cualquier forma que no incluya el lenguaje
articulado.

b) Por el espacio en que se encuentran el emisor y el 1. La comunicación, desde el punto de vista de presencia o
receptor ausencia de interacción, es _________.
YY Comunicación directa: Cuando el emisor y el receptor a) lingüística o no lingüística
se encuentran en el mismo espacio y/o tiempo. b) pública o privada
YY Comunicación indirecta: Cuando entre el emisor y el c) verbal o no verbal
receptor existe una barrera de tiempo y/o espacio. d) unilateral, bilateral y multilateral
e) directa o indirecta
c) Por la presencia o ausencia de interacción
YY Comunicación unilateral: Cuando no existe 2. Elemento concreto que es objeto del mensaje.
intercambio de roles entre emisor y receptor; no hay a) Receptor d) Mensaje
diálogo. b) Referente e) Ruido
YY Comunicación bilateral: Cuando dos personas c) Código
interactúan e intercambian los roles de receptor y
emisor. 3. Se le conoce como el proceso inverso al de la comunica-
YY Comunicación multilateral: Cuando más de dos ción que realiza el receptor.
personas interactúan y ponen en discusión sus ideas. a) Decodificación
b) Codificación
d) Por el tipo de receptor c) Retroalimentación
YY Comunicación privada: Cuando el receptor es d) Recepción
conocido o determinado. Generalmente dentro de un e) Canalización
ámbito limitado.
YY Comunicación pública o social: Cuando no se 4. Es el medio físico portador del mensaje en la comunica-
conoce al receptor y su cantidad es indeterminada. ción lingüística y directa.
a) Las ondas hertzianas
4. Formas principales de comunicación b) Los gestos del rostro
a) Oral: Aquella en la que nos comunicamos por medio c) El aparato fonador
de sonidos (palabras). d) Las ondas sonoras
b) Escrita: Cuando se emplean signos escritos. e) Las ondas magnéticas

118 Mejores Personas, Mejores Familias


5º AÑO DE SECUNDARIA
Colegio

LINGÜÍSTICA  Mejores Personas, Mejores Familias

5. Son los seres del mundo concreto, del pensamiento ficti-


cio o concepciones abstractas que tanto el emisor como el
receptor conocen de una manera semejante, más o menos
idéntica, y que son representados en el mensaje mediante 1. Es un sistema de signos comunes al emisor y receptor para
signos lingüísticos u otros signos igualmente útiles. una comunicación exitosa.
a) Contexto d) Código a) Código
b) Mensaje e) Referente b) Canal
c) Canal c) Contexto
d) Mensaje
6. Las personas gramaticales (yo, tú y él) se sustentan por e) Emisor
la existencia de dos elementos importantes de la co-
municación. Estos son respectivamente __________y
___________. 2. Es el elemento abstracto de la comunicación que contiene
las ideas a transmitir en el proceso comunicativo.
a) emisor - código
a) El canal
b) mensaje - receptor
b) El código
c) emisor - referente
c) El mensaje
d) canal - emisor
d) El emisor
e) emisor - receptor
e) El receptor

7. En: Juan no trajo el periódico, predomina el elemento de la 3. Tipo de comunicación en la cual se realiza un intercambio
comunicación llamado ________. de ideas al mismo tiempo que interactúan, en semejantes
rangos, un emisor y un receptor. Además, este tipo de co-
a) emisor municación se basa en el continuo intercambio de roles.
b) referente a) Pública
c) receptor b) Privada
d) código c) Bidireccional
e) canal d) Unidireccional
e) Lingüística
8. Son los contenidos, es decir; las ideas, pensamientos o sen-
timientos que se transmiten. 4. Cuando enfocamos el proceso comunicativo desde el pun-
a) Código to de vista del tiempo y del espacio en el cual se realiza el
b) Canal diálogo, nos referimos a la comunicación _________.
c) Contexto a) lingüística y no lingüística.
d) Mensaje b) pública y privada.
e) Receptor c) unidireccional y multidireccional.
d) unipersonal y bipersonal.
9. Señala la situación donde se utilice, evidentemente, un có- e) directa e indirecta.
digo lingüístico.
a) Una señal de tránsito 5. Las agencias publicitarias, ¿qué elemento de la comunica-
b) El logotipo de una revista ción constituirían específicamente?
c) Un comercial a) Decodificador
d) Una recomendación b) Canal
e) Una solicitud de empleo c) Contexto
d) Mensaje
10. Tipo de comunicación en el cual se usan gestos, señales, e) Encodificador
dibujos y cualquier forma que no incluya el lenguaje arti-
culado. 6. Se le conoce como idea codificada.
a) No lingüística a) El referente
b) Privada b) El emisor
c) Unidireccional c) El mensaje
d) Bipersonal d) El código
e) Directa e) El contexto

5º AÑO DE SECUNDARIA
Mejores Personas, Mejores Familias 119
Colegio

Mejores Personas, Mejores Familias LINGÜÍSTICA

7. Elemento de la comunicación que descifra el mensaje y la 2. Es el factor que interfiere con el proceso comunicativo
transforma en información: y, obstaculiza la viabilidad del canal que se utiliza en el
a) Código diálogo.
b) Canal a) Feedback
c) Contexto b) Referente
d) Receptor c) Código
e) Emisor d) Mensaje
e) Ruido
8. Suele relacionarse con el ambiente que rodea al diálogo.
a) Código 3. Es el proceso que realiza el emisor antes de enviar el
b) Canal mensaje:
c) Contexto a) Descodificación
d) Mensaje b) Ruido
e) Receptor c) Retroalimentación
d) Codificación
9. Es el factor que siempre debe compartir el emisor con el e) Canalización
receptor.
a) El canal 4. Cuando se da una comunicación oral, sea una conversa-
b) El código ción cotidiana o una charla de lingüística aplicada, el canal
c) El mensaje utilizado será _______.
d) El emisor a) las ondas hertzianas
e) El receptor b) los gestos del rostro
c) el aparato fonador
10. Tipo de comunicación que se basa en el continuo inter- d) las ondas sonoras
cambio de roles: e) las ondas magnéticas
a) Pública
b) Privada 5. Son representados en el mensaje mediante signos lingüís-
c) Bidireccional ticos u otros signos igualmente útiles.
d) Unidireccional a) Contextos
e) Lingüística b) Mensajes
c) Canales
d) Códigos
e) Referentes

1. Teniendo en cuenta la cantidad de personas que intervie-


nen en el diálogo con intercambio de roles, la comunica-
ción puede ser _________.
a) lingüística o no lingüística
b) pública o privada
c) verbal o no verbal
d) unilateral, bilateral y multilateral
e) directa o indirecta

120 Mejores Personas, Mejores Familias


5º AÑO DE SECUNDARIA
Colegio

LINGÜÍSTICA  Mejores Personas, Mejores Familias

MA
TE

02   LENGUAJE: CARACTERÍSTICAS Y FUNCIONES

El lenguaje como facultad humana


Cuando hablamos del lenguaje nos referimos exclusivamente a la facultad que tienen los hombres de comunicarse por medios
vocálicos.
En este sentido, se dice que el canal utilizado es la vía vocal-auditiva: las ideas, una vez elaboradas, estimulan las cuerdas vocales
y los sonidos emitidos (acto físico) son percibidos a través del oído, y retraducidos al plano mental del oyente.
Como sistema complejo del cerebro (o mente), el lenguaje humano se manifiesta formalmente a través de la lengua. Es decir,
estimulado por una experiencia adecuada, en un medio sociolingüístico determinado, el lenguaje humano determina el
surgimiento de una lengua natural: la española, la quechua, la inglesa, etc.

Características
a) El lenguaje humano es doblemente articulado
Se refiere al procedimiento que el ser humano realiza inconscientemente. La primera articulación se refiere a los monemas
(las palabras) y la decodificación llevada a cabo para entender un mensaje. La segunda articulación comprende la descompo-
sición de los monemas (morfemas) en sus fonemas constituyentes.
La doble articulación es un aspecto de la articulación lingüística consistente en la emisión de signos con significado (palabras
u oraciones) mediante la emisión de unidades que en sí mismas carecen de significado (alófonos o fonemas).

b) Desplazamiento
El lenguaje nos permite hacer referencia a tiempos y espacios distintos a los del acto del habla.

c) Innato
Según Chomsky: «El lenguaje es un producto de la inteligencia humana, creado de nuevo en cada individuo mediante opera-
ciones que están fuera del alcance de la voluntad o la conciencia».
Sin embargo lingüistas como Sapir afirman que el lenguaje es aprendido en su aspecto cultural, es decir, se aprende en la
interrelación dinámica de una colectividad humana.

d) Productividad
El lenguaje nos permite crear o enviar mensajes nuevos que nunca antes hemos emitido. Esta característica se percibe con
mejor nitidez en el campo literario.

e) Universal
El lenguaje es una facultad exclusivamente humana; todo ser humano, en condiciones normales, posee lenguaje, ya que el
habla es inherente a cualquier persona en cualquier lugar.

5º AÑO DE SECUNDARIA
Mejores Personas, Mejores Familias 121
Colegio

Mejores Personas, Mejores Familias LINGÜÍSTICA

f) Racional y creativo los dedos con la tapa del piano; cuando decimos a nues-
Para Descartes es lo que diferencia al hombre del animal. tra novia que la echamos de menos o cuando decimos que
Para Humboldt y Chomsky «el hombre es capaz de cons- odiamos las espinacas.
truir mensajes siempre nuevos a partir de los medios fi-
nitos que le da la lengua». El lenguaje es creativo porque 3. Función apelativa o conativa
podemos comprender y producir conceptos o ideas jamás Mediante el uso de esta función normalmente pretende-
realizados. Y, según Chomsky, se da de dos maneras; como mos provocar una reacción en  el receptor, que es el ele-
creatividad que sigue las reglas o que se sale de las reglas mento fundamental. Es decir, queremos que haga algo, o
(por factores socioculturales); así podemos distinguir cier- que deje de hacerlo. Es la función principal cuando, por
tos parámetros de acuerdo a la aceptabilidad semántica, ejemplo, decimos: «vete a tomar el aire»; «abre la ventana,
pueden ser gramaticales o agramaticales (gramática y se- por favor” o «cállate».
mánticamente inaceptables), anómalos o impropios.
4. Función fática
Funciones Básicas La usamos para comprobar que  el canal  (elemento fun-
a) De comunicación: permite la interacción social damental) sigue abierto, es decir; que la comunicación es
b) De sostén del pensamiento: pensamiento y palabra van físicamente posible. Por ejemplo, cuando hablando por el
emparejados celular preguntamos si nos oyen, o cuando usamos mule-
tillas. Ejemplos de muletillas: «Te quedas ahí quieto, ¿eh?»;
Otras Funciones «ayer lo pasé genial en la fiesta, ¿sabes?».

1. Función representativa o referencial 5. Función poética o estética


Se usa cuando pretendemos meramente transmitir una Se pretende crear belleza usando el lenguaje. Es la función
información, sin hacer valoraciones sobre ella ni pre- principal en poemas, novelas, obras de teatro y canciones.
tender reacciones en nuestro interlocutor, por ejemplo También es una de las principales funciones en los refra-
cuando decimos «está lloviendo», o «la capital de Ma- nes. Esta función, al igual que la representativa, se cen-
rruecos es Rabat». Esta función se centra, dentro de los tra en el mensaje, pero a diferencia de esta, en su forma y
elementos de la comunicación, en  el mensaje, aunque no en su contenido. Cualquier poema es un ejemplo de la
también hay quien dice que se centra en la realidad exterior función estética del lenguaje. Por ejemplo: «Mariposa de
o referente (los elementos de la comunicación están explica- sueño, te pareces a mi alma, y te pareces a la palabra me-
dos al final de este artículo). lancolía.» (Pablo Neruda)

2. Función expresiva o emotiva 6. Función metalingüística


Es utilizada cuando el emisor (elemento en el que se cen- Se utiliza cuando se usa la lengua para hablar de la misma
tra esta función) pretende dar cuenta de su estado físico o lengua u otra cualquiera. Por ejemplo, cuando decimos
anímico, como cuando soltamos un «¡ay!» al presionarnos «burro se escribe con b», o «the es el artículo en inglés».
Esta función se centra en el código, es decir, en la lengua
de la que se habla.

Referente
función referencial

Canal

Emisor Mensaje Receptor


Función expresiva Función poética Función apelativa

Función fática

Código
Función metalingüística

122 Mejores Personas, Mejores Familias


5º AÑO DE SECUNDARIA
Colegio

LINGÜÍSTICA  Mejores Personas, Mejores Familias

8. Elemento de la comunicación que se relaciona con la fun-


ción emotiva del lenguaje:
a) Código
b) Canal
1. Como sistema genético, el lenguaje humano __________. c) Contexto
a) se manifiesta en algunas lenguas d) Mensaje
b) se adquiere en la sociedad e) Emisor
c) es de carácter innato
d) no es de carácter universal 9. Las situaciones lingüísticas o teorías del lenguaje se comu-
e) depende de la lengua nican a través de la función ___________del lenguaje.
a) conativa
2. Indica V o F según corresponda. b) fática
Para Chomsky, el lenguaje es toda forma de c) metalingüística
comunicación. ( ) d) poética
El lenguaje humano y el animal son idénticos. ( ) e) referencial
Todo lenguaje es innato. ( )
El lenguaje es monoarticulado. ( ) 10. La función apelativa busca___________.
a) VVVF a) la necesidad del emisor de hacer productivo el tiempo
b) FFFF b) comprobar el buen uso de los signos lingüísticos
c) VVFF c) convencer o persuadir al receptor para que actúe de una
d) FFVV manera determinada
e) FFVF d) comprobar que el canal utilizado esté libre de cualquier
ruido
3. La definición: «Facultad exclusivamente huma- e) información objetiva
na para adquirir un sistema lingüístico» corresponde
a____________.
a) lengua
b) jerga
c) lenguaje
d) idioma 1. El lenguaje es biarticulado porque____________.
e) norma a) se expresa en morfemas y fonemas
b) es capaz de simbolizar
4. El lenguaje humano se manifiesta formalmente a través c) está en la mente
de_________. d) tiene variaciones
a) la lengua e) se expresa como acto social
b) el habla
c) el dialecto 2. A diferencia de la comunicación, el lenguaje
d) el sociolecto es___________.
e) el idiolecto a) un acto involuntario
b) un acto de todo ser vivo
5. La lingüística es una ciencia que tiene por propósito el es- c) exclusivo del hombre
tudio de______________. d) concreto
a) la forma de hablar de cada persona e) social
b) el estilo de cada uno de los escritores
c) la estructura y funcionamiento del lenguaje 3. Es una característica del lenguaje propuesta por Chomsky.
d) las lenguas europeas a) Innatismo
e) la lengua castellana b) Biarticulación
6. Es el elemento que se prioriza en la función fática o de con- c) Linealidad
tacto. d) Productividad
a) Receptor d) Mensaje e) Universalidad
b) Referente e) Ruido
c) Código 4. La capacidad de proyección y de poder formar recuerdos se
debe la característica del lenguaje llamada____________.
7. Son los seres del mundo concreto que se relacionan con la a) innatismo
función informativa del lenguaje. b) biarticulación
a) Contexto d) Código c) desplazamiento
b) Mensaje e) Referente d) productividad
c) Canal e) universalidad

5º AÑO DE SECUNDARIA
Mejores Personas, Mejores Familias 123
Colegio

Mejores Personas, Mejores Familias LINGÜÍSTICA

5. El rasgo que define al lenguaje humano es su carác-


ter___________.
a) escrito
b) gestual
c) articulado 1. Es la ciencia que estudia el lenguaje articulado en general.
d) mímico a) Gramática
e) gestual y escrito b) Lingüística
c) Semántica
6. La función que se relaciona directamente con el elemento d) Comunicación
de la comunicación que codifica un mensaje es la _______. e) Lenguaje
a) poética
b) apelativa 2. Señala la afirmación correcta con respecto a los asuntos
del lenguaje humano:
c) emotiva
a) El lenguaje es necesariamente concreto.
d) fática
b) El lenguaje es tripartito.
e) referencial c) Se considera que el lenguaje es innato porque constitu-
ye una herencia biológica.
7. En: En este momento, debes salir a la calle y buscarla, la d) No siempre el lenguaje es humano.
función predominante es la _____. e) Se considera que el idioma no es un lenguaje.
a) poética
b) apelativa 3. Unidades correspondientes a la segunda articulación del
c) emotiva lenguaje:
d) fática a) Fonemas d) Escritura
e) referencial b) Morfemas e) Tonos
c) Semas
8. Indica la afirmación correcta.
a) La función poética se vincula con el mensaje. 4. Dentro del terreno lingüístico, es una característica del
lenguaje.
b) La función metalingüística se vincula directamente con
a) Psicofísica d) Personal
el ruido y el canal.
b) Abstracta e) Biarticulado
c) La función fática se vincula con el elemento que deco-
c) Concreta
difica el mensaje.
d) La función apelativa se vincula con la función metalin-
5. Con relación al estudio del lenguaje, se puede afirmar
güística.
_____________.
e) La función referencial no toma en cuenta los seres de la
a) que los animales aprenden su lenguaje.
realidad.
b) que lengua es igual a lenguaje.
c) que la lingüística se ocupa de los asuntos del lenguaje
9. En: Elsa, guarda el juguete, predomina la función articulado.
________________. d) que el lenguaje se adquiere.
e) que los asuntos del lenguaje le corresponde a la ortogra-
a) apelativa o conativa fía.
b) representativa o denotativa
c) expresiva o emotiva 6. Es una función que utiliza el lenguaje para describir la rea-
d) estética o poética lidad objetiva.
e) fática o de contacto a) Representativa
b) Metalingüística
10. Función del lenguaje que consiste en la explicación del sig- c) Poética
nificado de una palabra. d) Conativa
e) Expresiva
a) Informativa
7. Si un profesor dice: «Haz el bien sin mirar a quién», la fun-
b) Expresiva
ción del lenguaje que predominada es la ___________.
c) Apelativa
a) poética d) fática
d) Fática b) apelativa e) referencial
e) Metalingüística c) emotiva

124 Mejores Personas, Mejores Familias


5º AÑO DE SECUNDARIA
Colegio

LINGÜÍSTICA  Mejores Personas, Mejores Familias

8. Señala la alternativa donde predomina la función apelati- 9. En: Dame contacto, ¿cuál es la función que predomina?
va. a) Apelativa d) Estética
a) Los morfemas son las unidades de estudio de la morfo- b) Cognitiva e) Emotiva
logía. c) Metalingüística
b) Los jóvenes a la obra; los viejos a la tumba.
c) Hace un frío horrible. 10. Agrupar las funciones fue un aporte del estudioso
d) Los sustantivos son seres de la realidad o de la inteligen- ________________.
cia. a) Sapir d) Coseriu
e) La fonética estudia los componentes del aparato fona- b) Saussure e) Jakobson
dor. c) Hjemslev

5º AÑO DE SECUNDARIA
Mejores Personas, Mejores Familias 125
Colegio

Mejores Personas, Mejores Familias LINGÜÍSTICA

M A
TE

03   PLANOS DEL LENGUAJE

Los planos del lenguaje están constituidos por dos partes: lengua y habla. Ferdinand de Saussure, desde un principio, estableció 
la distinción entre langue y parole (lengua y habla).
En esta parte, desarrollamos cada una de las características de esta dicotomía.

Lengua Habla

Es el uso del código


Es un código
Porque es la realización concreta, por parte de un
Porque es un sistema de signos convencionales
individuo, del acto de la pronunciación. Los signos
organizado en base a una serie de reglas acordadas
ideales de materialización en la emisión de sonidos
(tácita o explícitamente) por una comunidad lingüística.
articulados.

Casi fija
Variable
Porque el sistema de la lengua permanece estable, por lo
Porque el habla difiere de persona en persona e incluso
menos en el momento presente.
en la misma persona, según su edad, su interés y otras
Aunque, desde un punto de vista histórico, todas las
condiciones particulares en un momento dado.
lenguas cambian.

Teórica Práctica
Porque puede ser descrita y analizada en función a Porque es acción concreta y se desarrolla en el momento
normas y principios, planteados por una comunidad en que un individuo transmite su mensaje a otro. Es una
lingüística. realización efímera.

Psíquica Psicofísica
Porque son signos virtuales (la asociación de imágenes Porque el acto del habla implica el conjunto de selecciones
conceptuales e imágenes acústicas) que tienen su asiento que un individuo hace de los elementos del sistema y
en el cerebro. Además, aquí se codifican y decodifican la activación de los órganos del habla que permitirán la
todos los mensajes. emisión física (audible) de sonidos articulados.

Social Individual
Porque es un producto creado por la colectividad y es Porque es un acto producido por una sola persona a
propiedad compartida de todos sus miembros. través de un acto de voluntad.

Variaciones del uso de la lengua


1. Idioma
YY El idioma es un concepto más político que lingüístico y se refiere a la oficialización de una o más lenguas, de entre todas
las existentes en un territorio.
YY En el Perú (país en cuyo territorio conviven más de setenta lenguas) tradicionalmente se afirma que se considera idiomas
al castellano, quechua y aimara; sin embargo, la Constitución deja abierta la posibilidad de considerar idiomas a todas las
lenguas que existen dentro de su territorio.
YY El Perú es un país multilingüe y pluricultural.
2. Dialecto
YY Variante regional de una lengua general, que difiere de ella en ciertos aspectos gramaticales, fonéticos o léxicos.
YY Dialecto geográfico o regional es el que se configura en los límites de una localidad, región o área geográfica.
YY Hay lenguas que se hablan en distintos países, como el árabe, el inglés, el español o el francés. En otros casos, aunque la
lengua sea la misma, existen ciertas variaciones léxicas, fónicas y sintácticas menores por motivos históricos y estrictamente
evolutivos; sin embargo, todos los hablantes se entienden entre sí.

126 Mejores Personas, Mejores Familias


5º AÑO DE SECUNDARIA
Colegio

LINGÜÍSTICA  Mejores Personas, Mejores Familias

YY C
uando una lengua tiene varias manifestaciones dia- 3. En Lingüística; término que se define como propio de una
lectales, y todavía no se ha fijado la norma escrita, el comunidad lingüística.
dialecto oral en el que más se escriba y desarrolle una a) Jerga d) Sociolecto
literatura que sobresalga en calidad y cantidad, se ele- b) Habla e) Dialecto
vará como lengua modelo y fijará la norma lingüística c) Lengua
y literaria.
4. Dado que la lengua es propiedad de una comunidad, se
afirma que es ___________.
3. Sociolecto
a) psicofísica
Es la variación de una lengua de acuerdo con el grupo
b) abstracta
social que lo habla.
c) colectiva
Ejemplo de variantes: sexo, nivel académico, edad, etc.
d) individual
e) sistemática
4. Jerga
YY Vocabulario o léxico que emplea un determinado gru- 5. Se entiende por dialecto a _________________.
po social y profesional y que solo lo entiende parcial- a) un sistema de signos exclusivamente gráficos
mente el resto de la comunidad lingüística. b) un sistema de dibujos representativos de ideas
YY Caben bajo la denominación de jerga -o lenguajes c) una variación de la lengua por cuestiones geográficas
específicos- el léxico que se utiliza en la medicina, el d) cualquier combinación de signos
derecho, la banca, la ciencia y otros (jerga profesional e) las recomendaciones de uso del sistema articulado de
u ocupacional). Por ejemplo los vocablos patología, cada comunidad
hematoma y aneurisma, son palabras técnicas de la
medicina para nombrar la enfermedad, la inflama- 6. Las expresiones: «Vos sos mi nene»; «Tú eres mi bebé»
ción y la ceguera, respectivamente. Otras actividades constituyen casos de ___________.
también tienen expresiones específicas que no están a) dialectos
incluidas en el nivel culto del idioma. b) interlectos
YY En algunos ambientes delictivos o marginales se uti- c) idiolectos
liza un «lenguaje peculiar», llamado también jerga, d) lenguas distintas
marginal o germanía. Son ejemplos las palabras y e) distintos idiomas
frases como mermelada, (dinero), tasar (mirar). En
el Perú, en ciertos sectores académicos, a la jerga de 7. Enunciado que define mejor a un dialecto:
carácter vulgar se le denomina replana. a) Una variación de corte fonético.
YY El lenguaje de determinados grupos sociales, como el b) La manera propia del habla de un grupo profesional.
de los jóvenes, también suele denominarse jerga juve- c) La variación regional de una lengua general.
nil. d) Una manera arcaica de hablar.
e) Una lengua indígena sin escritura.

8. Es una característica de la lengua que la describe como


mental.
a) Psicofísica
b) Abstracta
1. La definición correcta de idioma es _____________. c) Concreta
a) un dialecto menor en zonas alejadas d) Personal
b) cualquier lengua se considera idioma e) Momentánea
c) una lengua reconocida como oficial por un Estado
d) un código particular que usan los humanos 9. Sistema de signos articulados con propósito comunicati-
e) un vocabulario jurídico vos que desarrollan las comunidades lingüísticas:
a) Lenguaje
2. En consideración al castellano estándar, se puede afirmar b) Lengua
que la expresión: «¡Qué bony, amix!», es un claro ejemplo c) Habla
de: d) Dialecto
a) Dialecto e) Gramática
b) Norma estándar
c) Jerga (juvenil) 10. Vocabulario que desarrollan ciertos grupos por razones
d) Arcaísmo profesionales o sociales:
e) Neologismo a) Lengua d) Lenguaje
b) Dialecto e) Habla
c) Jerga

5º AÑO DE SECUNDARIA
Mejores Personas, Mejores Familias 127
Colegio

Mejores Personas, Mejores Familias LINGÜÍSTICA

9. El habla es ____________.
a) la corrección de la lengua
b) la asimilación de las jergas
c) la realización del sistema - lengua
1. Variación regional de una lengua que puede presentar as- d) la situación invariable de la lengua
pecto fonético, morfológico, sintáctico y semántico. e) el uso técnico de la lengua
a) Lengua d) Dialecto
b) Idioma e) Norma 10. Se entiende por replana _________.
c) Lenguaje a) a la variación regional que afecta a una lengua
b) al conjunto de palabras que caracteriza el habla de un
2. La lengua es ________ porque puede ser descrita y anali- país
zada en función a normas y principios. c) al istema de signos de doble articulación con nivel de
a) social d) más o menos fija lengua
b) individual e) teórica d) al vocabulario denominado vulgar que desarrollan gru-
c) histórica pos afines dentro de una comunidad lingüística
e) al modo particular que tiene cada individuo de usar la
3. El lenguaje de determinados grupos sociales, como el de lengua colectiva
los jóvenes, también es llamado __________.
a) jerga d) habla
b) lengua e) idioma
c) dialecto

4. El prestigio de una lengua está determinado por factores 1. En el campo de la lingüística, se entiende por lengua
________. _____________.
(UNMSM 2009 II) a) al conjunto de signos acústicos que conforman un sistema
a) lingüísticos b) a la relación de sonidos que conforman el sistema lin-
güístico
b) sociales
c) a la materialización individual de las posibilidades de
c) geográficos
la lengua
d) biológicos
d) a las variaciones específicas y regionales de una lengua
e) raciales
e) a las variaciones individuales de cada vocablo dentro de
un sistema
5. La variante sociorregional de una lengua natural es reco-
nocida como _____________. 2. La lengua es abstracta y el habla es concreto; por lo tanto, si
(UNMSM 2012 II) la lengua es social, entonces el habla es ______________.
a) habla d) lenguaje a) individual d) pasajero
b) lengua e) idioma b) momentáneo e) psicofísico
c) dialecto c) la materialización del sistema

6. El nivel de la lengua que agrupa la forma de hablar con 3. En lingüística, se afirma que la lengua es mental porque
incorrecciones gramaticales, y palabras poco consideradas las posibilidades del sistema están en el común de los
socialmente, se las enmarca dentro del __________. hablantes. En ese sentido, el habla es psicofísica porque
a) idioma d) idiolecto ______________.
b) sociolecto e) dialecto a) su uso es efímero
c) lenguaje b) se materializa cada vez que el hablante hace uso dela
lengua
7. El nivel del habla que recurre a términos simbólicos y c) para su realización, aparte del esfuerzo mental, general-
giros idiomáticos, llamados académicos, se denomina mente hay un movimiento de los órganos fonadores
____________. d) es una secuencia de sonidos en el tiempo
a) dialecto c) idioma e) replana e) depende de las posibilidades de la lengua
b) lengua d) jerga
4. Se le conoce como germanía o jerigonza.
8. Al vocabulario que -dentro de la comunidad lingüística- de- a) Idioma c) Lenguaje e) Dialecto
sarrollan algunos grupos por razones distintivas y, que el co- b) Replana d) Idiolecto
mún de la sociedad considera formas vulgares, se denomina
_______. 5. Es la forma de hablar de ciertos sectores laborales, por la
necesidad del trabajo:
a) lengua d) replana
a) Dialecto c) Idioma e) Replana
b) idioma e) habla
b) Lengua d) Jerga
c) dialecto

128 Mejores Personas, Mejores Familias


5º AÑO DE SECUNDARIA
Colegio

LINGÜÍSTICA  Mejores Personas, Mejores Familias

MA
TE

04   FONÉTICA Y FONOLOGÍA

Fonética: ●● Velares: se producen con desplazamiento del


ZZ Rama de la Lingüística que se encarga del estudio de los postdorsal de la lengua hacia la zona velar: [k],
sonidos del habla, es decir, estudia los sonidos materiales o [x], [g]; se consideran consonantes posterio-
concretos, llamados también articulados. res.
ZZ Su mínima unidad de estudio es el fono.
b) Según el modo de articulación
El fono o sonido articulado ●● Oclusivas o explosivas: se articulan con obstruc-
Es la materialización del fonema a través del aparato fonador. ción total y momentánea del aire en algún punto
El fono se representa entre corchetes; lo cual significa que de la cavidad bucal: [p], [b], [d], [t], [k], [g].
hablamos de un sonido concreto (que se puede escuchar), ●● Fricativas o continuas: se articulan con obstruc-
material y no de un fonema. ción parcial del aire pulmonar en algún punto de
la cavidad bucal: [f], [ѳ], [s], [x].
Clases de fonos ●● Africadas: se articulan mediante una oclusión
1. LAS VOCALES seguida, casi simultáneamente, por una fricación:
YY [a] [č].
En la articulación correspondiente al fono [a]; la boca ●● Líquidas
y los labios se abren más que en cualquier otra articu- LL Laterales: se articulan con salida del aire pul-
lación, y la lengua se mantie­ne plana y en reposo, y en monar por uno o ambos lados de la cavidad
el centro. bucal: [λ], [l].
YY [e], [i] LL Vibrantes: se articulan mediante el contacto
Para los fonos [e], [i] el dorso de la len­gua se aproxima del ápice de la lengua, una o más veces, con la
ligeramente al paladar, y los labios forman una abertu- zona alveolar: [ř], [r].
ra alargada.
YY [u], [o] c. Según la acción de las cuerdas vocales:
En las articulaciones que corresponden a los fonos [u], ●● Sordas: se articulan sin vibración de las cuerdas
[o] la parte posterior de la lengua se aproxima ligera- vocales: [p], [t], [k], [f], [ѳ], [s], [x], [č].
mente al velo del paladar y los labios forman una aber- ●● Sonoras: se articulan con vibración de las cuerdas
tura redondeada. vocales: [b], [d], [g], [y], [l], [λ], [ř], [r], [m], [n],
[ñ].
2. CONSONANTES
a) Según el punto o zona de articulación: d. Según la resonancia:
●● Bilabiales: se articulan desplazando el labio infe- ●● Nasales: cuando la resonancia del segmento se
rior hacia el labio superior: [p], [b], [m] produce en la cavidad nasal: [m], [n], [ñ].
●● Labiodentales: se articulan aproximando el labio ●● Orales: cuando la resonancia se produce en la ca-
inferior a los bordes de los incisivos superiores: vidad bucal: [b], [d], [g], [y], [l], [l], [ř], [r], [p],
[f] [t], [k], [f], [ѳ], [s], [x], [č].
●● Dentales: se articulan al contacto del ápice de la
lengua con la cara interna de los incisivos superio- La articulación
res: [d], [t] Es la variación que sufre el fono al momento de pronunciarse
●● Interdentales: se articulan ubicando el ápice de la debido a la influencia de otros sonidos.
lengua entre los bordes de los incisivos superiores
e inferiores: [ѳ] En el Perú no empleamos este fo- ZZ E l aire espirado sale desde la laringe, pasa por la faringe y
nema, es sustituido por el fonema [s] (seseo). llega a la boca. El campo total de la articulación lo cons-
●● Alveolares: se producen con desplazamiento del tituyen la cavidad bucal, la cavidad faríngea y la cavidad
ápice de la lengua hacia la zona alveolar: [s], [l], nasal.
[ř] (rr), [r], [n]. ZZ Los movimientos de los labios, de la mandíbula inferior, de
●● Palatales: se articulan con desplazamiento del las mejillas, de la lengua y del velo del paladar modifican la
dorso de la lengua hacia la zona alveolar: [č] (Ch), forma y el espacio de la cavidad bucal, haciendo que el aire
[y],[ñ], [λ] (ll). Se consideran consonantes poste- produzca a su paso efectos acústicos más o menos diferentes.
riores. ZZ A la especial posición adoptada conjuntamente por dichos
órganos en el momento de producir un sonido, se le cono-
ce como articulación.

5º AÑO DE SECUNDARIA
Mejores Personas, Mejores Familias 129
Colegio

Mejores Personas, Mejores Familias LINGÜÍSTICA

Alófono
[n] (interdental) enfado Fonemas y grafemas de la lengua española
[n] (dental) diente
/n/ [n] (alveolar) ancestro
[n] (palatal) cancha
fonema [n] (velar) tango
Fonema : Grafemas
[n]
01. /b/ B – V /bérbo/
02. /p/ P /pélo/
alófonos
03. /d/ D /dédo/
04. /f/ F /fóka/
05. /t/ T /téla/
Definición
06. /l/ L /lána/
Es la ciencia lingüística que estudia los sonidos desde el 07. /λ/ LL /λáma/
punto de vista de su función en el sistema de comunicación 08. /y/ Y /yúka/
lingüística, es por eso que le interesa el papel distintivo de un 09. /k/ C, K, Q /kása//késo//kílo/
sonido:
Su unidad de estudio es el fonema.
ZZ /pata/ /peso/ /mesa/
ZZ /bata/ /beso/ /misa/ Fonema : Grafemas
ZZ P/b P/b e/i 10. /č/ CH / čančo /
11. /s/ S, C (ce, ci) /sélos/ /karísia/
12. /q/ Z /aqúkar/
El fonema 13. /g/ G (gue, gui) /géřa //gáta/
ZZ C
onstituye la imagen mental del sonido. Son limitados: 14. /x/ J (ge, gi) /xénio/ /xamón/
cada lengua contiene un número particular de sonidos 15. /r/ R,RR /taréa/
mentales que comparten todos los hablantes. 16. /ř/ R /buříto/
17. /m/ M /máno/
ZZ L
os fonemas son unidades mínimas, abstractas, autóno- 18. /n/ N /néna/
mas y distintivas de una lengua. 19. /ñ/ N /píña/
ZZ E
n español, se considera que existen veinticuatro fonemas
segmentales. Entendiéndose con ello la división de la cade-
na sonora en sonidos mínimos posibles con capacidad de
estructurar un sistema (vocales y consonantes); sin embar- Los fonemas suprasegmentales
go, para algunos lingüistas, por el acento y la entonación, en Son  llamados así porque son capaces de ocupar segmentos ya
el Perú, el número de fonemas se reduce a 22 por el peculiar ocupados por fonemas segmentales.
seseo (confusión entre la s y la z) y el yeísmo (confusión en-
Por ejemplo, en la palabra árbol, los sonidos de a y r nunca
tre la ll y la y).
sonarán a la vez, pero el sonido de a y el acento que hay en esa
ZZ Se llama fonemas suprasegmentales del castellano a los fo- misma vocal, si sonarán juntos, por lo tanto el acento es un
nemas que se superponen a otros fonemas segmentales para fonema suprasegmental.
darles la intensidad necesaria para diferenciar significados: / Otro fonema suprasegmental es la entonación. En la escritura,
papa/ /papá/, /mama/ /mamá/. los fonemas suprasegmentales son representados con los
ZZ G
rafema: representación del fonema en la escritura; tam- signos de puntuación. Pues, no es lo mismo decir:
bién se le conoce como grafía o letra. Según el OLE 10, el ZZ ¿Lo hiciste?   /  ¡Lo hiciste!
español presenta veintinueve letras y cinco dígrafos (ll, rr,
ch, gu, qu).
Tampoco significan lo mismo estas expresiones:
ZZ no sé si… lo haré   /  no sé si lo haré.
Clases de fonemas
ZZ Señor, Carlos es un irresponsable   /   Señor Carlos, es un
Los fonemas se clasifican en dos grupos: segmentales y irresponsable
suprasegmentales. Los primeros se concretizan en sucesión,
mientras que los segundos ocurren en simultaneidad con los
segmentos vocálicos.
Fonemas segmentales
Estos fonemas presentan las siguientes características:

130 Mejores Personas, Mejores Familias


5º AÑO DE SECUNDARIA
Colegio

LINGÜÍSTICA  Mejores Personas, Mejores Familias

Punto de articulación Bilabiales Labiodentales Dentales Interdentales Alveolares Palatales Velares


modo por la sono-
de articulación sordo sonoro sordo sonoro sordo sonoro sordo sonoro sordo sonoro sordo sonoro sordo sonoro
ridad
Oclusivos /p/ /b/ /t/ /d/ /k/ /g/
No líquidos
Consonántidos

Fricativos /f/ /θ/ /s/ /y/ /x/

No nasales Africado /c/


(orales)
Vibrante simple /r/
Líquidos

Vibrante múltiple /ř/

Laterales /l/ /λ/

Nasales /m/ /n/ /ñ/

Anteriores Centrales Posteriores


Vocálidos

Alta /I/ /u/

Media /e/ /o/

Baja /a/

5. Alternativa que contiene palabras que tienen consonantes


africadas.
a) Ella compró una toalla.
b) Yo iré con Yolanda.
1. Respecto a las consonantes, todas las vocales se articulan c) El sábado saldremos solos.
__________________. d) Ocho muchachas tienen chompas
a) con bloqueo total del aire en la cavidad bucal e) Felipe fue muy feliz con ella.
b) con la vibración de las cuerdas vocales
c) con bloqueo parcial del aire en la cavidad bucal.
d) con la no vibración de las cuerdas vocales 6. ¿Cuál es la palabra que contiene distinto fonema, pero con
e) con salida del aire pulmonar por la cavidad nasal. el mismo grafema?
a) Acueducto
b) Jorgito
2. La alternativa en la que aparecen consonantes vibrantes c) Gorgojo
es______________________. d) Acontece
a) llama mala e) Sartenazo
b) derrama la cera
c) gancho chino 7. Alternativa que presenta un dígrafo.
d) dame el bombo negro a) Pienso más en ella.
e) gaucho chinche b) Viajaremos a Chimbote.
c) Es un sinvergüenza.
3. La alternativa en la que aparece una consonante sorda es d) Trajo un tecito.
___________________. e) Reventaron cinco.
a) beben agua
b) mi mamá bebió 8. ¿Cuántos fonemas distintos hay en La china lo sabe?
c) un buen dedal de oro a) Siete b) Ocho c) Diez
d) el niño compró libro d) Nueve e) Once
e) ella ganó un gran libro
9. Indica la relación correcta:
4. Alternativa en la que aparecen consonantes laterales: a) Cuaderno /cuadérno/
a) Madre mía, ven b) Corazoncito /coraѳonsíto/
b) Iremos pronto c) Aventajado /Abentaxádo/
c) Beberemos agua d) Feo /xéo/
d) Tenemos más panes e) Pozo /poѳo/
e) Llamen a los niños

5º AÑO DE SECUNDARIA
Mejores Personas, Mejores Familias 131
Colegio

Mejores Personas, Mejores Familias LINGÜÍSTICA

10. Palabra que contiene al dígrafo /gu/. 8. Oración que contiene dos palabras diferenciadas por el fo-
a) Gusano d) Vergüenza nema acento.
b) Guisante e) Exiguo a) Estas limas son de Lima.
c) Aguaitar b) Tienes aptitud, pero no actitud.
c) Juan Mata llegó a la meta antes que tú.
d) Lanzó la olla a una profunda hoya.
e) Circulo por el círculo.

9. Indica la representación correcta.


1. Es un sonido oclusivo bilabial sordo. a) /buscaron/
a) [b] d) [p] b) /karákter/
b) [x] e) [k] c) /velocidad/
c) [t] d) /ciguena/
e) /cuaderno/
2. Palabra en la que aparece el sonido [r].
a) Ratón 10. Número de fonemas que corresponden, respectivamente,
b) Enrique a las palabras axila y ancho.
c) Honrado
d) Vereda
a) Seis – cuatro
e) Perro
b) Cinco – cinco
c) Seis – cinco
3. Palabras que contienen el sonido [x].
d) Cinco – cuatro
a) Taxi - auxilio
b) Excesivo - sexualidad e) Cinco - seis
c) Jarana - genial
d) Garra - anticuado
e) Guisante - guerrero

4. Palabra con sonido palatal.


a) Carnicería d) Enano 1. ¿Cuántos sonidos distintos hay en la palabra «chucherías»?
b) Cuchillos e) Mamá a) Seis
c) Mono b) Dos
c) Tres
5. Alternativa en la que los sonidos vocálicos son todos ante- d) Cuatro
riores. e) Siete

a) Mazamorra morada 2. No forma parte del aparato fonador o tracto vocal.


b) Lengua natural a) Lengua
c) Octavo tomo b) Dientes
d) Ciudad norteña c) Alveolos
e) Veinte tenientes d) Esófago
6. Alternativa en la que el acento cumpla función distintiva. e) Pared faríngea
a) Dijo que regresaría pronto.
b) La fe mueve montañas.
3. Por el modo de articulación, [f] es __________.
c) Es un muchacho muy cortés.
a) africado
d) Lava todos tus pantalones.
b) vibrante
e) El perro es blanco.
c) velar
7. Con respecto a la relación fonema-grafema del español, d) oclusivo
señala lo correcto. e) fricativo
a) Los fonemas son diecinueve, más la vocales suman
veinticuatro grafemas. 4. Señala la palabra que contenga el fono [č].
b) Los fonemas son veinticuatro y los grafemas también. a) Sancho
c) Los grafemas son veinticuatro y los fonemas, veintisiete. b) Excepto
d) Los fonemas son veinticuatro y los grafemas, veintisiete. c) Mono
e) Son veinticuatro fonemas incluyendo la h con valor se- d) Gota
miconsonante. e) Magia

132 Mejores Personas, Mejores Familias


5º AÑO DE SECUNDARIA
Colegio

LINGÜÍSTICA  Mejores Personas, Mejores Familias

5. Unidad de estudio de la fonética: 8. Señala la palabra con grafema dígrafo.


a) Morfema a) Agüita
b) Fonema b) Vehemencia
c) Morfo c) Rodrigo
d) Alomorfo d) Anguila
e) Fono e) Lingüística

6. Clases de fonemas suprasegmentales: 9. ¿Cuántos grafemas tienen las palabras «el chanchito«? (No
a) Acento y entonación tomar en cuenta las reiteraciones)
b) Tónico y átono a) Cinco
c) Simples y compuestos b) Nueve
d) Alargamiento y supresión c) Ocho
e) Consonantes y vocales d) Siete
e) Seis
7. Un grafema es ____________.
a) la representación gráfica de un fonema 10. Elija la palabra en que dos grafemas distintos representan
b) la segmentación que se ha materializado a través del el mismo fonema.
aparato fonador a) Jengibre
c) el sonido que se ha materializado a través del aparato b) Fumador
fonador c) Agujero
d) la variante que sufre el sonido al ser pronunciado d) Gitanita
e) cualquier unidad sonora e) Real

5º AÑO DE SECUNDARIA
Mejores Personas, Mejores Familias 133
Colegio

Mejores Personas, Mejores Familias LINGÜÍSTICA

M A
TE

05   MULTILINGÜISMO

Introducción ●● Q uechua norteño: quechua de Chachapoyas,


El perfil lingüístico de nuestro país presenta, como una de sus quechua de Cajamarca, quechua de Ferreñafe
particularidades esenciales y evidentes, un multilingüismo o (Incahuasi – Cañaris) y quechua de Lambayeque.
plurilingüismo manifiesto a través del empleo de dieciocho ●● Quechua central: quechua de Conchucos,
familias lingüísticas además del castellano. Las dieciséis quechua del Callejón de Huaylas, quechua del Alto
familias lingüísticas habladas en nuestra región amazónica la Pativilca, quechua de Huánuco y del Huallaga,
convierten en la de mayor complejidad lingüística; seguida de quechua Yaru, quechua Huanca y quechua de
la Sierra con dos familias lingüísticas vigentes. Tras la supuesta Yauyos.
extinción de sus lenguas amerindias (mochica, olmos, sechura, ●● Quechua costeño central: quechua de Pacaraos.
tallana, quingnam, colán, catacaos, pescadora, etc.; habladas ●● Quechua sureño: quechua ayacuchano (Chanca),
todavía a la llegada de los españoles), se considera actualmente y quechua cusqueño (Collao).
a la Costa como una zona totalmente castellanizada. Sin ●● Quechua de la Selva: quechua del Napo, quechua
embargo, debido a los fuertes movimientos migratorios de Pastaza, quechua de San Martín (Lamas), que-
peruanos del siglo XX, sobre todo de la Sierra a la Costa, chua santarrosino y quechua del Tigre.
encontramos miles de hablantes de lenguas amerindias todavía
vigentes en la Sierra o la Selva. b) Familia Aru:
Dos lenguas aru se consideran vigentes en la actua-
Las lenguas del Perú lidad: aimara y cauqui. A la lengua jaqaru se la con-
En el Perú existen hablantes monolingües y hablantes bilingües sidera extinta, aunque algunos todavía la consideran
o plurilingües. vigente, mientras que otros la consideran como un
En el territorio peruano se hablan dos tipos de lenguas: las dialecto de la lengua cauqui.
lenguas amerindias y las lenguas no amerindias.
●● A imara: También conocida como aru sureña o
A. Lenguas amerindias
aimara sureña. Tiene una gran trascendencia no
También conocidas como aborígenes, vernáculas, nativas,
solo en el país sino en Sudamérica, pues, además
oprimidas, etc. Son las lenguas de la América indígena,
de usarse en el Perú se usa en Bolivia y Chile. En
aparecidas en nuestros territorios antes de la llegada de los
nuestro país se usa en Puno, Moquegua, Tacna y,
occidentales. En nuestro país, estas lenguas se clasifican en
debido a la fuerte migración de puneños, también
lenguas andinas y lenguas amazónicas.
en Arequipa. Esta lengua no solo tiene como ene-
migo, en su lucha por la sobrevivencia, al castella-
1. Lenguas andinas
no; sino también al quechua cusqueño.
Estas lenguas tienen como territorio hegemónico los
●● Cauqui: Se encuentra en proceso de extinción,
accidentados relieves de la Sierra; aunque poco a poco vienen
como la gran mayoría de las lenguas amerindias
tomando clandestinamente las ciudades de la Costa. En
de todo el continente. Viene siendo desplazada
nuestro país tenemos dos importantes familias lingüísticas
por un dialecto castellano del sur de Lima. La
andinas: la familia lingüística quechua y la familia lingüística
gran mayoría de sus hablantes son bilingües. Se
aru.
usa todavía en los pueblos de Tupe, Colca, Aiza,
a) Familia Quechua:
Cotahuasi, Chavín y Canchán; todos ellos en la
Una gran variedad de lenguas clasificadas con la
provincia de Yauyos, al sur de Lima.
etiqueta de quechua o runa simi se usan a lo largo
y ancho de nuestro país. Veinte de los veinticuatro 2. Lenguas amazónicas
departamentos del Perú tienen quechuófonos. Son tantas y tan disímiles que han sido agrupadas en die-
No solo en el Perú se habla el quechua. En Bolivia, ciséis familias lingüísticas.
Argentina, Brasil, Ecuador, Colombia y Chile también a) Familia Arahua: lengua culina
se usan lenguas de la misma familia lingüística. b)  Familia Arahuaca: lengua campa–asháninca, lengua
Según datos del INEI, la lengua amerindia más campa caquinte, lengua chamicuro, lengua iñapari, len-
importante de nuestro país es el quechua, siendo el gua machigüenga, lengua campa nomatsigüena, lengua
quechua sureño el más expandido y con más hablantes piro, lengua resígaro y lengua yanesha (amuesha).
monolingües o bilingües.
c) Familia Bora: lengua bora
La investigadora Inés Pozzi Escot, en su libro  El
multilingüismo en el Perú, plantea la siguiente d)  Familia Cahuapana: lengua chayahuita y lengua jebero.
distribución de la familia lingüística quechua: e) Familia Candoshi: lengua candoshi–shapra

134 Mejores Personas, Mejores Familias


5º AÑO DE SECUNDARIA
Colegio

LINGÜÍSTICA  Mejores Personas, Mejores Familias

f) Familia Harakmbut: lengua Harakmbut 2. Socialmente, el bora es considerado un(a)__.


g) Familia Huitoto: lengua huitoto y lengua ocaina. a) dialecto geográfico
b) lengua sin prestigio
h)  Familia Jíbaro: lengua achuar–shiwiar, lengua agua-
runa y lengua huambisa. c) dialecto ágrafo
d) dialecto social
i)   Familia Pano: lengua amahuaca, lengua capanahua,
e) lengua natural
lengua cashibo–cacataibo, lengua cashinagua, lengua
matsés–mayoruna, lengua sharanahua–marinahua,
lengua shipibo–conibo, lengua yaminagua y    lengua 3. Alternativa en la que aparecen nombres de lenguas ame-
nagua. rindias.
a) Urarina, secoya, vasco
j) Familia Peba–yahua: lengua yahua
b) Yaminagua, portugués, iquito
k)  Familia Shimaco: lengua urarina c) Romanche, provenzal, catalán
l) FamiliaTacana: lengua ese eja d) Bora, quechua, aimara
m)  Familia Ticuna: lengua ticuna e) Iñapari, arabela, inglés
n)   Familia Tucano: lengua orejón y lengua secoya
4. El castellano estándar es el que____________.
o)  Familia Tupí–guaraní: lengua cocama–cocamilla y
a) se habla en Junín
lengua omagua
b) se habla en la selva
p)  Familia Záparo: Lengua arabela, lengua iquito y c) se usa en la educación
lengua taushiro d) se habla en Piura
e) se usa en todo Lima
Estas cuarenta y dos lenguas amazónicas conocidas se
usan a lo largo y ancho de nuestra región amazónica. Sin
5. La lengua chayahuita pertenece a la familia ________.
embargo, dialectos de estas lenguas se usan en países veci-
nos como Brasil, Colombia, Argentina o Bolivia. a) aru
b) jíbaro
B. Lenguas no amerindias  c) cahuapana
Las lenguas amerindias constituyen el cuerpo de lenguas d) bora
que ingresaron a nuestro territorio a partir del siglo XVI. e) candoshi
De todas ellas, el castellano aparece como la más impor-
tante; sin embargo, existe presencia significativa de otras 6. El aimara es _____________.
lenguas no amerindias como la portuguesa, la china y la a) una variedad o dialecto del quechua
japonesa que; a pesar de no tener mucha relevancia en el b) un dialecto sin gramática
mapa lingüístico peruano, tienen una valiosísima presen- c) el habla de los mestizos
cia cultural y económica. d) una lengua amerindia
e) una lengua amazónica
Situación de las lenguas en el Perú
Si la lengua es, como afirma Íbico Rojas, el último refugio 7. Es una lengua extinta.
de libertad que tiene el ser humano, esta libertad se respira a) Quechua d) Shipibo
irónicamente en nuestro país; pues, la Constitución Política
b) Asháninca e) Jaqaru
del Perú en su artículo segundo, inciso décimo noveno, dice:
c) Puquina
«Toda persona tiene derecho: (...) A su identidad étnica y
cultural. El Estado reconoce y protege la pluralidad étnica y
cultural de la Nación». 8. Familia lingüística amerindia con mayor distribución geo-
gráfica en el Perú.
a) Pano d) Arahuaca
b) Aimara e) Bora
c) Quechua

1. ¿Cuál es la lengua con mayor cantidad de hablantes en la 9. El castellano y el aguaruna están en relación
Amazonía? de____________.
a) Cocama a) lengua frente a lengua
b) Iñapari b) lengua frente a dialecto
c) Resígaro c) lengua frente a jerga
d) Quechua d) lengua frente a argot
e) Asháninca e) dialecto frente a dialecto

5º AÑO DE SECUNDARIA
Mejores Personas, Mejores Familias 135
Colegio

Mejores Personas, Mejores Familias LINGÜÍSTICA

10. ¿Qué lenguas son oficiales en el Perú? 6. La variedad quechua que cuenta actualmente con el mayor
a) Español porcentaje de hablantes es el ________.
b) Quechua a) quechua central
b) quechua norteño
c) Castellano
c) quechua costeño central
d) Aimara
d) quechua sureño
e) Todas las lenguas. e) quechua huanca

7. Señalala afirmación correcta:


a) El guaraní es un idioma oficial del Perú.
b) El quechua solo se habla en la Sierra peruana.
c) La caída del Imperio incaico propició la extinción de las
1. Actualmente, el Perú es considerado como un país multi- lenguas andinas.
lingüe, porque dentro de su territorio ____________. d) El quechua es conocido también como runasimi.
a) existen muchos dialectos e) En Chile no se habla quechua.
b) existen muchos analfabetos y mucha gente culta
8. Señala si las siguientes afirmaciones son verdaderas o falsas.
c) el castellano es la lengua dominante
En la Amazonía, hay más lenguas amerindías. ( )
d) se advierte empobrecimiento lingüístico
La lengua española evolucionó a partir del latín
e) se habla más de una lengua vulgar.  ( )
El quechua es la primera lengua nativa en números
2. Países en los que se habla aimara tradicionalmente. de habitantes en el Perú. ( )
a) Perú, Ecuador La familia Arú pertenece a la Amazonía. ( )
b) Uruguay, Bolivia a) FFVV d) VVVF
c) Bolivia, Perú b) VFFF e) VVFF
c) FVFV
d) Brasil, Bolivia
e) Colombia, Chile 9. Señala la alternativa en la que aparecen solo nombres de
lenguas amazónicas.
3. Señala el enunciado conceptualmente correcto. a) arahuaca, nagua, cauqui
a) El bilingüismo aparece solo en los países subdesarrollados. b) bora, chayahuita, huitoto
b) La lengua española es, demográficamente, la más im- c) yahua, portugués, omagua
portante del mundo. d) quechua, Ticuna, aimara
e) aimara, quechua, cauqui
c) En el Perú, la Amazonía es la región más compleja lin-
güísticamente.
10. Indica cuál no es una lengua selvática:
d) Las lenguas amerindias no evolucionan desde el siglo
a) Bora d) Asháninka
XVI.
b) Aguaruna e) Machiguenga
e) Demográficamente, lengua aimara es la más importan-
c) Jacaru
te de América.

4. El aimara también se habla en____________.


a) Argentina
b) Ecuador
c) Chile 1. Lingüísticamente, la sociedad peruana es ________.
a) bicultural d) plurilingüe
d) Brasil
b) multiétnico e) pluricultural
e) Uruguay
c) monolingüe

5. Son lenguas andinas. 2. Las lenguas nacionales se clasifican en lenguas


a) Mochica – cutle – castellano andino ______________.
b) Quechua – chipaya – aguaruna a) amerindias y no amerindias
c) Yunga – castellano – shipibo b) andinas y amazónicas
c) románicas y naturales
d) Aimara – jaqaru – puquina
d) castellano y quechua
e) Quechua – jibaro - cocama e) costeñas y nativas

136 Mejores Personas, Mejores Familias


5º AÑO DE SECUNDARIA
Colegio

LINGÜÍSTICA  Mejores Personas, Mejores Familias

3. Las lenguas amerindias no son denominadas, además len- 5. Señala la opción en la que figuran nombres de países en los
guas ___________. que se habla quechua tradicionalmente.
a) vernáculas a) Bolivia, Chile, Uruguay
b) aborígenes b) Ecuador, Colombia, Venezuela
c) oprimidas c) Brasil, Argentina, Panamá
d) nativas d) Argentina, Paraguay, Chile
e) lingüísticamente menos importantes e) Perú, Bolivia, Ecuador

4. Señala la afirmación incorrecta:


a) El quechua es lengua andina.
b) El quechua y el aimara no son lenguas indoeuropeas.
c) El quechua no es dialecto del castellano
d) El quechua y el castellano son lenguas indígenas perua-
nas.
e) El quechua y el castellano son lenguas oficiales del
Perú.

5º AÑO DE SECUNDARIA
Mejores Personas, Mejores Familias 137
Colegio

Mejores Personas, Mejores Familias LINGÜÍSTICA

M A
TE

06   MORFOLOGÍA: FORMACIÓN DE PALABRAS

Es la disciplina de la gramática que tiene como objeto de estudio la estructura interna de la palabra. (Del griego morphe: forma
y logos: tratado).

I. EL MORFEMA
Es la unidad mínima significativa del análisis morfológico. El morfema expresa conceptos significativos básicos, constituyén-
dose en la unidad gramatical mínima (indivisible) de una palabra.

NIÑ - O - S IN - QUIET - UD
Niñ
= infante In = negación
O
= masculino Quiet = estado de inmovilidad
S
= plural Ud = abstracción sustantiva

II. CLASES DE MORFEMAS


1. Lexical
Llamado también núcleo, lexema o raíz. Soporta la significación de la palabra. El significado léxico corresponde a «qué
significa una palabra». Pertenece al inventario lexicológico, por tal motivo, tiene su lugar en el diccionario. Su número es
ilimitado.

MESITA, CANTAREMOS, LIBROS, DESLEALTAD


2. Gramatical
Son las unidades mínimas que se anexan al morfema lexical y que aportan cargas significativas gramaticales. Al poseer
solo significado gramatical, no tienen lugar en el diccionario. Su número es limitado.

ALGUNOS, LLAMAREMOS, PANERA, MESITA

A. Morfemas Flexivos
Aportan los valores gramaticales de género, número, persona, tiempo y modo determinando a las categorías varia-
bles. No forman una nueva palabra.

SIMPLES AMALGAMA

Solo expresan un concepto: Expresan simultáneamente varios accidentes. Se


Género o número presenta en verbos.
GATA PENSÉ,
MUCHACHO 1ra persona
PAREDES singular
LIBROS pretérito indefinido
OSCUR —O –S modo indicativo

B. Morfemas Derivativos
Son aquellos que al anexarse a la raíz permiten la generación de una nueva palabra. Además, tienen la capacidad de
indicar a qué categoría gramatical pertenece la palabra. Se le suele llamar afijos.

Existen cuatro tipos importantes:


●● PREFIJOS: Son los que se anteponen a la raíz.

ANORMAL INÚTIL PREUNIVERSITARIO REHACER

138 Mejores Personas, Mejores Familias


5º AÑO DE SECUNDARIA
Colegio

LINGÜÍSTICA  Mejores Personas, Mejores Familias

●● SUFIJOS: Son los que se posponen a la raíz. Pueden ser:

AMANTE HÁBILMENTE PERUANO LAVAR

●● INFIJOS: Son aquellos que se incrustan dentro del lexema interrumpiéndolo.

ALVARITO ANITA LEONIDITAS

●● INTERFIJO: Carece de significado gramatical. Se inserta entre dos raíces o entre una raíz y un sufijo.

POLV(AR)EDA AUR(I)VERDE

DATOS IMPORTANTES:
●● MORFEMA : unidad mínima significativa (idea).
●● MORFO : representación gráfica de un morfema.
●● ALOMORFO : se denomina así a las diversas formas o posibilidades para expresar el mismo morfema (idea).
●● MORFEMAS LIBRES : pueden aparecer aisladamente en el habla.
Ejemplos : mar, sol, ayer, sí, no, hoy.


NOTA : los morfemas libres pueden ser ligados. Mar (libre) / Marino (ligado)

INÚTIL
MORFEMA MORFO ALOMORFOS IMPOSIBLE
Idea de negación INSANO
ANORMAL
ILEGAL



●● MORFEMAS LIGADOS : aparecen asociados a otros morfemas también ligados.
Ejemplos: in, des, pre.

CLASIFICACIÓN POR DISTRIBUCIÓN DE MORFEMAS


Clasificación por accidentes gramaticales

Raíz + morfema derivativo Insano, sanidad


DERIVACIÓN
(prefijos o sufijos) Subsuelo, impropio

Yuxtaposición Lexema + lexema (sin variar) Sacacorchos, abrelatas.


COMPOSICIÓN
Propiamente dicha Lexema + lexema (hay variación) Blanquirrojo, auriverde.

5º AÑO DE SECUNDARIA
Mejores Personas, Mejores Familias 139
Colegio

Mejores Personas, Mejores Familias LINGÜÍSTICA

Ropavejero
PARASÍNTESIS Composición + Derivación
Quinceañero

Ruidos Imitación seres inanimados Tictac, tintineo, chirriar


ONOMATOPEYA
Sonidos Imitación seres animados Zurear, aullar, maullar.

Otros procesos
1. Siglación: Palabra o término formado por las letras correspondientes a una sigla, que se ha lexicalizado: SARS (síndrome
respiratorio agudo y severo)
2. Acronimia: Palabra o término formado por la combinación de segmentos de palabras que forman una estructura sintagmá-
tica: automóvil + ómnibus = autobús.
3. Acortamiento: Se denomina en general acortamiento al procedimiento de creación de nuevas palabras o neologismos que
consiste en abreviar palabras mediante diversos metaplasmos de supresión: bici por bicicleta, boli por bolígrafo, pelu por pe-
luquería, insti por instituto…

6. Es el proceso que resulta de la combinación de la compo-


sición y la derivación.
a) Flexión
b) Prefijación
1. ¿Cuál de los siguientes enunciado es correcto con relación c) Sufijación
a la morfología? d) Onomatopeya
a) Disciplina que estudia las características sonoras de las e) Parasíntesis
palabras
b) La rama de la gramática que estudia la composición 7. Señala la palabra que se ha formado por derivación.
oracional a) Camposanto d) Rehuir
c) La unidad de las sintaxis b) Abrelatas e) Honesto
d) Disciplina que estudia la estructura interna de las pala- c) Lustrabotas
bras
e) Ciencia que estudia la relación entre las palabras
8. Señala la palabra que se ha formado por prefijación.
a) Cartero d) Prehistoria
2. ¿Cuál es la denominación del elemento subrayado de la si-
guiente palabra rebuscar? b) Antiquísimo e) Caprichoso
a) Morfema derivativo d) Infijo c) Cortaplumas
b) Raíz e) Morfema flexivo
c) Lexema 9. Señala la palabra que se ha formado por composición en la
forma de yuxtaposición.
3. Señala la palabra que carece de morfema gramatical. a) Prehistoria d) Agridulce
a) Paces c) Ojera e) Arboleda
b) Inutilizable e) Cielorraso
b) Leona d) Tos
c) Pordiosero

4. Señala la palabra que presenta morfema flexivo simple.
a) Soy c) Bestial e) Fulano 10. Señala la palabra que se ha formado por composición pro-
b) Fuiste d) Amé piamente dicha.
a) Inútil d) Alicaído
5. Señala la palabra que contiene morfema derivativo. b) Ternura e) Pisapapeles
a) Pies c) Alabaré e) Calle c) Rompecabezas
b) Id d) Balonazo

140 Mejores Personas, Mejores Familias


5º AÑO DE SECUNDARIA
Colegio

LINGÜÍSTICA  Mejores Personas, Mejores Familias

9. El proceso de unión de dos lexemas con variación morfo-


lógica en uno de ellos es ______.
a) composición por yuxtaposición
b) composición propiamente dicha
1. ¿Cuál de las siguientes palabras presenta infijo? c) derivación
a) Tigrillo d) Javiercito d) parasíntesis
b) Omarcito e) Osquitar e) onomatopeya
c) Leoniditas
10. Indicar la alternativa en la que hay una palabra derivada.
2. Los sustantivos son incapaces de presentar_____.
a) Blanquiazul
a) morfema lexema
b) Submarino
b) morfema flexivo
c) morfema derivativo c) Agridulce
d) morfema flexivos de género d) Tocadiscos
e) morfema amalgama e) Cortaplumas

3. ¿Cuál es la denominación del elemento subrayado de la si-


guiente palabra: gallina?
a) Morfema flexivo de género
b) Morfema flexivo de número
c) Morfema derivativo 1. Rama lingüística que estudia la estructura interna de las
d) Morfema lexema palabras:
e) Morfema derivativo simple a) Sintaxis d) Fonética
b) Fonología e) Lexicología
4. Morfológicamente una palabra es _______.
c) Morfología
a) unidad sonora separable de otras unidades
b) unidad funcional dentro de la oración
c) unidad portadora de base significativa 2. Identifica la palabra monomorfemática.
d) unidad compuesta de unidad o unidades significativas a) Vio d) Mujeres
e) unidad separable en fonos b) Iban e) Destapar
c) Corazón
5. Señala la opción en donde la palabra presenta dos morfe-
mas: uno derivativo y otro flexivo. 3. Reconoce la palabra polimorfemática.
a) Brisas c) Lealtad e) Rojizo a) Tesis d) Pies
b) Rosal d) Relojes b) Síntesis e) Viernes
c) Sabor
6. Señala la palabra que no se ha formado por parasíntesis.
a) Sietemesino d) Trapecista
4. Señala lo incorrecto.
b) Quinceañera e) Picapedrero
I. Lo mínimo que tiene una palabra es morfema lexical.
c) Pordiosero
Resolución: II. Los morfemas gramaticales son derivativos y flexivos.
En una palabra parasintética debe presentar como mínimo III.Las categorías invariables presentan morfemas flexivos.
dos lexemas o raíces. La palabra trapecista solo presenta a) I y II d) Solo II
uno (trapec-). Rpta.: d. b) II y III e) Solo III
c) I, II y III
7. Es un proceso que consiste en formar palabras a partir de
los sonidos de animales o cosas. 5. Identifica las categorías invariables.
a) Derivación I. Sustantivo
b) Composición II. Adverbio
c) Parasíntesis III.Adjetivo
d) Onomatopeya
IV.Preposición
e) Prefijación
V. Conjunción
8. La voz onomatopéyica del toro es _____. VI.Pronombre
a) crasita
b) gorgoritea a)I, III y VI d) IV, V y VI
c) zurea b) II, IV y V e) I, II y III
d) otila c) III, IV y VI
e) brama

5º AÑO DE SECUNDARIA
Mejores Personas, Mejores Familias 141
Colegio

Mejores Personas, Mejores Familias LINGÜÍSTICA

6. Palabra formada por yuxtaposición. 9. La composición propia es un procedimiento morfológico


a) Desteñir que consiste en _________.
b) Quinceañero a) la unión de dos palabras sin modificación genética.
c) Yegua b) la unión de las iniciales de determinadas palabras.
d) Rompecabezas c) reducción silábica de una palabra.
e) Luna d) combinación de composición y derivación.
e) unión de dos lexemas con modificación genética.
7. Alternativa que presenta acronimia:
a) CGTP 10. Palabra formada por composición propia.
b) Hombre lobo a) Agridulce
c) Luna de miel b) Quitamanchas
d) ovni c) Volante
e) salchipapa d) Investigando
e) Aguadito
8. La parasíntesis es un proceso que consiste en ___.
a) añadir solo morfemas derivativos.
b) unión únicamente de dos lexemas.
c) la reducción de sílabas de la palabra.
d) la imitación de sonidos de los seres.
e) la combinación de derivación y composición.

142 Mejores Personas, Mejores Familias


5º AÑO DE SECUNDARIA
Colegio

LINGÜÍSTICA  Mejores Personas, Mejores Familias

MA
TE

07   TILDE DIACRÍTICA

Existen algunos monosílabos y polisílabos que, a pesar de tener la misma escritura y pronunciación, no tienen la misma función
gramatical. Para diferenciar el valor gramatical de cada uno de ellos, se usa la tilde diacrítica.

ZZ Tú recuperarás muy pronto tu dinero.


Pron. Adj.

I. MONOSÍLABOS: (con tilde diacrítica)

YY Él Pronombre Las cosas las trajo él.


El Artículo El amor de mi vida has sido tú.

YY Tú Pronombre Tú y solo tú puedes entenderme.


Tu Adjetivo Me gusta tu manera de hablar.

YY Mí Pronombre Las cosas serán todas para mí.


Mi Adjetivo Mi casa es tu casa, amiga mía.

YY Sí Afirmación Sí, yo escribí aquello.


Sí Pronombre Lo quiere todo para sí.
Si Condicional Volveré, si tú me lo pides.

YY Sé Verbo “ser” Sé tú mismo, no imites.


Sé Verbo “saber” Solo sé que nada sé.
Se Pronombre El hombre se quedó callado.

YY Té Sustantivo Sírveme un té de buena calidad.


Te Pronombre Yo te necesito, en verdad, mucho.

YY Dé Verbo “dar” Quiere que le dé un premio.
De Preposición Viene de Barranco.

YY Más Cantidad Cada día te quiero más.


Mas Conjunción Hazlo, mas no te enojes.

II. BISÍLABO
YY Aún Adverbio (todavía) NOTA: El monosílabo que también se tilda es TÉS como
Aún sigo esperando a mi madre. plural de TÉ.
Ejemplo:
YY Aun Adverbio (incluso, hasta, también) Cada día encontramos tés aromatizados de más y más sabores:
Aun dormida, te recita el poema de Wilder. florales, afrutados, dulces, especiados...

5º AÑO DE SECUNDARIA
Mejores Personas, Mejores Familias 143
Colegio

Mejores Personas, Mejores Familias LINGÜÍSTICA

9. ¿En qué alternativa SE deberá tildarse?


a) Se que tú te marcharás
b) Se muere de vergüenza.
c) Se pasó de vueltas.
1. Se que tu me perdonaras. En la expresión se han omitido d) Ese anillo se oxidó por ser barato.
_____ tildes diacríticas. e) Es necesario, que se dé cuenta.
a) tres c) dos e) siete
10. Caso en que la palabra AUN debe llevar tilde:
b) cuatro d) seis
a) ¿Todos, aun ella, me odian?
b) Sé que aun me espera una oportunidad.
2. Señale qué palabra corresponde al acento diacrítico. c) Ni aun en sueños te olvido.
a) Ti c) Vi e) Ni d) Ella no sonríe aun en fiestas
b) Di d) Si e) Seremos aun más que amigos.

3. Seleccione la opción donde se presenta escritura correcta.
a) Dijo que si vendría a la reunión.
b) Me envió veintiséis álbumes y tres tés.
c) Se que no hubo reunión.
d) Yo sé por que se retiró anoche. 1. ¿Cuál de las siguientes oraciones presenta tilde diacrítica y
general?
e) a y b
a) Eran unos dóciles jóvenes: muy confiables.
b) Jamás me di cuenta que me sonreía siempre.
4. ¿Qué monosílabos jamás se tilda? c) Pensé en lo que sería de nosotros si no siguiera mis prin-
a) Su, te, di d) O, ti, des cipios.
b) De, mi ,tu e) Si, se, vi d) El oidor no pudo quejarse de mí ni de mi labor.
c) Fue, fe , se e) Volveré, y tú y solo tú lo sabes por ahora.

5. Elija una alternativa correctamente tildada. 2. Marque la opción en la que la palabra DE esté incorrecta-
a) Solo tu y nadie más que tu nos salvará. mente tildada.
b) Ganamos 30 ó 40 soles de más. a) Lo cierto es que quiero que dé más de sí.
c) Yo sé que todo irá mejor. b) Aún te amo y aún te espero, aunque no dé señales de
d) Lo haré aún cuando tenga miedo. vida.
c) Así dé muestras de cariño, yo lo amo.
e) Pedro le dió un beso a María.
d) Si un día dé sol llega, es que tú ya tocaste mi puerta.
e) Para mí, dé hasta lo que no tiene.
6. La ____________ es aquella que permite distinguir pala-
bras que tienen idéntica forma, pero pertenecen a distinta 3. Presenta correcto uso del acento ortográfico.
categoría gramatical. a) Sólo a aquella joven pertenece mi corazón.
a) tilde d) diéresis b) No és aun hora para salir.
b) acentuación e) ortografía c) Yo se bien que sé dice.
c) tilde diacrítica d) Julia sé caso de blanco.
e) María no conoce la noticia aún, pero él, sí.
7. ¿En qué alternativa la palabra “si” no lleva tilde?
a) Volvió en si. 4. En: “Yo se bien lo que ha sufrido mas no por eso debo ser
b) Si aceptó. mas condescendiente con tu hermano; al contrario, debere
c) “El si de las niñas” ser mas exigente si cabe”, hace falta __________ tildes.
d) No voy, si me apuras. a) tres d) seis
b) cuatro e) siete
e) Quisiera decir si.
c) cinco

8. Señale la oración que contenga tilde correcta. 5. Presenta uso incorrecto de la tilde.
a) Yo por tí soy capaz de todo. a) Sé caritativo con los dé aquí.
b) Hace más de un siglo que el peruano aguarda. b) No me dé explicaciones de lo ocurrido.
c) Hubo grandes compositores que tu debes tener en tú c) No me dé usted tanta importancia por el hecho de ha-
colección. ber pasado el examen.
d) Sólo no es una buena manera de vivir. Debes buscar d) No quiero desayunar aún.
compañía. e) Si supiéramos el porqué de esta decisión, podríamos ir
e) La situación más ruín de todas, fue ésta. a París.

144 Mejores Personas, Mejores Familias


5º AÑO DE SECUNDARIA
Colegio

LINGÜÍSTICA  Mejores Personas, Mejores Familias

6. Presenta uso incorrecto de la tilde. 3. Monosílabo que nunca se tilda:


a) Habiéndose escapado el animal, decidiose ir en su bús- a) De c) Te e) Di
queda. b) Se d) Tes
b) Ella y él son amigos de mi hermano.
c) Esta pluma es mucho mejor que esa, pero yo prefiero 4. En: “A mi nadie me dijo que tu no ibas a poder venir a tomar
aquella. el te”, hacen faltan ____________ tildes diacríticas.
d) Esta sonata es en sí bemol. a) tres c) cinco e) siete
e) El decímetro es diez veces mayor que el centímetro y b) cuatro d) dos
cien veces mayor que el milímetro.
5. Es un monosílabo diacrítico:
7. Presenta uso incorrecto de la tilde. a) Fe c) Le e) Aun
a) Aun el inválido salió precipitadamente. b) Ti d) Si
b) Yo sé la lección de Química.
c) Aurelio dio la conferencia y luego se fue solo al campo. 6. ¿Cuál de las siguientes palabras no debió tildarse?
d) En este libro hay prologo; en aquél no. a) Tu razón d) Es táctil
e) Solo me lo dijo a mí, a él, no. b) Se feliz e) Dieciséis
c) Se volvió
8. Presenta uso incorrecto de la tilde sobre la palabra MAS.
a) Intenté asistir, más no pude. 7. ¿Cuál es la palabra que lleva tilde en la sílaba incorrecta?
b) Antonio quiere más mantequilla. a) Dí la verdad.
c) Hoy trabajé más que ayer, porque soy más. b) Quiero un té de canela.
d) Aun los más alejados pudieron verlo. c) Dé la noticia.
e) Dos más dos son cuatro. d) Oblíguesele a él.
e) Sí se puede.
9. Señale la oración en la que se debe colocar la mayor canti-
dad de tildes. 8. ¿Cuál es la palabra que no debió tildarse?
a) No se si se fue a la casa de Victor. a) Vuelve en tí.
b) Cesar me vendio el baúl a buen precio. b) Solo me gustas tú.
c) A ti o a el le daran te en la mañana. c) Sé valiente.
d) Ines vendra el 28 o 29 de julio. d) En sí, nada se sabe.
e) Se que si tu lo pides que te de, lo hara. e) Ella eres tú.

10. Presenta correcta tildación. 9. «Tu y el, llenos de ímpetu, decidieron bajar colgados de un
a) Vi que tú y ella huían del lugar. arnés hacia el fondo del volcán; mas no tomaron en cuenta
b) Ayer leí dos ó tres páginas más. un pequeño túnel abierto en la ladera del cráter. Al llegar
c) Él siempre elogia mí esfuerzo. al fondo, se tomaron varios tes calientes para el frío».
d) Sé que, sí me empeño, lo haré. ¿Cuántas tildes faltan en el texto anterior?
e) Pedro le dió un beso a María. a) Tres c) Cinco e) Siete
b) Cuatro d) Cero

10. «En estos veinte días hemos consumido las cuatro cabras
que teníamos y nuestras reservas de dátiles prácticamente
han desaparecido. Además, ya estoy harta de que siempre
1. En: “Tu aún debes hacer lo que diga mi tia”, hacen faltan nos cuente las mismas historias y no nos deje trabajar».
____________ tildes diacríticas. ¿Cuántas tildes faltan en el texto anterior?
a) tres c) cinco e) siete a) Tres c) Cinco e) Siete
b) cuatro d) dos b) Cuatro d) Cero

2. En: “Yo si le pedi que me comprara un manometro, pero se


vino sin el”, hacen faltan ____________ tildes diacríticas.
a) tres c) cinco e) siete
b) cuatro d) dos

5º AÑO DE SECUNDARIA
Mejores Personas, Mejores Familias 145
Lingüística
Colegio

Mejores Personas, Mejores Familias LINGÜÍSTICA

MA
TE

08   EL SUSTANTIVO

LOS SUSTANTIVOS O NOMBRES B. (POR SU ESTRUCTURA) Simples o


compuestos
1. Los sustantivos (Definición) ●● Simples: El sustantivo está formado por un lexe-
YY Semántica: Según la Nueva gramática de la lengua es- ma.
pañola, indica personas, animales, cosas reales o ima- ●● Compuestos: El sustantivo está formado por dos
ginarias, grupos, entidades materiales o inmateriales, o más lexemas.
sucesos» Los sustantivos compuestos pueden estar formados
YY morfológica: variable por la unión de dos sustantivos:
YY sintáctica: núcleo del sintagma nominal Ejemplo: balón + cesto = baloncesto.
YY lexicológico: inventario abierto lexema lexema
2. Las funciones de los sustantivos
●● Por un sustantivo y un verbo:
La función principal del sustantivo es la de Núcleo
Ejemplo: Sacar + muela = sacamuelas.
del Sujeto.
●● Por una frase entera: correveidile, compuesta de las
YY La pelota cayó en el suelo.
palabras: corre + ve + y + dile.
PELOTA (sustantivo) es el núcleo del sujeto de esta
Las palabras compuestas, aunque se compongan
oración.
de dos raíces, pasan a tener un solo significado.
YY El aeroplano aterrizó sin novedad. A E ROPL A NO
(sustantivo) es el núcleo del sujeto de la oración.
C. (Por su extensión) comunes o propios
●● Comunes: Nombran a cualquier ser u objeto de
También desempeña la función de núcleo de un sintagma
nominal o de un sintagma preposicional, aunque no sea una misma clase.
sujeto. ●● Propios: Nombran a un ser o a un objeto, distin-
Ejemplos: La amiga de mi tío ha comprado un loro. guiéndolo de los demás seres de su misma clase.
sust. sust. sust. Los nombres propios se escriben siempre con le-
tra mayúscula a principio de palabra.
3. Clases de sustantivos
D. (Por su cantidad) individuales o colectivos
A. (POR SU 0RIGEN) Primitivos o derivados ●● Individuales: Son aquellos que en singular nom-
●● Primitivos: Se trata de un sustantivo que no procede bran a un solo ser.
de otro sustantivo del castellano. ●● Colectivos: Son aquellos que en singular nom-
●● Derivados: Se trata de un sustantivo que se ha
bran a un conjunto de seres.
formado a partir de otro sustantivo. Los sustanti-
vos derivados se forman añadiendo al lexema, un
E. (Por su naturaleza) concretos o abstractos
morfema derivativo.
●● Concretos: Son aquellos que nombran seres u ob-
jetos que podemos tocar, ver, oír, oler, notar, etc...
●● Abstractos: Son aquellos que nombran ideas o
sentimientos, que no podemos percibir por nues-
tros cinco sentidos.

146 Mejores Personas, Mejores Familias


5º AÑO DE SECUNDARIA
Colegio

LINGÜÍSTICA  Mejores Personas, Mejores Familias

F. (Por su contabilidad) contables o incontables 4. Señala la alternativa que presenta solo sustantivos colecti-
●● Contables: Son aquellos que nombran seres u objetos vos derivados.
que se pueden contar por unidades.
a) Archipiélago, colonia, persona
●● Incontables: Son aquellos que nombran seres,
b) Cornamenta, festivo, feligresía
objetos o cosas que no pueden contarse por uni-
c) Sindicato, hueste, follaje
dades.
d) Fusiles, lodazal, borrasca
G. Animados o inanimados
e) Catacumbas, voces, borricada

●● A
nimados: Nombran seres que se mueven por 5. En el enunciado: Lo malo del día fue la pérdida de unos do-
sí mismos, que tienen vida (personas o animales, cumentos; mas lo bueno es que los encontré junto a los libros
u objetos personificados aunque sean de ficción del gabinete, la cantidad total de sustantivos es _____.
(unicornio)).
●● Inanimados: Nombran objetos que no se mueven a) seis b) ocho c) siete
por sí mismos, ni tienen vida propia. d) cuatro e) cinco

Ejemplo de análisis morfológico 6. Por su naturaleza, «astucia» es un sustantivo ____.


En el análisis morfológico, cuando aparezca un
SUSTANTIVO, hay que describirlo con todas las a) colectivo b) abstracto c) concreto
características anteriormente estudiadas. Así, si te- d) derivado e) primitivo
nemos el sustantivo perro, lo haremos de la manera
siguiente: 7. La alternativa en la que hay más sustantivos comunes es
YY Perro: sustantivo común, masculino, singular, _________
concreto, individual, contable, simple, primitivo,
animado a) Edith y sus amigos llegaron ayer.
b) Los jugadores viajarán a Huancayo.
c) El poeta de la soledad escribirá muchas poesías.
d) Los padres de Ariana tendrán una reunión.
e) La doctora García no logró viajar a Ica.
1. La categoría léxica que cumple la función de núcleo del sin-
tagma nominal es el _________. 8. Señala la alternativa en la que hay solo sustantivos colectivos.

a) verbo b) adjetivo c) adverbio


a) Gentío, clero, jauría
d) artículo e) nombre o sustantivo
b) Maniobra, piara, pinar
2. Morfológicamente, el nombre _______. c) Roquedal, terruño, rebaño
d) Bohío, sanatorio, viñedo
a) describe acción o estado e) Tonel, rosal, mar
b) determina cuantitativamente
c) es una categoría variable 9. La concordancia que se establece entre el sustantivo y sus
d) determina cualitativamente modificadores directos es de ______.
e) es una categoría invariable
a) persona y número
b) número y género
3. Señala la alternativa en la que todos los sustantivos son
c) persona y género
abstractos.
d) número solamente
e) persona solamente
a) Tristeza, luz, valor
b) Color, oscuridad, aire
10. Señala la alternativa que presenta solo sustantivos patronímicos.
c) Pecado, sudor, dolor
d) Olvido, caridad, maldad a) Lima, Iglesia, Chávez
e) Gemido, respiración, arte. b) Álvarez, Martín, José
c) Rodrigo, Álvaro, Domingo
d) Rodrígez, Álvarez, Domínguez
e) Fernández, Fernando, Benítez

5º AÑO DE SECUNDARIA
Mejores Personas, Mejores Familias 147
Colegio

Mejores Personas, Mejores Familias LINGÜÍSTICA

8. Señala el sustantivo que en singular designan un grupo.

a) Contados b) Contables c) Colectivos


d) Continuos e) Infinitos
1. Señala los enunciados correctos sobre el sustantivo.
9. ¿Cuál es el sustantivo gentilicio correcto?
I. También se le llama nombre.
II. Carece de género y número. a) Finlandeño b) Hungarano c) Chipriano
III.Su inventario es abierto. d) Portorricense e) Chalaco
a) I y II b) I y III c) I, II y III
d) Solo I e) Solo III 10. Señala cuál no es derivado aumentativo.

2. ¿Qué alternativa presenta solo sustantivos compuestos? a) Islote b) Barcaza c) Papelote


d) Cucharón e) Tazón
a) Asimismo – rápidamente
b) Debe – maremoto
c) Cortaúñas – hazmerreír
d) Saltamontes – memorial
e) Desaguadero – amicísimo
1. Señala los sustantivos primitivos.
3. ¿Qué alternativa solo presenta sustantivos?
a) Escritor, hielo, cualidad
a) Ciprés – computó – sedentario b) Sed, tierra, mar
b) Descanso – desolaron – bibliográfica
c) Odio, flor, maldad
c) Rutina – rubro – abatimiento
d) Goloso – adelante – esperanzas d) Ganancia, amor, río
e) Encarará – inexorable – tregua e) Candor, paloma, plumaje

4. Los diminutivos de Dios, bribón y carreta son, respectiva- 2. ¿Cuántos sustantivos hay en el fragmento?
mente: ¡Qué pura, Platero, y qué bella esta flor del camino! Pasan a
su lado todos los tropeles - los toros, las cabras, los potros, los
a) Diócesis – bribonzuelo – carretilla hombres.
b) Diosito – briboncito – carretilla
c) Diosdado – bribonito – carretita a) Seis b) Nueve c) Siete
d) Diosote – bribonzuelo - carretín d) Cuatro e) Cinco
e) Diosín – briboncito – carretita
3. Señala la relación incorrecta.
5. Señala la alternativa que presenta solo nombres topóni-
a) Hijo: simple b) Aire: concreto
mos.
c) Niñera: primitivo d) Tunecino: gentilicio
a) Rímac, Rodríguez, Carlos e) Persona: individual
b) Álvarez, Martínez, Pérez
c) Álvaro, Martín, Pedro 4. ¿Cuál de ellos es un sustantivo colectivo?
d) José, Amazonas, Sánchez
e) Loreto, Abancay, Tacna a) Colmena b) Concilio c) Concierto
d) Librería e) Establo
6. Uno de los siguientes sustantivos es singular, concreto y
colectivo. 5. Señala el enunciado en el que el sustantivo «motivos» no
forma parte del predicado.
a) Atenuante b) Sartén c) Agua
d) Hada e) Macolla a) Me preocupan los motivos de su enojo.
b) Tenía motivos para enojarse.
7. ¿Cuál es el sustantivo que designa en forma genérica a to- c) Hoy nos explicará los motivos de su huida.
dos los seres de una misma especie?
d) Tenía duda de los motivos expuestos.
a) Propio b) Denominativo c) Común e) Aseguraba que los motivos eran ciertos.
d) No connotativo e) Cardinal

148 Mejores Personas, Mejores Familias


5º AÑO DE SECUNDARIA
Colegio

LINGÜÍSTICA  Mejores Personas, Mejores Familias

MA
TE

09   EL ADJETIVO CALIFICATIVO

I. DEFINICIÓN 2. Adjetivo pospuesto o especificativo


YY E
l adjetivo calificativo es aquella palabra que semántica-
mente indica características o propiedades del nombre: juez Señala una especificación que restringe la referencia
honesto, noche romántica. propia del sustantivo.
YY Morfológicamente es variable (género y número). Ejemplos:
YY Sintácticamente funciona como modificador directo. n «El duro mármol», no se designa una clase o
●● E
YY Lexicológicamente es de inventario abierto. especie de «mármol» que se oponga a otro que
YY El adjetivo calificativo es dependiente semántica y no sea «duro», sino que simplemente se describe
funcionalmente del nombre. cómo es el mármol; a este adjetivo antepuesto, le
llamaremos explicativo o epíteto.
Ejemplos: En el pasadizo nebuloso ●● En «La casa vieja», la intención del hablante es la de
cual mágico sueño de Estambul señalar una clase opuesta a otras, como podrían ser:
su perfil presenta destelloso la casa nueva, la casa bella, la casa remodelada; a este
la niña de la lámpara azul adjetivo pospuesto lo llamaremos especificativo.
●● Más ejemplos de adjetivos explicativos o epítetos: La
II. POSICIÓN DEL ADJETIVO EN EL GRUPO NOMINAL oscura noche, una alada mariposa, el encendido fue-
El adjetivo adyacente de un sustantivo puede anteponerse go, el árido desierto.
o posponerse a este. El valor del adjetivo es variable según ●● M
ás adjetivos especificativos: La mujer morena, el
su posición. hombre inteligente, el cabello limpio, el régimen
democrático.
1. Adjetivo antepuesto:

También a veces llamado epíteto o explicativo, revela que Pero, el valor de la anteposición o proposición del adje-
el hablante tiende a explicar y describir la realidad suge- tivo depende muchas veces de la intención del hablante
rida por el sustantivo. que de criterios objetivos.
ZZ En ocasiones, la posición del adjetivo genera cambios en
su significado:

Confrontemos

ANTEPUESTO PROPUESTOS

YY Una verdadera alegría (= gran) YY Una elegría verdadera (= cierta)


YY Buen amigo (= gran) YY Amigo bueno (= bondadoso)
YY Gran jefe (= con grandeza) YY Jefe grande (= de tamaño)
YY nuevo libro (= recien editado) YY Libro nuevo (= apenas usado)
YY Pobre hombre (miserable) YY Hombre pobre (= sin recursos)
YY Rara cualidad (= no frecuente) YY Cualidad rara (= extravagante)

Mejores Personas, Mejores Familias 4 149


5º AÑO DE SECUNDARIA
Colegio

Mejores Personas, Mejores Familias LINGÜÍSTICA

III. GRADOS DE SIGNIFICACIÓN DE LOS ADJETIVOS CALIFICATIVOS

GRADO DEFINICIÓN EJEMPLO

Es aquel grado en el cual la cualidad se ZZ Mujer bonita


POSITIVO
expresa de forma natural. ZZ Animal feroz

Comparativo o de igualdad
Es aquel grado en el cual la cualidad se ZZ Ella es tan alta como él.
exagera o señala de una forma superior.
Comparativo de inferioridad
COMPARATIVO
ZZ Ella es menos alta que él.
Comparativo de superioridad
ZZ Ella es más alta que él.

Es aquel grado en el cual la cualidad se Relativo: Se expresa la cualidad en forma


exagera o señala de una forma superior. superior, pero existe un límite semántico que
empieza con la preposición «de»
ZZ Ela es la más alta del salón
ZZ Ella es la menos alta del salón

Absoluto: Se expresa la cualidad en el grado


SUPERLATIVO máximo. Puede ser de dos clases:
1. Perifrásico: Con el apoyo de adverbios.
YY Muy alta, sumamente alta, excesiva-
mente alta.
2. Sintético: Colocando al adjetivo en grado po-
sitivo las terminaciones «ísimo», «énimo» o
«entísimo»
YY Crudelísimo (de cruel)
YY Misérrimo (de mísero)
YY Ardentísimo (de ardiente)

4. ¿Qué alternativa presenta un adjetivo en grado comparati-


vo de igualdad?

a) Es el más hábil de todos los alumnos.


1. Indica cuántos adjetivos calificativos hay en el siguiente b) Tiene más prestigio que otros.
texto: Pisaré las tristes calles y en una hermosa plaza re- c) Es tan popular como su compañero.
cordaré nuestros buenos momentos. d) Tiene menos bienes que su hermano.
e) Es demasiado bueno contigo.
a) Dos b) Cuatro c) Tres
d) Uno e) Cinco
5. ¿Qué adverbios intervienen en la formación del grado
comparativo del adjetivo?
2. Reconocer el adjetivo en la siguiente oración: Los enfermos
rápidamente fueron atendidos por el hábil colaborador.
a) Muy, suavemente
b) Más, menos, tan
a) Rápidamente b) Enfermos c) Hábil
c) Que, como
d) Los e) Colaborador
d) Mucho, también
e) Menos, demasiado
3. Señala la alternativa que presenta un adjetivo en grado su-
perlativo.
6. ¿Cuántos adjetivos hay en: Las turbulentas aguas del río
caudaloso envolvieron con ímpetu salvaje a las débiles bal-
a) Eso está lejísimo. sas que luchaban por mantenerse a flote?
b) Es tardísimo para eso.
c) Esos niños están muy hambrientos.
a) Uno b) Cuatro c) Dos
d) Habla muy claro.
d) Cinco e) Tres
e) Regresa lo más rápido posible.

150 Mejores Personas, Mejores Familias


5º AÑO DE SECUNDARIA
Colegio

LINGÜÍSTICA  Mejores Personas, Mejores Familias

7. Sobre el adjetivo es verdadero que __________. 3. ¿Qué adjetivo señala cualidades inherentes del sustantivo
al que modifica?
a) modifica al sustantivo
a) Calificativo b) Gentilicio c) Epíteto
b) reemplaza al pronombre
d) Explicativo e) Especificativo
c) es complemento del verbo
d) tiene accidente de tiempo 4. ¿En qué frase hay un adjetivo explicativo?
e) acompaña siempre al determinante
a) Rosas hermosas
b) Cristales de muy buena calidad
8. Señala la alternativa que presenta un adjetivo en grado su-
c) Comida rica y barata
perlativo.
d) Entusiasmados jóvenes
e) Instrumento afinado
a) Tiene más dinero que su hermano.
b) Es muy fácil mentir para ti. 5. ¿Qué grado del adjetivo permite comparar las cualidades
c) La vida es dura. de dos o más sujetos?
d) Es más alto que Carlos.
a) El grado superlativo
e) Vámonos rápidamente. b) El grado positivo
c) El grado comparativo
9. En la oración: Ningún hombre debe ser indiferente a su en- d) El grado superlativo relativo
torno ni a su realidad, la palabra subrayada es _________. e) El grado superlativo absoluto

a) pronombre b) adjetivo c) verbo 6. ¿En qué opción hay un adjetivo calificativo?


d) adverbio e) sustantivo
a) Entusiasmo b) Conspiración c) Ironía
d) Amabilidad e) Sarcástico
10. Reconoce el adjetivo en la siguiente oración: Muy tarde lle-
gó el triste hombre calladamente.
7. ¿En qué alternativa el adjetivo se ha utilizado en grado su-
perlativo relativo?
a) Muy b) Hombre c) Triste
d) Calladamente e) Tarde
a) Los eventos de esta semana fueron mejores que los tu-
yos.
b) Estas son las chicas más hermosas del país.
c) Fue un día terrible.
d) Todos son más amables que tú.
e) Ninguno es mayor que tú.
1. Escribe el grado superlativo absoluto perifrásico de los si-
guientes adjetivos. 8. ¿Qué grado del adjetivo determina una cualidad sin exage-
ración ni alteración?
ZZ Amable ___________________
ZZ Mísero ___________________ a) El grado superlativo
ZZ Afable ___________________ b) El grado positivo
ZZ Bueno ___________________ c) El grado comparativo
ZZ Malo ___________________ d) El grado superlativo relativo
e) El grado superlativo absoluto
2. Completa las siguientes frases con un adjetivo epíteto.
9. ¿En qué oración hay un adjetivo superlativo irregular?

ZZ Pasto ___________________ a) Es un ser supremo.


ZZ Nieve ___________________ b) Ha sido amabilísimo con nosotras.
ZZ Noche ___________________ c) Su alma paupérrima conmueve a todo el barrio.
ZZ Sufrimiento ___________________ d) La purísima Virgen nos guiará.
ZZ Rocas ___________________ e) Sus comentarios fueron bonísimos.

5º AÑO DE SECUNDARIA
Mejores Personas, Mejores Familias 151
Colegio

Mejores Personas, Mejores Familias LINGÜÍSTICA

10. ¿Qué adjetivo es el que de manera subjetiva se antepone al 3. Indica el adjetivo que no sea calificativo.
sustantivo?
a) Bueno b) Amable c) Mártir
a) El adjetivo especificativo d) Poco e) Feo
b) El adjetivo explicativo
c) El adjetivo epíteto 4. Indica la oración que presenta adjetivo en grado positivo.
d) El adjetivo gentilicio
e) El adjetivo calificativo
a) Julito, échale crema a esas manos maltratadas.
b) Wílber es demasiado exigente con los trabajos.
c) Martín es muy hablador.
d) Tus ojos son más grandes que el de Rosita.
e) Estuvo bastante callada durante la última reunión.
1. Indica cuántos adjetivos hay en el siguiente texto: Feliz, el
gran día cuando conocí a tan bella mujer. 5. Marca la alternativa en la que el significado del adjetivo
aparece en grado superlativo absoluto.
a) Dos b) Cinco c) Tres
d) Uno e) Cuatro a) Sé mucho más que tú.
b) Comprobó que María era tan hábil como Sofía.
2. El adjetivo es una categoría gramatical variable. La defini- c) Su biblioteca era la más grande de la ciudad.
ción anterior es de tipo _______. d) Todos creen que Olga es muy diferente.
e) Tu bicicleta es tan bonita como la mía.
a) sintáctico b) lexicológico c) semántico
d) fonológico e) morfológico

152 Mejores Personas, Mejores Familias


5º AÑO DE SECUNDARIA
Colegio

LINGÜÍSTICA  Mejores Personas, Mejores Familias

MA
TE

10   DETERMINATIVOS: ADJETIVOS DETERMINATIVOS

I. LOS ADJETIVOS DETERMINATIVOS


Son los adjetivos que limitan o restringen la significación del sustantivo.

II. ADJETIVOS DEMOSTRATIVOS


Son aquellos adjetivos que se encargan de señalar la ubicación del sustantivo mencionado en relación con el hablante.

Cerca del hablante Este, esta, esto, estas

Cerca del oyente Ese, esa, esos, esas

Lejos de ambos Aquel, aquella, aquellos, aquellas


ZZ Esta noche veremos ese programa de televisión. ZZ Esos alumnos entrarán a aquel salón.
Adj. Adj. Adj. Adj.
Dem. Dem. Dem. Dem.

III.ADJETIVOS POSESIVOS
Son aquellos que modifican al sustantivo señalando pertenencia o posesión.

Para un solo poseedor Para varios poseedores

Mío, mía, míos, mías Nuestro, nuestra, nuestros, nuestras

Tuyo, tuya, tuyos, tuyas Vuestro, vuestra, vuestros, vuestras

Suyo, suya, suyos, suyas Suyo, suya, suyos, suyas

Apocopados: mi, tu, su, mis, tus, sus (Solo funcionan como adjetivos)

ZZ Esos caprichos tuyos son imposibles de cumplir. ZZ Nuestras intenciones son nobles.
Adj. Adj.
Pos. Pos.

5º AÑO DE SECUNDARIA
Mejores Personas, Mejores Familias 153
Colegio

Mejores Personas, Mejores Familias LINGÜÍSTICA

IV. ADJETIVOS NUMERALES


Son aquellos que indican cantidad u orden en relación con el sustantivo por este modificado.

ZZ Cardinales: Indican número exacto (uno, dos, tres, etc.)

ZZ Ordinales: Indican sucesión numérica (primero, último, vigésimo, etc.)

ZZ Partitivos: Indican fracción (media, octava, doceava, etc.)

ZZ Múltiplos: Indican multiplicidad (doble, triple, quíntuple, etc.)

ZZ Distributivos: Indican repartición (sendos, cada, ambos)


Nota: «Sendos» significa «uno para cada uno»

Ejemplos:

ZZ Tres puertas hay abiertas. ZZ Siempre queda en último lugar.


Adj. Adj.
Num. Num.

V. ADJETIVOS INDEFINIDOS
Son aquellos que modifican al sustantivo de una forma vaga o imprecisa. Indican cantidad, cualidad, existencia, identidad;
pero de forma indeterminada.

Clasificación

ZZ Indefinidos identificativos: Mismo, otro, demás (femeninos y plurales)

ZZ Indefinidos cuantitativos: Poco, mucho, demasiado, harto, todo, varios, bastante, más, menos, cierto
(femeninos y plurales)

ZZ Indefinidos intensivos: Tanto, tal (femeninos y plurales)

ZZ Indefinidos existenciales: Algún, ningún, cualquier (femeninos)

ZZ Sin muchas dificultades pasaron a la siguiente ronda.


Adj.
Ind.

ZZ Lo evaluaron con las mismas preguntas del examen anterior.


Adj.
Ind.

154 Mejores Personas, Mejores Familias


5º AÑO DE SECUNDARIA
Colegio

LINGÜÍSTICA  Mejores Personas, Mejores Familias

7. Señala la serie correcta.


El distraído individuo no supo contestar aquella pregunta.
1 2 3

1. Señala lo correcto. a) Sustantivo - adjetivo - sustantivo


I. El adjetivo calificativo determina al sustantivo. b) Sustantivo - adjetivo - adjetivo
II. Jamás puede haber un verbo entre un sustantivo y un c) Adjetivo - sustantivo - adjetivo
adjetivo. d) Sustantivo - sustantivo - pronombre
III.Sendos es un adjetivo numeral.
e) Adjetivo - sustantivo - sustantivo
a) Solo I b) I y II c) Solo II 8. Relaciona correctamente.
d) II y III e) Solo III
I. Mío A. Demostrativo
2. Relacione correctamente.
II. Otro B. Indefinido
I. Triple A. Cardinal III.Esas C. Posesivo
II. Dos B. Ordinal
III. Décimo C. Múltiplo a) IC, IIB, IIIA
b) IB, IIA, IIIC
a) IC, IIA, IIIB b) IB, IIC, IIIA c) IA, IIC, IIIB
c) IA, IIB, IIIC d) IC, IIB, IIIA d) IC, IIA, IIIB
e) IB, IIA, IIIC e) IB, IIC, IIIA

3. «Cualquier caballero que venga para tocar estos escudos 9. «Logró establecer con claridad ciertas metas. Solo dos días
ha de traer dos escudos con las armas pintadas». antes de que venciera el breve plazo». Señala la clase de
El número de sustantivos modificados por adjetivos determi- adjetivos.
nativos en la oración anterior es ____.
a) Posesivo, numeral, calificativo
a) dos b) cinco c) tres b) Indefinido, ordinal, ordinal
d) seis e) cuatro c) Demostrativo, cardinal, calificativo
d) Indefinido, numeral, calificativo
4. «Se notifica a los alumnos de este centro de estudios que si e) Indefinido, indefinido, indefinido
en este examen no aprueban Lenguaje, se verán en grandes
apuros». La clase de adjetivos determinativos que predomi- 10. «Tu estancia en el campo te separará un buen tiempo de las
na en el texto anterior es _______. frecuentes amanecidas». Señala a qué clase corresponde la
palabra subrayada.
a) demostrativo b) calificativo c) numeral
d) gentilicio e) posesivo a) Adjetivo demostrativo
b) Adjetivo posesivo
5. Señala la oración en la cual «medio» funcione como adjetivo. c) Adjetivo numeral
d) Adjetivo indefinido
I. Medio país se volcó a las calles. e) Adjetivo calificativo
II. Me expuso una teoría medio confusa.
III.He comprado medio pollo.

a) Solo I b) I y III c) Solo II
d) II y III c) Solo III

6. Señala lo correcto. 1. ¿En qué oración se ha subrayado un determinante nume-


I. Un adjetivo epíteto señala lugar de origen. ral?
II. Esto es un adjetivo demostrativo.
III.Mi es apócope de mío. a) Aquella pobre muchacha engañada cerró los ojos.
b) Son solo cinco los que esperan.
a) Solo I b) I y II c) Solo II c) Ambos serán elegidos.
d) II y III e) Solo III d) Tres profesoras presentarán su proyecto.
e) Los nueve serán premiados.

5º AÑO DE SECUNDARIA
Mejores Personas, Mejores Familias 155
Colegio

Mejores Personas, Mejores Familias LINGÜÍSTICA

2. ¿En qué oración la palabra subrayada funciona como de- 10. ¿Cuál es un determinante numeral múltiplo?
terminante?
a) Tercio b) Ambas c) Tres
a) Pocos son los afortunados. d) Cuarto e) Triple
b) Me regresaré solo con unos pocos.
c) Si me vendes unos pocos, te lo agradeceré.
d) Pocos comentarios escuché de ti.
e) Me dijo que eran suficientes unos pocos.

3. ¿En qué oración la palabra subrayada funciona como de- 1. «Cesaremos algunos mecanismos que tenemos para apar-
terminante? tarte de esa frivolidad en que vives». El adjetivo indefinido
del texto anterior es _______.
a) Con unos cinco estoy feliz.
b) Solo son cinco. a) usaremos b) era c) algunos
c) Cinco minutos me bastarán. d) que e) tenemos
d) Estuvieron en la fila los cinco solos.
e) Cinco más cinco es diez.
2. Señala lo correcto.
4. ¿En qué oración la palabra subrayada funciona como de-
terminante? I. Si un adjetivo determinativo va antes del sustantivo, ne-
cesariamente sufre una apócope.
a) Los nuestros son mejores. II. Los adjetivos posesivos señalan pertenencia.
b) Espérame con el nuestro listo. III.El adjetivo determinativo epíteto señala la cualidad innata del
c) Vendimos nuestro departamento. sustantivo.
d) Si seguimos a los nuestros, ganaremos.
e) Con los nuestros llegaremos. a) Solo I b) I y II c) Solo II
d) I, II y III e) Solo III
5. ¿En qué oración la palabra subrayada funciona como de-
terminante?
3. El sustantivo y el adjetivo concuerdan en _________ y
a) Aún espera al primero. __________.
b) Debes ser el primero en todo.
c) Lo primero eres tú. a) número y persona
d) El primer lugar lo ocupó su perro. b) género y persona
e) Los primeros no fueron vacunados. c) tiempo y modo
d) género y número
6. ¿Cuál es un determinante posesivo? e) masculino y femenino

a) Ambos b) Muchos c) Unos 4. «Encontró un tercer pájaro muerto en el escaño que eligió
d) Demasiados e) Sus para sentarse». El sustantivo que tiene más modificadores
es ______.
7. ¿Cuál es un determinante indefinido?
a) tercer b) muerto c) pájaro
a) Un b) Su c) Algunos d) escaño e) sentarse
d) Medio e) Tres
5. Indica la alternativa que presenta más determinantes.
8. ¿Cuál es un determinante numeral cardinal?
a) Ese vendió los tres últimos ejemplares de su colección.
a) Medio b) Su c) Diez b) Aquel niño travieso rompió los vidrios de la ventana.
d) Poco e) Hartas c) Ellos no tienen el proverbial espíritu triunfador.
9. ¿Cuál es un determinante numeral partitivo? d) Tus mejores caballos murieron en la estampida.
e) Los dos pajarillos posaron sobre la mesa de mármol.
a) Mucha b) Doble c) Tercera
d) Medio e) Cada

156 Mejores Personas, Mejores Familias


5º AÑO DE SECUNDARIA
Colegio

LINGÜÍSTICA  Mejores Personas, Mejores Familias

MA
TE

11   EL ARTÍCULO

I. CRITERIO SEMÁNTICO
Es una palabra vacía de significado, solo posee valor gramatical debido a que indica el género y el número de los nombres.

II. CRITERIO MORFOLÓGICO


Es una categoría gramatical variable, con accidentes gramaticales de género y número, que deben de concordar con los del
nombre.

III.CRITERIO LEXICOLÓGICO
Es una categoría de inventario cerrado (ya que se consideran solo nueve artículos en el español; difícilmente se incorporará más
palabras a su lista o inventario).

IV. CRITERIO SINTÁCTICO


Funciona como modificador directo (MD) del sustantivo.

CLASIFICACIÓN DE LOS ARTÍCULOS

ARTÍCULOS DEFINIDOS

NÚMERO
GÉNERO
SINGULAR PLURAL

Masculino El Los
Femenino La Las
Neutro Lo

ARTÍCULOS INDEFINIDOS

NÚMERO
GÉNERO SINGULAR PLURAL

Masculino Un Unos

Femenino Una Unas

Neutro

1. Posible confusión con algunos pronombres personales


Los artículos definidos son homónimos de los pronombres personales: él, la, los, las, lo. Al respecto, dos observaciones:
a) El pronombre «él» se distingue por el acento diacrítico.
b) Los pronombres «lo, los, la, las» se diferencian de los artículos porque acompañan a los verbos en posición proclítica
(antecedente).

5º AÑO DE SECUNDARIA
Mejores Personas, Mejores Familias 157
Colegio

Mejores Personas, Mejores Familias LINGÜÍSTICA

Ejemplos:
FUNCIÓN
PALABRA
ARTÍCULO PRONOMBRE
ZZ Lo bello ZZ Lo atendieron
LO
Art Adj. sust Pron. Vb
ZZ Los animales ZZ Los sentenciaron
LOS
Art. sust. Pron Vb
ZZ La mañana ZZ La encontró
LA
Art. sust. Pron. Vb
ZZ Las palomas ZZ Las venció
LAS
Art. sust. Pron Vb.

2. El artículo neutro «lo» y la sustantivación


El artículo neutro «lo» otorga calidad de nombre a los adjetivos calificativos precedidos por este.
Ejemplos:

COMO ADJETIVO COMO SUSTANTIVO

ZZ Su bello rostro ZZ Lo bello de su rostro

ZZ Fue un suceso maravilloso ZZ Lo maravilloso del suceso

ZZ Tomamos un trago amargo ZZ Lo amargo de aquel trago

d) pronombre - artículo
e) pronombre - pronombre

5. Señala cuál de las siguientes palabras no puede cumplir la


1. «Lo vi nuevamente y descubrí lo curioso, lo incomprensi- función de adjetivo determinante.
ble de su dolor». ¿Cuál es un caso de sustantivación?
I. Lo vi a) Estos b) Mío c) Nuestro
II. Lo curioso d) Pocos e) Tiempo
III. Lo incomprensible
a) Solo I b) Solo II c) Solo III 6. Señala cuál es uno de los accidentes gramaticales del artí-
d) I y II e) II y III culo.

2. Señala la alternativa que incluye artículo definido. a) Persona b) Tiempo c) Modo


I. La imagen atormenta continuamente a Luisa. d) Aspecto e) Género
II. Lo que te hace falta es práctica.
III. La verdad salió a relucir. 7. ¿Cuántos sustantivos modificados por artículos hay en el
a) Solo I b) Solo III c) I, II, III siguiente fragmento?
d) Solo II e) I y III «Tú me levantas, tierra de Castilla, en la rugosa palma de
tu mano».
3. Señala la oración que incluya un artículo definido.
a) No tarda en llegar. a) Dos b) Cinco c) Tres
b) ¿A qué volviste? d) Uno e) Cuatro
c) Dime la verdad ahora mismo. 8. Es una contracción.
d) Ya no la quiero, es cierto.
e) La quise de todas maneras. a) Él b) Del c) De
d) Le e) Ante
4. «Entonces lo seguí con la mirada». Las palabras subraya-
das son respectivamente _______. 9. La cantidad de artículos en la gramática española es _____.
a) adjetivo - artículo
b) pronombre - adjetivo a) dos b) nueve c) tres
c) artículo - artículo d) diez e) seis

158 Mejores Personas, Mejores Familias


5º AÑO DE SECUNDARIA
Colegio

LINGÜÍSTICA  Mejores Personas, Mejores Familias

10. Algunos artículos definidos se pueden confundir con cier- 7. ¿Cuál es la función que cumple el artículo en la frase nominal?
tos _________.
a) Objeto directo
a) adjetivos numerales b) Núcleo del sujeto
b) sustantivos abstractos c) Modificador indirecto
c) pronombres posesivos d) Modificador directo
d) pronombres personales e) Núcleo del predicado
e) verbos en voz pasiva
8. ¿Qué artículo es correcto para la siguiente frase?
“_______ águila rápida”.
a) La b) El c) Unas
d) Unos e) Los

1. ¿En qué alternativa la palabra subrayada es un artículo? 9. ¿Qué artículo es correcto para la siguiente frase?
“_________ agua contaminada”.
a) Lo mejor del paseo fue la piscina.
a) La b) El c) Los
b) Lo encontramos en el paseo.
d) Las e) Una
c) Lo dejé triste y desesperado.
d) Lo dijo ayer.
10. ¿En qué frase encontramos al artículo neutro?
e) Lo descubrí en ese oscuro cuarto.
a) El pueblo de los hombres
2. ¿En qué alternativa la palabra subrayada no es un artículo? b) La casa de mi tía
c) La bonita profesora
a) Busca siempre la sinrazón. d) Lo malo de los vicios
b) Adelante veremos la luna. e) Unas fresas deliciosas
c) Adelante la veremos.
d) La mejor alumna del salón era su vecina.
e) Quizás la vecina no quiera.

3. ¿En qué alternativa la palabra subrayada no es un determi-


1. Señala la oración que presenta un artículo.
nante?
a) Tuvo que irse de prisa.
a) Con una como ella, aprenderemos. b) Uno de nosotros no irá.
b) Ella lo sigue como una fanática. c) Era tarde cuando una voz me llamó.
c) Sírveme una copa más. d) Entonces la trajo aquí.
d) Por una semana, paseé en el Cusco. e) Esa cabaña se derrumbará.
e) Empezaremos una nueva vida.
2. El artículo y el _______ modifican al sustantivo.
4. ¿En qué alternativa la palabra subrayada es un artículo? a) adverbio b) verbo c) pronombre
d) adjetivo e) sustantivo
a) Uno fue mejor que otro.
b) Con uno es más tranquilo. 3. Presenta artículo indefinido.
c) Es un buen tipo mi papá.
d) Sigue siendo solo una. a) Uno para todos y todos para uno.
e) Unas se irán por la izquierda. b) Sí, fui el único para ti.
c) Una mirada bastó; así sucedió.
5. Señala en qué opción hay un artículo contracto. d) Uno no sabe lo que sucederá mañana.
e) Unos van y otros vienen.
a) Una b) El c) Lo
d) Del e) Las 4. En la oración: Una buena cosecha da un buen vino, las
palabras subrayadas son ______.
6. ¿Qué accidente gramatical tiene el artículo? a) artículos neutros
b) artículos indefinidos
a) Tiempo b) Cantidad c) Género c) pronombres personales
d) Número e) C y D d) determinantes posesivos
e) números ordinales

5º AÑO DE SECUNDARIA
Mejores Personas, Mejores Familias 159
Colegio

Mejores Personas, Mejores Familias LINGÜÍSTICA

MA
TE

12   PRONOMBRE Y SUS CLASES

I. DEFINICIÓN
1. Criterio semántico
El pronombre es una categoría que no presenta un significado fijo, sino meramente ocasional. Es decir, su contenido
dependerá del contexto lingüístico o extralingüístico.

2. Criterio morfológico
El pronombre es una categoría gramatical variable que presenta accidentes gramaticales de género, número y persona.

3. Criterio sintáctico
El pronombre cumple con todas las funciones propias del sustantivo, como las de núcleo del sujeto, objeto directo, objeto
indirecto, etc.

4. Criterio lexicológico
El pronombre es una categoría de inventario cerrado; por lo tanto no admite incorporación de nuevos pronombres.

II. CLASIFICACIÓN
1. Pronombres personales
Son aquellos que señalan a las personas intervinientes en una conversación.

Clasificación sintáctica de los pronombres personales


Se trata de una clasificación que atiende a la función que dichos pronombres cumplen dentro de la oración.

ZZ Subjetivos: Son aquellos que funcionan como Yo, tú, vos, él, ella, ello, nosotros, nosotras, vosotros,
sujeto. vosotras, ellos, ellas, usted, ustedes
ZZ Objetivos: Son aquellos que funcionan como
Lo, los, la, las, le, les, me, te, se, nos, os
objetos directos o indirectos.
ZZ Terminales: Son aquellos que funcionan como
Mí, ti, sí, conmigo, contigo, consigo
término.

NOTA: Los pronombres objetivos (denominados átonos porque carecen de acento en su estructura)
de acuerdo con su posición pueden ser proclíticos o enclíticos.
YY Proclíticos: Si están precediendo al verbo.
– Se lo dijo. Me la compré.
YY Enclíticos: Si se anexan al verbo o verboide.
– Con verbos: Dímelo. Cómpraselo.
– Con infinitivo: Decírselo. Dejarlo.
– Con gerundio: Estudiándolo. Dándoselo.

2. Pronombres determinativos
A. Pronombres demostrativos
Son los que indican distancia temporal o espacial en relación con las personas gramaticales. Son los siguientes: este,
ese, aquel (femeninos y plurales)

B. Pronombres posesivos
Son los que indican pertenencia en relación con las personas gramaticales. Son los siguientes: Mío, tuyo, suyo,
mí, tú, nuestro, vuestro (plurales y femeninos)

160 Mejores Personas, Mejores Familias


5º AÑO DE SECUNDARIA
Colegio

LINGÜÍSTICA  Mejores Personas, Mejores Familias

C.
Pronombres numerales
Son los que indican cantidad, orden o repartición.

Clasificación:

ZZ Cardinales: Indican números exactos (uno, dos, tres, etc.)

ZZ Ordinales: Indican sucesión numérica (primero, último, vigésimo, etc.)

ZZ Partitivos: Indican fracción (media, octava, doceava, etc.)

ZZ Múltiplos: Indican multiplicidad (doble, triple, quíntuple, etc.)

ZZ Distributivos: Indican repartición, el único es «ambos».

D.
Pronombres indefinidos
Son los que indican cantidad indefinida. Son los siguientes: poco, mucho, alguno, otro… (femeninos y plurales).

3. Pronombres relativos
Son aquellos que hacen referencia a sustantivos antes mencionados.

CON SUSTANTIVO
CON SUSTANTIVO ANTECEDENTE
CONSECUENTE
ZZ (el), (la) Cual ZZ Quien ZZ Cuyo
QUE
ZZ (los, las) cuales ZZ Quienes ZZ Cuya
Admite todo tipo de Admite solo antecedente de ZZ Cuyos
Usan artículos
antecedente rasgos humanos ZZ Cuyas

4. Pronombres enfáticos
Son aquellos pronombres que indican pregunta o admiración. Son los siguientes: qué, quién (es), cuál (es), cuánto (a) (s).

4. Los pronombres _________ subordinan verbos.

a) demostrativos b) indefinidos c) posesivos


d) relativos e) numerales
1. Conocida como aquella categoría que reemplaza al nom-
bre. 5. Señala la oración que incluye un pronombre enclítico.

a) Sustantivo b) Pronombre c) Verbo a) No tarda en llegar.


d) Conjunción e) Adjetivo b) ¿A qué volviste?
c) Dime la verdad ahora mismo.
2. Categoría que posee género, número y persona: d) Ya no la quiero, es cierto.
e) Le compré un delicioso helado de fresa.
a) Verbo b) Preposición c) Adjetivo
d) Pronombre e) Nombre 6. Los pronombres personales objetivos que se anteponen al
verbo se denominan_____________.
3. Señala la oración que incluye pronombre personal.
I. La imagen atormenta continuamente a Luisa. a) relativos b) proclíticos c) posesivos
II. Lo que te hace falta es práctica. d) demostrativos e) enclíticos
III. La verdad salió a relucir.
7. Semánticamente el pronombre _____________ de signifi-
cado____________ .
a) Solo I b) I y II c) Solo II
d) I, II y III e) Solo III a) posee - propio b) alude - referencial
c) alude - aludido d) carece - propio
e) carece - referencial

5º AÑO DE SECUNDARIA
Mejores Personas, Mejores Familias 161
Colegio

Mejores Personas, Mejores Familias LINGÜÍSTICA

8. Los pronombres se clasifican en posesivos, _____ relati- 4. ¿En qué oración la palabra subrayada no funciona como
vos, interrogativos, exclamativos e _______. pronombre?
a) Con unos cinco estoy feliz.
a) cuantitativos - infinitivos b) Solo son cinco.
b) indefinidos - infinitivos c) Cinco horas estuve esperando a mi querido esposo,
c) demostrativos - indefinidos pero no llegó.
d) correlativos - interrogativos d) Estuvieron en la fila los cinco tutores solos.
e) demostrativos - interrogativos e) Solo me comí dos.

9. Los pronombres ______ se refieren a las personas del dis- 5. ¿En qué oración la palabra subrayada funciona como pro-
curso. nombre posesivo?

a) Los nuestros son mejores.


a) exclamativos b) demostrativos c) neutros b) Espérame con nuestro proyecto listo.
d) indefinidos e) personales c) Vendimos nuestro departamento.
d) Si seguimos a nuestros padres, ganaremos.
10. Los pronombres personales se subclasifican, a su vez, e) Con nuestros propios medios llegaremos.
en_______ y________.
6. ¿En qué oración la palabra subrayada funciona como pro-
nombre numeral?
a) tónicos y atómicos
b) átonos y tónicos a) Aún espera al primer puesto.
c) sonoros y sordos b) Debes ser el primer hombre en todo.
d) lentos y débiles c) El primero eres tú.
e) altos y débiles d) El primer lugar lo ocupó su hermana.
e) Los primeros concursantes mejoraron más.

7. ¿Qué palabra siempre funciona como un pronombre


demostrativo?

a) Esa b) Aquella c) Esta


1. ¿En qué oración la palabra subrayada no es un pronom- d) Mis e) Eso
bre?
8. ¿En qué alternativa la palabra subrayada es un pronombre?
a) Pocos son los damnificados.
b) Me regresaré solo con unos pocos. a) Lo mejor del paseo fue la piscina.
b) Lo encontramos en el paseo.
c) Si me vendes unos pocos, te lo agradeceré.
c) Lo más bello se lo pedimos.
d) Pocos comentarios escuché de ti. d) Lo bueno fue ayer.
e) Me dijo que eran suficientes unos pocos. e) Lo lindo de su mirada me atrae demasiado.
2. ¿En qué oración se ha subrayado un pronombre numeral? 9. ¿En qué alternativa la palabra subrayada no es un pronom-
bre?
a) Aquella pobre muchacha engañada cerró los ojos.
b) Son solo cinco los que esperan. a) Busca siempre la sinrazón.
c) Ambos primos serán elegidos. b) Adelante la veremos.
d) Cada profesora presentará su proyecto. c) La contrató su vecina.
e) Los nueve niños serán premiados. d) Quizás la quiera más.
e) Esta la guardaremos.
3. ¿En qué oración la palabra subrayada es un pronombre
indefinido? 10. ¿En qué alternativa la palabra subrayada no es un pronom-
bre?
a) Pocos hombres son los afortunados.
b) Me regresaré solo con unos pocos alumnos. a) Con una como esa, aprenderemos.
c) Si me vendes unos pocos polos, te lo agradeceré. b) Ella lo sigue como una loca.
c) Sírveme una más.
d) Pocos comentarios escuché de ti.
d) Con una, paseé en el Cusco.
e) Me dijo que eran suficientes unos pocos.
e) Empezaremos una de nuevo.

162 Mejores Personas, Mejores Familias


5º AÑO DE SECUNDARIA
Colegio

LINGÜÍSTICA  Mejores Personas, Mejores Familias

4. ¿Cuál es la diferencia entre los demostrativos: pronombre


- adjetivo?

1. Los pronombres___________ son aquellos que poseen a) Los pronombres son modificadores directos y los adje-
acento propio en su significante. tivos son núcleo de la F. N.
b) Los pronombres son núcleo de la F.V. y los adjetivos son
a) átonos b) tónicos c) posesivos núcleo de la F.N.
c) Los pronombres son modificadores indirectos y los ad-
d) demostrativos e) neutros
jetivos son modificadores del nombre.
d) Los pronombres pueden ser núcleo de la F.N. y los adjetivos
2. Los pronombres__________ son aquellos que carecen de
son modificadores directos del nombre.
acento en su significante, por lo que siempre se pronun-
e) No hay diferencia entre el pronombres y el adjetivo.
cian con el acento del verbo.
5. Señala la alternativa correcta con referencia a las palabras subra-
a) tónicos b) átonos c) posesivos yadas en María me dijo la verdad. Entréganos todo lo que tienes.
d) demostrativos e) neutros Avísame a la hora de salida.

3. Los pronombres neutros se hallan en los pronombres de- a) Unas son pronombres; otros, adjetivos.
nominados_______________. b) Todas son pronombres.
c) Todas son adjetivos.
a) indefinidos b) posesivos c) relativos d) Unas son pronombres; otras, adverbios.
d) demostrativos e) interrogativos e) Todas son preposiciones.

5º AÑO DE SECUNDARIA
Mejores Personas, Mejores Familias 163
Colegio

Mejores Personas, Mejores Familias LINGÜÍSTICA

MA
TE

13   EL VERBO Y SUS ACCIDENTES GRAMATICALES

I. DEFINICIÓN
Vocal
1. Morfológica Infinitivo Conjugación
temática
El verbo es una palabra variable: Esto significa que posee
accidentes gramaticales que son los siguientes: número,
cant-ar a primera
persona, tiempo y modo.
tem-er e segunda
2. Sintáctica
El verbo cumple la función sintáctica de núcleo del
part-ir i tercera
predicado.

3. Semántica IV. NÚMERO Y PERSONA


El verbo es una categoría gramatical encargada de indi-
car el comportamiento realizado por un sujeto. Existen 1. La persona gramatical
diferentes clases de verbos: verbos de acción, verbos de
estado, verbos de movimiento, verbos de percepción, etc.
●● L a primera persona se identifica con el hablan-
4. Lexicológica te, o hablantes. Puede llevar como sujeto los pro-
El verbo es una palabra de inventario abierto: Esto sig- nombres personales yo, nosotros, nosotras.
nifica que se pueden incorporar nuevos verbos a este ●● La segunda persona es la persona a la que se ha-
inventario. bla. Lleva como sujeto los pronombres personales
tú, vosotros, vosotras.
II. ESTRUCTURA ●● La tercera persona se refiere a todo ser u obje-
to que no es ni la primera ni la segunda persona.
El verbo admite muchas formas distintas, que son resulta- Puede llevar como sujeto los pronombres perso-
do de combinar dos partes: nales él, ella, ellos, ellas, usted, ustedes, y además
YY La raíz o lexema del verbo soporta el significado se- todos los sustantivos.
mántico: cant-ar, com-er, part-ir.
La raíz se obtiene quitando al infinitivo la terminación 2. El número
-ar, -er o -ir: cant-ar, beb-er, sal-ir.
YY Las desinencias son las terminaciones que se añaden Las tres personas gramaticales pueden aparecer en sin-
a una misma raíz para obtener las distintas formas de gular o en plural.
un verbo, y que aportan significados gramaticales:
cant-o, cant-abais, cant-arán. V. TIEMPO, MODO Y ASPECTO
YY Estos significados son: persona, número, tiempo, modo y 1. El tiempo
aspecto.
Las formas verbales sitúan la acción en un tiempo de-
III. CONJUGACIONES VERBALES terminado, que puede ser presente, pretérito (pasado)
Los verbos se agrupan en tres conjugaciones según la vocal y futuro.
temática que presenten al comienzo de sus desinencias. La
vocal temática se aprecia claramente en la terminación de
los infinitivos:

164 Mejores Personas, Mejores Familias


5º AÑO DE SECUNDARIA
Colegio

LINGÜÍSTICA  Mejores Personas, Mejores Familias

Modos Tiempos simples Tiempos compuestos

Presente (amo) Pretérito perfecto compuesto (he amado)

Pretérito imperfecto (amaba) Pretérito pluscuamperfecto (había amado)

Indicativo Pretérito perfecto simple (amé) Pretérito anterior (hube amado)

Futuro (amaré) Futuro perfecto (habré amado)

Condicional (amaría) Condicional perfecto (habría amado)

Presente (ame) Pretérito perfecto (haya amado)

Pretérito pluscuamperfecto (hubiera ama-


Subjuntivo Pretérito imperfecto (amara / amase)
do/ hubiese amado)

Futuro (amare) Futuro perfecto (hubiere amado)

Imperativo Presente (ama)


2. El modo

Manifesta la acción del verbo, y hay tres tiempos:

●● E l indicativo es el modo de la objetividad, de la 1. ¿Qué expresión presenta un verbo en aspecto perfectivo?


imparcialidad del hablante; este se limita a expo-
ner un hecho pasado, presente o futuro: Se acerca a) Todos festejaban en primer triunfo de Garcilaso.
una tormenta. b) Yo habré dado fin a esa relación tormentosa.
●● El subjuntivo es el modo de la subjetividad, de la c) Petra Celestina escribía sobre los pueblos clásicos ama-
participación emocional del hablante frente a la zónicos.
acción; expresa un deseo, temor, posibilidad, duda, d) Los problemas políticos creaban el caos.
etc.: Que tengas suerte. e) El campesino quería justicia real.
También se usa para formular órdenes en oracio-
nes negativas: No vengas mañana. 2. Es incorrecto afirmar con respecto al verbo que ____.
●● El imperativo es el modo del mandato; se utiliza
para dar órdenes o instrucciones en las oraciones a) Presenta accidente de género en todos los casos
afirmativas: Ven pronto. b) tiene tres accidentes privativos
c) es núcleo del predicado verbal
3. El aspecto d) pueden ser irregular o regular
e) presenta tiempo simple y compuesto
El verbo puede informar también sobre el desarrollo
de la acción. El aspecto caracteriza la acción desde el 3. ¿En qué oración encontramos un verbo en modo subjunti-
punto de vista de su transcurso o de su término. vo?
Hay dos aspectos:
a) Habría vuelto con rabo entre las piernas.
●● P erfectivo: presenta la acción terminada, tanto si es b) Tal vez haya otro lugar para mí.
presente, como si es pasada o futura: A las ocho habré c) Te podría decir que hay una lucha.
llegado a Londres. d) hubo salida por unas protestas.
●● Imperfectivo: muestra la acción sin terminar, en su e) Cantaba por la libertad.
transcurso: Mañana volaré a Londres.
El aspecto se expresa mediante desinencias, pero
también con algunas perífrasis verbales.

5º AÑO DE SECUNDARIA
Mejores Personas, Mejores Familias 165
Colegio

Mejores Personas, Mejores Familias LINGÜÍSTICA

4. El _______ se desarrolla en relación al momento en que se


realiza la acción verbal.

a) modo b) número c) persona


d) aspecto e) tiempo 1. Morfológicamente, parte del verbo que contiene los acci-
dentes de persona y número.
5. Indique la alternativa correcta respecto al verbo.
a) radical b) derivativo c) alomorfo
a) Posee tres accidentes, de los cuales dos son compartidos d) característica e) desinencia
con los pronombres.
b) Está constituido por un lexema y las desinencias. 2. Accidente gramatical del verbo que carece de marca mor-
c) Es de inventario cerrado. fémica y que se deduce de otro.
d) Presenta un significado relativo.
e) Funciona como núcleo del sujeto. a) Persona b) Aspecto c) Número
d) Modo e) Tiempo
6. ¿En qué expresión encontramos un verbo en modo sub-
juntivo? 3. No es un verbo en modo subjuntivo.

a) Hoy me habría enterado te tu engaño. a) Hayas leído b) Cantase c) Vuelve


b) Andad delante de los que necesitan guía. d) Hubiera salido e) Soñara
c) Volverá siempre que pueda unirse a los otros.
d) Unos veían, otros escuchaban, pocos hacían. 4. En el enunciado: «Pánfila, si me trataras más, no me mira-
e) Ven y dame lo que me quitaste. rías de esa manera», lo subrayado se encuentra en tiempo
___________.
7. ¿En qué alternativa encontramos un verbo en aspecto per-
fectivo? a) pretérito b) imperativo c) futuro
d) condicional e) subjuntivo
a) Huirá sin dejar huella.
b) Camina sin mirar atrás.
5. El enunciado: «Es modo verbal que expresa mandato» co-
c) Había olvidado la llave en el auto.
rresponde a la definición de__________.
d) ¿Fascine con mi voz?
e) Cantaría si tú me aplaudieses.
a) verbo imperativo
b) verbo subjuntivo
8. En: La Navidad, es la fiesta por la llegada del niño Jesús,
c) verbo infinitivo
quien nos recuerda que fue enviado puro, sin mancha, en
d) verbo participio
el seno de una familia, bendecida, unida y feliz a pesar de
e) verbo gerundio
las circunstancias, señala la cantidad de verbos conjuga-
dos.
6. Señala la cantidad de verbos del siguiente enunciado: «el
amor que alguna vez te tuve ha sido borrado por el tiempo
a) Cuatro b) Siete c) Cinco y poco a poco por otro sentimiento; creía que había supe-
d) Tres e) Seis rado tu engaño, mas solo me habré sanado cuando vuelva
sinceramente a creer en mí»
9. ¿Qué oración presenta verbo en modo imperativo?
a) cuatro b) siete c) cinco
a) Haber venido por ella fue su motivo. d) ocho e) seis
b) No creo que habiendo visto esto se atreva.
c) Ella llegó luchando por la causa. 7. Reconozca la oración que se encuentra en modo indicati-
d) Anda y vete: te está esperando; anda y vete. vo.
e) Hoy va, por una razón, a seguir en esta lucha.
a) Facundo escribe una historia de amor.
10. ¿Qué categoría gramatical presenta un morfema amalga- b) Ojalá entiendas este poema surrealista.
ma? c) No desperdicies tu vida, muchacho.
d) Compañera, respétese a sí misma.
a) Determinante b) Adjetivo c) Verbo e) Quizá nos acompañen a la ceremonia.
d) Adverbio e) Sustantivo

166 Mejores Personas, Mejores Familias


5º AÑO DE SECUNDARIA
Colegio

LINGÜÍSTICA  Mejores Personas, Mejores Familias

8. Elija el enunciado cuyo verbo se expresa en aspecto im- 2. ¿Qué alternativa presenta un verbo en tiempo pluscuam-
perfectivo. perfecto?

a) Lo anunció hace dos años en la universidad. a) Hube tardado.


b) He revisado numerosas tesis con mi equipo. b) He visitado a todas.
c) Comentaba lo mucho que aprendí contigo. c) Había ingresado a la carrera más rentable.
d) Me encantó tu forma de encarar la situación. d) Habré partido.
e) Por último, Manuela confesó su siniestro secreto. e) Había mucha gente allá.

9. Marque la opción en la que el verbo se expresa en aspecto 3. En: Tal vez fui yo que no puedo olvidar, el verbo se en-
perfectivo. cuentra en modo_________________.

a) No vino; observaba desde la cima. a) indicativo b) indefinido


b) Rolando tiene que ser más valiente. c) subjuntivo d) potencial
c) Ya debes de ir con la gitana estafadora. e) imperativo
d) Le tocarás la canción que antes era para mí.
e) Me querían hacer correr esos bribones. 4. En: Dejad que los niños vengan a mí, el verbo se encuentra
en modo___________.
10. Reconozca en que oraciones hallamos formas no persona-
les. a) potencial b) imperativo
c) subjuntivo d) compuesto
I. Un ladrido, en ese instante, nos alertó. e) indicativo
II. Nunca vi un bazar como este en mi vida.
III. Me has dicho que tu tesorito ya tiene dueño. 5. Elija la serie con accidentes gramaticales correspondientes
IV. Esta tarde, estaremos dibujándonos sonrisas. al verbo.
V. Jamás pienses robártelos: esos besos ya tienen dueña. UNI 2006 - II
a) Tiempo, número, género y modo
a) I - II - III b) IV - V c) I - II b) Número, género, aspecto y modo
d) Solo V e) III - IV - V c) Persona, número, tiempo y modo
d) Género, número, tiempo y voz
e) Modo, número, caso y tiempo

1. No me lo había presentado, sin embargo, ella ya había sos-


pechado de quien se trataba. Los verbos del texto anterior
están en tiempo__________.

a) pretérito anterior
b) pluscuamperfecto
c) indefinido
d) imperfecto
e) perfecto compuesto

5º AÑO DE SECUNDARIA
Mejores Personas, Mejores Familias 167
Colegio

Mejores Personas, Mejores Familias LINGÜÍSTICA

M A
TE

14   MAYÚSCULAS

I. GENERALIDADES YY Tras dos puntos que anuncian la reproducción de una


YY Cuando los dígrafos ch, gu, ll y que se emplean en ma- cita o palabras textuales:
yúscula al inicio de una palabra escrita con minúsculas, Juan dijo: «Me tengo que ir ahora mismo».
solo adopta forma de mayúscula el primero de sus com- III. EN FUNCIÓN DE LA CONDICIÓN O CATEGORÍA Y
ponentes: Chávez, Guinea, Llosa. DE OTRAS CIRCUNSTANCIAS
YY Cuando los dígrafos forman parte de una sigla, se es- YY Nombres propios de persona, animal o cosa singulari-
cribe en mayúscula solo el primero de sus componen- zada: José, Chita, Olifante.
tes: PCCh (Partido Comunista de China). YY Nombres de divinidades: Dios, Jehová, Alá, Júpiter.
YY La forma mayúscula de las letras “i” y “j” carece del YY Cuando el apellido lleva preposición o preposición y
punto que llevan en su grafía minúscula: JIRA- artículo, estos van con minúscula, a menos que no se
FAS. escriba el nombre: Señor Carlos de la Garza; Juan de
YY El empleo de mayúsculas no exime de ponerles tilde cuando así Ávalos; Señor De la Garza; Sr. De Ávalos; Ana de De
lo exigen las reglas: África, MÉXICO. la Torre.
YY Las siglas como CIA (Central Intelligence Agency) no se YY Cuando no lleva preposición, solo artículo, se escribe
acentúan. siempre con mayúscula: Señor La Plata; Carlos La
Orden.
II. USO DE MAYÚSCULA INICIAL YY Nombres de dinastías derivadas de apellidos: los Bor-
YY La primera palabra de un escrito y la que va después bones, los Capuleto, los Borgia, salvo que se utilicen
de un punto: Salieron a dar un breve paseo. La maña- como adjetivo: … los reyes borbones.
na era espléndida. YY Los sobrenombres, apodos y seudónimos: Manuel Be-
YY La palabra que sigue a los puntos suspensivos (…) nítez, el Cordobés, el Greco, el Libertador, el Benemé-
cuando estos cierran un enunciado: Vamos a comprar rito de las Américas.
un pastel… Pronto llegarán los invitados. YY Nombres abstractos personificados utilizados alegóri-
YY Cuando no cierra un enunciado porque queda en sus- camente: la Muerte, la Esperanza, el Mal.
penso, pero se cambia de tema: ¡Si Raúl no viene en 15 YY Nombres y accidentes geográficos: América, España,
minutos!... ¿Qué le pasaría? el Orinoco, el Ebro, los Andes, océano Pacífico, mar
YY Si los puntos suspensivos no cierran un enunciado, la Mediterráneo, volcán Misti.
primera palabra después de estos se escribe con mi- YY Espacios urbanos: avenida Perú, plaza de Armas, el
núscula: No sé… qué hacer. Tengo que… decidirme. puente de los Suspiros.
YY D espués de dos puntos (:) en los casos siguien- YY Cuando el artículo forma parte del nombre, también
tes: se escribe con mayúscula: La Mancha, La Habana.
Muy señor mío: YY Sustantivos y adjetivos que forman parte del nombre
Le agradezco… de determinadas zonas geográficas, que generalmente
YY Tras los dos puntos que siguen al verbo fundamental de un abarcan distintos países, pero que se conciben como
documento jurídico-administrativo. áreas geopolíticas con características comunes: Oc-
CERTIFICA: cidente, Oriente Medio, Lejano Oriente, Cono Sur,
Que D. José García… Hispanoamérica.

168 Mejores Personas, Mejores Familias


5º AÑO DE SECUNDARIA
Colegio

LINGÜÍSTICA  Mejores Personas, Mejores Familias

Sin embargo, en otros casos se escribe con minúscula.


Ejemplos: Mi casa está al oriente de la ciudad.
YY Nombres de galaxias, constelaciones, estrellas, planetas o sa-
télites en contextos astronómicos: … la Tierra gira en torno 1. Marca la opción donde se presenta el uso adecuado de las
al Sol; Hoy habrá luna llena. mayúsculas.
YY Nombres de signos del zodiaco también los nombres
alternativos que aluden a la representación icono- a) El Río Amazonas es caudaloso.
gráfica de cada signo: Tauro (Toro). Se escriben con b) Viajó hasta la Habana.
minúscula, en cambio, cuando dejan de ser nombres c) Volverá para la Fiesta.
propios por designar, genéricamente, a las personas d) Llevó a su perro fido al parque.
nacidas bajo cada signo: Panchito es sagitario; Los gé- e) Ayer encontré a José de la Torre.
minis son muy volubles.
2. Marca la opción donde se presenta el uso adecuado de las
YY L
os sustantivos y adjetivos que componen el nombre de letras mayúsculas.
entidades, organismos, departamentos o divisiones ad-
ministrativas, monumentos, locales o establecimientos a) La Selva es el Paraíso de los aventureros.
públicos, partidos políticos, etcétera: Ministerio de Ha- b) Marte y Venus son planetas del sistema solar.
cienda, la Facultad de Medicina, la Torre de Pisa, , el c) Le daré un Panetón.
Partido del Trabajo, Universidad de Lima. d) Mañana leeremos el diario la República.
ombres de libros sagrados: el Corán, la Biblia, las Sa-
YY N e) ¿Has leído la novela Crimen y Castigo?
gradas Escrituras, el Génesis, el Talmud.
3. Marca la opción donde se presenta el uso adecuado de las
YY L
os sustantivos y adjetivos que forman parte del nom- mayúsculas.
bre de publicaciones periódicas o colecciones: La
Vanguardia, El Norte, Revista de Medicina Interna. a) La luna fue conquistada por el hombre.
YY L
a primera palabra del título de cualquier obra de b) Ya se percibe el Espíritu de Competencia.
creación (libros, películas, cuadros, esculturas, piezas c) Rosa trabaja en el Banco de Crédito.
musicales, programas de radio o televisión, etcétera); d) Se demostró que la tierra no es plana.
el resto de las palabras que lo componen, salvo que e) El Ministro de trabajo no atenderá hoy.
se trate de nombres propios, deben escribirse con mi-
núscula: Las mil y una noches, Sueño de una noche 4. Marca la opción donde se presenta el uso adecuado de las
de verano, El manantial, El coleccionista de huesos. mayúsculas.
Salvo que se trate de abreviar títulos de determinados
textos literarios: el Quijote, la Celestina. a) Yo leí la Biblia y el Corán, Daniel.
b) José CHávez compró cuatro libros.
c) Leí la novela los Ríos Profundos.
IV. CASOS EN QUE NO DEBE USARSE MAYÚSCULA d) En el Río Amazonas, escribe sus inspiraciones.
INICIAL e) Carmen estudia Biología vegetal.
YY os nombres de los días de la semana
L 5. Marca la opción donde se presenta el uso adecuado de las
YY Los nombres de las estaciones del año mayúsculas.
YY Los nombres de los meses del año
YY Las notas musicales a) Roberto vive en América del Sur.
b) El Río Ucayali pasa por Iquitos.
YY Los nombres de vientos, salvo que estén personifica- c) Visité el Puente de Los Suspiros.
dos en poemas o relatos mitológicos: austro, bóreas, d) Ellos viajaron al Sur de Arequipa.
tramontana. e) La iglesia opinó sobre Economía.
YY Los nombres de religiones: budismo, catolicismo
YY Los gentilicios: mexicano, salvadoreño, ruso, español 6. Marca la opción donde se presenta el uso adecuado de las
YY Los tratamientos: usted, señor, don, fray, san, santo, mayúsculas.
sor, reverendo, salvo que se escriban en abreviatura,
en cuyo caso se escriben con mayúscula: Ud., Sr., D., a) Todos regresarán a leer Los Miserables.
Fr., Sto., Rvdo. b) Conocí la plaza de Armas.
c) Lee El quijote.
YY Los títulos, cargos y nombres de dignidad como rey,
d) Ella es sagitario.
papa, duque, presidente, ministro, alcalde, etc.
e) Vive cerca del Río Marañón.

5º AÑO DE SECUNDARIA
Mejores Personas, Mejores Familias 169
Colegio

Mejores Personas, Mejores Familias LINGÜÍSTICA

7. Marca la alternativa donde hay el uso correcto de las letras 3. Señala la alternativa en la que se ha usado las mayúsculas
mayúsculas. de modo correcto.

a) El Sr. de la Peña protege a los minusválidos. a) Tengo una vecina que siempre asiste a la Biblioteca Na-
b) En Abril visitaremos el Puerto de Ilo. cional del Perú.
c) El lago Titicaca Se encuentra en Puno. b) Vi ese cuadro en el Museo De La Nación durante la tar-
d) El solitario personaje se encomendó a Dios. de.
e) Rebeca y su hermano son Capricornio. c) Tengo los cuatro libros de la palabra del mudo de Ri-
beyro.
8. Marca la opción donde hay uso adecuado de las letras ma- d) La calle bolívar tiene más de 30 cuadras.
yúsculas. e) ¿Qué dijiste? ¡pero esto es el colmo!

a) Durante el Verano viajarán a las islas Ballestas. 4. ¿Qué oración presenta incorrección ortográfica en rela-
b) Zambo, wanka, gueso y pellejo eran perros pastores. ción a las mayúsculas?
c) Los Limeños festejaron el día de la canción criolla.
d) Es de conocimiento que la Tierra gira alrededor del Sol. a) ¿El planeta más cercano a la Tierra es Mercurio?
e) Los artistas se presentaron en el parque de la Exposición. b) Facundo Quiroga, el Tigre de los Llanos, asoló esas tierras.
c) Galileo sustentaba que la Tierra gira en torno al Sol.
9. Señala la opción donde hay uso correcto de las mayúsculas. d) El ministerio de Justicia está en reorganización.
e) Como muchos jóvenes solía estar en la luna.
a) Señores, ¿Qué es lo que han decidido?
b) ¡Qué sorpresa! no te esperaba hoy. 5. Identifica la opción en la que hay uso correcto de las letras
c) Aunque yo, ¿Sabes?, No lo considero así. mayúsculas.
d) ¿A qué hora has llegado?, ¿Has tenido un buen viaje?
e) ¿Qué me has preguntado? No estaba atento. a) Los trabajadores de la Papelera Paracas exigen aumentos.
b) El descubrimiento de América se llevó a Cabo en el s. XV.
10. Indica la alternativa que presenta uso incorrecto de las le- c) Papas, Reyes y Presidentes están sujetos a renovarse.
tras mayúsculas. d) En La Edad Media, los pontífices ejercían gran poder
político.
a) El Signo Cáncer está representado por el cangrejo. e) Rubén Darío es el autor de Cantos de Vida y Esperanza.
b) Solo Dios, el Todopoderoso, te protegerá.
c) Tu auto se vendió el miércoles a buen precio. 6. ¿En qué opción hay uso correcto de mayúsculas?
d) El alcalde del Rímac visitó Ancash el sábado.
e) Wílber tiene un Quijote de 1700. a) Miguel De Cervantes fue apodado El Manco de Lepanto.
b) Trujillo es la famosa Ciudad de la eterna primavera.
c) Aníbal cruzó Los Alpes y San Martín atravesó Los Andes.
d) Él me pidió que le hiciera una canción: Se sentía solo.
e) El Banco Central de Reserva del Perú denunció al co-
rrupto congresista.
1. ¿En qué caso se aprecia error en el uso de la mayúscula?
7. ¿En qué oración la palabra «occidente» debe escribirse con
a) Margot del Olmo vive en Los Olivos. inicial mayúscula?
b) París, la Ciudad Luz, es glamorosa.
c) Ayer leí hasta la página veintitrés del Quijote. a) Pintamos la fachada de occidente.
d) Celebraremos el Sábado mi ingreso en la universidad. b) El barco chocó por el lado de occidente.
e) El coronel Abad es muy famoso por sus conquistas. c) Los encontramos por el lado de occidente del estadio.
d) El lado de occidente de la casa ya está terminada.
2. El Ministro de Vivienda y el Presidente Ollanta Humala e) Egipto ejerció gran influencia en occidente.
lanzaron el programa «Agua para todos» en San Juan de
Lurigancho. 8. Hay uso normativo de la mayúscula.
El texto anterior presenta _____ errores en el uso de ma-
yúscula. a) Titicaca es el nombre de un Lago.
b) El Quechua es una lengua Amerindia.
a) seis b) tres c) dos c) El Viajero arribó a la capital Limeña.
d) cuatro e) cinco d) Cortázar polemizó con José M. Arguedas.
e) El Ministro del Ambiente es atinado en sus comentarios.

170 Mejores Personas, Mejores Familias


5º AÑO DE SECUNDARIA
Colegio

LINGÜÍSTICA  Mejores Personas, Mejores Familias

9. Señala la alternativa que denota correcto empleo de las 5. Se escribe con mayúsculas _____.
mayúsculas.
UNMSM 2010 - II a) todas las abreviaturas
a) Pedro De la Torre llegó tarde. b) los seudónimos
b) Nos recomendaron leer la Biblia. c) los meses del año y los días de la semana
c) A pesar de todo, Ana es Sagitario. d) después de un punto y coma
d) Elías nació en el Norte de Lima. e) luego de los dos puntos, necesariamente
e) No festejaron El Día del Maestro.
6. Señale la alternativa que no presenta errores en el uso de
10. Elija la alternativa en donde las letras mayúsculas están co- mayúsculas.
rrectamente utilizadas.
UNI 2010 - II a) Abelardo Gamarra, El Tunante, escribió la obra Cien
a) Es mérito de la Real academia española. Años de vida Perdularia.
b) El Museo de la Nación está de aniversario. b) El Estadio del Colegio Ricardo Bentín fue inaugurado
c) El Imperio Romano colapsó siglos después. por el Alcalde del Distrito del Rímac.
d) El Ministro de Educación nos visitará hoy. c) La Universidad Nacional Federico Villarreal está de
e) En cada Invierno, Manuel sufre de faringitis. aniversario.
d) El 26 de Diciembre es el cumpleaños de José.
e) El primero de Mayo es el día internacional de los traba-
jadores.

7. ¿Cuál es correcta, con relación a mayúsculas?

1. Uso correcto de mayúsculas: a) Pepe miró hacia la calle. todo ya había terminado.
b) Verdaderamente tú habías sido la culpable. pepe siem-
a) Aún no ha sido bien estudiado el señor de Sipán. pre lo supo.
b) El doctor donó lo siguiente: Lápiceros, papel y libros. c) La culpa la tuvo Usted.
c) En mesa Redonda, venden artículos a bajo precio. d) El río Urubamba estaba muy sucio.
d) Municipalidad de Lince dispuso la remoción de los es- e) Te digo que no irás a la Playa.
combros.
e) El Presidente respondió a las acusaciones de la comisión. 8. Presenta correcta redacción.

2. Presenta escritura correcta. a) Shirley tiene unos lindos Ojos.


b) El niño era feliz y Ustedes le quitaron esa felicidad.
a) El Jefe de Estado Chileno vendrá al Perú. c) El rey Eduardo tuvo un gobierno Trágico.
b) En Brasil, elegirán al próximo jugador de Fútbol del mes. d) Alfredo Bryce ha escrito novelas muy interesantes.
c) Cristo, el Hijo de Dios, murió en la cruz. e) En este colegio estudiará chávez.
d) San Martín, El Libertador, nació en Argentina.
e) Marta de La Cruz ganó el premio mayor. 9. Presenta correcta redacción.

3. Uso correcto de las grafías mayúsculas: a) Yo estudio en el Colegio Nacional La Virgen De la Con-
cepción.
a) El Señor Pérez no irá a la Conferencia. b) Él vive en CHiclayo.
b) La Reina Isabel apoyó a Colón. c) Vargas Llosa vive en Madrid (España).
c) El Caballero de los mares murió en Angamos. d) Cervantes era conocido como “El Manco De Lepanto”.
d) Voy a tomar el Sol en esta hermosa playa. e) Es enrique quien más sabe.
e) Fidel de la Rosa los esperará en El Cairo.
10. Presenta correcta redacción.
4. Uso correcto de mayúsculas:
a) El presidente Piñera volverá al Perú.
a) Ayer, Nosotros hicimos toda la tarea de lenguaje. b) Cerca de la avenida Grau está el museo de Bellas Artes.
b) Levantó el arma. había imaginado tantas veces esa escena. c) Alfonso, «El sabio», apoyó el resurgimiento cultural.
c) Ya ve Usted, siempre fue él así, extraño. d) Viva el Rey Felipe III.
d) El volcán Misti es una característica típica de Arequipa. e) Vamos al festival del cusco.
e) ¿Dinos cómo lo supiste? te juro que hasta ahora esta-
mos sorprendidos.

5º AÑO DE SECUNDARIA
Mejores Personas, Mejores Familias 171
Lingüística
Colegio

Mejores Personas, Mejores Familias LINGÜÍSTICA

M A
TE

15   CLASIFICACIÓN DE LOS VERBOS I

I. CLASIFICACIONES SINTÁCTICAS DEL VERBO

1. Copulativo / No copulativo
a) Verbos copulativos
Ser, estar, parecer, quedar, resultar, permanecer, yacer, semejar, volverse, hacerse
LL Los verbos copulativos tienen como función la unión entre el sujeto y el atributo.
LL Esquema básico: Sujeto + Verbo copulativo + Atributo
LL El atributo indica una característica del núcleo del sujeto.
LL Si la oración carece de predicativo, el verbo es no copulativo.
Juana es doctora.
SUJETO PREDICADO
(Con verbo copulativo)

b) Verbos no copulativos o predicativos
Amar, querer, soñar, saltar, jugar, caminar, leer, saber y miles de verbos más

LL Son los verbos que por sí solos forman predicado.
Las flores del jardín renacen.
SUJETO PREDICADO (con verbo no copulativo)

2. Transitivo/ Intransitivo
a) Verbos transitivos: Son los que presentan objeto directo.
b) Verbos intransitivos: Son los que no presentan objeto directo.

Método del reconocimiento

OBJETO DIRECTO OBJETO INDIRECTO



PREGUNTAS ¿Qué? ¿A quién? ¿A quién? ¿Para quién?
PRONOMBRES Lo, los, la, las Le, les

●● El poeta compone unos versos (Verbo transitivo) = El poeta los compone.


N OD

●● Su actitud disgustó a los presentes (Verbo intransitivo) = Su actitud les disgustó.
N OI

NOTA:
Los pronombres átonos ME, TE, SE, NOS, OS pueden funcionar como OD u OI
ZZ El organizador del evento me felicitó ⇒ Se puede decir «lo felicitó»
OD
ZZ Ella me dio su número telefónico ⇒ Se puede decir «le dio su número telefónico»
OI

172 Mejores Personas, Mejores Familias


5º AÑO DE SECUNDARIA
Colegio

LINGÜÍSTICA  Mejores Personas, Mejores Familias

3. Personal / Impersonal
a) Verbos personales: Son aquellos verbos que poseen sujeto, expreso o tácito.

LL P or el peso de los años, se cayeron las casas de adobe (cayeron = V. Personal)
PREDICADO SUJETO

LL Durante unos minutos, la sal de las lágrimas lo amargaba (amargaba = V. Personal)
PREDICADO SUJETO PREDICADO

b) Verbos impersonales: Son aquellos verbos que no presentan sujeto.

LL Hubo muchas dificultades LL Garuaba en la sierra


PREDICADO PREDICADO

II. CLASIFICACIÓN DE LOS VERBOS POR LA EXIGENCIA DE UNA FORMA PRONOMINAL


1. Verbo reflexivo
●● E s aquel cuyo sujeto recibe la acción que él mismo ha ejecutado.
●● El verbo está acompañado de una forma pronominal reflexiva (me, te, se, nos, os, se) en la que se reflejará la persona
gramatical que funciona, al mismo tiempo como sujeto de la oración.
●● Las formas pronominales funcionan como objeto directo o indirecto.
●● Acepta los refuerzos: «a mí mismo», «a ti mismo», «a sí mismo».

El vecino se afeita
Pronombre en 3° persona, singular, Verbo reflexivo
funciona como OD
SUJETO
PREDICADO
(3° persona singular)

2. Verbo cuasirreflejo
●● E l sujeto realiza la acción, pero esta no recae sobre el mismo sujeto.
●● El verbo está acompañado de una forma pronominal reflexiva (me, te, se, nos, os, se) en la que se reflejará la persona
gramatical que funciona, al mismo tiempo como sujeto de la oración.
●● Las formas pronominales no funcionan como objeto directo, ni como objeto indirecto; SINO COMO SIGNO CUA-
SIRREFLEJO.
●● No acepta los refuerzos: «a mí mismo», «a ti mismo», «a sí mismo»

Ella se cae
Pronombre en 3° persona, singular, Verbo cuasirreflejo
SIGNO CUASIRREFLEJO

SUJETO
PREDICADO
(3° persona singular)

Tú te alegras
Pronombre en 2° persona, singular, Verbo cuasirreflejo
SIGNO CUASIRREFLEJO

SUJETO
PREDICADO
(2° persona singular)

3. Verbo recíproco
●● E s aquel verbo que indica una acción que es intercambiada mutuamente entre los sujetos que la realizan.
●● Utiliza las formas pronominales (nos, os, se) que funcionan como objeto directo o indirecto.
●● Acepta el refuerzo «mutuamente».

5º AÑO DE SECUNDARIA
Mejores Personas, Mejores Familias 173
Colegio

Mejores Personas, Mejores Familias LINGÜÍSTICA

Ellos se abrazan
Pronombre en 3° persona, plural, Verbo recíproco
funciona como OD

SUJETO
PREDICADO
(3° persona plural)

Tú y yo nos dictamos claves


Pronombre en 1° persona, plural, Verbo recíproco O.D.
funciona como OI.

SUJETO
PREDICADO
(3° persona singular)

6. ¿En qué oración hay un verbo cuasirreflejo?

a) Recuerdas el número.
b) Se besaron con ternura y amor.
1. La oración Reclámaselo tiene verbo con objeto... respectivamente. c) Te vas hoy.
d) Se aman mucho.
a) directo e indirecto e) Lo mira fijamente.
b) directo y circunstancial
c) ambos directos 7. Señale la opción correcta:
d) indirectos y circunstancial
e) indirecto y directo a) El atributo puede ser un adverbio
b) Los verbos intransitivos pueden llevar OD.
2. ¿Qué oración presenta verbo transitivo? c) Los verbos reflexivos no son pronominales.
d) Los verbos recíprocos solo tienen objeto directo.
a) Puede irse a cualquier parte.
e) El verbo transitivo siempre lleva O.D.
b) Murió cuando trataba de escapar.
c) Soñaba con viajar muy lejos. 8. ¿En qué oración el verbo es cuasirreflexivo?
d) Estamos aún esperando.
e) Le compraron sus poesías. a) Nos veíamos constantemente.
b) Se tocó varias veces el rostro.
3. ¿Qué oración no presenta verbo copulativo? c) Se cepilla siempre los dientes.
d) Se escriben siempre.
a) Anduvo muy triste por la calle.
e) No te vayas sin decir adiós.
b) devolver eso fue la solución.
c) Parece medio asustada. 9. ¿Qué oración presenta verbo intransitivo?
d) Eso estuvo difícil.
e) Eulogio quedó estupefacto con la noticia. a) En el Perú, se cultiva un algodón de gran calidad.
b) Te garantizo que aprenderás.
4. Todo verbo intransitivo lleva como complemento ____. c) Dormía y soñaba constantemente.
d) Queremos casarnos, pero debemos esperar mucho aún.
a) un circunstancial e) Necesito el dinero urgente.
b) un objeto directo
c) un agente 10. La Navidad en el Perú se celebra el 24 y 25 de diciembre de
d) no obligatorio cada año, con una cena familiar y como una costumbre se
e) un atributo hace la entrega de regalos (que suelen ser muy variados y
de acuerdo a la posibilidad económica de cada familia) a
5. ¿Qué oración presenta verbo impersonal? los niños en representación de los dones que les fueron en-
tregados a Jesús como una muestra de cariño y amor a su
a) Devolverá unos libros. inocencia y pureza. En el texto hay ____ verbos transitivos.
b) Tomé de ellas lo mejor.
c) A veces le da por volverla a buscar. a) dos b) tres c) cuatro
d) Hubo la gran necesidad de contar con él. d) cinco e) seis
e) El mundo es ancho y ajeno.

174 Mejores Personas, Mejores Familias


5º AÑO DE SECUNDARIA
Colegio

LINGÜÍSTICA  Mejores Personas, Mejores Familias

8. ¿Cuál de las opciones presenta verbo reflexivo?


a) Lávate
b) Estoy entendiendo
c) Tengo que concentrarme
1. Señale la opción que presenta verbo transitivo. d) Voy a decírselo
e) Había elegido
a) Fue una gran medida.
b) Vendimos a precios justos. 9. Encontramos un verbo transitivo.
c) Dime dónde irás mañana.
d) Llegó corriendo. a) Viniste y venciste.
e) Ella llora por el pasillo. b) Mabel vendía flores.
c) Mañana habrá llegado.
2. En: «Esperar a que ella llegara era un suplicio para él, pues d) Fue su mayor tesoro.
siempre se demoraba y nunca se disculpaba cuando apa- e) El método era excelente.
recía», ¿cuántos verbos conjugados hay?
10. Identifica el verbo transitivo.
a) dos b) tres c) cuatro a) Ese resfriado no es problema, Manolo.
d) cinco e) seis b) Avelino viajó por la escarpada montaña.
c) Un amor no muere con solo un adiós.
3. Marque la oración en la que ¨estar¨ sea copulativo. d) Roxana se va al supermercado contigo.
e) Entonces compra un televisor de 21 pulgadas.
a) Ella está en busca de nuevas emociones.
b) Los precios de las verduras están por las nubes.
c) Ella está con vestido de estreno.
d) La primera dama está medio celosa de la presidenta argentina.
e) Había estado nevando en Nueva York.
1. Una de las oraciones tiene un verbo copulativo.
4. Presenta verbo recíproco. I. Juan está contento con su postre.
II. Carlos decidía con mucha prudencia.
a) Debemos partir mañana. III.Amalia está bailando con Carmelo.
b) Tenemos que ser más cuidadosos. IV. Me pareció audaz tu propuesta, amiga.
c) La pared parece débil. V. Mariana ha sido amable y muy inteligente.
d) Meditó toda la noche en su cuarto. a) I, II y III b) III, IV y V c) I, IV y
e) Los vecinos se volvieron a pelear. d) IV y V e) III y IV

5. Señale la oración que presenta verbo transitivo. 2. El cantar de aquella dama enamorada ha sido bellísimo.
¿Qué tipo de verbo presenta esta oración?
a) La niña estaba muy pálida.
a) Transitivo b) Recíproco c) Reflexivo
b) Los peregrinos llegaron muy tarde.
d) Copulativo e) Predicativo
c) Ellos regresaron anoche.
d) Mi equipo perdió el partido.
3. En: «Después del almuerzo, me fui a la iglesia del pueblo».
e) El forastero viajó temeroso.
¿Qué tipo de verbo presenta esa oración?
6. Sobre el verbo es falso que _________. a) Copulativo b) Reflejo c) Cuasirreflejo
d) Recíproco e) Transitivo
a) parecer es un verbo predicativo.
b) vas a sorprender es una perífrasis verbal. 4. Una de las oraciones presenta verbo intransitivo.
c) sus accidentes nominales son número y persona. a) Dímelo ahora con lujo de detalles, Hugo.
d) funciona como núcleo de la oración b) Facundo, lo vas a preparar sin demora.
e) llover es un verbo impersonal. c) Melissa y Cecilia tienen que dar su cuota.
d) El ermitaño volverá a la montaña sagrada.
7. Los verbos transitivos deben presentar_________. e) El nuevo experimento demostrará tu teoría.
a) objeto indirecto 5. Los verbos copulativos deben presentar complemento
b) objeto directo _______.
c) complemento indirecto
d) complemento directo a) directo b) atributo c) indirecto
e) b y d d) circunstancial e) agente

5º AÑO DE SECUNDARIA
Mejores Personas, Mejores Familias 175
Colegio

Mejores Personas, Mejores Familias LINGÜÍSTICA

M A
TE

16   CLASIFICACIÓN DE LOS VERBOS II

CLASIFICACIÓN MORFOLÓGICA DEL VERBO 2. Verbos defectivos


1. Verbos regulares e irregulares En morfología, un verbo defectivo es un verbo cuyo para-
Los verbos regulares deben cumplir con dos requisitos digma flexivo o conjugación carece de alguno o varios de
obligatoriamente; de lo contrario, el verbo es irregular: sus tiempos, modos o personas en su conjugación regular.
YY L a raíz debe mantenerse en todos sus sonidos. En español son los siguientes: adir, acaecer, amanecer,
YY Las desinencias deben ser iguales a las de su verbo atañer, balbucir, blandir, concernir, colorir, desabrir, em-
modelo. pedernir, garantir, incoar, llover, nevar, soler y usucapir.
YY Verbos modelos: Cantar, comer, vivir Por ejemplo, no se dice *acaezo ni *acaezco.
Ejemplo:
YY El verbo «escribir» es regular 3. Los verbos y los verboides
Diferencias entre verbo y verboides
ESCRIBIR VIVIR
El verbo se caracteriza por presentar persona, deben estar
escrib-o viv-o
debidamente conjugados en una de ellas.
escrib-í viv-í
escrib-iré viv-iré
Ejemplos:
YY El verbo «medir» es irregular.
YY C
omprábamos: Es un verbo porque se puede expresar
MEDIR VIVIR
en una determinada persona «nosotros», es decir, en
mid-o viv-o la primera persona del número plural.
med-í viv-í YY Dijiste: Es un verbo porque se puede expresar en se-
med-iré vivi-iré gunda persona «tú» , es decir, en la segunda persona
del número singlar.
YY El verbo «salir» es irregular YY Leyó: Es un verbo porque se puede expresar en la ter-
SALIR VIVIR cera persona del número singular «él».
salg-o viv-o
sal-í viv-í En cambio los verboides, carecen de persona y, por lo tan-
sal-dré viv-iré to, por sí solos no pueden funcionar.

4. Los verboides

VERBOIDE TIPO EJEMPLOS


Simple Termina en ar, er, ir amar, comer, vivir
Infinitivo
Es el nombre del haber amado
verbo Compuesto Unión de «haber» + participio pasivo haber comido
haber vivido
amando, comiendo,
Simple termina en ando, iendo
Gerundio viviendo
Indica la acción del habiendo amado
proceso Compuesto Unión de «habiendo» + participio pasivo habiendo comido
habiendo vivido
Participio Regular termina en ado, ido amado, comido, vivido
Indica una acción Pasivo
Irregular termina en cho, to, so hecho, escrito, impreso
realizada o al que
realiza la acción Activo termina en ante, iente amante, viviente

176 Mejores Personas, Mejores Familias


5º AÑO DE SECUNDARIA
Colegio

LINGÜÍSTICA  Mejores Personas, Mejores Familias

8. ¿Cuál es la oración que presenta participio irregular?

a) Confiaba solo en su familia.


b) Fue inaugurada la nueva comisaría
1. Presenta verbo defectivo. c) Estuve muerta de miedo con esa película.
a) Carmen está alegre. d) Varios clientes dejaron abandonados sus autos.
b) Ellos no fueron al cine. e) Vendía sin factura.
c) Castilla abolió la esclavitud.
d) El partido será en Cochabamba. 9. ¿Cuál es la oración que presenta un infinitivo?
e) Ellos no acaecieron muy rápidamente.
a) Me trae recuerdos esta noche.
2. Es un verbo irregular. b) Hoy no habrá repetición.
c) Me sabe convencer siempre.
a) Dejar b) Beber c) Pensar d) Esas listas han sido depuradas.
d) Vivir e) Bailar e) Encontramos a nuestro amigo maltrecho.

3. Indique la oración que contenga un participio con función 10. Oración con un verbo irregular - regular - irregular:
adjetiva.

a) Fue censurado el presidente del Congreso Nacional de a) Cenaré con la jefa: aprovecharé y le pediré aumento
la República. b) Yo no te pido que me firmes diez papeles grises.
b) Estoy a punto de irme. c) Quisiera que ames las palomas que suelo mirar.
c) Estoy muriendo de amor. d) Dime que vendrás temprano y almorzarás.
d) Hemos conseguido un pequeño reconocimiento por e) Ven sin medir las consecuencias.
nuestra labor.
e) El día soleado se hizo esperar.

4. Relaciona correctamente:
I. Conquistando A. Infinitivo
II. Rogar B. Gerundio 1. Las formas verbales no personales son ______.
III. Muerto C. Participio
a) Infinitivo, agente, gerundio.
a) IC - IIB - IIIA b) IB - IIA - IIIC b) Gerundio, participio, perífrasis.
c) IA - IIC - IIIB d) IA - IIB – IIIC c) Participio, infinitivo, gerundio.
e) IB - IIC - IIIA d) Agente, circunstancial, infinitivo.
e) Sustantivo, adjetivo, adverbio.
5. ¿En qué oración el participio funciona como adjetivo?
2. Marque la alternativa que presente una relación correcta:
a) Ya habrán pagado el aguinaldo.
b) Habíamos discutido sin sentido previsto. a) Infinitivo - adverbio
c) Siempre fue muy querido por nosotros. b) Participio - gerundio
d) Hemos ganado el concurso. c) Gerundio - adjetivo
e) Han salido un poco tarde. d) Gerundio - adverbio
e) Infinitivo - adjetivo
6. Oración con verbo regular.
3. Es el verboide que desempeña la función del adverbio.
a) Tenía una vida disipada.
b) No se volverían a ver en mucho tiempo. a) Infinitivo b) Gerundio c) Participio
c) No cuento contigo entonces. d) Verbo regular e) Verbo irregular
d) Se siente el muy muy.
e) Se corto el cabello y ahora todos se ríen de él. 4. ¿Cuántos verbos regulares existen en la siguiente oración:
«Fue un bonito compartir con quienes pasé tantos bellos
7. ¿En qué oración el infinitivo funciona como sustantivo? momentos: solíamos molestar a nuestros maestros de la
secundaria»?
a) ¿Hoy podré ir a verte?
b) Debí decirlo en su momento. a) Tres b) Ninguno c) Dos
c) A nadie le gustaba su cantar. d) Cuatro e) Uno
d) Vamos a plantear el ejercicio.
e) Quiso postular, pero no pudo.

5º AÑO DE SECUNDARIA
Mejores Personas, Mejores Familias 177
Colegio

Mejores Personas, Mejores Familias LINGÜÍSTICA

5. Marque la alternativa en la que hay verbo irregular.

a) Él trabaja en el nuevo teatro.


b) Nosotros amamos a nuestros padres.
c) Julito canta maravillosamente. 1. Sobre los verbos regulares es cierto.
d) Esa joven teme la oscuridad.
a) Todos siempre son verbos copulativos.
e) Ayer vino el profesor de Lenguaje.
b) No pueden ser verbos predicativos.
6. Marque la alternativa donde se presenta un verbo c) No alteran la pronunciación de su raíz.
impersonal. d) Siguen la desinencia de su verbo modelo.
e) c y d
a) Exploraba nuevas técnicas de defensa.
b) El entrenamiento no va a ser tan fácil. 2. Identificamos verbos cuasirreflejos.
c) Nuestros nuevos amigos llegaron bien. I. Se cayeron por descuidados.
d) Los aplausos llovieron tras la presentación. II. Nunca se besó con Fernando.
e) Había excelentes deportistas en el estadio. III. Consuelo levantó su promedio.
IV. Nos fuimos a la nueva sede.
7. Señala solo los verbos regulares.
a) I y II b) I, II y III c) II y III
a) Vagar – dormir d) III y IV e) I y IV
b) Estudiar – comer
c) Caer – morir 3. Elige la afirmación correcta.
d) Saltar – estar
e) Mentir – pasear a) Los verbos irregulares nunca varían su raíz.
b) El verbo es impersonal siempre está en singular.
8. Los verbos impersonales carecen de ______. c) Los verbos recíprocos señalan acción refleja.
d) El verbo copulativo tiene objeto directo.
a) OD b) sujeto c) OI e) Un verbo intransitivo debe presentar OD.
d) atributo e) circunstancial

9. Reconoce las oraciones con verbos personales. 4. Reconoce el enunciado que presente verbo irregular.
I. Llovía con insistencia. a) Todos estudiamos con ahínco.
II. Regresaría por tu propuesta. b) Sofía vestía un hermoso traje.
III. Así habrá nuevas glorias. c) Tu promedio fue sobresaliente.
IV. Moría de amor por ti. d) No llegaba según lo acordado.
a) I y II b) II y III c) III y IV e) Repetía la frase de su padre.
d) I, II y III e) II y IV
5. Señale lo correcto respecto del verbo subrayado:
10. Una oración tiene verbo impersonal. «No pescó ningún paiche».
I. Es verbo regular. II. Es verbo transitivo
a) Han buscado nuevos candidatos.
III. Es verbo irregular. IV. Es intransitivo.
b) Hubo una mejor oportunidad.
V. Es personal.
c) No han dicho todavía toda la verdad.
d) Sembramos más por la lluvia inesperada. a) I y IV b) I, II y V c) I y II
e) Cayó una nevada durante el paseo. d) II, III y III e) II y III

178 Mejores Personas, Mejores Familias


5º AÑO DE SECUNDARIA
Colegio

LINGÜÍSTICA  Mejores Personas, Mejores Familias

M A
TE

17   EL ADVERBIO Y SUS CLASES

I. DEFINICIÓN 8. Adverbios de orden


1. Definición semántica Primeramente, posteriormente, ulteriormente, sucesiva-
El adverbio es una palabra que puede indicar diversas mente, últimamente, respectivamente.
realidades: tiempo, lugar, modo, cantidad, etc.
9. Adverbios relativos
2. Definición morfológica Son los que subordinan verbos: cuando, donde, como,
El adverbio es una categoría gramatical invariable, por adonde, mientras, apenas.
lo tanto carece de accidentes gramaticales.
10. Adverbios interrogativos
3. Definición sintáctica Son los que indican interrogación o pregunta: cuándo,
El adverbio funciona como complemento del verbo, dónde, cómo, adónde.
del adjetivo y de otro adverbio. En el caso de que fun-
cione como complemento del verbo, dentro del predi- III. LOCUCIONES ADVERBIALES
cado funciona como CIRCUNSTANCIAL. Ejemplo:
YY Lo harán paulatinamente: palabra adverbial.
4. Definición lexicológica YY Lo harán poco a poco: locución adverbial.
El adverbio es una categoría de inventario abierto, por Son las frases que equivalen a la utilización de un solo adverbio.
lo tanto, admite la incorporación de nuevas palabras. A manos llenas, a boca de jarro, a bordo, a brazo partido, a
buen recaudo, a campo traviesa, a sabiendas, a cántaros, a pie
II. CLASIFICACIÓN juntillas, a menudo, a hurtadillas, a la buena de Dios, a tontas
1. Adverbios de lugar y a locas, a salto de mata, a capa y espada, a los cuatro vientos,
Adentro, afuera, adelante, atrás, arriba, abajo, dentro, a carta cabal, a duras penas, a escondidas, a gatas, de vez en
fuera, delante, detrás, debajo, cerca, lejos, encima, en- cuando, de buenas a primeras, en cuclillas, en un dos por tres,
frente, junto, aquí, acá, ahí, allí, allá, donde, donde- sin más ni más, sin ton ni son, contra viento y marea, en reali-
quiera. dad, por lo general, en resumen.

2. Adverbios de tiempo
Hoy, ayer, mañana, temprano, tarde, antes, después,
ahora, anoche, enseguida, ya, aún, todavía, pronto,
siempre, recién, recientemente, luego, cuando, mien-
tras. 1. Definición sintáctica del adverbio.

3. Adverbios de cantidad a) Categoría variable


Harto, demasiado, bastante, mucho, muy, tanto, tan, b) Inventario cerrado
más, medio, poco, algo, casi, nada, menos, excepto, c) Categoría independiente
apenas. d) Funciona como circunstancial
e) Es complemento del sustantivo
4. Adverbios de modo
Bien, mal, rápido, despacio, duro, ligero, claro, así, como; 2. Una de las siguientes alternativas presenta una locución
y la mayoría de adverbios terminados en «mente» : fácil- adverbial de modo.
mente, cruelmente, etc.
a) Fuente ovejuna b) Amor mío
5. Adverbios de afirmación c) Se fue. d) De acuerdo con
Sí, también, cierto, claro, efectivamente. e) Como si nada

6. Adverbios de negación 3. En: «Ella estudiaba mientras esperaba la hora de su cita»,


No, nunca, jamás, tampoco. hay un adverbio de _____.
7. Adverbios de duda a) lugar b) modo c) tiempo
Quizá (s), ojalá, acaso, posiblemente, probablemente, d) duda e) afirmación
tal vez.

5º AÑO DE SECUNDARIA
Mejores Personas, Mejores Familias 179
Colegio

Mejores Personas, Mejores Familias LINGÜÍSTICA

4. Presenta adverbio de modo. 4. No es adverbio.

a) Se quemaron todos d) Corría todos los días. a) Apenas b) Tanto c) De malas


b) Se quedó sin nada e) Vengan ya. d) Antes e) Media
c) Lo resolvió así.
5. ¿En qué oración aparece adverbio de modo?
5. No es adverbio.
a) Viajaremos pronto a Piura.
a) Así b) Porque c) Mientras b) Los niños se quedarán aquí.
d) Donde e) A tontas y a locas c) Ella contestó correctamente.
d) Ellos retornaron anoche.
6. El adverbio es modificador de__________. e) Iré a la biblioteca mañana.
a) pronombre b) adjetivo c) verbo 6. Señale las afirmaciones correctas.
d) adverbio e) b, c y d I. El adverbio puede modificar al adjetivo.
II. El sustantivo puede modificar al adjetivo.
7. ¿Qué función cumple el elemento subrayado de la siguien- III. El adverbio puede modificar a otro adverbio.
te oración: «Dio vueltas de campana al caer del morro des-
pués del choque»? a) Solo I b) I, III c) Solo II
d) II, III e) I, II, III
a) Adjetivo b) Pronombre c) Verbo
d) Adverbio e) Sustantivo 7. El adverbio concuerda con el verbo en_________.

a) Número d) género y número


8. ¿Cuál de las siguientes palabras es categoría invariable?
b) número y persona e) género, número y persona
c) no concuerda
a) Dama b) Mirada c) Siempre
d) Ella e) Caminé
8. ¿Cuántos adverbios hay en la siguiente oración: «Ella juga-
ba muy mal, pero se esforzó mucho por mejorar»?
9. Una de las siguientes expresiones puede funcionar como
adverbio. a) Uno b) Dos c) Tres
d) Cuatro e) Cinco
a) Somos b) Demuestra c) Deprisa
d) Pero e) Adiós 9. Sobre el adverbio podemos afirmar que______.

10. ¿Qué categoría es el elemento subrayado de la siguiente a) admite morfemas flexivos.


oración: «Si me ibas a dejar, por qué esperaste tanto»? b) siempre se pospone al verbo.
c) admite morfemas derivativos.
a) Conjunción b) Preposición c) Adverbio d) concuerda en accidentes con el verbo.
d) Pronombre e) Sustantivo e) su inventario es limitado.

10. ¿En qué oración no hay adverbio?

a) Dame media naranja.


b) Algo le preocupa mucho.
c) Poco es lo que puedo hacer por ti.
1. ¿Cuántos adverbios hay en: «Los últimos serán los prime-
d) Más trabajo es lo que tengo ahora.
ros»? e) Trabajo más cada día.
a) Uno b) Cero c) Tres
d) Dos e) Cuatro

2. Presenta adverbio relativo.


1. ¿Cuántos adverbios hay en el siguiente texto: «Cuando vuel-
a) Donde b) Siempre c) Como
vas a mi lado, quizás hoy o tal vez mañana; pero si vuelves, te
d) Aunque e) En verdad
diré lo que siento por ti»
3. No es adverbio. a) Uno b) Cero c) Tres
d) Dos e) Cuatro
a) Tanto b) Mucho c) Pocos
d) Algo e) Más

180 Mejores Personas, Mejores Familias


5º AÑO DE SECUNDARIA
Colegio

LINGÜÍSTICA  Mejores Personas, Mejores Familias

2. ¿Cuántos adverbios hay en el siguiente texto: «Ayer, no más, 4. Una de las siguientes oraciones presenta adverbio de cantidad.
me dijo cuánto necesitaba ese empleo. Ahora, sin explicacio-
nes, se va corriendo, cansado de trabajar. ¡Qué ocioso! a) Trabaja bien.
Muchos no tienen trabajo y él se da el lujo de abandonarlo b) Nunca se cansa.
así»? c) Las cosas están cerca.
d) Espera poco de ti.
a) Seis b) Siete c) Ocho e) Es verdad, todo eso.
d) Nueve e) Diez
5. Presenta adverbio de modo.
3. ¿En qué oración encontramos una locución adverbial de
a) Juan vive feliz sirviendo a su pueblo.
cantidad?
b) Los saludó cortésmente.
a) Este problema lo resuelvo en un dos por tres. c) Llévalo todo allá.
b) Come tanto como una ballena y va a reventar. d) Mira cuánto ha crecido.
c) Eres terca como una mula. e) Delante de ella, confesó todo.
d) Esa mujer habla hasta por los codos.
e) Anda cabizbajo por lo que pasó allá

5º AÑO DE SECUNDARIA
Mejores Personas, Mejores Familias 181
Colegio

Mejores Personas, Mejores Familias LINGÜÍSTICA

M A
TE

18   LA PREPOSICIÓN

I. DEFINICIÓN
1. Definición semántica
No tiene significación por sí sola; es decir, las palabras
que relaciona son las que determinan el sentido de esta 1. ¿Cuántas preposiciones simples en uso tiene el castellano
categoría (su significado es de carácter contextual). actualmente?
2. Definición morfológica a) 32 preposiciones b) 26 preposiciones
Categoría gramatical invariable, es decir, que no sufre c) 20 preposiciones d) 17 preposiciones
variaciones formales, esto es, carece de morfemas lexi- e) 23 preposiciones
cológicos.
2. Serie compuesta únicamente de preposiciones:
3. Definición sintáctica
Funciona como conectivo, conector o nexo subordi- a) Desde - y - por - si - di - de
nante, es decir, puede enlazar un elemento sintáctico b) Porque - según - sobre - pero - más - sin
cualquiera como un sustantivo o elemento de valor c) Y - que - con - en - por - sin
equivalente. d) Según - contra - hasta - bajo - a - tras
e) Sí - porque - el - ve - la - y
4. Definición lexicológica
Es una clase de palabra de inventario cerrado porque 3. ¿Cuál es la definición morfológica de la preposición?
no admite la incorporación de nuevas palabras.
a) Es invariable. b) Es un enlace.
II. CATÁLAGO c) No modifica. d) Es de inventario cerrado.
La lista tradicional de preposiciones del idioma español es e) Es no connotativa.
a, ante, bajo, cabe, con, contra, de, desde, en, entre, hacia,
hasta, para, por, según, sin, so, sobre, tras. La lista oficial de 4. ¿En qué opción no aparece ninguna preposición?
la RAE y ASALE para el español actual es: a, ante, bajo, cabe,
con, contra, de, desde, durante, en, entre, hacia, hasta, me- a) Ahora solo queda lamentos después de lo pasado.
diante, para, por, según, sin, so, sobre, tras, versus y vía. b) Habla de todo y no comunica nada.
YY Cabe: Es una preposición arcaica que significa «junto a» c) Y que nadie sepa la verdad, mi vida.
YY so: Es una preposición arcaica que significa «bajo» d) En todas hay preposición.
e) En ninguna hay preposición.
III. FUNCIÓN
La preposición sirve para conectar a un núcleo con un término. 5. La preposición encabeza a un______.

a) verbo b) elemento nominal


NÚCLEO TÉRMINO
c) elemento adverbial d) artículo
Un verbo Un sustantivo
e) cualquier palabra
Un sustantivo Un adjetivo
PREPOSICIÓN
Un adjetivo Un adjetivo
6. En: «Mi vida estuvo llena de pequeños detalles que al final
Un adverbio Un adverbio
solo fueron para tí», hay_________ preposición(es)
Un pronombre Un pronombre
a) Cero b) Una c) Dos
IV. LOCUCIONES PREPOSITIVAS d) Cinco e) Tres
Al igual que el adverbio, se pueden formar grupos de palabras
que cumplen la misma función de una preposición. 7. La estructura de la preposición se compone de _____.
Ejemplo:
YY Lo hará según tu opinión a) conjunción + elemento verbal
YY Lo haría de acuerdo con tu opinión b) adverbio + preposición
Otras locuciones: detrás de, acerca de, en contra de, por c) preposición + elemento nominal
encima de, por debajo de, a través de, en medio de, por d) preposición + elemento adverbial
delante de, etc. e) preposición + preposición

182 Mejores Personas, Mejores Familias


5º AÑO DE SECUNDARIA
Colegio

LINGÜÍSTICA  Mejores Personas, Mejores Familias

8. «En la vida hay amores que nunca pueden olvidarse y si- 6. ¿En cuál de las siguientes oraciones la preposición a expre-
tuaciones que llevan de una cosa a la otra». sa dirección?
En el texto anterior, ¿cuántas preposiciones hay?
a) Saldremos a la diez.
a) Dos b) Cinco c) Tres b) Cocinaba a la norteña.
d) Cuatro e) Seis c) Compró el libro a diez soles.
d) Iré a Iquitos.
9. ¿Cuáles de las siguientes preposiciones ya no forman parte e) Vimos a Rojas.
del repertorio del castellano?
I. Mediante IV. Cabe 7. Marcar la cantidad de frases prepositivas (locuciones) que
II. Allende V. Ante hay en: Después de hacer su ingreso, se puso en medio
III. Pro VI. So de los invitados y habló acerca de su vida privada a fin de
amenizar la reunión.
a) Solo II b) I, III y V c) II y I
d) Todas menos V e) Todas a) Dos b) Tres c) Cinco
d) Seis e) Cuatro
10. ¿En qué opción hay menos preposiciones?
8. Es correcto con respecto a la preposición.
a) A continuación tenemos que resolver los ejercicios con
prontitud. a) Se pospone a un sustantivo para subordinarlo.
b) Después de ti no hay nada. b) Hay más de 30 preposiciones.
c) Me pareció que vi a un lindo gatito. c) Sufre accidentes de género y número.
d) Hoy tengo un enorme disgusto. d) Si no existieran las preposiciones, los sustantivos no
e) Con la esperanza de ganar, voy a la carrera. podrían modificar.
e) La preposición es siempre un morfema lexemático.

9. AL y DEL son contracciones compuestas de ____.

a) Conjunción + preposición.
1. En: «Todo tiene su final; nada dura para siempre», ¿cuán- b) Preposición + artículo.
tas preposiciones hay? c) Preposición + conjunción.
d) Adverbio + preposición.
a) Cero b) Dos c) Tres e) Interjección + artículo.
d) Una e) Cinco
10. ¿Dónde está mal usada la preposición hasta?
2. En una frase preposicional, sus dos elementos funcionan
como ________. a) Hasta los operadores de limpieza ganan más que él.
b) Hoy trabajaré hasta la medianoche.
a) modificador directo + modificador indirecto c) El avión saldrá hasta mañana en la tarde.
b) enlace + modificador indirecto d) Se congregaron desde la ciudad de Dan hasta Beerseba.
c) núcleo + término e) No lo haré hasta que me des una buena razón.
d) enlace + término
e) enlace + núcleo

3. Es una de las preposiciones menos usadas del castellano actual.



a) Sobre b) Bajo c) Delante 1. ¿Dónde hay, al menos, una preposición?
d) Ante e) So
a) Ante semejante situación, mejor no digo nada.
4. Preposición que indica materia: b) Solo sé que ya no lo soporto.
c) Ni el mejor ha logrado recibir un sí.
a) A b) Hasta c) Para d) ¡... y pensar que hay una iglesia tan cerca!
d) Desde e) De e) Fue así tal y como te lo conté.

5. ¿Qué preposición no indica posición? 2. ¿Cuál de las siguientes preposiciones tiene más significados?

a) Bajo b) Hacia c) Ante a) Para b) Sin c) De


d) Tras e) Sobre d) En e) Según

5º AÑO DE SECUNDARIA
Mejores Personas, Mejores Familias 183
Colegio

Mejores Personas, Mejores Familias LINGÜÍSTICA

3. ¿Qué preposición indica tiempo aproximado? 5. Son dos palabras recientemente reconocidas como preposi-
ciones.
a) Hacia b) Sin c) Hasta
d) En e) A a) Ni – como
b) Mediante – durante
4. No es preposición. c) Vía – pro
d) So – cabe
a) Para b) Desde c) Por e) Eso – esto
d) Nunca e) Tras

184 Mejores Personas, Mejores Familias


5º AÑO DE SECUNDARIA
Colegio

LINGÜÍSTICA  Mejores Personas, Mejores Familias

M A
TE

19   LA CONJUNCIÓN

I. DEFINICIÓN c) Adversativas
1. Definición semántica: Carece de significado por sí LL Hazlo, pero con cuidado.
mismo. LL Quisiera comprar ese auto, mas no tengo
2. Definición morfológica: Es una categoría grama- dinero.
tical invariable, por lo tanto, carece de accidentes d) Ilativas (consecutiva coordinada)
gramaticales. LL Pienso, luego existo.
3. Definición sintáctica: Es un nexo coordinante y LL Cobraste, por lo tanto, págame.
subordinante.
4. Definición lexicológica: Es una categoría de e) Distributivas
inventario cerrado, por lo tanto, no admite la incorpo- LL Bien está jugando, bien está durmiendo.
ración de nuevas palabras.
LL O ra practica álgebra, ora repasa lenguaje.
II. CLASIFICACIÓN
f) Explicativas
1. Conjunciones coordinantes
LL Siempre te quejas, o sea, eres inconforme.
Son aquellas que unen elementos del mismo valor sin-
LL Obtuviste el segundo lugar; es decir, clasifica-
táctico.
ción a la siguiente etapa.

Tipo Definición Catálogo 2. Conjunciones subordinantes
Expresan las ideas de Son aquellas que encabezan proporciones subordina-
Copulativas Y, e, ni, que
unión y sucesión. das. Cumplen la función de subordinar verbos, gene-
Dan las ideas de opción y ralmente.
Disyuntivas O, u
exclusión.
Pero, mas, sino,
Tipo Definición Catálogo
Adversativas Dan la idea de oposición. sin embargo, sino
que, no obstante. Porque, pues, ya
Indican el valor
Luego, entonces, Causales que, puesto que,
semántico de causa.
Dan la idea de ilación o conque, así que, como.
Ilativas Si, como si,
consecuencia. por lo tanto, en Dan el valor
consecuencia. siempre que, con
Condicionales semántico de
Dan el valor semántico Ora…ora, bien… tal que, en caso de
Distributivas condición.
de distribución bien, ya…ya que
Aportan la idea Aunque, así, a pesar
Sirven de nexo entre dos
Concesivas de objeción, pero de que, por más
proposiciones, siendo la Es decir, o sea,
Explicativas superable. que, si bien.
segunda una explicación esto es.
de la primera. Aporta el valor Que (precedida
Consecutiva semántico de por «tanto», «tan»,
consecuencia. «tal»)
Ejemplos: Para que, a fin de
Insertan la idea de
Finales que, con el objeto
finalidad o propósito.
a) Copulativas de que
LL Marido y mujer se dieron el sí. Insertan proposiciones
LL Mario e Inés se lo contaron. subordinadas que
LL No come ni deja comer. Completivas funcionan como Que, si
LL Está llora que llora. sujeto u objeto
directo.
b) Disyuntivas Comparan y actúan
LL ¿Quieres té o café? Comparativas como subordinantes Que, como
LL ¿Quieres que te acompañe Javier u Óscar? de nominales.

5º AÑO DE SECUNDARIA
Mejores Personas, Mejores Familias 185
Colegio

Mejores Personas, Mejores Familias LINGÜÍSTICA

Ejemplos: 5. ¿Cuántas conjunciones hay en: «Árbol que nace doblado


jamás su tronco endereza»?
a) Causales
LL S e retiró del partido porque estaba a) Dos b) Tres c) Cinco
lesionado. d) Cero e) Una
LL Ya que no me tomas en cuenta, prefiero no
opinar. 6. ¿Cuántas conjunciones coordinadas hay en: «Odioso y
cobarde en toda circunstancia, pero así has logrado con-
b) Condicionales quistar mi corazón que ya no me pertenece»?
LL Si hubiéramos hablado, hoy sería diferente.
LL Siempre que hablemos la verdad, nos a) Cero b) Una c) Dos
llevaremos bien. d) Tres e) Cinco

c) Concesivas 7. Relacionar:
LL Aunque me cueste la vida, sigo buscando tu I. Copulativas A. Pero, mas
amor. II. Disyuntiva B. Y, e, ni
LL A pesar de que se lo advertí, viajó por la III. Adversativa C. O, u
noche. IV. Consecutiva D. Luego, pues

d) Consecutiva a) IB, IIA, IIID, VC b) IA, IIC, IIIB, IVD


LL Se molestó tanto que lo golpeó. c) IB, IIC, IIIA, IVD d) IC, IIB, IIID, IVC
e) IA, IIB, IIIC, IVD
e) Finales
LL Para que me oigas, mis palabras se adelgazan 8. En qué oración no hay frase conjuntiva.
a veces.
a) Ya que no quieres, yo lo tomaré.
f) Completiva b) En caso que no viniese, tú lo reemplazarás.
LL Quiero que me digas la verdad. c) Dicen que llegó tarde sin embargo no lo vi.
LL Es necesario que trabajes con nosotros. d) Luego lo visitaré, pero no creo que lo encuentre.
e) Iré a pesar de que no lo deseo.
g) Comparativas
LL Es más alta que tu hermana. 9. En: «La tribuna del coloso se encontraba totalmente saturada:
LL Su piel como pétalos de rosas. ya no entraba ni un alfiler en esta final entre Gracilaso y los
cremas», encontramos__________ conjunciones.

a) Cero b) Una c) Dos


d) Tres e) Cinco

1. No es conjunción subordinante. 10. ¿Cuántos conjunciones copulativas hay en: «Es toda forma
de comunicación (palabras, gestos, mímicas, colores, luces
a) Aunque b) Así que c) Pero y otros) que emplea el hombre para la expresión de pensa-
d) Si e) Porque mientos»?

2. Alternativa que presenta solo conjunciones. a) Cero b) Una c) Dos


d) Tres e) Cinco
a) Y, o, que, porque b) Por, entre, padre, con
c) A, sin, pues, ya d) Supe, doy, dé, sal
e) Hoy, ayer, tú, ven

3. Es una conjunción subordinante.

a) Si b) Solo c) No 1. ¿Cuántos conjunciones adversativas hay en: «La fiesta es-


d) Ya e) Mas tuvo divertida hasta que llegó Manuel e hizo que todas se
aburrieran y se fueran»?
4. ¿Cuántas conjunciones hay en: «Ven cuando todos se ha-
yan ido, pero dime antes si te quedarás, amada elegante»? a) Cero b) Una c) Dos
d) Tres e) Cinco
a) Cero b) Tres c) Cinco
d) Dos e) Una

186 Mejores Personas, Mejores Familias


5º AÑO DE SECUNDARIA
Colegio

LINGÜÍSTICA  Mejores Personas, Mejores Familias

2. En: «El agua de la fuente estaba muy helada y los animales 10. Completa: El profesor escribió en la pizarra; ____, ningún
sufrían por la escasez. Un águila surcaba el cielo también alumno escribía.
sedienta», ¿cuántas conjunciones hay?
a) porque b) sin embargo c) pues
a) Cero b) Una c) Dos d) y e) de vez en cuando
d) Tres e) Cinco

3. «La lingüística es la ciencia que estudia el lenguaje humano


y el proceso comunicativo». En el texto hay __________.
a) una conjunción disyuntiva
b) dos conjunciones adversativas 1. Es una conjunción subordinante.
c) una conjunción copulativa
d) una conjunción consecutiva a) Aunque b) Pero c) Sin embargo
e) no hay conjunción d) Ni e) que

4. Es locución conjuntiva: 2. Ten cuidado con esa mujer: ____es hermosa ____tiene un
trato delicado, esa apariencia esconde un ser traicionero
a) Estadio Nacional b) Sin censura
____ carente de moral.
c) Hasta por d) En realidad
e) Sin embargo a) si bien – y – y
b) dado que – por lo tanto – también
5. ¿Cuál de las siguientes oraciones presenta una conjunción c) ya que – pero – y
adversativa?
d) si bien – y – además
a) Pedro es inteligente, luego es descuidado. e) por lo que – mas – y
b) Pedro es inteligente y, desde luego, descuidado.
c) Pedro es inteligente, aunque es descuidado. 3. El tiempo hace olvidar ____ reviste; _____, da hermosura
d) Pedro es inteligente, pero descuidado. a los malos momentos pasados.
e) Pedro es inteligente, y luego es descuidado.
a) si – por ciento b) pero – además
6. Es una conjunción adversativa. c) y – en cambio d) pues – luego
e) y – inclusive
a) Más b) y c) o
d) por lo tanto e) mas 4. ¿Qué clase de conjunción hay en: «Esa película me hizo
recordar mis días pasados con ustedes, porque era muy
7. No es conjunción divertida»?
a) Como b) O sea c) Es decir
d) Sin e) Que a) Copulativa b) Disyuntiva c) Condicional
d) Causal e) Concesiva
8. Oración que presenta conjunción comparativa.
5. La función sintáctica de la conjunción es_____.
a) Vino y se fue.
b) Salió pero dejó el encargo. a) Modificador b) Circunstancial c) Núcleo
c) Trabajas o estudias. d) Objeto e) Nexo
d) Es tan alto como tú.
e) Ninguna vino hoy.

9. Muestra conjunción equivalente a pero.

a) Cantas o bailas.
b) Ni canta ni baila.
c) Saluda, pero rápido.
d) Abunda el colibrí o pájaro mosca.
e) Trabaja, no obstante es lenta.

5º AÑO DE SECUNDARIA
Mejores Personas, Mejores Familias 187
Colegio

Mejores Personas, Mejores Familias LINGÜÍSTICA

M A
TE

20   LA ORACIÓN: CLASIFICACIÓN SEGÚN


LA ACTITUD DEL HABLANTE
I. INTRODUCCIÓN a) Actitud enunciativa
La sintaxis es la parte de la gramática que se encarga de El hablante expresa un hecho o una idea; además, el
estudiar el funcionamiento de las palabras dentro de la hablante nos da a entender que su mensaje es una rea-
oración. lidad objetiva.
En términos más sencillos, se encarga de estudiar a la Por ejemplo: Tienes el pelo ardiendo. (El verbo apare-
oración y sus elementos en funcionamiento. ce en su modo indicativo)
Su unidad es el SINTAGMA Pueden ser de dos tipos:
●● Afirmativas: Afirman un hecho o una idea.
II. UNIDADES SINTÁCTICAS : DIFERENCIA ENTRE Por ejemplo: Tienes el pelo ardiendo. / Sí tienes el
ORACIÓN Y PROPOSICIÓN pelo ardiendo.
●● Negativas: Niegan un hecho o una idea, y lo ha-
SENTIDO AUTONOMÍA cen por medio de algún adverbio de negación (no,
nunca, jamás…) o de los pronombres «nadie», «nin-
ORACIÓN Sí Sí guno» y «nada».
Por ejemplo:
PROPOSICIÓN Sí No
No tienes el pelo ardiendo. / Nadie tiene el pelo
Autonomía: Independencia, que no esté conectada a otra ardiendo.
expresión.
YY Ejemplos de oraciones: La lengua es un sistema de signos. b) Actitud interrogativa
El hablante expresa una pregunta. Pero puede estar
III. LA ORACIÓN haciéndolo de una forma directa o indirecta.
●● Oraciones interrogativas directas: Emplean signos de
nexo interrogación.
La lengua es un sistema de signos y se realiza en el habla. Por ejemplo:
¿Tienes el pelo ardiendo?
Proposición Proposición
Oración compuesta ●● Oraciones interrogativas indirectas: Se presen-

tan por medio de los llamados verbos de lengua
1. Definición de oración
(decir, preguntar, etc.)
Una definición tradicional es «La oración es la palabra
Por ejemplo:
o conjunto de palabras que tiene sentido completo y
Me pregunto si tienes el pelo ardiendo.
autonomía sintáctica» que aunque es imprecisa, tra-
ta de reflejar el hecho de que la oración pragmática-
●● Oraciones interrogativas totales: El hablante
mente es el fragmento más pequeño del discurso que
pregunta por la totalidad de lo que dice, es decir,
comunica una idea completa y posee independencia
busca una respuesta afirmativa o negativa sobre lo
(es decir, podría sacarse del contexto y seguir comuni-
que pregunta.
cando).
Por ejemplo:
¿Tienes el pelo ardiendo? (La respuesta que se
2. Clasificación de las oraciones según la actitud del
busca es «sí» o «no»).
hablante
¿Qué es la actitud del hablante?
●● Oraciones interrogativas parciales: El hablante
Imagina cualquier cosa que puedas decirle a otra per-
pregunta por una parte del contenido de lo que
sona, recuerda algo que le hayas dicho hoy a alguien.
dice, es decir, busca una información en particu-
Piensa en ese mensaje que le estás transmitiendo a
lar acerca de su mensaje. O, dicho de otro modo,
quien te oye. Date cuenta de que no solo le estás ha-
puede preguntar por el sujeto de su enunciado, o
ciendo llegar una información determinada, sino que,
por la causa, o por el momento, etc. Para esto usa
al hablar, estás manifestando una posición ante lo que
partículas interrogativas.
dices, estás dando esa información de un modo concre-
Por ejemplo:
to; y esto añade cosas a ese mensaje, le da una forma,
¿Quién tiene el pelo ardiendo? / ¿Por qué tienes el
y hace que el oyente lo interprete de un modo u otro.
pelo ardiendo? / ¿Cuándo has tenido el pelo ar-
Cuando hablas, tomas una actitud ante lo que dices.
diendo?

188 Mejores Personas, Mejores Familias


5º AÑO DE SECUNDARIA
Colegio

LINGÜÍSTICA  Mejores Personas, Mejores Familias

c) Actitud exclamativa
El hablante transmite sentimientos y emociones (ale-
gría, tristeza, admiración, temor, etc.) en su mensaje,
con lo que consigue darle énfasis. El principal elemen-
to gráfico que distingue estas oraciones es el uso de 1. La oración: «No olvidaré lo que me hiciste» es______.
los signos de exclamación. Aunque también se pue-
den encontrar estos signos en oraciones desiderativas a) enunciativa b) apelativa c) interrogativa
o imperativas para darle más énfasis a los deseos y a d) dubitativa e) desiderativa
las órdenes.
Por ejemplo: ¡Tienes el pelo ardiendo! 2. ¡Si no te marcharas!, es una oración ________.

d) Actitud imperativa a) enunciativa. b) imperativa. c) desiderativa.


d) dubitativa. e) interrogativa.
El hablante transmite una orden o un ruego. Por eso el
verbo suele aparecer en modo imperativo.
3. Presenta interrogación indirecta.
Por ejemplo: Quémate el pelo.
Aunque también se pueden usar otras fórmulas con a) Tu tienes lo mío y yo lo tuyo.
las que el hablante puede manifestar su actitud impe- b) Ojalá que llueva café en el campo.
rativa: c) Me sube la bilirrubina.
●● Deber + infinitivo: Debes quemar tu pelo. d) No sé como pedírselo.
●● Tener que + infinitivo: Tienes que quemarte el e) Al pobre Marito le cuesta creerlo.
pelo.
●● Hay que + infinitivo: Hay que tener el pelo ardien- 4. Presenta proposiciones.
do.
a) Yo no te conozco.
●● No + infinitivo: No quemarse el pelo.
b) Cuenta la historia que a mi encuentro volvió.
●● A + infinitivo: ¡A quemarse el pelo!
c) Ambos bailábamos una canción de moda.
d) ¿Por qué no?
Otras formas de expresar órdenes o ruegos, pero que necesi- e) Algo retrasada llegó.
tan estar dentro de un contexto para comprenderlas ade-
cuadamente como imperativas: 5. La diferencia entre una proposición y una oración es
●● Presente de indicativo: ¡Tú te quemas el pelo ya! el (la) _____.
●● Futuro de indicativo: ¡Te quemarás el pelo ahora
mismo! a) sentido completo b) entonación propia
●● Presente de subjuntivo: ¡Que todo el mundo tenga c) dependencia léxica d) dependencia sintáctica
el pelo ardiendo! e) presencia de modificadores

6. No es oración desiderativa.
e) Actitud desiderativa
El hablante manifiesta su deseo. Puede expresarlo por a) Desearía verlo caminar ya.
medio de varias fórmulas: b) ¡Si dieras un beso!
●● Ojalá + subjuntivo: Ojalá tengas el pelo ardiendo. c) Te extraño como el mar extraña al sol.
●● Verbo de deseo en condicional: Me gustaría que d) Ojalá acierte.
tuvieras el pelo ardiendo. e) Ojalá no fume tanto.
●● Querer + infinitivo: Quiero quemar tu pelo.
●● Si + subjuntivo: ¡Si ardiera tu pelo! 7. «Ahora tengo clara la noción del amor y del dolor aunque
fue en un lejano pasado. Esta lección ya no es para mí. Dí-
ganle adiós si hay que hacerlo. Cierren su historia de amor.
f) Actitud dubitativa Terminen su libro»,
El hablante expresa una duda sobre lo que dice. Para ¿cuántas oraciones hay?
ello dispone de varias fórmulas:
●● Deber de + infinitivo: Debes de tener el pelo ar- a) Cuatro b) Siete c) Cinco
diendo. d) Ocho e) Seis
●● Puede que + subjuntivo: Puede que tengas el pelo
ardiendo. 8. Es una oración desiderativa.
●● Algunos adverbios y locuciones adverbiales +
subjuntivo: Quizás tengas el pelo ardiendo / Tal a) Vuelve. b) Conquístame. c) Sí.
vez tengas el pelo ardiendo. d) A veces. e) Caminaré.

5º AÑO DE SECUNDARIA
Mejores Personas, Mejores Familias 189
Colegio

Mejores Personas, Mejores Familias LINGÜÍSTICA

9. «No me digas que es muy tarde ya», es una oración ____. 7. Señale la oración imperativa

a) unimembre. b) desiderativa. a) Ojalá puedas venir a buscarme más temprano, Ifigenia.


c) interrogativa. d) exhortativa. b) No me hagas caso, estoy tan ebrio que no sé lo que digo.
e) enunciativa.
c) Vamos, muchachos, ustedes pueden ganar.
10. «Con cada día que pasa me doy cuenta que todo lo tuyo d) Muévete, combi del demonio, ¡déjame pasar!
fue una vil mentira», es una oración_______. e) ¡Cómo me gustaría darte una mano en tu problema,
querida!
a) enunciativa b) dubitativa c) desiderativa
d) interrogativa e) exhortativa 8. ¿Qué oración es interrogativa indirecta?

a) ¿Qué hora ves en el reloj de la torre?


b) Cuéntame qué te dijo el médico, amor.
c) ¿Viniste anoche al recital?
d) Tal vez sea mejor discutir esto luego ¿no?
1. ¿Qué oración es aclarativa o enunciativa?
e) Nadie va a dejar de apresurarte si te retrasas.
a) No iré a tu casa.
b) Quizá llegue hoy. 9. ¿Qué oración es dubitativa?
c) ¿Dónde vives?
d) Terminen la tarea. a) ¿Vienes o no?
e) ¡Cuánto quisiera que esto cambie! b) Y si le ponemos este otro vestido… ¿como se verá?
c) Aún los más fieros tiemblan ante el león, ¿no?
2. «Busca por dentro, amor, que pongo en juego todos mis
sentimientos»; es una oración_________. d) A lo peor ya terminó la ceremonia y tal vez no llegamos.
e) Ja,ja,ja…no sé si será cierto lo que cuentas, pero es di-
a) enunciativa b) dubitativa c) exhortativa vertido.
d) interrogativa e) desiderativa
10. ¿En qué oraciones predomina la función apelativa?
3. Enunciado correcto:
a) Desiderativas b) Exclamativas c) Informativas
a) La sintaxis estudia la estructura de la oración. d) Imperativas e) Enunciativas
b) Toda frase es siempre una oración.
c) Un verbo por sí solo no puede formar una oración.
d) Proposición y oración son lo mismo.
e) Toda oración tiene proposiciones.

4. En relación a la sintaxis, señale lo incorrecto.

a) La estructura sintagmática es estudiada por la sintaxis. 1. Señale la oración exclamativa.


b) Su unidad mínima es el sintagma.
c) Estudia las funciones de las palabras dentro de un a) Ella avanzó tranquilamente por el bosque.
enunciado.
d) Estudia la estructura interna de las palabras. b) ¡Ese es mi hijo!
e) Forma parte de la gramática. c) Ya dejaron muchos cabos sueltos en la investigación.
d) Dime qué puedo hacer por ti.
5. Alternativa que corresponde a una oración declarativa: e) Ojalá jueguen con más ganas esta vez.

a) Nunca creí en nada de lo que decía. 2. Relaciona y marca lo correcto:


b) ¡Si pudiera saber lo que pasó! I. Lávate la cara con agua y jabón.
c) Acaso nuestro padre venga hoy. II. Tus ojos son fascinantes.
d) Quizá no sabe hasta dónde llegará.
III. Quizá asista a la conferencia.
e) ¡Haga bien las cuentas, por favor!
1. Dubitativa 2. Imperativa
3. Desiderativa 4. Enunciativa
6. Se denomina oración desiderativa___________.
a) I3, II4, III1
a) Cuando contiene más de un verbo.
b) I2, II4, III1
b) Cuando no se halla al sujeto.
c) I3, II2, III4
c) Cuando el sujeto es tácito.
d) I1, II2, III3
d) Cuando niega o afirma un hecho.
e) I1, II3, III4
e) Cuando expresa un deseo.

190 Mejores Personas, Mejores Familias


5º AÑO DE SECUNDARIA
Colegio

LINGÜÍSTICA  Mejores Personas, Mejores Familias

3. Oración interrogativa directa total. 5. Acerca de la oración podemos afirmar que _____.

a) No sabe qué puede comprar con ese dinero. a) siempre tiene sujeto y predicado.
b) Dime quién es el culpable. b) no tiene clasificación semántica.
c) ¿Estudió lo que quería? c) es todo enunciado con autonomía sintáctica.
d) ¿Cuándo será el reencuentro? d) es un grupo de palabras con verbo.
e) ¿De quién es ese libro? e) de acuerdo a la clasificación sintáctica pueden ser dubi-
tativas, desiderativas, enunciativas, etc.
4. No es oración imperativa.

a) Váyanse inmediatamente.
b) Ve y dile que lo extraño.
c) Regrésala pronto a casa.
d) Me molesta escuchar esa música.
e) Aléjate de ella.

5º AÑO DE SECUNDARIA
Mejores Personas, Mejores Familias 191
Colegio

Mejores Personas, Mejores Familias LINGÜÍSTICA

M A
TE

21   USO DEL PUNTO Y COMA, EL


PUNTO Y LOS DOS PUNTOS

I. EL PUNTO Ejemplos:
Signo de puntuación (.) cuyo uso principal es señalar grá- ●● Cada grupo irá por un lado diferente: el primero,
ficamente la pausa que marca el final de un enunciado — por la izquierda; el segundo, por la derecha; el ter-
que no sea interrogativo o exclamativo—, de un párrafo o cero, de frente.
de un texto. ●● Se dieron cita el presidente ejecutivo, Francisco
Ruiz; el consejero delegado, Pedro García; el vo-
Clases cal, Antonio Sánchez; y el secretario general, Juan
Recibe distintos nombres, según marque el final de un González.
enunciado, un párrafo o un texto:
a) Si se escribe al final de un enunciado y a continuación, 2. Ideas completas
en el mismo renglón, se inicia otro, se denomina punto Para separar oraciones sintácticamente independien-
y seguido, nombre más lógico y recomendable que el tes entre las que existe una estrecha relación semántica.
también usual de punto seguido. Si el punto y seguido Ejemplo:
coincide con el final de una línea, se comienza a escribir ●● Era necesario que el hospital permaneciese abier-
en la siguiente con el mismo margen, sin sangría inicial. to toda la noche; hubo que establecer turnos.
El punto y seguido es, pues, el que separa los enuncia- ●● Todo el mundo a casa; ya no hay nada más que
dos que integran un párrafo. hacer.

b) Si se escribe al final de un párrafo y el enunciado si- 3. Delante de conectores


guiente inicia un párrafo nuevo, se denomina punto Se escribe punto y coma delante de conectores de sen-
y aparte, aunque en algunas zonas de América se dice tido adversativo, concesivo o consecutivo, como pero,
punto aparte. mas, aunque, sin embargo, por tanto, por consiguien-
Estuvo rondando la casa varias horas, silbando claves te, etc., cuando las oraciones que encabezan tienen
privadas, hasta que la proximidad del alba lo obligó a cierta longitud.
regresar. En el cuarto de su madre, jugando con la her- Ejemplos:
manita recién nacida y con una cara que se le caía de ●● Los jugadores se entrenaron intensamente durante
inocencia, encontró a José Arcadio. todo el mes; sin embargo, los resultados no fueron
Úrsula había cumplido apenas su reposo de cuarenta los que el entrenador esperaba.
días, cuando volvieron los gitanos. Eran los mismos sal-
timbanquis y malabaristas que llevaron el hielo. 4. Reemplazando conectores de contraste
El punto y coma puede reemplazar a conectores de
(Ga Márquez Años [Col. 1967]). contraste cuando separa dos ideas contrarias yuxta-
puestas.
El punto y aparte es, pues, el que separa dos párrafos Ejemplo:
distintos, que suelen desarrollar, dentro de la unidad ●● Sufría un dolor indecible; mantuvo hasta el final
del texto, ideas o contenidos diferentes. una expresión serena.

c) Si se escribe al final de un escrito o de una división III. LOS DOS PUNTOS


importante del texto, se denomina punto final. No es Signo de puntuación (:) que representa una pausa mayor
correcta la denominación punto y final, creada por que la de la coma y menor que la del punto. Detienen el
analogía de las correctas punto y seguido y punto. discurso para llamar la atención sobre lo que sigue, que
siempre está en estrecha relación con el texto precedente.
II. PUNTO Y COMA Se escriben pegados a la palabra o el signo que los antece-
Signo de puntuación (;) que indica una pausa mayor que la de, y separados por un espacio de la palabra o el signo que
marcada por la coma y menor que la señalada por el punto. los sigue:
a) Preceden a una enumeración de carácter explicativo:
1. Enumeraciones complejas ●● Ayer me compré dos libros: uno de Jesús Huamán y
Para separar los elementos de una enumeración cuan- otro de María Torres.
do se trata de expresiones complejas que incluyen co-
mas.

192 Mejores Personas, Mejores Familias


5º AÑO DE SECUNDARIA
Colegio

LINGÜÍSTICA  Mejores Personas, Mejores Familias

b) Cuando, por interés, se anticipan los elementos de la


enumeración, los dos puntos sirven para cerrarla y dar
paso al concepto que los engloba:
●● Natural, sana y equilibrada: así debe ser una bue-
na alimentación. 1. Señala qué opción presenta adecuada puntuación.
a) Corazón, qué hice para merecer esto: no me miras, no
c) Preceden a la reproducción de citas o palabras textua- me hablas ni devuelves mi saludo.
les, que deben escribirse entre comillas e iniciarse con b) Corazón: que hice para merecer esto; no me miras, no
mayúscula: me hablas, ni devuelves mi saludo.
●● Ya lo dijo Venancio Benites: «La claridad es la cor- c) Corazón, qué hice para merecer esto: no me miras; no
tesía del filósofo». me hablas; ni devuelves mi saludo.
d) Corazón: Qué hice para merecer esto; no me miras, no
d) Se emplean tras las fórmulas de saludo en el encabeza- me hablas ni devuelves mi saludo.
miento de cartas y documentos. e) Corazón, que hice para merecer esto: no me miras, no
En este caso, la palabra que sigue a los dos puntos, y me hablas, ni devuelves mi saludo.
que inicia el cuerpo de la carta, se escribe con inicial
mayúscula y en renglón aparte: 2. Señala en qué alternativa encontramos una oración inade-
●● Muy señor mío: / Le agradeceré que en el plazo más cuadamente escrita.
breve posible... a) Cuando la tarde languidece, renace la sombra; todo se
apaga.
e) Sirven para separar una ejemplificación del resto de la b) Dime, Zoila, qué haremos por la noche; sin tu hermani-
oración: to.
●● De vez en cuando tiene algunos comportamientos c) Esos jóvenes que van para allá; son mis amigos.
inexplicables: hoy ha venido a la oficina en zapatillas. d) Podrías, mi pequeña, dejar de ser tan problemática; así
me gustas.
f) En textos jurídicos y administrativos, como decretos, e) El oso de anteojos sufre porque su osa lo dejó; ya se
sentencias, bandos, edictos, certificados o instancias, olvidará.
se colocan después del verbo que presenta el objetivo
fundamental del documento y que va escrito con to- 3. Si estás apurada ______ vete sola _____ a mí no me im-
das sus letras en mayúscula. La primera palabra que porta que te quedes conmigo ______ exnovia.
sigue a dicho verbo se escribe con inicial mayúscula y a) , - ; - , b) , - , - ; c) ,-,-,
en párrafo aparte: d) , - : - , e) ; - : - ;
●● CERTIFICA: Que D. José Álvarez García ha segui-
do con aprovechamiento el Curso... Solamente en 4. Si me lo pide, se quedará acá ___señor bodeguero___ su
este caso los dos puntos son compatibles con la con- pequeña hija ____ solo si ella promete portarse bien.
junción subordinante que. a) , - , - ; b) ,- - - ; b) , - , - ,
d) ; - ; - , e) ( - ) - ,
g) Se usan también para conectar oraciones relacionadas
entre sí sin necesidad de emplear otro nexo. Son varias
5. Sabía _______ iba a llegar ________ no sabía con
las relaciones que pueden expresar:
________ lo haría _______ tuve que tomar previsiones.
●● Causa-efecto: Se ha quedado sin trabajo: no po-
a) cuándo – ; – quien – :
drá ir de vacaciones este verano.
b) cuando – : – quién – ;
●● Conclusión, consecuencia o resumen de la ora-
c) cuando – ; – quien – :
ción anterior: El arbitraje fue injusto y se cometie-
d) cuándo – : – quien – ,
ron demasiados errores: al final se perdió el parti-
e) cuándo – ; – quién – :
do. En este caso se usa también el punto y coma.
●● Verificación o explicación de la oración anterior,
6. Presenta correcta puntuación.
que suele tener un sentido más general: El ceviche
a) A quién, venga dígale qué acordamos: el laser será para
es un plato muy completo y nutritivo: tiene la pes-
mí.
cado, camote y limón, y la fuerza de la cebolla. En
b) A quién venga dígale; qué acordamos: el laser será para
este caso se usa también el punto y coma.
mí.
c) A quién venga dígale qué acordamos: el láser, será para
h) En títulos y epígrafes es frecuente su uso para separar
mí.
el concepto general del aspecto parcial del que va a
d) A quién venga dígale qué acordamos, el láser será para
tratarse:
mí.
●● La literatura medieval: estudio comparativo de los
e) A quién venga dígale qué acordamos: el láser será para
principales motivos recurrentes.
mí.

5º AÑO DE SECUNDARIA
Mejores Personas, Mejores Familias 193
Colegio

Mejores Personas, Mejores Familias LINGÜÍSTICA

7. Es una oración con adecuada puntuación. 2. Soy alto ___ mido 1.70 ___ tengo ojitos caramelo ___ no
a) Nunca más, mujer, podrás hacerme daño: desde hoy, he tengo novia.
decidido que te dejaré de amar. a) , –; – , b) ; – ; – ; c) ; – ; – ;y
b) Nunca más, mujer; podrás hacerme daño: desde hoy, he d) , – , – ; e) , – , – ,y
decidido que te dejaré de amar.
c) Nunca más; mujer, podrás hacerme daño; desde hoy he 3. Tiene correcta redacción.
decidido que te dejaré de amar. a) Desde que te vi; solo puedo pensar en ti, Natacha.
d) Nunca más, mujer, podrás hacerme daño, desde hoy he b) Se acercó a Pancha, sin embargo no le dijo una palabra.
decidido que te dejaré de amar. c) No conseguiré lo que quiero sin tu ayuda.
e) Nunca más, mujer; podrás hacerme daño, desde hoy he d) Ese viejo amigo tuyo; sabe escribir bellos versos.
decidido que te dejaré de amar. e) Dile a Raquel: que no se vaya sin mí.
4. Hace falta dos puntos.
8. Presenta correcta redacción. a) No crea don Cecilio que me gustó.
a) Aunque pasen cien años, yo, querida amiga; no te podré b) Para poder acercarse cambió su personalidad.
olvidar: eres y serás mi único amor. c) Hoy no verás a Inés se fue sin decir a dónde.
b) Aunque pasen cien años, yo, querida amiga, no te podré d) Todos incluso Damián irán a las olimpiadas.
olvidar; eres y serás mi único, amor. e) Amiga David José el padre de tu hijo volvió.
c) Aunque pasen cien años, yo; querida amiga; no te po-
dré olvidar: eres y serás mi único amor. 5. Para que te ame ___ escucha mi amistoso consejo ___
d) Aunque pasen cien años yo, querida amiga, no te podré hazle un embrujo y será para ti____ Monillo.
olvidar; eres y serás mi único amor. a) ; – , – , b) : – pero – , c) , – ; – ;
e) Aunque pasen cien años, yo, querida amiga, no te podré d) , – : – , e) , – ya – ,
olvidar: eres y serás mi único amor.
6. En Cerca de aquí vive Daniela la diosa del sabor sin em-
9. Señala la opción que presenta una adecuada escritura. bargo no está sola vive con ella Reyner su esposo, es cierto
a) Cuando era un chiquillo qué alegría, jugando a la gue- que_____________.
rra noche y día. a) faltan 3 comas.
b) Cuando era un chiquillo, qué alegría, jugando a la gue- b) se necesitan 2 punto y coma.
rra noche y día. c) faltan guiones.
c) Cuando era un chiquillo, que alegría; jugando a la gue- d) se necesita de los dos puntos.
rra noche y día. e) a, y d.
d) Cuando era un chiquillo; que alegría, jugando a la gue-
rra noche y día. 7. Nosotros estábamos muy inquietos ___ por ello ___ deci-
e) Cuando era un chiquillo: qué alegría, jugando a la gue- dimos pasear un rato ___ no obstante ___ no conseguimos
rra noche y día. sosiego.
a) , – , – , – , b) ; – ( – ; – ) c) ; – , – ; – ,
10. Le falta los dos puntos necesariamente. d) , – ; – , – ; e) ( – , – , – )
a) No sé por qué aún continúas portándote así.
b) María si tu papá regresa dile que me busque. 8. Tiene correcta puntuación.
c) La señora belleza de cuatro décadas me ama. a) Tienes la bemba colorada pero el alma ennegrecida.
d) José Marlon y Juan aman a Juanacha. b) Por fin, trajo: la pelota, la camiseta y los chimpunes.
e) Él o tú cualquiera se las verá conmigo soy lo máximo. c) Puedes traer; por ejemplo, un tarro de leche.
d) Le pediste mucho más amor; no le importas ni un tan-
tito.
e) Alegre como nunca; Elvirina abandonó el albergue.

9. En No estaremos ocupados el sábado así que puedes visi-


1. Indica en qué opción hace falta un punto y coma. tarnos Camilo tu tía Tina no parece estar a pesar de todo
a) Todos los que no hayan comprado los ingredientes no muy alegre si puedo convencerla nos acompañará, faltan
pasarán por acá. ___ punto y coma.
b) Esa vieja canción de amor me recuerda a mis años en La a) dos b) seis c) cinco
Habana Cuba. d) cuatro e) tres
c) Ana y Cleo copiaron fueron descubiertas.
d) Los niños que no hicieron su tarea se quedarán hasta las 10. Marca la alternativa que presenta puntuación correcta.
cuatro o cinco. a) José estudia Química en San Marcos; Elías, Medicina.
e) Una vez más estoy esperando que me envíes esas cartas b) La cantante Sonia, visito París, Roma y España.
de nuestro tío. c) Señor este producto es bueno y económico; cómprelo.
d) El Dr. Reyes, lo explicó con claridad y precisión.
e) El que persevera tiene asegurado el éxito hermano.

194 Mejores Personas, Mejores Familias


5º AÑO DE SECUNDARIA
Colegio

LINGÜÍSTICA  Mejores Personas, Mejores Familias

7. Tiene correcta puntuación.


a) Mantengan, siempre chicos, en todo momento, sus
grandes sueños; estudiar, ingresar y triunfar.
b) Mantengan siempre, chicos, en todo momento; sus
1. En esa avenida ____ venderás papita con huevo ____ mi grandes sueños; estudiar, ingresar y triunfar.
amada ____ de ahora en adelante. c) Mantengan, siempre, chicos; en todo momento, sus
a) , - : - , b) , - , - , c) , - , - ; grandes sueños: estudiar, ingresar y triunfar.
d) , - - - , e) - - - - , d) Mantengan siempre, chicos, en todo momento, sus
grandes sueños: estudiar, ingresar y triunfar.
2. Le dio muchas recomendaciones_____consejos___ estra- e) Mantengan siempre, chicos en todo momento, sus
tegias_____al final no le hizo caso. grandes sueños: estudiar, ingresar y triunfar.
a) , - , - ; b) ; - y - : c) : - , - .
d) : - , - y e) , - y - , pero 8. En el enunciado antes de aceptar sus propuestas amigas
vean las propuestas detenidamente, se ha omitido el em-
3. Tengamos en cuenta que ___ a pesar de todo lo sucedi- pleo de____________.
do___ estos problemas nos ayudaran a mejorar nuestra a) dos comas
actitud ___ pero no podemos descuidar los otros aspec- b) el punto y coma
tos___ c) los dos puntos
a) , - , - ¡ - ! b) : - ; - , - . c) - - - - , - , d) una coma
d) , - , - ; - . e) ( - ) - , - . e) los puntos suspensivos

4. El mejor padre es Dios el mejor maestro el tiempo el mejor 9. En la expresión Los padres de ese colegio actuaron con
libro el mundo. apresuramiento los nuestros decidieron evaluar plantear so-
¿Qué signos de puntuación deben figurar? lucionar bien los problemas, el punto y coma va después de
a) ; - , - : - , b) , - : - , - ; c) , - ; - , - ; la palabra____________.
d) ; - , - ; - , e) : - , - ; - , a) colegio d) plantear
b) apresuramiento e) bien
5. Milagros Esther y Anatolia tres jóvenes muy bellas conocie- c) nuestros
ron a un muchacho culto elegante y muy simpático el cual
conquistó el corazón de una de ellas. 10. Los signos de puntuación que deben ir en la siguiente ora-
En el texto se han omitido_______ coma(s) y________ ción son_____________.
punto y coma. Los amantes nos ofrecen pasión los árboles frutas las relacio-
a) 5 - 1 b) 4 - 1 c) 5 - 0 nes serias complicaciones.
d) 4 - 3 e) 3 - 1 a) punto y coma, coma, punto y coma, coma.
b) coma, punto y coma, coma, coma.
6. En qué alternativa se ha empleado incorrectamente los dos c) punto y coma, coma, punto y coma, punto y coma.
puntos. d) coma, coma, punto y coma, coma.
a) Hay una cruel forma de publicidad: el ataque por medio e) punto y coma, coma, coma, punto y coma.
de forma escrita y radial.
b) El cuerpo tiene tres partes: cabeza, tronco y extremida-
des.
c) En los signos se producen dos tipos de clasificación: na-
turales y artificiales.
d) Las palabras de Marco fueron: «Te quiero más que a mi
vecina».
e) El lobo perdió los pelos: caperucita, las mañas.

5º AÑO DE SECUNDARIA
Mejores Personas, Mejores Familias 195
Lingüística
Colegio

Mejores Personas, Mejores Familias LINGÜÍSTICA

M A
TE

22   EL PREDICADO I

I. EL PREDICADO Para reconocerlo:


Es lo que se dice del sujeto. YY Se le pregunta ¿Qué es lo que? al verbo.
Ejemplo: La pregunta a veces no sirve.
YY La higuera no dio fruto este año.
S P ●● Juan peinaba a su hijo.
Para localizarlo: Idetentificando a aquello que no es ¿Qué es lo que peinaba? o ¿A quién peinaba?
sujeto.
Otras maneras de reconocerlo:
II. CLASES DE PREDICADO YY Hay que recurrir a sustituir por los pronombres perso-
1. Predicado nominal (PN) nales átonos lo, la, los las.
Está formado por los verbos copulativos SER, ESTAR, YY La sustitución puede no servirnos pues podemos ser
PARECER y un ATRIBUTO. leístas.
●● La chica es inteligente. ●● Juan le peinaba o Juan lo peinaba.
PN
YY La solución es poner el complemento en femenino y
2. Predicado verbal (PV) luego sustituirlo.
Está formado por todos los demás verbos. ●● Juan peinaba a su hija.
●● Juan la peinaba.
●● El niño come manzanas.
N OD YY S i para el femenino es la para el masculino es lo (Juan
PV lo peinaba) y por lo tanto es OD.
●● Juan peinaba a su hijo.
OD
¡CUIDADO!
SER y ESTAR pueden engañarnos y funcionar
2. El complemento u objeto indirecto (OI).
como PV cuando van con CC (complemento Definición: Complemento indirecto es la persona o cosa que
circunstancial) y no con atributo. recibe indirectamente la acción del verbo.

El piloto dio un regalo a su mujer.
●● El niño está en Madrid. OD OI
Det N N CC
S PV Para localizarlo:
YY Se pregunta ¿A quién? o ¿Para quién? al verbo.
●● La boda es en la catedral. ●● Antonio preguntó la lección a su compañera.
Det N N CC OI
S PV YY Se puede sustituir por le, les o se.
●● Antonio le preguntó la lección. Antonio se la pre-
guntó.
III. COMPLEMENTOS DEL VERBO � No confundir con el OD. Poner en femenino si es ne-
cesario.
1. El complemento u objeto directo (OD).
Definición: Complemento directo es la persona o
cosa que recibe directamente la acción del verbo.

●● Ángela pintaba un cuadro.


OD
●● Ángel lavaba a su hermano.
OD

196 Mejores Personas, Mejores Familias


5º AÑO DE SECUNDARIA
Colegio

LINGÜÍSTICA  Mejores Personas, Mejores Familias

9. En: «A los lectores entrega el bibliotecario los libros», halla-


mos la siguiente estructura:
a) S, NP, OD, OD
1. Oración con OI. b) OD, NP, OD, S
a) Escribe al atardecer. c) OI, NP, OD, S
b) Escribe a su madre. d) OI, NP, S, OD
c) Escribe una carta. e) S, NP, OI, OD
d) Escribe a contraluz.
e) Escribe las últimas noticias. 10. Cuando HABER es empleado como impersonal, la construc-
ción nominal que aparece junto a él y que aparentemente es
sujeto, en realidad es ___.
2. Oración con dos OI.
a) objeto directo.
a) Destruyó su finca para distraer a los ladrones.
b) objeto indirecto.
b) Escogió a los mejores para ese trabajo.
c) atributo.
c) Nos sorprendió con ese gesto.
d) complemento agente.
d) Iremos a verte hoy.
e) aposición.
e) A Pedro le gusta cantar boleros.

3. No puede funcionar como OD.


a) El libro
b) A Mariela 1. Realiza el análisis sintáctico de Cuéntaselo.
c) Lo
a) NP , OD, OI
d) Con Domingo
b) NP, OI, OD
e) Cantar óperas
c) NP, C. Circuns., OD
4. En: «Dime si te sientes bien», el OD es _____. d) NP, C. Agente, OI
a) me e) Solo NP
b) si
c) sientes bien 2. La frescura del agua da a las flores un aroma primaveral. El
d) si te sientes bien OD es______.
e) te a) la frescura del agua
b) a las flores
5. En: «Le exigió controlar sus nervios», señale lo correcto. c) las flores
a) Le: OD d) una belleza
b) Sus nervios: OD
e) un aroma primaveral
c) Controlar: NP
d) Le: OI
e) Más de una 3. En: «Se la lavó», lo subrayado es respectivamente___.
a) PVO – OD
6. En: «Él nos mostraba afición», señale lo verdadero. b) OI – OI
a) Él: OD – Nos: OD c) OD – OD
b) Nos: OI – Afición: OD d) OI – OD
c) Nos: OD – Afición: OI
e) PVO – OI
d) Él: OD – Nos: OI
e) Afición: sujeto – Nos: OI
4. «El caminar es saludable». Lo subrayado es ___.
7. Reconozca el OD en: Pensó que la muerte venía a coronar a) OI
su vejez. b) PVO
a) Que la muerte. c) atributo
b) Coronar su vejez.
d) OD
c) Que la muerte venía a coronar su vejez.
d) Su vejez. e) AG
e) La muerte venía a coronar.
5. Sobre la expresión ¡Cántamelo!, señala lo verdadero o falso.
8. En: «El ruiseñor cantaba a las flores su dulce victoria», el OI I. Tiene sujeto tácito.
es__________. II. El OD es lo
a) su dulce victoria
III. El IO es me
b) flores
c) a las flores IV. No es oración.
d) su dulce a) VVVV b) VVVF c) FVVF
e) el ruiseñor d) VFVF e) FVVV

5º AÑO DE SECUNDARIA
Mejores Personas, Mejores Familias 197
Colegio

Mejores Personas, Mejores Familias LINGÜÍSTICA

6. ¿Cuál de las oraciones no tiene OD? 2. En: «Vinieron los médicos y le dieron a su majestad un prepa-
a) La mujer lavó la ropa en la tina. rado para que vomitase». Las palabras subrayadas______.
b) Ana compró, en la esquina, fruta. a) ambas son OI.
c) Jonás limpió el carro para salir. b) son modificadores diferentes.
d) No acabaremos ni en un año. c) son parte del sujeto.
e) Repartiremos las entradas mañana. d) uno pertenece al sujeto y el otro al predicado.
e) uno es objeto y el otro complemento.
7. ¿Qué oración tiene predicado nominal?
3. «Cuando tú te hayas ido, me envolverán las sombras».
a) Ingresé en la universidad.
La estructura de la oración es ____.
b) La noche ha estado tranquila.
a) C. Circ. – OI – NP – MD – NS
c) Te amo intensamente.
b) C. Circ. – NS – NP – MD
d) Regresa pronto, por favor.
c) C. Circ. – NS – NP – OI – OD
e) ¡No me dejes morir!
d) C. Circ. – OI – NP – OD
e) C. Circ. – NP – S
8. En: «Maritza bailó el tango como ninguna», no aparece
____. 4. «Nos vieron en el parque».
a) CC b) OI c) NP «Nos dieron la buena noticia».
d) OD e) NS Las palabras subrayadas de las oraciones anteriores ____.
a) cumplen la misma función sintáctica.
9. En: «Los encontraron al amanecer». Es falso que ____. b) la primera es OI; la segunda, OD.
a) el OD es un pronombre. c) la primera es OD; la segunda, OI.
b) aparece un C.C de tiempo. d) la primera es sujeto; la segunda, OD.
c) el verbo es el núcleo. e) ambas son pronombres posesivos.
d) el sujeto es tácito.
e) aparece el OI en su forma pronominal. 5. Señala la alternativa que presenta objeto indirecto.
UNMSM 2008 – II
a) Allí se vende caramelos grandes.
10. ¿En cuál (es) de las siguientes oraciones me funciona como b) Tú se lo regalaste ayer mismo.
OD? c) En el fútbol, se batalla hasta final.
a) Me dijo que no me ama. d) Margarita se arrepentirá de todo.
b) ¿Me lo traerás el viernes? e) Luisa no se pintó ayer en la noche.
c) Mi novio me recogerá temprano.
d) Cántame una balada de Reik.
e) Me visitó el viernes.

1. Cuántos objetos directos hay en: Me prometió que esta vez si


iba a cumplir sus sueños más preciados.
a) Uno b) Cuatro
c) Dos d) Cero
e) Tres

198 Mejores Personas, Mejores Familias


5º AÑO DE SECUNDARIA
Colegio

LINGÜÍSTICA  Mejores Personas, Mejores Familias

M A
TE

23   EL PREDICADO II

I. COMPLEMENTO PREDICATIVO (PVO) «Cruda» modifica al verbo «comen» y al


Es el modificador bivalente en el predicado. objeto directo «carne»
Existen dos tipos de predicativo:

1. Predicativo subjetivo (ps) II. COMPLEMENTO ATRIBUTO (AT.)


Se presenta cuando modifica al verbo y al núcleo del Es un modificador bivalente que presenta la mayor carga
sujeto. significativa. Aparece solo con verbos copulativos; por eso,
●● Los atletas nacionales llegaron (agotados). es obligatorio.
SUJETO PREDICADO Ejemplo:
el predicativo es no obligatorio.
MD NS MD NP
«Agotados» modifica al verbo «llegaron» y YY Los caballeros medievales eran (valientes).
al núcleo del sujeto «atletas» SUJETO PREDICADO
El atributo es obligatorio

2. Predicativo objetivo (po) «Valientes» modifica al verbo «eran» y al
Se presenta cuando modifica al verbo y al objeto di- núcleo del sujeto «caballeros»
recto.
●● Los ermitaños comen (cruda) (la carne).
SUJETO PREDICADO

III. COMPLEMENTO CIRCUNSTANCIAL (CC)


Circunstancias
YY Las palabras que expresan circunstancias del verbo se llaman complementos circunstanciales (CC).

Ejemplo:
Mi abuelo camina despacio los domingos en el parque dos kilómetros con sus amigos.

YY Los circunstanciales más habituales en español son los siguientes:

Tipo Abreviatura Pregunta para reconocerlo Ejemplo


CC de lugar CCL ¿Dónde? Ha estacionado la moto allí.
CC de tiempo CCT ¿Cuándo? Iremos a la piscina el lunes.
CC de modo CCM ¿Cómo? No me gusta que contestes así.
CC de causa CCC ¿Por qué? No trabajamos por una huelga.
CC de finalidad CCF ¿Para qué? Compró quesos para cenar.
CC de compañía CCCo ¿Con quién? Salió a comer con sus amigos.
CC de instrumento CCI ¿Con qué? Cortó el jamón con un cuchillo de sierra.
CC de material CCM ¿Con qué material? Hizo una escultura con hierro fundido.
CC de cantidad CCCa ¿Cuánto? Aquel año trabajamos mucho.

5º AÑO DE SECUNDARIA
Mejores Personas, Mejores Familias 199
Colegio

Mejores Personas, Mejores Familias LINGÜÍSTICA

IV. COMPLEMENTO AGENTE (C. AG.) 5. ¿Qué clase de complemento es el elemento subrayado de la
Aparece en la oración pasiva. siguiente oración: Por la madre fueron protegidos aquellos
YY Su amo abandonó a ese perro. (Voz activa) niños?
YY Ese perro fue abandonado por su amo. (Voz pasiva) a) Objeto directo
b) Objeto indirecto
1. Voz pasiva c) Complemento circunstancial
Está formada por el verbo SER + PARTICIPIO. d) Complemento agente
●● El paciente ha sido operado por el doctor. e) Complemento predicativo
Voz pasiva
6. Indica la alternativa que presente complemento atributo.
2. Sujeto paciente (S. Pac.) a) Nosotros ya estamos en casa.
Es el único sujeto que no realiza la acción del verbo, b) Iba a casa después de la cena.
sino que la recibe. c) Tú eres bastante comprensivo.
d) Trajeron nuevos ayudantes.
3. Complemento agente (C. Ag.)
e) Iremos después.
Es el sintagma preposicional que realiza la acción en
las construcciones pasivas en la lengua española. 7. ¿Qué clase de complemento es el elemento subrayado
Por lo general va encabezado por la preposición por o de la siguiente oración: «La cifra parece exagerada»?
de, o una locución prepositiva equivalente: a) Objeto directo
b) Objeto indirecto
●● E l paciente ha sido operado por el doctor. c) Complemento circunstancial
Det N N C.Ag d) Complemento agente
S. Pac PV e) Complemento atributo

8. ¿Cuál de las siguientes oraciones presenta complemento


atributo?
a) Ven a casa temprano.
b) Los estudiantes están muy cansados.
c) Ellos suponen especiales a los chicos.
d) Trabaja muy bien.
1. Lo condenaron por hablar mal. La estructura es _____. e) Dejaremos abierta la casa.
a) OD, NP, C b) MD, NP, C c) OD, NP, OI
d) OI, NP, C e) MI, NP, C 9. ¿Cuál de las siguientes oraciones NO presenta atributo?
a) La casa era antigua.
2. Para que haya complemento agente, es necesario _______. b) Bella será tu vida.
a) que la oración esté en voz activa c) La felicidad no es completa.
b) que la oración esté en voz pasiva d) Ahora tenemos una nueva oportunidad.
c) que el pasado presente objeto directo e) El cuerpo yace inerte.
d) que el predicado presente objeto indirecto
e) que la oración sea compuesta 10. ¿Cuál de las siguientes alternativas es correcta con relación
al complemento predicativo?
a) Los adjetivos no pueden funcionar como complemento
3. ¿Cuál de las siguientes alternativas presenta complemento
predicativo.
agente en el predicado? b) Para que haya complemento predicativo siempre habrá
a) Por su causa, hemos perdido parte del capital. verbo no copulativo.
b) Yo soy capaz de todo por ti. c) Los complementos predicativos necesitan indispensa-
c) Si fuera por ella, las cosas cambiarían. blemente de voz pasiva.
d) Ella es amada por ser buena. d) No puede haber complemento predicativo en oraciones
e) Será devuelta la mercadería. bimembres.
e) Los complementos predicativos son monovalentes.
4. ¿Qué clase de complemento es el elemento subrayado de la
siguiente oración: «Ella llegó a casa temprano»?
a) Objeto directo
b) Objeto indirecto
c) Complemento circunstancial
d) Complemento agente 1. Lo sospeché desde un principio, pero no lo revelé porque tenía
e) Complemento predicativo miedo. ¿Cuántos circunstanciales hay en el enunciado ante-
rior?
a) Uno b) Tres c) Cero
d) Dos e) Cuatro

200 Mejores Personas, Mejores Familias


5º AÑO DE SECUNDARIA
Colegio

LINGÜÍSTICA  Mejores Personas, Mejores Familias

2. El presidente acordó con sus asesores implementar el regla- 10. ¿Cuál de las siguientes es una oración con complemento
mento. predicativo subjetivo?
El CC es de__________________. a) El pantalón quedó abandonado sobre la cama.
a) modo b) compañía c) tiempo b) El crimen fue descubierto por la mucama.
d) finalidad e) afirmación c) Ella está en su nueva casa.
d) El guardia está trabajando en la comisaría.
3. A las dos de la mañana, el ruido de dos balazos hizo desper- e) Presenta dificultades.
tar a los habitantes de un edificio de veinte pisos. Al cambiar
la oración a voz pasiva, ¿qué elemento se transformará en
sujeto?
a) El ruido de dos balazos
b) Las dos de la mañana
c) Los habitantes de un edificio de veinte pisos 1. Oración que presenta la siguiente estructura :
d) Un edificio de veinte pisos MD - NS - OI - OD - NP – CIRC.
e) Dos balazos a) Ese joven sabe la verdad.
b) Tus hermanos me lo contaron durante el almuerzo.
4. El método universitario se interesa por la constante y progresiva c) Deja ese objeto sobre la mesa.
ascensión del alumno a una meta. El predicado presenta los si- d) Se le fue la última oportunidad.
guientes complementos______. e) Eso amigos no se lo esperaban.
a) 1 circunstancial
b) 1OI y 1 circunstancial 2. Una oración presenta complemento circunstancial de
c) 1OI y 2 circunstancial modo.
d) 1OI a) Me recibió amablemente.
e) 1OD y 1OI
b) Me recibió todo frío y altanero.
5. En este país, hoy en día, hay necesidad de mucho sacrificio. c) ¿Por cuánto tiempo te quedarás?
La estructura del predicado es ______. d) La reunión estuvo mal organizada.
a) C, C, NP, OD b) C, NP, OD c) C, NP, C, OD e) Si deseas, puedes hacerlo.
d) C, C, NP e) C, NP
3. No pertenece a la estructura del predicado.
6. Entrégamelo pronto o devuélvelo. Existen _____. a) El atributo
a) 2NP, 2OI, IC, 1OD b) El complemento agente
b) 2NP, 2OD, 1C, 1OI c) El predicativo
c) 2NP, 1OD, 1OI, IC d) El circunstancial
d) 2NP, 1C e) El modificador indirecto
e) 2NP, IC, 2OD
4. Solo una opción carece de circunstancial.
7. Ana considera divertida la obra de García Márquez.
a) Tomaremos una decisión inmediata.
La palabra subrayada es________.
a) predicativo b) Perdieron el partido fácilmente.
b) complemento circunstancial. c) Recogeré los resultados hoy.
c) sujeto d) Rápidamente ingresaron al vestíbulo.
d) objeto directo e) Los preparó con dedicación.
e) objeto indirecto
5. Identifica la oración que contiene un complemento predi-
8. El C. C. de la oración: Me veo en el muelle, siempre esperan- cativo.
do el regreso de mi padre es_________. a) María lo observa atentamente.
a) tácito b) en el muelle b) Marta contó historias agradables.
c) me d) el regreso de mi padre c) Los chicos compraron caramelos.
e) esperando el regreso d) Fernando leyó la carta con lentitud.
e) Mis hermanas almorzaron contentas.
9. Señala la oración que presenta atributo.
a) Luisa está feliz en Roma.
b) En ese cajón está mi testamento.
c) Ella está en su nueva casa.
d) Necesitarás un buen abogado para que te defienda.
e) Es de madera.

5º AÑO DE SECUNDARIA
Mejores Personas, Mejores Familias 201
Colegio

Mejores Personas, Mejores Familias LINGÜÍSTICA

M A
TE

24   ORACIÓN COMPUESTA

I. ORACIÓN SIMPLE Y ORACIÓN COMPUESTA


Son aquellas que están unidas por signos de puntuación. La relación entre ellas puede ser de total independencia o una se
puede subordinar en sentido a la otra, es el hablante quien restituye el significado del signo de puntuación (coma, punto y
coma o los dos puntos). Dado este margen para la subjetividad, en sintaxis se prefiere analizar las dos frases como indepen-
dientes, marcando simplemente el nexo gráfico que hace que formen una oración compuesta y no dos oraciones separadas
(como cuando hay un punto y seguido). A veces la elección entre un signo u otro responde a cuestiones de estilo y no semán-
ticas.
Como sabemos, las oraciones tienen un verbo en forma personal que funciona como núcleo del predicado. Ahora bien, hay
oraciones que tiene un solo verbo y otras, en cambio, tienen más de un verbo. Este hecho nos permite diferenciar entre ora-
ciones simples y compuestas.

a) Son oraciones simples aquellas que tienen un solo verbo (ya sea este una forma verbal simple o compuesta).

●● Enrique comió en abundancia.



Forma simple

●● Paola ha festejado su santo ayer.

Forma compuesta

b) Son oraciones compuestas aquellas que tienen dos o más verbos, ya sean estos formas simples o compuestas.

●● Alejandra regresó de la playa temprano, pero recién se ha bañado.

Forma simple Forma compuesta

II. ELEMENTOS DE LA ORACIÓN COMPUESTA: LAS PROPOSICIONES


En la oración compuesta, cada verbo puede admitir un sujeto y unos modificadores.

Oración compuesta


SUJETO
___________ PREDICADO
_______________ SUJETO PREDICADO
_____ _______________
YY El maquinista tomó los mandos y el tren se deslizó por la vía
N N S OD N N CIRC.

Cada uno de estos grupos de palabras es una proposición. Por lo tanto, llamamos proposición a cada grupo de palabras con
sentido completo, pero que depende sintácticamente de una estructura mayor: la oración compuesta.

III. CLASES DE ORACIONES COMPUESTAS.


Según la forma en que las proposiciones se relacionan entre sí, se pueden distinguir dos clases de oraciones compuestas: por
coordinación o por subordinación. Sin embargo, para algunos gramáticos la clasificación sería por yuxtaposición, por coor-
dinación o por subordinación. –Torrego,(2002). Gramática didáctica del español–.
Oraciones compuestas por yuxtaposición
Observa las oraciones 1 y 2. Ambas son oraciones compuestas, pues cada una contiene dos verbos. Sin embargo, existen
diferencias entre ellas. Lee con atención:
1. Pablo entrenó durante cinco meses y ganó el campeonato.

ENLACE COORDINANTE

202 Mejores Personas, Mejores Familias


5º AÑO DE SECUNDARIA
Colegio

LINGÜÍSTICA  Mejores Personas, Mejores Familias

2. Pablo entrenó durante cinco meses ; ganó el campeonato.



SIGNO DE PUNTUACIÓN

●● L
a oración 1 tiene un enlace coordinante explícito; en cambio, en la oración 2, la relación de coordinación está sobreentendida. Y
el enlace se sustituye por un signo de puntuación (en este caso, el punto y coma).

Observa y compara ahora estas otras oraciones compuestas:


3. Cierra la ventana porque entra mucho aire.

ENLACE SUBORDINANTE

4. Cierra la ventana: entra mucho aire.



SIGNO DE PUNTUACIÓN

●● E n estas oraciones sucede algo muy parecido. En la oración 3 hay un enlace subordinante presente. En cambio, en la
oración 4 este enlace está sobreentendido y su ausencia se marca con otro signo de puntuación.
●● Las oraciones 2 y 4 son compuestas por yuxtaposición: no tienen un enlace presente o explícito. En el lenguaje ha-
blado, el oyente descubre este enlace por el contexto. En el lenguaje escrito, además el enlace suele sustituirse por un
signo de puntuación (coma, punto y coma, dos puntos). Observa este otro ejemplo:

Proposición 1 proposición 2
___________________________ ______________________
Por las mañanas mi abuelo camina , por las tardes lee el periódico.

Oraciones Compuestas por Coordinación: b) Las proposiciones coordinadas adversativas indi-


Las oraciones compuestas por coordinación son aquellas can contraposición. Llevan como enlace las con-
que se forman cuando dos o más proposiciones se unen junciones pero, mas, sino o sino que.
por medio de una conjunción coordinante (y, o, mas, pero, Ejemplo: Es muy rico, pero no gasta nada.
etc).
c) La proposiciones coordinadas disyuntivas indi-
Por ejemplo: can opción entre dos o más posibilidades. Llevan
como enlace la conjunción o (u).
ENLACE COORDINANTE
Ejemplo: ¿Te quedas con ella o vienes conmigo?
PROPOSICIÓN 1 PROPOSICIÓN 2

__________________________________ ___________________________________
d) Las proposiciones coordinadas conjuntivas pue-
Todos salieron corriendo pero el ómnibus los dejó atrás. den ser de tres tipos: ilativas, distributivas o expli-
SUJETO PREDICADO SUJETO PREDICADO cativas.

Oración Compuesta por Coordinación Conjuntiva Ila-


YY A veces, las proposiciones de una oración compues- tiva:
ta por coordinación comparten el mismo sujeto. Por Se da cuando la segunda proposición expresa consecuen-
ejemplo: cias de lo dicho en la primera. Llevan como enlace por lo
tanto, luego, por consiguiente, por ello, entonces.
ENLACE COORDINANTE
Ejemplo:
PROPOSICIÓN 1 PROPOSICIÓN 2

________________________________________ _________________________
P P
_____________ ________________
Kevin entró en la computadora y se llevó los archivos (Es mi padrino), por lo tanto (respeto sus consejos).
SUJETO PREDICADO (S. TÁCITO) (PREDICADO)
Prop. 1 Prop. 2
_________________________________________
YY Según la clase de conjunción que funcione como enla- Orac. Comp. Coord. Conj. Ilativa.
ce, las proposiciones coordinadas pueden ser copula- YY El censo ya comenzó, por lo consiguiente, no pode-
tivas, adversativas, disyuntivas/o conjuntivas: mos salir de casa.
a) Las proposiciones coordinadas copulativas indi- YY Enrique ya cobró, por ello, me hizo un préstamo.
can suma o acumulación. Llevan como enlace las
conjunciones y (e), ni.
Ejemplo: No fui ni llamé tampoco.

5º AÑO DE SECUNDARIA
Mejores Personas, Mejores Familias 203
Colegio

Mejores Personas, Mejores Familias LINGÜÍSTICA

Oración Compuesta por Coordinación Conjuntiva Dis-


tributiva: 5. Oración compuesta:
Cuando el enlace se realiza por palabras correlativas o por a) Quiérete más.
repetición de una palabra bien – bien, ya – ya, unos – otros, b) Va a estar mejor sin mí.
algunos – otros, ora – ora. No excluyen ni contradicen. c) Ven en cuanto puedas.
Ejemplos: d) Con mentiras, no vas a ganar.
YY Unos cocinan, otros duermen. e) Toma mi mano sincera.
YY Ya escuchaba la clase, ya toma apuntes.
YY Ora toca la quena, ora canta huaynos. 6. Oración compuesta coordinada copulativa:
a) Yo trabajo aquí porque lo necesito.
Oración Compuesta por Coordinación Conjuntiva Ex- b) A pesar de todo, sigo esperando por una oportunidad.
plicativa: c) Hicimos lo necesario; pero no fue suficiente.
Es aquella en la que la segunda proposición es una explica- d) Te advertimos, luego no debes quejarte.
ción de la primera. Llevan con nexo es decir, así pues, o sea, e) No lo hice yo ni tampoco lo hiciste tú.
esto es, etc.
Ejemplos: 7. Oración compuesta coordinada disyuntiva:
a) Nos amanecimos con la tarea, pero no logramos termi-
S P P narla.
_________ _____________ __________ b) Estudia, escucha tus clases, haz tu tarea: eso es todo.
Las lenguas también mueren, es decir (desparecen) c) Ya habían acabado la tarea; no obstante no pudieron sa-
Prop. 1 prop. 2 lir.
_________________________________________ d) Vencemos ahora o perderemos todo de una vez.
e) Siento que todo está mejorando.
Orac. Comp. Coord.. explicativa
8. Llamamos compuestas coordinadas a ______.
YY T
erminó su contrato, o sea está sin empleo. a) dos o más proposiciones unidas con conjunción subor-
YY La empresa te aceptó, esto es, ya tienes trabajo. dinante.
b) dos o más enunciados unidos con pronombre relativo.
c) dos o más enunciados unidos con adverbio subordi-
nante.
d) dos o más enunciados unidos con conjunción coordi-
nante.
1. «Hubo mejores ofertas». La oración es _______. e) más de una es correcta.
a) dubitativa b) bimembre c) compuesta
d) simple e) personal 9. Oración con proposición coordinada:
a) La clase que más cansa es la de lengua.
2. «Mis tíos, muy bien». La oración anterior ____. b) Quien quiera ingresar debe hacer lo que debe hacer.
a) es unimembre d) no tiene predicado c) Para mentir y comer pescado hay que tener mucho cui-
b) es bimembre e) no tiene sujeto dado.
c) es simple d) Ella viene y yo me voy.
e) Dame de beber.
3. Oración compuesta yuxtapuesta:
10. Marca la alternativa en la que hay proposición coordinada
a) Los peruanos pelearon por su libertad; pero América
conjuntiva explicativa.
nos contemplaba. a) Está contento porque obtuvo una beca.
b) Bailamos toda la noche, cantamos hasta el amanecer, b) Él escribió poesías, pero no las ha publicado.
estábamos felices. c) Él no se da por vencido, o sea, es perseverante.
c) La tarea, aun cuando nos fatigó, nos agradó bastante. d) La gente va y viene asustada por las calles.
d) La poesía es locura y la narración es la verdad de las e) Lucia viajará en tren y Tania irá en avión.
mentiras.
e) Ninguna de las anteriores.

4. Oración simple:
a) Yo soy el que te dio el primer beso en una noche de
verano. 1. Presenta oración simple.
b) Salió temprano de su casa. a) Martín y Pedro se han dormido durante el viaje
c) Todo lo que tengo es tuyo. b) Mi papá me lleva al colegio y me regreso en micro.
d) Sé cuánto te debo. c) Carlos comprendió que se había equivocado.
e) Me iré de tu lado cuando sea necesario. d) El chofer hizo una maniobra, pero no logró esquivar al
peatón.
e) Tengo que visitar tres familias y nadie quiere ir conmigo.

204 Mejores Personas, Mejores Familias


5º AÑO DE SECUNDARIA
Colegio

LINGÜÍSTICA  Mejores Personas, Mejores Familias

2. Presenta tres proposiciones. 10. En: «Llamó por teléfono a su casa, pero nadie respondió la
a) No conozco Tumbes, pero he estado en Piura. llamada», se aprecia una oración compuesta __________.
b) He leído un rato, he dado una vuelta y ahora me voy a a) yuxtapuesta b) adversativa c) disyuntiva
dormir. d) adversativa e) condicional
c) No has tendido la cama ni te has duchado.
d) Ni llamó ni vino.
e) Llegó, vio, venció.

3. Presenta oración simple. 1. Presenta oración compuesta yuxtapuesta.


a) Sara me dijo que la esperara aquí. a) Ven pronto por favor.
b) Todos empezaron a correr cuando oyeron los ladridos. b) No vino a la reunión.
c) Quien llegue primero recibirá un premio. c) Sopa le dieron al niño y no se la quiso tomar.
d) Ya sabremos la verdadera historia. d) Hay una época del año feliz.
e) ¿Me puedes decir dónde queda tu colegio? e) Ellos cantan, nosotros escuchamos: está bien.

4. Señala la oración en la que hay oración bimembre com- 2. Los enunciados: «Ella tiene dinero, por lo tanto, viajará» y
puesta. «Juan canta, pero no baila» constituyen, respectivamente,
a) Nos alegra tu gran triunfo personal. oraciones compuestas por coordinación conjuntiva_____.
b) Creo que llegará hoy a nuestro país. a) Adversativa y disyuntiva.
c) La realidad es variable y compleja. b) Ilativa y adversativa.
d) He recibido el mensaje de mi tío. c) Ilativa y explicativa.
e) Los rayos de luz hirieron mis ojos. d) Ilativa y disyuntiva.
e) Copulativa y adversativa.
5. Alternativa donde se presenta oración bimembre simple.
a) Llegó muy cansado.
b) Espero que me visites. 3. Marca la oración compuesta por coordinación conjuntiva
c) Le di lo que pidió. adversativa.
d) Dice que te quiere. a) Regué las plantas, por lo tanto, florecerán.
e) Usa el reloj que te di. b) Apresúrate o no conseguirás los periódicos.
c) Ya selecciona estampillas, ya atiende al público.
6. Marca la oración compuesta por coordinación conjuntiva d) Gané un concurso, así que estoy muy feliz.
disyuntiva. e) Ese anciano no usa anteojos, mas emplea bastón.
a) Creo que ingresarás a este instituto.
b) Deben ayudarlos o perderán su casa. 4. Llamamos oraciones compuestas yuxtapuestas a ____.
c) José escribe poemas y Ana estudia. a) enunciados unidos por medio de un punto y coma.
d) Busqué la medalla, mas no la encontré. b) enunciados unidos por medios de dos puntos.
e) Si te retiras, él también se irá. c) enunciados unidos por medio de coma.
d) enunciados unidos por medio de punto seguido.
7. Señala la oración compuesta por coordinación conjuntiva e) más de una es correcta.
copulativa.
a) José María y Marcial se fueron a Chile. 5. Señala cuál de las siguientes oraciones es una oración
b) Todos lo felicitaron: ganó la Tinka. compuesta yuxtapuesta.
c) No te desesperes ni te aflijas más. a) Vargas jugara de volante, Guerrero de delantero.
d) Hacemos más cuando estamos unidos. b) El tren eléctrico tendrá cinco tipos de tarjetas: A, B, C,
e) Pronuncia bien o no te entenderé. D y E.
c) El tren Macho y el metro cruzan el río Rímac.
8. En: «Miró a los que llegaban, pero no reconoció a ninguno», d) El ejército llevo a menores de la comunidad nativa
se aprecia una oración compuesta ______. Waya.
a) yuxtapuesta b) adversativa c) disyuntiva e) El presidente que fue elegido en junio es él.
d) adversativa e) condicional

9. En: «Ana llamó a su amiga y la invitó al cine», se aprecia


una oración compuesta ______________.
a) yuxtapuesta b) adversativa c) disyuntiva
d) adversativa e) copulativa

5º AÑO DE SECUNDARIA
Mejores Personas, Mejores Familias 205
Colegio

Mejores Personas, Mejores Familias LINGÜÍSTICA

M A
TE

25   ORACIÓN SUBORDINADA SUSTANTIVA

Las Oraciones Subordinadas Sustantivas son aquellas oraciones que desempeñan funciones sintácticas propias del sustantivo.

1. Según la función sintáctica que desempeñan, se clasifican según:


YY S. S. de Sujeto → Me gusta que vengas pronto.
YY S. S. de C. Directo → Le pregunté si vendría. Me dijo que no vendría.
YY S. S. de C. Indirecto → Dieron los permisos a los que los solicitaron.
YY S. S. de C. del Nombre → Tengo la certeza de que volverá.
YY S. S. de C. del Adjetivo → Parecía feliz de que la hubiese recuperado.
YY S. S. de C. del Adverbio → Ella estaba muy lejos de los que amaba.
YY S. S. de C. Circunstancial → Iré sin que me lo pidas.
YY S. S. de C. Agente → Los cuadros son expuestos por sus creadores.
YY S. S. de Suplemento o Régimen → Habló de que era mejor no venir.
YY S. S. de Atributo → El hecho es que no lo hizo.

2. Las Subordinadas Sustantivas se unen a la Oración Principal mediante:


YY nexos → que, el que, el hecho de que
YY pronombres interrogativos → qué, cuál, quién
YY adverbios interrogativos → cuánto, cómo, cuándo, dónde etc.

3. Ejemplos de Oraciones Subordinadas Sustantivas:


YY Me gusta que siempre estés contento → funciona como sujeto

Me gusta que siempre estés contento (Tú)


N N
C.I. N.V. Nexo C.C. V.C. N
Tiempo Atributo

S.V. (Predicado Nominal) S.N. (Suj.)



S.V. (Pred. Verbal) Oración Subordinada Sustantiva (Sujeto)

YY Fumar mucho causa cáncer → funciona como sujeto

Fumar mucho causa cáncer

N.
N. Det. N.V.
C.D.

Or. Sub. Sustantiva (Sujeto) S.V. (Pred. Verbal)

YY Quiero que vengas → funciona como C. Directo


(Yo) Quiero que vengas (Tú)

N.V. N.
N. N.V. Nexo
S.V. (P.V.) S.N. (Suj.)

Or. Subordinada Sustantiva de C.D.

S.N. (Suj.) S.V. (Predicado Verbal)


206 Mejores Personas, Mejores Familias


5º AÑO DE SECUNDARIA
Colegio

LINGÜÍSTICA  Mejores Personas, Mejores Familias

YY No sé qué me dices → funciona como C. Directo

(Yo) No sé qué me dices (Tú)

Nexo C.I. N.V. N


N. Neg. N.V.
S.V. (Pred. Verbal) S.N. (Suj.)

Or. Subordinada Sustantiva de C.D.

S.N. (Suj.) S.V. (Predicado Verbal)



YY Me pregunto cuánto dinero tendrá → funciona como C. Directo

(Yo) Me pregunto cuánto dinero tendrá (Él)


N.
N.V. N
N. C.I. N.V. Nexo C.D.
S.V. (Pred. Verbal) S.N. (Suj.)
Or. Subordinada Sustantiva de C.D.
S.N. (Suj.) S.V. (Predicado Verbal)

YY No tengo la seguridad de que sea inocente → funciona como C. Nombre



(Yo) No tengo la seguridad de que sea inocente (Él)
N
V.C. N
N. Neg. N.V. Det. N. Prep Nexo Atributo
S.V. S.N.
(Pred. Nominal) (Suj.)
Or. Subordinada Sustantiva de C.N.
C.D.
S.N. S.V. (Predicado Verbal)
(Suj.)

YY Ella es quien lleva la voz cantante → funciona como atributo

Ella es quien lleva la voz cantante


N.
Det. N.
N. V.C. N. N.V. Adyacente
C.D.
S.N. (Suj.) S.V. (Predicado Verbal)
Oración Subordinada Sustantiva de Atributo
S.N. (Suj.) S.V. (Predicado Nominal)

5º AÑO DE SECUNDARIA
Mejores Personas, Mejores Familias 207
Colegio

Mejores Personas, Mejores Familias LINGÜÍSTICA

7. De la oración: «Solo sé que nada sé» se puede afirmar ___.


a) que la oración es compuesta subordinada
b) que la oración es compuesta coordinada
c) que la oración es compuesta yuxtapuesta
1. En la oración: «El café está que arde», lo subrayado cumple d) que la oración es simple
la función de_______. e) que la oración presenta coordinación y subordinación
a) OD b) OI c) agente
d) predicativo e) atributo 8. Funciona como nexo de proposiciones subordinadas:
a) Conjunción coordinante adversativa.
2. Oración compuesta subordinada sustantiva: b) Conjunción coordinante copulativa.
a) Lo que yo tengo es muy serio. c) Preposición.
b) Las cosas pasaron inesperadamente. d) Interjección.
c) Pedro vendió el auto que tanto le gustaba. e) Conjunciones subordinantes.
d) Donde sea, yo te encontraré.
e) Las cosas que dices no me agradan. 9. «Un alumno de 5.° preguntó si Carlos tenía enamorada». La
subordinada cumple función de_______.
3. La oración compuesta no es subordinada sustantiva: a) CIRC b) OD c) AP
a) Cuando la proposición subordinada cumple función d) OI e) PVO
modificador directo.
b) Cuando la proposición subordinada cumple función de
aposición.
c) Cuando la proposición subordinada cumple función
objeto indirecto.
d) Cuando la proposición subordinada cumple función de
objeto directo. 1. No es una función de la subordinada sustantiva:
e) Cuando la proposición subordinada cumple función de a) Núcleo del sujeto
modificador indirecto. b) Objeto directo
c) Predicativo
4. En la oración; «Vende lo que no sirve», encontramos d) Núcleo del agente
________. e) Modificador directo
a) Proposición subordinada sustantiva.
b) Proposición subordinada adjetiva. 2. Señale la oración en la que la subordinada sustantiva fun-
c) Proposición subordinada adverbial. ciona como núcleo del sujeto.
d) Proposición coordinada. a) Pidió que lo lleven cargado.
e) Proposición yuxtapuesta. b) Ella está que no puede con los celos.
c) Regaló víveres a quienes se lo pedían.
d) Quien calla otorga.
5. Escribe dos ejemplos de oración subordinada sustantiva.
e) Observó que estábamos equivocados.
• __________________________________________
3. ¿Qué función cumple la proposición subordinada subra-
__________________________________________
yada?
Dios da barbas a quien no tiene quijada.
• __________________________________________ a) N. S b) OD c) M. I
__________________________________________ d) OI e) MD

6. Oración con más de una proposición subordinada sustan- 4. ¿Cuál de las siguientes alternativas presenta proposición
tiva: subordinada sustantiva?
a) La clase que más cansa es la de lengua. a) Cuando llegó, nosotros ya lo habíamos solucionado.
b) Quien quiera ingresar debe hacer lo que deba hacer. b) Como si nada, me has vuelto loco.
c) Para mentir y comer pescado hay que tener mucho cui- c) Nací para triunfar en la vida.
dado. d) Eres apreciado por los que te rodean.
d) No por mucho madrugar, amanece más temprano. e) Ya dejamos las costumbres que teníamos.
e) En cuanto puedas, ven a casa.

208 Mejores Personas, Mejores Familias


5º AÑO DE SECUNDARIA
Colegio

LINGÜÍSTICA  Mejores Personas, Mejores Familias

5. ¿Cuál de las siguientes alternativas presenta oración no su-


bordinada?
a) Anda y ven.
b) Eres lo que necesito para este trabajo.
c) Quien mucho abarca poco aprieta. 1. Señale la oración que carezca de subordinación sustantiva.
d) Todo se hará como se nos ha indicado. a) Le gustaron las medias que le regalaste.
e) Porque te necesitamos, te lo pedimos. b) Quien termine último lavará los platos.
c) Le pidió que vaya temprano.
6. Señale la opción que presenta oración compuesta por
d) Ella está que salta de la alegría.
subordinación sustantiva.
e) Ese conductor transmitía alegría a quienes sufrían por
a) Ellos han de señalar “el desafío”
la pobreza.
b) Unos estudian, otros pierden el tiempo.
c) Ella pensó que él regresaría.
2. Señale la oración en la que la proposición subordinada
d) Inés baila samba; Ana, vals.
sustantiva cumpla la función de objeto directo.
e) Ellos pagarán el SOAT.
a) Quien con niños se acuesta mojado amanece.
b) La razón de que tengamos problemas no es económica.
7. La oración compuesta: «Nuestro deseo es que ella esté sana
y salva» contiene una proposición subordinada en función c) Ella pasaba la voz a quienes querían ir a la fiesta.
de__________. d) Los ladrones serán delatados por quienes fueron sus
a) sujeto b) O. Directo c) atributo cómplices.
d) O. Indirecto e) agente e) Detrás de los muebles, observaba que los regalos eran
colocados por un hombre de barbas y traje rojo.
8. En el enunciado: «Dime si viajarás mañana», la proposición
subordinada sustantiva está en función de ___________. 3. ¿Qué clase de oración es la siguiente: «No sé qué piensas tú
a) O. Directo b) Atributo c) O. Indirecto que habría que hacer»?
d) Circunstancial e) Agente a) oración simple
b) subordinada sustantiva
9. En la oración: «Fue recibido por quienes los estimaban», la c) subordinada adverbial
subordinada sustantiva funciona como ____. d) subordinada adjetiva
a) O. Directo b) agente c) aposición e) yuxtapuesta
d) sujeto e) atributo

4. Señale la oración que presente una proposición subordina-
10. En la oración: «Se lo prometió a quien nunca volvería a ver»,
da sustantiva.
la proposición subordinada cumple función de_______.
a) complemento agente a) Cuando tú no estás, me pongo triste.
b) atributo b) Tú estás donde te conviene.
c) objeto indirecto c) La canción que escuchaste resultó ganadora.
d) complemento de nombre d) Es verdad que nadie lo conoce.
e) sujeto e) Mientras lo piensas, me tomo una copa.

5. Marca la alternativa que muestra una oración subordinada


sustantiva.
a) El delincuente saltó por donde pudo.
b) El amigo, que fue condenado, es de Cuzco.
c) Te reías bien y me haces reír.
d) Pensé que Magnolia me hablaría de su dolor.
e) Puesto que no has sacado buena nota, no viajarás.

5º AÑO DE SECUNDARIA
Mejores Personas, Mejores Familias 209
Colegio

Mejores Personas, Mejores Familias LINGÜÍSTICA

M A
TE

26   ORACIÓN SUBORDINADA ADJETIVA

La Oración de Relativo o Subordinada Adjetiva es aquella que desempeña la función propia de un Adjetivo con respecto a una
palabra de la oración principal:
ZZ El alumno que estudia aprobará → funciona como adjetivo: alumno estudioso
ZZ Mira la estrella que brilla → funciona como adjetivo: estrella brillante

Utiliza los siguientes nexos (la oración de relativo en negrita):


ZZ que → Necesito un camarero que sepa francés.
ZZ cual → Acaban de pintar esa pared sobre la cual te apoyas.
ZZ cuyo(a) → Esa es la casa cuya fachada se desplomó ayer.
ZZ quien → La chica en quien confío me mintió.

Nota: el sustantivo al que se refiere la or. de relativo se denomina antecedente.

Tipos y Ejemplos de Subordinada Adjetiva

Adjetiva Especificativa → delimitan y restringen el significado del antecedente.


ZZ Un perro que está solo produce lástima → solo se refiere a perros solitarios.

Un perro que está solo produce lástima


N N.
N V.C.
Atributo
Det. N. S.N. (Suj.) (Pred. S.V.
Nominal) N.V. C.D.
Or. Subordinada Adjetiva
Especificat.


S.N. (Sujeto) S.V. (Pred. Verbal)

ZZ Los jugadores que habían sido seleccionados se concentraron ayer → solo nos referimos a jugadores seleccionados.

Los jugadores que habían sido seleccionados se concentraron ayer


N V.Aux. V.Aux. N.V. N.

S.N. Reflex N.V


Det. N. S.V. (Pred. Verbal) C.C.
(Suj.)
Tiempo
Or. Subord. Adjetiva Especificativa


S.N. (Sujeto) S.V. (Pred. Verbal)

YY Los alumnos que estudian aprobarán el examen → solo los que estudian
YY El jersey que me compré ayer se estropeó → solo el que me compré ayer
YY Retiraron los coches que aparcaron mal → solo los mal aparcados
YY El cine que está en la plaza estrena mañana → solo estrena ese cine

Adjetiva Explicativa → explica una característica del antecedente.

Nota: No limita el significado del antecedente y se escribe entre comas.

210 Mejores Personas, Mejores Familias


5º AÑO DE SECUNDARIA
Colegio

LINGÜÍSTICA  Mejores Personas, Mejores Familias

YY Los jugadores, que habían sido seleccionados, se concentraron ayer → todos fueron seleccionados

Los jugadores que habían sido seleccionados se concentraron ayer
N V.Aux. V.Aux. N.V. N.

S.N. Reflex N.V


Det. N. S.V. (Pred. Verbal) C.C.
(Suj.)
Tiempo
Or. Subord. Adjetiva Explicativa


S.N. (Sujeto) S.V. (Pred. Verbal)

YY R etiraron los coches, que aparcaron mal → retiraron todos por aparcar mal
YY Los alumnos, que estudiaron, aprobarán → aprobarán todos porque estudiaron

6. Es oración compuesta subordinada adjetiva:


a) Cuando la proposición subordinada cumple función de
modificador indirecto.
1. ¿Qué función cumple la proposición subordinada subra- b) Cuando la proposición subordinada cumple función de
yada en la oración: Lentamente, el hombre de sombrero de modificador directo.
paja y poncho gris miraba, como si fueran las últimas, las c) Cuando la proposición subordinada cumple función de
huellas que dejaban sus pies sobre el barro? circunstancial.
a) Objeto directo b) Objeto indirecto d) Cuando la proposición subordinada cumple función de
c) Circunstancial d) Modificador directo complemento predicativo.
e) Predicativo objetivo e) Cuando la proposición subordinada cumple función de
complemento agente.
2. El agua que trae Juan está purificada, ¿qué función cumple
lo subrayado dentro de la proposición subordinada? 7. En: «Dicen que tiene una pena; dicen que tiene una pena
a) objeto indirecto b) objeto directo que la hace llorar». Encontramos subordinadas de las si-
c) circunstancial d) modificador directo guientes clases:
e) sujeto a) 2 sustantivas – 1 adjetiva.
b) 3 sustantivas.
3. Señale la proposición subordinada adjetiva en la siguiente c) 1 sustantiva – 2 adjetivas.
oración: Cuando menos lo esperábamos, quien nos conocía, d) 1 sustantiva – 2 adjetivas.
trajo los libros que estábamos buscando. e) 3 adjetivas.
a) cuando menos lo esperábamos
b) quien nos conocía 8. En: «En La vida hay amores que nunca pueden olvidarse;
c) que estábamos buscando imborrables momentos que siempre guarda el corazón». Las
d) los libros subordinadas son de las siguientes clases:
e) nos conocía a) 1 sustantiva – 1 adjetiva
b) 2 adjetivas
4. En la oración: «Los hijos del capataz, que son solteros, via- c) 2 sustantivas
jaran al exterior», lo subrayado es una proposición subor- d) 1 sustantiva – 1 adverbial
dinada______________. e) 1 adversativa
a) sustantiva
b) adjetiva explicativa 9. «Ana saldrá con Manuel porque él se lo ha pedido; no im-
c) adjetiva especificativa porta que sea humilde porque ella lo ayudará para que sur-
d) adverbial de modo ja». La proposición subordinada subrayada cumple la fun-
e) adverbial de lugar ción de_________.
a) AP b) OD c) NS
5. Oración compuesta subordinada adjetiva: d) CIRC e) MD
a) El amigo por quien te preocupas te traicionó.
b) La vida que pasamos juntos fue muy especial. 10. En: «Miénteme con un beso que parezca de amor». La su-
c) Ellos compraron todo lo que pudieron. bordinada es__________.
d) Anita fue contratada por quien antes la despidió. a) Sust. b) Adj. c) Adv.
e) Yo soy quien más te quiere. d) Pron enclítico e) Verbo

5º AÑO DE SECUNDARIA
Mejores Personas, Mejores Familias 211
Colegio

Mejores Personas, Mejores Familias LINGÜÍSTICA

8. ¿Cuál es la oración que contiene una proposición subordi-


nada adjetiva?
a) Es cierto que conoce la historia que trataste de ocultar.
1. Las oraciones compuestas subordinadas adjetivas también b) Eso es lo que tú siempre quisiste.
son llamadas proposiciones_____. c) No importa que vengas tarde.
a) completivas b) apositivas c) de relativo d) Lo que he ganado ha sido muy poco.
d) sustantivas e) informativas e) Me entristece que mientas tanto.

2. Las oraciones compuestas subordinadas adjetivas se divi- 9. En la oración compuesta: «La joven, que te ama, está se-
den en________. gura de ti», la proposición subordinada está en función
a) apositivos y sustantivos de_______________.
b) completivas y de relativos a) aposición b) O. directo c) sujeto
c) explicativa y especificativa d) M. directo. e) complemento agente
d) explicativa y completivas
e) informativas y especificativas 10. Señale la oración compuesta que presenta proposición su-
bordinada adjetiva.
3. En la oración compuesta: «Los perros que no ladran no les a) Escribo mis poemas para los que son sensibles.
gustan a los alpaqueros», Hay una proposición subordina- b) Ellos serán quienes hagan el trabajo que debías hacer tú.
da______. c) Ya veremos quién se lo lleva.
a) sustantiva de sujeto b) sustantiva de complemento d) Estoy convencido de que lo lograré.
c) sustantiva atributiva d) adjetiva especificativa e) José es el que dirige a su hermano menor.
e) adjetiva explicativa

4. Señale la oración compuesta subordinada adjetiva especi-


ficativa.
a) Te suplico que me digas la verdad. 1. «La muchacha que le gusta a Sergio fue enamorada de
b) Las flores que me regalaron están secas. Enrique». La prop. sub. subrayada cumple la función
c) Escribo como me han enseñado. de________.
d) Eso es lo que busco. a) AP b) MD c) PVO
e) Quisiera salir, pero no puedo. d) MI e) NS

5. Señale la oración compuesta que presenta proposición su- 2. ¿Qué tipo de subordinación existe en la siguiente oración:
bordinada adjetiva. La carretera, serpiente que se extiende por la verde selva,
a) Permitid que por ahora calle su nombre. se ve desde el helicóptero?
b) Fue preciso que su prima la cogiese. a) Sustantiva b) Adjetiva c) Adverbial
c) Ha dicho que lo llames ahora. d) Condicional e) Coordinada
d) Él no es lo que parece.
e) No compró el carro que mencionó el comercial. 3. Oración compuesta subordinada adjetiva:
a) El llorar muchas veces ayuda a relajarse.
6. Marque la oración compuesta que contiene una proposición su- b) Todos deben de haber estado esperando la noticia.
bordinada en función de objeto directo: c) Soy lo que la vida me ha hecho.
a) Las papas están que queman. d) Con mentir nada vas a ganar.
b) No creo que haya leído ese libro. e) La vida que vivimos ha sido buena.
c) Es inútil que intentes verla.
d) Quien llora ahora reirá después. 4. Crea 2 oraciones subordinadas adjetivas.
e) El niño que te digo es pelirrojo.
• __________________________________________
7. Indique la oración compuesta que contiene una propo- __________________________________________
sición subordinada en función de modificador direc-
to. • __________________________________________
a) Me preguntaron quién había venido. __________________________________________
b) Lo busco y no lo encuentro.
c) El auto que compré será reparado por mi cuñado. 5. Señala la alternativa que presenta una proposición subor-
d) Fui castigado enérgicamente por quien tú sabes. dinada sustantiva.
e) Es una pena que Roberto sufra por mi culpa. a) Ana leyó el poema que escribe.
b) Que no vinieras me entristeció.
c) Julia volverá cuando tu regreses.
d) corrió tanto que quedó exhausto.
e) Olga sabe que Luis fuma mucho.

212 Mejores Personas, Mejores Familias


5º AÑO DE SECUNDARIA
Colegio

LINGÜÍSTICA  Mejores Personas, Mejores Familias

M A
TE

27   ORACIONES SUBORDINADAS ADVERBIALES

Las Oraciones Subordinadas Adverbiales son aquellas que desempeñan funciones sintácticas propias del adverbio. Se
clasifican en:

ZZ T
emporales (o de tiempo) → indican circunstancia temporal. Nexos: cuando, mientras, antes que, antes de que, des-
pués de que, etc.
– Lo hice cuando me dijiste.

ZZ Locales (o de lugar) → indican circunstancia espacial. Nexos: donde, precedida o no de preposición, etc.
– Lo coloqué donde me dijiste.

ZZ M
odales (o de modo) → indican el modo como se ejecuta la proposición principal. Nexos: como, según, conforme,
como si, etc.
– Lo hice como me dijiste.

ZZ C
omparativas → indican comparación de igualdad, inferioridad o superioridad respecto a la proposición principal.
– Compraremos tantos libros como quieras.

ZZ C
ondicionales → indican una condición respecto a la proposición principal. Nexos: si, en el caso de que, de + inf. etc.
– No conduzcas si bebes.

ZZ C
ausales → indican el origen lógico de la proposición principal. Nexos: porque, ya que, puesto que, por + inf, etc.
– Bebo porque tengo sed.

ZZ C
onsecutivas → indican la consecuencia de la principal. Nexos: por tanto, así que, conque, de manera/modo/forma
que, así pues, etc.
– Tengo sed, así que bebo.

ZZ Concesivas → indican un hecho que no impide el cumplimiento de la principal. Nexos: aunque, por más que, a pesar
de que, etc.
– Iré aunque llueva.

ZZ F
inales → indican la consecuencia última. Nexos: para que, a que, a fin de que, con la intención de, etc.
– Vino para arreglar la puerta.

2. Señale la relación incorrecta.


a) El té está que parece un caldero- atributo.
b) El cantante que escuchamos por la radio está de moda -
1. En la oración: Vino cuando se le solicitó, encontra- modificador directo.
mos________________. c) Quizás te preguntes por qué no asistí a tu boda - objeto
a) proposición subordinada sustantiva. directo.
b) proposición subordinada adjetiva. d) El área donde pateará el tiro libre es ideal para su pierna
c) proposición subordinada adverbial. zurda - circunstancial de lugar.
d) proposición coordinada. e) Me interrogó sobre cómo logramos esos efectos espe-
e) proposición yuxtapuesta. ciales - parte del circunstancial.

5º AÑO DE SECUNDARIA
Mejores Personas, Mejores Familias 213
Colegio

Mejores Personas, Mejores Familias LINGÜÍSTICA

3. Mientras no tomen bebidas alcohólicas, la fiesta será de lo me-


jor. En la oración anterior, encontramos una__________.
a) subordinada sustantiva.
b) subordinada adjetiva. 1. Marque la oración que no contiene proposición subordi-
c) subordinada adverbial. nada adverbial.
a) Cuando suene el primer disparo, corres.
d) coordinada ilativa.
b) La que se fue regresará pronto.
e) yuxtapuesta. c) Antes de que amanezca, ya estaba de pie.
d) Si no das el último examen, desaprobarás el curso.
4. ¿Cuántas proposiciones subordinadas encontramos en la e) Aunque no lo creas, dijo la verdad.
siguiente oración: «Yo pienso que no son tan inútiles las
cosas que vivimos ambos; sin embargo no puedo dejar que 2. En la oración: «Estuvo donde venden libros usados», la pro-
terminemos como enemigos porque siempre fuimos buenos posición es_________.
amigos»? a) sustantiva
a) uno b) cuatro c) dos b) adverbial de lugar
d) cinco e) tres c) adjetiva
d) adverbial de tiempo
e) adverbial de modo
5. «Cuando nadie me ve, puedo ser o no ser: cuando nadie me
ve, pongo el mundo al revés».
3. En la oración: «Ya que tengo sed, beberé algo», la proposi-
Las subordinaciones son______________. ción subordinada es____________.
a) 2 sustantivas b) 2 adjetivas c) 2 adverbiales a) adverbial consecutiva
d) 3 adverbiales e) 1 sustantiva y 1 adjetiva b) sustantiva
c) adverbial de finalidad
6. «Te vas porque yo quiero que te vayas». Encontramos su- d) adjetiva
bordinada _______. e) adverbial de causa
a) adverbial b) sustantiva c) adjetiva
d) yuxtapuesta e) de régimen 4. Las oraciones compuestas: «Cuando llegue la primavera,
todo florecerá» y «Ellos gritaron tanto que perdieron la voz»,
7. En: «La chica que te dijo que no le gusta que la beses y que son respectivamente, subordinadas adverbiales de______.
no te volverá a hablar si lo haces nuevamente ha regresado a) causa y modo
hoy», encontramos subordinación sustantiva en número b) consecutiva y finalidad
de ________. c) concesiva y modo
d) tiempo y consecutiva
a) uno b) cuatro c) dos
e) tiempo y concesiva
d) cinco e) tres
5. En la oración compuesta: «Saludó a sus profesores como le
8. En la oración; «Se lo dije porque estaba molesto», lo subra- había enseñado», lo subrayado constituye proposición su-
yado es una proporción subordinada adverbial de______. bordinada
a) modo b) causa c) tiempo a) adverbial concesiva
d) finalidad e) lugar b) adjetiva especificativa
c) adverbial modal
9. La función que cumple las subordinadas adverbiales es d) sustantiva o directo
de_____________. e) adverbial de finalidad
a) OD b) predicativo c) OI
d) agente e) circunstancial 6. En la oración compuesta: «Está preocupado porque le roba-
ron sus libros», lo subrayado constituye proposición subor-
dinada adverbial _________.
10. En la oración: «Para que seas feliz, me alejo de ti», lo
a) de finalidad b) modal c) concesiva
subrayado es una proposición subordinada adverbial
d) causal e) consecutiva
de______________.
a) lugar b) Cantidad c) Tiempo
7. El enunciado: La señora que vino encontró el lapicero que
d) Finalidad e) Modo
perdí cuando salimos de paseo, contiene _____________
proposiciones subordinadas.
a) dos b) cuatro c) seis
d) tres e) cinco

214 Mejores Personas, Mejores Familias


5º AÑO DE SECUNDARIA
Colegio

LINGÜÍSTICA  Mejores Personas, Mejores Familias

8. En la oración anterior encontramos: 2. En: «La oración compuesta es mi tema favorito; contestaré
a) dos subordinadas adjetivas. todas las preguntas; obtendré un hermoso veinte y mi papá
b) una subordinada adverbial. me dará permiso para ir a la reunión». Encontramos pro-
c) cuatro proposiciones. posiciones yuxtapuestas en número de _______.
d) una subordinada sustantiva. a) uno b) cuatro c) dos
e) a, b y c son correctas. d) cinco e) tres

9. Presenta proposición subordinada de lugar 3. En: «He tratado de salirme de esta historia, porque entien-
a) Limpia los ceniceros que están sucios. do que fui yo el último en llegar». Encontramos subordi-
b) Estudio donde estudió mi papá. nación de las siguientes clases _____.
c) Siento que se hayan molestado. a) 2 adv b) 2 adj c) 1 adv - 1 sust
d) Cuando llegue la primavera todo florecerá. d) 3 sust e) 2 sust
e) La persona a la que llamas está ausente.
4. En: «Yo sé que no lo harás pues dicen la verdad». Encontra-
10. Relación correcta. mos subordinada en número de____.
a) Saludó a su padre como le habían enseñado: sustantiva a) uno b) cuatro c) dos
b) Apenas llegó a la fiesta, se puso a bailar: adv. tiempo d) cinco e) tres
c) Quiero que escuches este poema: adjetiva.
d) Este es el lugar que me gusta mucho: adv. modo 5. Marca la alternativa presenta una proposición subordina-
e) El caminó hasta donde se lo indicaron: adv. tiempo da adverbial.
a) Me regalaron un auto que me gustó mucho.
b) Si mañana llegas temprano, iremos al cine.
c) Lo que más me gusta es leer.
d) Creen que María debe volver pronto.
1. Señale la oración que tiene tres proposiciones.
e) He esperado en vano; el carro no fue encontrado.
a) He terminado mi examen, así que trataré de soplar a
mis amigos.
b) Ha terminado el bimestre; sin embargo, aún no tengo
mis promedios, por tanto a corretear profesores.
c) A Martha le gusta un chico de su salón, por tanto lo
invitará a la graduación.
d) No sé la respuesta y estoy nervioso, pero trataré de co-
piarme.
e) El costo de la vida sube otra vez.

5º AÑO DE SECUNDARIA
Mejores Personas, Mejores Familias 215
Colegio

Mejores Personas, Mejores Familias LINGÜÍSTICA

216 Mejores Personas, Mejores Familias


5º AÑO DE SECUNDARIA

También podría gustarte